ΠšΠΎΠ½Ρ‚Ρ€ΠΎΠ»ΡŒΠ½Π°Ρ Ρ€Π°Π±ΠΎΡ‚Π° 1 ΠΌΠ°Π³Π½ΠΈΡ‚Π½ΠΎΠ΅ ΠΏΠΎΠ»Π΅: ΠšΠΎΠ½Ρ‚Ρ€ΠΎΠ»ΡŒΠ½Π°Ρ Ρ€Π°Π±ΠΎΡ‚Π° β„– 1 «ΠœΠ°Π³Π½ΠΈΡ‚Π½ΠΎΠ΅ ΠΏΠΎΠ»Π΅. ЭлСктромагнитная индукция» (11 класс)

Π‘ΠΎΠ΄Π΅Ρ€ΠΆΠ°Π½ΠΈΠ΅

ΠšΠΎΠ½Ρ‚Ρ€ΠΎΠ»ΡŒΠ½Π°Ρ Ρ€Π°Π±ΠΎΡ‚Π° β„– 1 «ΠœΠ°Π³Π½ΠΈΡ‚Π½ΠΎΠ΅ ΠΏΠΎΠ»Π΅. ЭлСктромагнитная индукция» (11 класс)

ДАВА УРОК β„– 11 ΠšΠ›ΠΠ‘Π‘ 11-Π°,Π±

Π’Π΅ΠΌΠ° ΡƒΡ€ΠΎΠΊΠ°:

ΠšΠΎΠ½Ρ‚Ρ€ΠΎΠ»ΡŒΠ½Π°Ρ Ρ€Π°Π±ΠΎΡ‚Π° β„–1 Β«ΠœΠ°Π³Π½ΠΈΡ‚Π½ΠΎΠ΅ ΠΏΠΎΠ»Π΅. ЭлСктромагнитная индукция»

Π¦Π΅Π»ΠΈ ΡƒΡ€ΠΎΠΊΠ°:

  • ΠžΠ±Ρ€Π°Π·ΠΎΠ²Π°Ρ‚Π΅Π»ΡŒΠ½Π°Ρ: ΠΏΡ€ΠΎΠ²Π΅Ρ€ΠΈΡ‚ΡŒ знания ΠΏΠΎ Ρ‚Π΅ΠΌΠ΅ Β«ΠœΠ°Π³Π½ΠΈΡ‚Π½ΠΎΠ΅ ΠΏΠΎΠ»Π΅. ЭлСктромагнитная индукция».

  • Π’ΠΎΡΠΏΠΈΡ‚Π°Ρ‚Π΅Π»ΡŒΠ½Π°Ρ: ΡΠΎΠ·Π΄Π°Ρ‚ΡŒ условия для ΠΏΠΎΠ»ΠΎΠΆΠΈΡ‚Π΅Π»ΡŒΠ½ΠΎΠΉ ΠΌΠΎΡ‚ΠΈΠ²Π°Ρ†ΠΈΠΈ ΠΏΡ€ΠΈ ΠΈΠ·ΡƒΡ‡Π΅Π½ΠΈΠΈ Ρ„ΠΈΠ·ΠΈΠΊΠΈ, ΠΈΡΠΏΠΎΠ»ΡŒΠ·ΡƒΡ Ρ€Π°Π·Π½ΠΎΠΎΠ±Ρ€Π°Π·Π½Ρ‹Π΅ ΠΏΡ€ΠΈΠ΅ΠΌΡ‹ Π΄Π΅ΡΡ‚Π΅Π»ΡŒΠ½ΠΎΡΡ‚ΠΈ, сообщая интСрСсныС свСдСния; Π²ΠΎΡΠΏΠΈΡ‚Ρ‹Π²Π°Ρ‚ΡŒ чувство уваТСния ΠΊ собСсСднику, ΠΈΠ½Π΄ΠΈΠ²ΠΈΠ΄ΡƒΠ°Π»ΡŒΠ½ΠΎΠΉ ΠΊΡƒΠ»ΡŒΡ‚ΡƒΡ€Ρ‹ общСния.

  • Π Π°Π·Π²ΠΈΠ²Π°ΡŽΡ‰Π°Ρ: Ρ€Π°Π·Π²ΠΈΠ²Π°Ρ‚ΡŒ умСния ΡΡ‚Ρ€ΠΎΠΈΡ‚ΡŒ Π»ΠΎΠ³ΠΈΡ‡Π΅ΡΠΊΡƒΡŽ Ρ†Π΅ΠΏΡŒ рассуТдСний, Ρ€Π°Π·Π²ΠΈΡ‚ΠΈΠ΅ логичСского ΠΌΡ‹ΡˆΠ»Π΅Π½ΠΈΡ, ΠΎΡ‚Ρ€Π°Π±ΠΎΡ‚Π°Ρ‚ΡŒ ΡƒΠΌΠ΅Π½ΠΈΠ΅ ΠΏΡ€ΠΈΠΌΠ΅Π½ΡΡ‚ΡŒ ΠΏΡ€ΠΈΠ΅ΠΌΡ‹ Π°Π½Π°Π»ΠΈΠ·Π°, сравнСния, Π΄ΠΎΠΊΠ°Π·Π°Ρ‚Π΅Π»ΡŒΡΡ‚Π²Π°, обобщСния, выдвиТСния Π³ΠΈΠΏΠΎΡ‚Π΅Π·, пСрСноса Π·Π½Π°Π½ΠΈΠΉ Π² Π½ΠΎΠ²ΡƒΡŽ ΡΠΈΡ‚ΡƒΠ°Ρ†ΠΈΡŽ.

ВрСбования ΠΊ знаниям, умСниям ΠΈ Π½Π°Π²Ρ‹ΠΊΠ°ΠΌ:

Π£Ρ‡Π΅Π½ΠΈΠΊ Π΄ΠΎΠ»ΠΆΠ΅Π½ Π·Π½Π°Ρ‚ΡŒ: ΠΎΠΏΡ€Π΅Π΄Π΅Π»Π΅Π½ΠΈΠ΅ ΠΌΠ°Π³Π½ΠΈΡ‚Π½ΠΎΠ³ΠΎ поля, элСктромагнитная индукция, ΠΌΠ°Π³Π½ΠΈΡ‚Π½Ρ‹ΠΉ ΠΏΠΎΡ‚ΠΎΠΊ, самоиндукция, сила АмпСра ΠΈ сила Π›ΠΎΡ€Π΅Π½Ρ†Π°.

Π£Ρ‡Π΅Π½ΠΈΠΊ Π΄ΠΎΠ»ΠΆΠ΅Π½ ΡƒΠΌΠ΅Ρ‚ΡŒ: ΠΏΡ€ΠΈΠΌΠ΅Π½ΡΡ‚ΡŒ ΠΏΠΎΠ»ΡƒΡ‡Π΅Π½Π½Ρ‹Π΅ знания ΠΏΡ€ΠΈ Π²Ρ‹ΠΏΠΎΠ»Π½Π΅Π½ΠΈΠΈ ΠΊΠΎΠ½Ρ‚Ρ€ΠΎΠ»ΡŒΠ½ΠΎΠΉ Ρ€Π°Π±ΠΎΡ‚Ρ‹.

ΠžΠ±ΠΎΡ€ΡƒΠ΄ΠΎΠ²Π°Π½ΠΈΠ΅ ΠΊ ΡƒΡ€ΠΎΠΊΡƒ: Ρ‚Π΅Ρ‚Ρ€Π°Π΄ΡŒ для ΠΊΠΎΠ½Ρ‚Ρ€ΠΎΠ»ΡŒΠ½Ρ‹Ρ… Ρ€Π°Π±ΠΎΡ‚, Ρ€Π°Π·Π΄Π°Ρ‚ΠΎΡ‡Π½Ρ‹ΠΉ ΠΌΠ°Ρ‚Π΅Ρ€ΠΈΠ°Π»

Π₯ΠΎΠ΄ ΡƒΡ€ΠΎΠΊΠ°:

  1. ΠžΡ€Π³Π°Π½ΠΈΠ·Π°Ρ†ΠΈΠΎΠ½Π½Ρ‹ΠΉ ΠΌΠΎΠΌΠ΅Π½Ρ‚

Π’Π·Π°ΠΈΠΌΠ½ΠΎΠ΅ привСтствиС учитСля ΠΈ учащихся. Π£Ρ‡ΠΈΡ‚Π΅Π»ΡŒ ΠΎΡ‚ΠΌΠ΅Ρ‡Π°Π΅Ρ‚ ΠΎΡ‚ΡΡƒΡ‚ΡΡ‚Π²ΡƒΡŽΡ‰ΠΈΡ… Π½Π° ΡƒΡ€ΠΎΠΊΠ΅.

ΠšΡ€ΠΈΡ‚Π΅Ρ€ΠΈΠΈ оцСнивания ΠΎΡ‚Π²Π΅Ρ‚ΠΎΠ²

Задания β„–1 – 5 ΠΎΡ†Π΅Π½ΠΈΠ²Π°ΡŽΡ‚ΡΡ ΠΏΠΎ 1 Π±Π°Π»Π»Ρƒ, Π° задания 6 ΠΈ 7 ΠΏΠΎ 3 Π±Π°Π»Π»Π°.

ΠšΡ€ΠΈΡ‚Π΅Ρ€ΠΈΠΈ ΠΎΡ†Π΅Π½ΠΊΠΈ ΠΎΡ‚Π²Π΅Ρ‚Π° ΠΊ заданиям 6, 7

Π‘Π°Π»Π»

ΠŸΡ€ΠΈΠ²Π΅Π΄Π΅Π½ΠΎ ΠΏΠΎΠ»Π½ΠΎΠ΅ ΠΏΡ€Π°Π²ΠΈΠ»ΡŒΠ½ΠΎΠ΅ Ρ€Π΅ΡˆΠ΅Π½ΠΈΠ΅, Π²ΠΊΠ»ΡŽΡ‡Π°ΡŽΡ‰Π΅Π΅ ΡΠ»Π΅Π΄ΡƒΡŽΡ‰ΠΈΠ΅ элСмСнты:

  • Π²Π΅Ρ€Π½ΠΎ записано ΠΊΡ€Π°Ρ‚ΠΊΠΎΠ΅ условиС Π·Π°Π΄Π°Ρ‡ΠΈ;

  • записаны уравнСния ΠΈ Ρ„ΠΎΡ€ΠΌΡƒΠ»Ρ‹, ΠΏΡ€ΠΈΠΌΠ΅Π½Π΅Π½ΠΈΠ΅ ΠΊΠΎΡ‚ΠΎΡ€Ρ‹Ρ… Π½Π΅ΠΎΠ±Ρ…ΠΎΠ΄ΠΈΠΌΠΎ ΠΈ достаточно для Ρ€Π΅ΡˆΠ΅Π½ΠΈΡ Π·Π°Π΄Π°Ρ‡ΠΈ Π²Ρ‹Π±Ρ€Π°Π½Π½Ρ‹ΠΌ способом;

  • Π²Ρ‹ΠΏΠΎΠ»Π½Π΅Π½Ρ‹ Π½Π΅ΠΎΠ±Ρ…ΠΎΠ΄ΠΈΠΌΡ‹Π΅ матСматичСскиС прСобразования ΠΈ расчёты, приводящиС ΠΊ ΠΏΡ€Π°Π²ΠΈΠ»ΡŒΠ½ΠΎΠΌΡƒ числовому ΠΎΡ‚Π²Π΅Ρ‚Ρƒ, ΠΈ прСдставлСн ΠΎΡ‚Π²Π΅Ρ‚

3

ΠŸΡ€Π°Π²ΠΈΠ»ΡŒΠ½ΠΎ записаны Π½Π΅ΠΎΠ±Ρ…ΠΎΠ΄ΠΈΠΌΡ‹Π΅ Ρ„ΠΎΡ€ΠΌΡƒΠ»Ρ‹, ΠΏΡ€ΠΎΠ²Π΅Π΄Π΅Π½Ρ‹ вычислСния, ΠΏΠΎΠ»ΡƒΡ‡Π΅Π½ ΠΎΡ‚Π²Π΅Ρ‚ (Π²Π΅Ρ€Π½Ρ‹ΠΉ ΠΈΠ»ΠΈ Π½Π΅Π²Π΅Ρ€Π½Ρ‹ΠΉ), Π½ΠΎ Π΄ΠΎΠΏΡƒΡ‰Π΅Π½ ошибка Π² записи ΠΊΡ€Π°Ρ‚ΠΊΠΎΠ³ΠΎ условия ΠΈΠ»ΠΈ ΠΏΡ€ΠΈ ΠΏΠ΅Ρ€Π΅Π²ΠΎΠ΄Π΅ Π΅Π΄ΠΈΠ½ΠΈΡ† Π² БИ.

Π˜Π›Π˜

ΠŸΡ€Π΅Π΄ΡΡ‚Π°Π²Π»Π΅Π½ΠΎ ΠΏΡ€Π°Π²ΠΈΠ»ΡŒΠ½ΠΎΠ΅ Ρ€Π΅ΡˆΠ΅Π½ΠΈΠ΅ Ρ‚ΠΎΠ»ΡŒΠΊΠΎ Π² ΠΎΠ±Ρ‰Π΅ΠΌ Π²ΠΈΠ΄Π΅, Π±Π΅Π· ΠΊΠ°ΠΊΠΈΡ…-Π»ΠΈΠ±ΠΎ числовых расчСтов.

Π˜Π›Π˜

Записаны уравнСния ΠΈ Ρ„ΠΎΡ€ΠΌΡƒΠ»Ρ‹, ΠΏΡ€ΠΈΠΌΠ΅Π½Π΅Π½ΠΈΠ΅ ΠΊΠΎΡ‚ΠΎΡ€Ρ‹Ρ… Π½Π΅ΠΎΠ±Ρ…ΠΎΠ΄ΠΈΠΌΠΎ ΠΈ достаточно для Ρ€Π΅ΡˆΠ΅Π½ΠΈΡ Π·Π°Π΄Π°Ρ‡ΠΈ Π²Ρ‹Π±Ρ€Π°Π½Π½Ρ‹ΠΌ способом, Π½ΠΎ Π² матСматичСских прСобразованиях ΠΈΠ»ΠΈ вычислСниях Π΄ΠΎΠΏΡƒΡ‰Π΅Π½Π° ошибка

2

Записаны ΠΈ ΠΈΡΠΏΠΎΠ»ΡŒΠ·ΠΎΠ²Π°Π½Ρ‹ Π½Π΅ всС исходныС Ρ„ΠΎΡ€ΠΌΡƒΠ»Ρ‹, Π½Π΅ΠΎΠ±Ρ…ΠΎΠ΄ΠΈΠΌΡ‹Π΅ для Ρ€Π΅ΡˆΠ΅Π½ΠΈΡ Π·Π°Π΄Π°Ρ‡ΠΈ.

Π˜Π›Π˜

Записаны всС исходныС Ρ„ΠΎΡ€ΠΌΡƒΠ»Ρ‹, Π½ΠΎ Π² ΠΎΠ΄Π½ΠΎΠΉ ΠΈΠ· Π½ΠΈΡ… Π΄ΠΎΠΏΡƒΡ‰Π΅Π½Π° ошибка

1

ВсС случаи Ρ€Π΅ΡˆΠ΅Π½ΠΈΡ, ΠΊΠΎΡ‚ΠΎΡ€Ρ‹Π΅ Π½Π΅ ΡΠΎΠΎΡ‚Π²Π΅Ρ‚ΡΡ‚Π²ΡƒΡŽΡ‚ Π²Ρ‹ΡˆΠ΅ΡƒΠΊΠ°Π·Π°Π½Π½Ρ‹ΠΌ критСриям выставлСния ΠΎΡ†Π΅Π½ΠΎΠΊ Π² 1,2,3 Π±Π°Π»Π»Π°

0

БистСма ΠΎΡ†Π΅Π½ΠΊΠΈ тСстов Π½Π΅ являСтся ΡΠ°ΠΌΠΎΡ†Π΅Π»ΡŒΡŽ. Она лишь ΠΎΡ€ΠΈΠ΅Π½Ρ‚ΠΈΡ€ΠΎΠ²Π°Π½Π° Π½Π° систСму ΠΎΡ†Π΅Π½ΠΎΠΊ Π·Π°Π΄Π°Π½ΠΈΠΉ Π•Π“Π­, с Ρ‚Π΅ΠΌ Ρ‡Ρ‚ΠΎΠ±Ρ‹ ΡƒΡ‡Π΅Π½ΠΈΠΊΠΈ постСпСнно ΠΏΡ€ΠΈΠ²Ρ‹ΠΊΠ»ΠΈ ΠΊ Π΄Ρ€ΡƒΠ³ΠΎΠΌΡƒ Π²ΠΈΠ΄Ρƒ ΠΎΡ†Π΅Π½ΠΊΠΈ Π·Π½Π°Π½ΠΈΠΉ ΠΈ ΡƒΠΌΠ΅Π½ΠΈΠΉ ΠΈ ΠΏΠΎΠ½ΠΈΠΌΠ°Π»ΠΈ соотвСтствиС этой ΠΎΡ†Π΅Π½ΠΊΠΈ ΠΎΡ†Π΅Π½ΠΊΠ΅ ΠΏΠΎ Ρ‚Ρ€Π°Π΄ΠΈΡ†ΠΈΠΎΠ½Π½ΠΎΠΉ, ΠΏΡΡ‚ΠΈΠ±Π°Π»Π»ΡŒΠ½ΠΎΠΉ систСмС.

75% ΠΎΡ‚ максимальной суммы Π±Π°Π»Π»ΠΎΠ² – ΠΎΡ†Π΅Π½ΠΊΠ° Β«5Β»;

50-74% — ΠΎΡ†Π΅Π½ΠΊΠ° Β«4Β»;

30-49% — ΠΎΡ†Π΅Π½ΠΊΠ° Β«3Β»;

0-29% — ΠΎΡ†Π΅Π½ΠΊΠ° Β«2Β».

  1. Актуализация Π·Π½Π°Π½ΠΈΠΉ

Π’Π°Ρ€ΠΈΠ°Π½Ρ‚ 1

1. ΠœΠ°Π³Π½ΠΈΡ‚Π½ΠΎΠ΅ ΠΏΠΎΠ»Π΅ создаСтся

1) элСктричСскими зарядами

2) ΠΌΠ°Π³Π½ΠΈΡ‚Π½Ρ‹ΠΌΠΈ зарядами

3) двиТущимися элСктричСскими зарядами

4) Π»ΡŽΠ±Ρ‹ΠΌ Ρ‚Π΅Π»ΠΎΠΌ

2. Π›ΠΈΠ½ΠΈΠΈ ΠΌΠ°Π³Π½ΠΈΡ‚Π½ΠΎΠΉ ΠΈΠ½Π΄ΡƒΠΊΡ†ΠΈΠΈ Π²ΠΎΠΊΡ€ΡƒΠ³ ΠΏΡ€ΠΎΠ²ΠΎΠ΄Π½ΠΈΠΊΠ° с Ρ‚ΠΎΠΊΠΎΠΌ ΠΏΡ€Π°Π²ΠΈΠ»ΡŒΠ½ΠΎ ΠΏΠΎΠΊΠ°Π·Π°Π½Ρ‹ Π² случаС

1) А 3) Π’

2) Π‘ 4) Π“

3. ΠŸΡ€ΡΠΌΠΎΠ»ΠΈΠ½Π΅ΠΉΠ½Ρ‹ΠΉ ΠΏΡ€ΠΎΠ²ΠΎΠ΄Π½ΠΈΠΊ с Ρ‚ΠΎΠΊΠΎΠΌ I находится ΠΌΠ΅ΠΆΠ΄Ρƒ полюсами ΠΌΠ°Π³Π½ΠΈΡ‚Π° (ΠΏΡ€ΠΎΠ²ΠΎΠ΄Π½ΠΈΠΊ располоТСн пСрпСндикулярно плоскости листа, Ρ‚ΠΎΠΊ Ρ‚Π΅Ρ‡Π΅Ρ‚ ΠΊ Ρ‡ΠΈΡ‚Π°Ρ‚Π΅Π»ΡŽ). Π‘ΠΈΠ»Π° АмпСра, Π΄Π΅ΠΉΡΡ‚Π²ΡƒΡŽΡ‰Π°Ρ Π½Π° ΠΏΡ€ΠΎΠ²ΠΎΠ΄Π½ΠΈΠΊ, Π½Π°ΠΏΡ€Π°Π²Π»Π΅Π½Π°

1) Π²ΠΏΡ€Π°Π²ΠΎ 3) Π²Π²Π΅Ρ€Ρ…

2) Π²Π»Π΅Π²ΠΎ 4) Π²Π½ΠΈΠ·

4. ВраСктория ΠΏΠΎΠ»Π΅Ρ‚Π° элСктрона, Π²Π»Π΅Ρ‚Π΅Π²ΡˆΠ΅Π³ΠΎ Π² ΠΎΠ΄Π½ΠΎΡ€ΠΎΠ΄Π½ΠΎΠ΅ ΠΌΠ°Π³Π½ΠΈΡ‚Π½ΠΎΠ΅ ΠΏΠΎΠ»Π΅ ΠΏΠΎΠ΄ ΡƒΠ³Π»ΠΎΠΌ 60˚

1) прямая 3) ΠΏΠ°Ρ€Π°Π±ΠΎΠ»Π°

2) ΠΎΠΊΡ€ΡƒΠΆΠ½ΠΎΡΡ‚ΡŒ 4) винтовая линия

5. Π’ ΠΊΠ°Ρ‚ΡƒΡˆΠΊΠ΅ ΠΈΠ½Π΄ΡƒΠΊΡ‚ΠΈΠ²Π½ΠΎΡΡ‚ΡŒΡŽ 0,08 Π“Π½ ΠΏΡ€ΠΎΡ…ΠΎΠ΄ΠΈΡ‚ Ρ‚ΠΎΠΊ силой 20 А. ΠžΠΏΡ€Π΅Π΄Π΅Π»ΠΈΡ‚Π΅ Π­Π”Π‘ самоиндукции, которая Π²ΠΎΠ·Π½ΠΈΠΊΠ°Π΅Ρ‚ Π² ΠΊΠ°Ρ‚ΡƒΡˆΠΊΠ΅ ΠΏΡ€ΠΈ исчСзновСнии Π² Π½Π΅ΠΉ Ρ‚ΠΎΠΊΠ° Π·Π° 0,002 с.

6. ΠŸΡ€ΠΎΠ²ΠΎΠ΄Π½ΠΈΠΊ Π΄Π»ΠΈΠ½ΠΎΠΉ 15 см подвСшСн Π³ΠΎΡ€ΠΈΠ·ΠΎΠ½Ρ‚Π°Π»ΡŒΠ½ΠΎ Π½Π° Π΄Π²ΡƒΡ… нСвСсомых нитях Π² ΠΌΠ°Π³Π½ΠΈΡ‚Π½ΠΎΠΌ ΠΏΠΎΠ»Π΅ ΠΈΠ½Π΄ΡƒΠΊΡ†ΠΈΠ΅ΠΉ 60 ΠΌΠ’Π», ΠΏΡ€ΠΈ Ρ‡Π΅ΠΌ Π»ΠΈΠ½ΠΈΠΈ ΠΈΠ½Π΄ΡƒΠΊΡ†ΠΈΠΈ Π½Π°ΠΏΡ€Π°Π²Π»Π΅Π½Ρ‹ Π²Π²Π΅Ρ€Ρ… пСрпСндикулярно ΠΏΡ€ΠΎΠ²ΠΎΠ΄Π½ΠΈΠΊΡƒ.

Π°) По ΠΏΡ€ΠΎΠ²ΠΎΠ΄Π½ΠΈΠΊΡƒ пропустили Ρ‚ΠΎΠΊ. Π‘ΠΈΠ»Π° Ρ‚ΠΎΠΊΠ° 2 А. Π‘ ΠΊΠ°ΠΊΠΎΠΉ силой ΠΌΠ°Π³Π½ΠΈΡ‚Π½ΠΎΠ΅ ΠΏΠΎΠ»Π΅ дСйствуСт Π½Π° ΠΏΡ€ΠΎΠ²ΠΎΠ΄Π½ΠΈΠΊ? Π£ΠΊΠ°ΠΆΠΈΡ‚Π΅ Π½Π°ΠΏΡ€Π°Π²Π»Π΅Π½ΠΈΠ΅ этой силы Π½Π° рисункС.

Π±) На ΠΊΠ°ΠΊΠΎΠΉ ΡƒΠ³ΠΎΠ» ΠΎΡ‚ Π²Π΅Ρ€Ρ‚ΠΈΠΊΠ°Π»ΠΈ отклонятся Π½ΠΈΡ‚ΠΈ, Π½Π° ΠΊΠΎΡ‚ΠΎΡ€Ρ‹Ρ… висит ΠΏΡ€ΠΎΠ²ΠΎΠ΄Π½ΠΈΠΊ? Масса ΠΏΡ€ΠΎΠ²ΠΎΠ΄Π½ΠΈΠΊΠ° 10 Π³.

7. ΠŸΡ€ΠΎΡ‚ΠΎΠ½ Π²Π»Π΅Ρ‚Π°Π΅Ρ‚ Π² ΠΌΠ°Π³Π½ΠΈΡ‚Π½ΠΎΠ΅ ΠΏΠΎΠ»Π΅ ΠΈΠ½Π΄ΡƒΠΊΡ†ΠΈΠ΅ΠΉ 20 ΠΌΠ’Π» со ΡΠΊΠΎΡ€ΠΎΡΡ‚ΡŒΡŽ 10 ΠΊΠΌ/с ΠΏΠΎΠ΄ ΡƒΠ³Π»ΠΎΠΌ 30˚ ΠΊ линиям ΠΌΠ°Π³Π½ΠΈΡ‚Π½ΠΎΠΉ ΠΈΠ½Π΄ΡƒΠΊΡ†ΠΈΠΈ.

Π°) Π‘ ΠΊΠ°ΠΊΠΎΠΉ силой ΠΌΠ°Π³Π½ΠΈΡ‚Π½ΠΎΠ΅ ΠΏΠΎΠ»Π΅ дСйствуСт Π½Π° ΠΏΡ€ΠΎΡ‚ΠΎΠ½? Заряд ΠΏΡ€ΠΎΡ‚ΠΎΠ½Π° Π΅ = 1,6 βˆ™ 10-19 Кл.

Π±) Π—Π° ΠΊΠ°ΠΊΠΎΠ΅ врСмя ΠΏΡ€ΠΎΡ‚ΠΎΠ½ ΡΠΎΠ²Π΅Ρ€ΡˆΠΈΡ‚ ΠΎΠ΄ΠΈΠ½ ΠΏΠΎΠ»Π½Ρ‹ΠΉ ΠΎΠ±ΠΎΡ€ΠΎΡ‚ Π²ΠΎΠΊΡ€ΡƒΠ³ Π»ΠΈΠ½ΠΈΠΉ ΠΌΠ°Π³Π½ΠΈΡ‚Π½ΠΎΠΉ ΠΈΠ½Π΄ΡƒΠΊΡ†ΠΈΠΈ? Масса ΠΏΡ€ΠΎΡ‚ΠΎΠ½Π° 1,67 βˆ™ 10-27 ΠΊΠ³

Π’Π°Ρ€ΠΈΠ°Π½Ρ‚ 2

1. ДвиТущийся элСктричСский заряд создаСт

1) Ρ‚ΠΎΠ»ΡŒΠΊΠΎ элСктричСскоС ΠΏΠΎΠ»Π΅

2) Ρ‚ΠΎΠ»ΡŒΠΊΠΎ ΠΌΠ°Π³Π½ΠΈΡ‚Π½ΠΎΠ΅ ΠΏΠΎΠ»Π΅

3) ΠΊΠ°ΠΊ элСктричСскоС, Ρ‚Π°ΠΊ ΠΈ ΠΌΠ°Π³Π½ΠΈΡ‚Π½ΠΎΠ΅

4) Ρ‚ΠΎΠ»ΡŒΠΊΠΎ Π³Ρ€Π°Π²ΠΈΡ‚Π°Ρ†ΠΈΠΎΠ½Π½ΠΎΠ΅ ΠΏΠΎΠ»Π΅

2. На рисункС ΠΈΠ·ΠΎΠ±Ρ€Π°ΠΆΠ΅Π½ цилиндричСский ΠΏΡ€ΠΎΠ²ΠΎΠ΄Π½ΠΈΠΊ, ΠΏΠΎ ΠΊΠΎΡ‚ΠΎΡ€ΠΎΠΌΡƒ ΠΈΠ΄Π΅Ρ‚ элСктричСский Ρ‚ΠΎΠΊ. НаправлСниС Ρ‚ΠΎΠΊΠ° ΡƒΠΊΠ°Π·Π°Π½ΠΎ стрСлкой. Как Π½Π°ΠΏΡ€Π°Π²Π»Π΅Π½ Π²Π΅ΠΊΡ‚ΠΎΡ€ ΠΌΠ°Π³Π½ΠΈΡ‚Π½ΠΎΠΉ ΠΈΠ½Π΄ΡƒΠΊΡ†ΠΈΠΈ Π² Ρ‚ΠΎΡ‡ΠΊΠ΅ Π‘?

1) Π² плоскости Ρ‡Π΅Ρ€Ρ‚Π΅ΠΆΠ° Π²Π²Π΅Ρ€Ρ…

2) Π² плоскости Ρ‡Π΅Ρ€Ρ‚Π΅ΠΆΠ° Π²Π½ΠΈΠ·

3) ΠΎΡ‚ нас пСрпСндикулярно плоскости Ρ‡Π΅Ρ€Ρ‚Π΅ΠΆΠ°

4) ΠΊ Π½Π°ΠΌ пСрпСндикулярно плоскости Ρ‡Π΅Ρ€Ρ‚Π΅ΠΆΠ°

3. Π‘ΠΊΠΎΡ€ΠΎΡΡ‚ΡŒ элСктрона Π½Π°ΠΏΡ€Π°Π²Π»Π΅Π½Π° пСрпСндикулярно ΠΌΠ°Π³Π½ΠΈΡ‚Π½ΠΎΠΉ ΠΈΠ½Π΄ΡƒΠΊΡ†ΠΈΠΈ. Π‘ΠΈΠ»Π° Π›ΠΎΡ€Π΅Π½Ρ†Π° Π½Π°ΠΏΡ€Π°Π²Π»Π΅Π½Π°

1) Π²ΠΏΡ€Π°Π²ΠΎ 3) Π²Π²Π΅Ρ€Ρ…

2) Π²Π»Π΅Π²ΠΎ 4) Π²Π½ΠΈΠ·

4. Π›Π΅Π³ΠΊΠΎΠ΅ мСталличСскоС ΠΊΠΎΠ»ΡŒΡ†ΠΎ подвСшСно Π½Π° Π½ΠΈΡ‚ΠΈ. ΠŸΡ€ΠΈ Π²Π΄Π²ΠΈΠ³Π°Π½ΠΈΠΈ Π² ΠΊΠΎΠ»ΡŒΡ†ΠΎ постоянного ΠΌΠ°Π³Π½ΠΈΡ‚Π° ΠΎΠ½ΠΎ отталкиваСтся ΠΎΡ‚ Π½Π΅Π³ΠΎ. Π­Ρ‚ΠΎ ΠΎΠ±ΡŠΡΡΠ½ΡΠ΅Ρ‚ΡΡ

1) Π½Π°ΠΌΠ°Π³Π½ΠΈΡ‡ΠΈΠ²Π°Π½ΠΈΠ΅ΠΌ ΠΊΠΎΠ»ΡŒΡ†Π°

2) элСктризациСй ΠΊΠΎΠ»ΡŒΡ†Π°

3) Π²ΠΎΠ·Π½ΠΈΠΊΠ½ΠΎΠ²Π΅Π½ΠΈΠ΅ΠΌ Π² ΠΊΠΎΠ»ΡŒΡ†Π΅ ΠΈΠ½Π΄ΡƒΠΊΡ†ΠΈΠΎΠ½Π½ΠΎΠ³ΠΎ Ρ‚ΠΎΠΊΠ°

4) Π²ΠΎΠ·Π½ΠΈΠΊΠ½ΠΎΠ²Π΅Π½ΠΈΠ΅ΠΌ Π² ΠΌΠ°Π³Π½ΠΈΡ‚Π΅ ΠΈΠ½Π΄ΡƒΠΊΡ†ΠΈΠΎΠ½Π½ΠΎΠ³ΠΎ Ρ‚ΠΎΠΊΠ°

5. ΠžΠΏΡ€Π΅Π΄Π΅Π»ΠΈΡ‚Π΅ ΠΈΠ½Π΄ΡƒΠΊΡ‚ΠΈΠ²Π½ΠΎΡΡ‚ΡŒ ΠΊΠ°Ρ‚ΡƒΡˆΠΊΠΈ, Ссли извСстно, Ρ‡Ρ‚ΠΎ сила Ρ‚ΠΎΠΊΠ° Π² Ρ†Π΅ΠΏΠΈ Π·Π° 0,02 с возрастаСт Π΄ΠΎ максимума ΠΈ Ρ€Π°Π²Π½Π° 4 А., создавая ΠΏΡ€ΠΈ этом Π­Π”Π‘ самоиндукции 12 Π’.

6. На Π΄Π²ΡƒΡ… Π³ΠΎΡ€ΠΈΠ·ΠΎΠ½Ρ‚Π°Π»ΡŒΠ½Ρ‹Ρ… Ρ€Π΅Π»ΡŒΡΠ°Ρ…, расстояниС ΠΌΠ΅ΠΆΠ΄Ρƒ ΠΊΠΎΡ‚ΠΎΡ€Ρ‹ΠΌΠΈ 50 см, Π»Π΅ΠΆΠΈΡ‚ мСталличСский ΡΡ‚Π΅Ρ€ΠΆΠ΅Π½ΡŒ, сила Ρ‚ΠΎΠΊΠ° Π² ΠΊΠΎΡ‚ΠΎΡ€ΠΎΠΌ 5 А. Π Π΅Π»ΡŒΡΡ‹ ΠΈ ΡΡ‚Π΅Ρ€ΠΆΠ΅Π½ΡŒ находятся Π² ΠΎΠ΄Π½ΠΎΡ€ΠΎΠ΄Π½ΠΎΠΌ ΠΌΠ°Π³Π½ΠΈΡ‚Π½ΠΎΠΌ ΠΏΠΎΠ»Π΅ ΠΈΠ½Π΄ΡƒΠΊΡ†ΠΈΠ΅ΠΉ 50 ΠΌΠ’Π», Π½Π°ΠΏΡ€Π°Π²Π»Π΅Π½Π½ΠΎΠΌ пСрпСндикулярно Ρ€Π΅Π»ΡŒΡΠ°ΠΌ ΠΈ ΡΡ‚Π΅Ρ€ΠΆΠ½ΡŽ.

Π°) Π‘ ΠΊΠ°ΠΊΠΎΠΉ силой ΠΌΠ°Π³Π½ΠΈΡ‚Π½ΠΎΠ΅ ΠΏΠΎΠ»Π΅ дСйствуСт Π½Π° ΡΡ‚Π΅Ρ€ΠΆΠ΅Π½ΡŒ? На рисункС ΡƒΠΊΠ°ΠΆΠΈΡ‚Π΅ Π½Π°ΠΏΡ€Π°Π²Π»Π΅Π½ΠΈΠ΅ этой силы.

Π±) ΠŸΡ€ΠΈ ΠΊΠ°ΠΊΠΎΠΌ Π·Π½Π°Ρ‡Π΅Π½ΠΈΠΈ коэффициСнта трСния стСрТня ΠΎ Ρ€Π΅Π»ΡŒΡΡ‹ ΠΎΠ½ Π±ΡƒΠ΄Π΅Ρ‚ Π΄Π²ΠΈΠ³Π°Ρ‚ΡŒΡΡ прямолинСйно ΠΈ Ρ€Π°Π²Π½ΠΎΠΌΠ΅Ρ€Π½ΠΎ? Масса стСрТня 125 Π³.

7. Π­Π»Π΅ΠΊΡ‚Ρ€ΠΎΠ½ Π²Π»Π΅Ρ‚Π°Π΅Ρ‚ Π² ΠΌΠ°Π³Π½ΠΈΡ‚Π½ΠΎΠ΅ ΠΏΠΎΠ»Π΅ ΠΈΠ½Π΄ΡƒΠΊΡ†ΠΈΠ΅ΠΉ 10 ΠΌΠ’Π» пСрпСндикулярно линиям ΠΌΠ°Π³Π½ΠΈΡ‚Π½ΠΎΠΉ ΠΈΠ½Π΄ΡƒΠΊΡ†ΠΈΠΈ со ΡΠΊΠΎΡ€ΠΎΡΡ‚ΡŒΡŽ 1 Мм/с.

Π°) Π§Π΅ΠΌΡƒ Ρ€Π°Π²Π΅Π½ радиус ΠΊΡ€ΠΈΠ²ΠΈΠ·Π½Ρ‹ Ρ‚Ρ€Π°Π΅ΠΊΡ‚ΠΎΡ€ΠΈΠΈ, ΠΏΠΎ ΠΊΠΎΡ‚ΠΎΡ€ΠΎΠΌΡƒ двиТСтся элСктрон? ΠœΠΎΠ΄ΡƒΠ»ΡŒ заряда элСктрона Π΅ = 1,6 βˆ™ 10-19 Кл, Π΅Π³ΠΎ масса m = 9,1 βˆ™ 10-31 ΠΊΠ³.

Π±) Π‘ ΠΊΠ°ΠΊΠΎΠΉ частотой обращаСтся элСктрон?

  1. БистСматизация Π·Π½Π°Π½ΠΈΠΉ

Π£Ρ‡ΠΈΡ‚Π΅Π»ΡŒ собираСт Ρ‚Π΅Ρ‚Ρ€Π°Π΄ΠΈ для ΠΊΠΎΠ½Ρ‚Ρ€ΠΎΠ»ΡŒΠ½Ρ‹Ρ… Ρ€Π°Π±ΠΎΡ‚, Π° ΡƒΡ‡Π΅Π½ΠΈΠΊ — листы с заданиями.

  1. ПодвСдСниС ΠΈΡ‚ΠΎΠ³ΠΎΠ²

ΠŸΡ€ΠΎΠ²Π΅Π΄Ρ‘ΠΌ Π°Π½Π°Π»ΠΈΠ· своСй Π΄Π΅ΡΡ‚Π΅Π»ΡŒΠ½ΠΎΡΡ‚ΠΈ ΠΏΠΎ ΠΈΡ‚ΠΎΠ³Π°ΠΌ ΡƒΡ€ΠΎΠΊΠ°:

-Ρ‡Ρ‚ΠΎ Π²Ρ‹ ΠΌΠΎΠΆΠ΅Ρ‚Π΅ Π΄Π΅Π»Π°Ρ‚ΡŒ сами Π±Π΅Π· ΠΏΠΎΠΌΠΎΡ‰ΠΈ учитСля?

-ΠΊΠ°ΠΊΠΈΠ΅ трудности Π²Ρ‹ испытывали Π² процСссС Ρ€Π°Π±ΠΎΡ‚Ρ‹?

-Ρ‚Π΅ΠΏΠ΅Ρ€ΡŒ я знаю, ΠΏΠΎΠ½ΠΈΠΌΠ°ΡŽβ€¦β€¦.

-Ρ‚Π΅ΠΏΠ΅Ρ€ΡŒ я ΠΌΠΎΠ³Ρƒ ΠΎΠ±ΡŠΡΡΠ½ΠΈΡ‚ΡŒ как……., почСму…..?

-Ρ‚Π΅ΠΏΠ΅Ρ€ΡŒ я ΠΌΠΎΠ³Ρƒ ΠΏΡ€ΠΈΠΌΠ΅Π½ΠΈΡ‚ΡŒ ……….

— ΠΊΠ°ΠΊΠΈΠ΅ задания Π²Π°ΠΌ показались Π½Π°ΠΈΠ±ΠΎΠ»Π΅Π΅ интСрСсными? Π’Ρ€ΡƒΠ΄Π½Ρ‹ΠΌΠΈ? Π’Π°ΠΆΠ½Ρ‹ΠΌΠΈ?

  1. Π”ΠΎΠΌΠ°ΡˆΠ½Π΅Π΅ Π·Π°Π΄Π°Π½ΠΈΠ΅

Β§1-11 ΠΏΠΎΠ²Ρ‚ΠΎΡ€ΠΈΡ‚ΡŒ

ΠšΠΎΠ½Ρ‚Ρ€ΠΎΠ»ΡŒΠ½Π°Ρ Ρ€Π°Π±ΠΎΡ‚Π° β„– 1. Π’Π΅ΠΌΠ°. ΠœΠ°Π³Π½ΠΈΡ‚Π½ΠΎΠ΅ ΠΏΠΎΠ»Π΅. ЭлСктромагнитная индукция. 11 класс

ΠšΠΎΠ½Ρ‚Ρ€ΠΎΠ»ΡŒΠ½Π°Ρ Ρ€Π°Π±ΠΎΡ‚Π° β„– 1. Π’Π΅ΠΌΠ°. ΠœΠ°Π³Π½ΠΈΡ‚Π½ΠΎΠ΅ ΠΏΠΎΠ»Π΅. ЭлСктромагнитная индукция.

I Π²Π°Ρ€ΠΈΠ°Π½Ρ‚

β„–β„– 1- 11 Π²Ρ‹Π±Ρ€Π°Ρ‚ΡŒ ΠΎΠ΄ΠΈΠ½ ΠΏΡ€Π°Π²ΠΈΠ»ΡŒΠ½Ρ‹ΠΉ ΠΎΡ‚Π²Π΅Ρ‚ (1 Π±Π°Π»Π»)

1. ΠœΠ°Π³Π½ΠΈΡ‚Π½ΠΎΠ΅ ΠΏΠΎΠ»Π΅ сущСствуСт…

А. Π²ΠΎΠΊΡ€ΡƒΠ³ двиТущихся элСктричСских зарядов.

Π‘. Π²ΠΎΠΊΡ€ΡƒΠ³ Π»ΡŽΠ±Ρ‹Ρ… элСктричСских зарядов.

Π’. Π²ΠΎΠΊΡ€ΡƒΠ³ ΠΌΠ°Π³Π½ΠΈΡ‚Π½Ρ‹Ρ… зарядов. Π“. Π²ΠΎΠΊΡ€ΡƒΠ³ любого Ρ‚Π΅Π»Π°.

2. На ΠΊΠ°ΠΊΠΎΠΌ ΠΈΠ· рисунков ΠΏΡ€Π°Π²ΠΈΠ»ΡŒΠ½ΠΎ ΠΏΠΎΠΊΠ°Π·Π°Π½ΠΎ Π½Π°ΠΏΡ€Π°Π²Π»Π΅Π½ΠΈΠ΅ ΠΈΠ½Π΄ΡƒΠΊΡ†ΠΈΠΈ ΠΌΠ°Π³Π½ΠΈΡ‚Π½ΠΎΠ³ΠΎ поля, созданного прямым ΠΏΡ€ΠΎΠ²ΠΎΠ΄Π½ΠΈΠΊΠΎΠΌ с Ρ‚ΠΎΠΊΠΎΠΌ.

А. рис. А Π‘. рис. Π‘ Π’. рис. Π’

3. ΠžΠΏΡ€Π΅Π΄Π΅Π»ΠΈΡ‚Π΅ ΠΏΠΎΠ»ΡŽΡΡ‹ ΠΊΠ°Ρ‚ΡƒΡˆΠΊΠΈ с Ρ‚ΠΎΠΊΠΎΠΌ.

А. сСвСр Π²Π²Π΅Ρ€Ρ…Ρƒ Π‘. сСвСр Π²Π½ΠΈΠ·Ρƒ

Π’. сСвСр справа Π“. сСвСр слСва

4. Как Π½Π°ΠΏΡ€Π°Π²Π»Π΅Π½Π° сила, Π΄Π΅ΠΉΡΡ‚Π²ΡƒΡŽΡ‰Π°Ρ Π½Π° ΠΏΡ€ΠΎΠ²ΠΎΠ΄Π½ΠΈΠΊ с Ρ‚ΠΎΠΊΠΎΠΌ Π² ΠΌΠ°Π³Π½ΠΈΡ‚Π½ΠΎΠΌ ΠΏΠΎΠ»Π΅.

А. Π²Π²Π΅Ρ€Ρ… Π‘. Π²Π½ΠΈΠ·

Π’. ΠΊ Π½Π°ΠΌ Π“. ΠΎΡ‚ нас

5. ΠŸΡ€ΡΠΌΠΎΠ»ΠΈΠ½Π΅ΠΉΠ½Ρ‹ΠΉ ΠΏΡ€ΠΎΠ²ΠΎΠ΄Π½ΠΈΠΊ c Ρ‚ΠΎΠΊΠΎΠΌ ΠΏΠΎΠΌΠ΅Ρ‰Ρ‘Π½ Π² ΠΎΠ΄Π½ΠΎΡ€ΠΎΠ΄Π½ΠΎΠ΅ ΠΌΠ°Π³Π½ΠΈΡ‚Π½ΠΎΠ΅ ΠΏΠΎΠ»Π΅ пСрпСндикулярно линиям ΠΈΠ½Π΄ΡƒΠΊΡ†ΠΈΠΈ. Как измСнится сила АмпСра, Π΄Π΅ΠΉΡΡ‚Π²ΡƒΡŽΡ‰Π°Ρ Π½Π° ΠΏΡ€ΠΎΠ²ΠΎΠ΄Π½ΠΈΠΊ, Ссли ΠΈΠ½Π΄ΡƒΠΊΡ†ΠΈΡŽ ΠΌΠ°Π³Π½ΠΈΡ‚Π½ΠΎΠ³ΠΎ поля ΡƒΠ²Π΅Π»ΠΈΡ‡ΠΈΡ‚ΡŒ Π² 3 Ρ€Π°Π·Π°?

А. увСличится Π² 3 Ρ€Π°Π·Π° Π‘. увСличится Π² 9 Ρ€Π°Π·

Π’. ΡƒΠΌΠ΅Π½ΡŒΡˆΠΈΡ‚ΡΡ Π² 3 Ρ€Π°Π·Π° Π“. ΡƒΠΌΠ΅Π½ΡŒΡˆΠΈΡ‚ΡΡ Π² 9 Ρ€Π°Π·

6. ΠŸΡ€ΡΠΌΠΎΠ»ΠΈΠ½Π΅ΠΉΠ½Ρ‹ΠΉ ΠΏΡ€ΠΎΠ²ΠΎΠ΄Π½ΠΈΠΊ Π΄Π»ΠΈΠ½ΠΎΠΉ 10 см находится Π² ΠΎΠ΄Π½ΠΎΡ€ΠΎΠ΄Π½ΠΎΠΌ ΠΌΠ°Π³Π½ΠΈΡ‚Π½ΠΎΠΌ ΠΏΠΎΠ»Π΅ с ΠΈΠ½Π΄ΡƒΠΊΡ†ΠΈΠ΅ΠΉ 4 Π’Π» ΠΈ располоТСн ΠΏΠΎΠ΄ ΡƒΠ³Π»ΠΎΠΌ 30° к Π²Π΅ΠΊΡ‚ΠΎΡ€Ρƒ ΠΌΠ°Π³Π½ΠΈΡ‚Π½ΠΎΠΉ ΠΈΠ½Π΄ΡƒΠΊΡ†ΠΈΠΈ. Π§Π΅ΠΌΡƒ Ρ€Π°Π²Π½Π° сила, Π΄Π΅ΠΉΡΡ‚Π²ΡƒΡŽΡ‰Π°Ρ Π½Π° ΠΏΡ€ΠΎΠ²ΠΎΠ΄Π½ΠΈΠΊ со стороны ΠΌΠ°Π³Π½ΠΈΡ‚Π½ΠΎΠ³ΠΎ поля, Ссли сила Ρ‚ΠΎΠΊΠ° Π² ΠΏΡ€ΠΎΠ²ΠΎΠ΄Π½ΠΈΠΊΠ΅ 3 А?

А. 1,2 Н Π‘. 0,6 Н Π’. 2,4 Н Π“. 60 Н

7. Π­Π»Π΅ΠΊΡ‚Ρ€ΠΎΠ½ Π²Π»Π΅Ρ‚Π°Π΅Ρ‚ Π² ΠΌΠ°Π³Π½ΠΈΡ‚Π½ΠΎΠ΅ ΠΏΠΎΠ»Π΅. ΠžΠΏΡ€Π΅Π΄Π΅Π»ΠΈΡ‚Π΅ Π½Π°ΠΏΡ€Π°Π²Π»Π΅Π½ΠΈΠ΅ силы Π›ΠΎΡ€Π΅Π½Ρ†Π°, Π΄Π΅ΠΉΡΡ‚Π²ΡƒΡŽΡ‰Π΅ΠΉ Π½Π° элСктрон.

А. Π²Π²Π΅Ρ€Ρ… Π‘. Π²Π½ΠΈΠ·

Π’. ΠΊ Π½Π°ΠΌ Π“. ΠΎΡ‚ нас

8. ΠœΠ°Π³Π½ΠΈΡ‚ вводится Π² алюминиСвоС ΠΊΠΎΠ»ΡŒΡ†ΠΎ Ρ‚Π°ΠΊ, ΠΊΠ°ΠΊ ΠΏΠΎΠΊΠ°Π·Π°Π½ΠΎ Π½Π° рисункС. НаправлСниС Ρ‚ΠΎΠΊΠ° Π² ΠΊΠΎΠ»ΡŒΡ†Π΅ ΡƒΠΊΠ°Π·Π°Π½ΠΎ стрСлкой. Каким полюсом ΠΌΠ°Π³Π½ΠΈΡ‚ вводится Π² ΠΊΠΎΠ»ΡŒΡ†ΠΎ?

А. ΠΏΠΎΠ»ΠΎΠΆΠΈΡ‚Π΅Π»ΡŒΠ½Ρ‹ΠΌ Π‘. ΠΎΡ‚Ρ€ΠΈΡ†Π°Ρ‚Π΅Π»ΡŒΠ½Ρ‹ΠΌ

Π’. сСвСрным Π“. ΡŽΠΆΠ½Ρ‹ΠΌ

9. Π—Π° 5 с ΠΌΠ°Π³Π½ΠΈΡ‚Π½Ρ‹ΠΉ ΠΏΠΎΡ‚ΠΎΠΊ, ΠΏΡ€ΠΎΠ½ΠΈΠ·Ρ‹Π²Π°ΡŽΡ‰ΠΈΠΉ ΠΏΡ€ΠΎΠ²ΠΎΠ»ΠΎΡ‡Π½ΡƒΡŽ Ρ€Π°ΠΌΠΊΡƒ, увСличился ΠΎΡ‚ 3 Π΄ΠΎ 8 Π’Π±. Π§Π΅ΠΌΡƒ Ρ€Π°Π²Π½ΠΎ ΠΏΡ€ΠΈ этом Π·Π½Π°Ρ‡Π΅Π½ΠΈΠ΅ Π­Π”Π‘ ΠΈΠ½Π΄ΡƒΠΊΡ†ΠΈΠΈ Π² Ρ€Π°ΠΌΠΊΠ΅?

А. 0,5 Π’ Π‘. 2,5 Π’ Π’. 1 Π’ Π“. 25 Π’

10. На рисункС ΠΏΠΎΠΊΠ°Π·Π°Π½ΠΎ ΠΈΠ·ΠΌΠ΅Π½Π΅Π½ΠΈΠ΅ силы Ρ‚ΠΎΠΊΠ° Π² ΠΊΠ°Ρ‚ΡƒΡˆΠΊΠ΅ индуктивности ΠΎΡ‚ Π²Ρ€Π΅ΠΌΠ΅Π½ΠΈ. ΠœΠΎΠ΄ΡƒΠ»ΡŒ Π­Π”Π‘ самоиндукции ΠΏΡ€ΠΈΠ½ΠΈΠΌΠ°Π΅Ρ‚ наибольшСС Π·Π½Π°Ρ‡Π΅Π½ΠΈΠ΅ Π² ΠΏΡ€ΠΎΠΌΠ΅ΠΆΡƒΡ‚ΠΊΠ΅ Π²Ρ€Π΅ΠΌΠ΅Π½ΠΈ

А. 0-1 с Π‘. 5-6 с

Π’. 1-5 с Π“. 6-8 с

11. На рисункС прСдставлСн Π³Ρ€Π°Ρ„ΠΈΠΊ измСнСния силы Ρ‚ΠΎΠΊΠ° с Ρ‚Π΅Ρ‡Π΅Π½ΠΈΠ΅ΠΌ Π²Ρ€Π΅ΠΌΠ΅Π½ΠΈ Π² ΠΊΠ°Ρ‚ΡƒΡˆΠΊΠ΅ ΠΈΠ½Π΄ΡƒΠΊΡ‚ΠΈΠ²Π½ΠΎΡΡ‚ΡŒΡŽ 6 ΠΌΠ“Π½. ΠžΠΏΡ€Π΅Π΄Π΅Π»ΠΈΡ‚Π΅ Π·Π½Π°Ρ‡Π΅Π½ΠΈΠ΅ Π­Π”Π‘ самоиндукции.

А. 3 ΠΌΠ’ Π‘. 6 ΠΌΠ’ Π’. 9 ΠΌΠ’ Π“. 12 ΠΌΠ’

β„– 12 установитС соотвСтствиС (2 Π±Π°Π»Π»Π°)

12. Частица массой m, нСсущая заряд q, двиТСтся Π² ΠΎΠ΄Π½ΠΎΡ€ΠΎΠ΄Π½ΠΎΠΌ ΠΌΠ°Π³Π½ΠΈΡ‚Π½ΠΎΠΌ ΠΏΠΎΠ»Π΅ с ΠΈΠ½Π΄ΡƒΠΊΡ†ΠΈΠ΅ΠΉ Π’ ΠΏΠΎ окруТности радиуса R со ΡΠΊΠΎΡ€ΠΎΡΡ‚ΡŒΡŽ Ο…. Π§Ρ‚ΠΎ ΠΏΡ€ΠΎΠΈΠ·ΠΎΠΉΠ΄Ρ‘Ρ‚ с радиусом ΠΎΡ€Π±ΠΈΡ‚Ρ‹, ΠΏΠ΅Ρ€ΠΈΠΎΠ΄ΠΎΠΌ обращСния ΠΈ кинСтичСской энСргиСй частицы ΠΏΡ€ΠΈ ΡƒΠ²Π΅Π»ΠΈΡ‡Π΅Π½ΠΈΠΈ ΠΈΠ½Π΄ΡƒΠΊΡ†ΠΈΠΈ ΠΌΠ°Π³Π½ΠΈΡ‚Π½ΠΎΠ³ΠΎ поля?

ЀизичСская Π²Π΅Π»ΠΈΡ‡ΠΈΠ½Π°

А. радиус ΠΎΡ€Π±ΠΈΡ‚Ρ‹
Π‘. ΠΏΠ΅Ρ€ΠΈΠΎΠ΄ обращСния
Π’. кинСтичСская энСргия

Π•Π΅ ΠΈΠ·ΠΌΠ΅Π½Π΅Π½ΠΈΠ΅

1. увСличится
2. ΡƒΠΌΠ΅Π½ΡŒΡˆΠΈΡ‚ΡΡ
3. Π½Π΅ измСнится

β„– 13 Ρ€Π΅ΡˆΠΈΡ‚ΡŒ Π·Π°Π΄Π°Ρ‡Ρƒ (3 Π±Π°Π»Π»Π°)

13. Π‘ ΠΊΠ°ΠΊΠΎΠΉ ΡΠΊΠΎΡ€ΠΎΡΡ‚ΡŒΡŽ Π²Ρ‹Π»Π΅Ρ‚Π°Π΅Ρ‚ Π°-частица ΠΈΠ· Ρ€Π°Π΄ΠΈΠΎΠ°ΠΊΡ‚ΠΈΠ²Π½ΠΎΠ³ΠΎ ядра, Ссли ΠΎΠ½Π°, попадая Π² ΠΎΠ΄Π½ΠΎΡ€ΠΎΠ΄Π½ΠΎΠ΅ ΠΌΠ°Π³Π½ΠΈΡ‚Π½ΠΎΠ΅ ΠΏΠΎΠ»Π΅ ΠΈΠ½Π΄ΡƒΠΊΡ†ΠΈΠ΅ΠΉ 2 Π’Π» пСрпСндикулярно Π΅Π³ΠΎ силовым линиям, двиТСтся ΠΏΠΎ Π΄ΡƒΠ³Π΅ окруТности радиусом 1 ΠΌ? (Масса Π°-частицы 6,7βˆ™10-27 ΠΊΠ³, Π΅Ρ‘ заряд Ρ€Π°Π²Π΅Π½ 3,2βˆ™10-19 Кл)

1-6 Π±Π°Π»Π»ΠΎΠ²

7-12 Π±Π°Π»Π»ΠΎΠ²

13-14

15-16 Π±Π°Π»Π»ΠΎΠ²

2

3

4

5

ΠšΠΎΠ½Ρ‚Ρ€ΠΎΠ»ΡŒΠ½Π°Ρ Ρ€Π°Π±ΠΎΡ‚Π° β„– 1. Π’Π΅ΠΌΠ°. ΠœΠ°Π³Π½ΠΈΡ‚Π½ΠΎΠ΅ ΠΏΠΎΠ»Π΅. ЭлСктромагнитная индукция.

II Π²Π°Ρ€ΠΈΠ°Π½Ρ‚

β„–β„– 1- 11 Π²Ρ‹Π±Ρ€Π°Ρ‚ΡŒ ΠΎΠ΄ΠΈΠ½ ΠΏΡ€Π°Π²ΠΈΠ»ΡŒΠ½Ρ‹ΠΉ ΠΎΡ‚Π²Π΅Ρ‚ (1 Π±Π°Π»Π»)

1. Π’ΠΎΠΊΡ€ΡƒΠ³ двиТущСгося элСктричСского заряда сущСствуСт…

А. Волько ΠΌΠ°Π³Π½ΠΈΡ‚Π½ΠΎΠ΅ ΠΏΠΎΠ»Π΅.

Π‘. Волько элСктричСскоС ΠΏΠΎΠ»Π΅.

Π’. ЭлСктричСскоС ΠΈ ΠΌΠ°Π³Π½ΠΈΡ‚Π½ΠΎΠ΅ поля. Π“. Никакого поля Π½Π΅ сущСствуСт.

2. На ΠΊΠ°ΠΊΠΎΠΌ ΠΈΠ· рисунков ΠΏΡ€Π°Π²ΠΈΠ»ΡŒΠ½ΠΎ ΠΏΠΎΠΊΠ°Π·Π°Π½ΠΎ Π½Π°ΠΏΡ€Π°Π²Π»Π΅Π½ΠΈΠ΅ ΠΈΠ½Π΄ΡƒΠΊΡ†ΠΈΠΈ ΠΌΠ°Π³Π½ΠΈΡ‚Π½ΠΎΠ³ΠΎ поля, созданного прямым ΠΏΡ€ΠΎΠ²ΠΎΠ΄Π½ΠΈΠΊΠΎΠΌ с Ρ‚ΠΎΠΊΠΎΠΌ.

А. рис. А Π‘. рис. Π‘ Π’. рис. Π’

3. ΠžΠΏΡ€Π΅Π΄Π΅Π»ΠΈΡ‚Π΅ ΠΏΠΎΠ»ΡŽΡΡ‹ ΠΊΠ°Ρ‚ΡƒΡˆΠΊΠΈ с Ρ‚ΠΎΠΊΠΎΠΌ.

А. сСвСр Π²Π²Π΅Ρ€Ρ…Ρƒ Π‘. сСвСр Π²Π½ΠΈΠ·Ρƒ

Π’. сСвСр справа Π“. сСвСр слСва

4. Как Π½Π°ΠΏΡ€Π°Π²Π»Π΅Π½Π° сила, Π΄Π΅ΠΉΡΡ‚Π²ΡƒΡŽΡ‰Π°Ρ Π½Π° ΠΏΡ€ΠΎΠ²ΠΎΠ΄Π½ΠΈΠΊ с Ρ‚ΠΎΠΊΠΎΠΌ Π² ΠΌΠ°Π³Π½ΠΈΡ‚Π½ΠΎΠΌ ΠΏΠΎΠ»Π΅.

А. Π²Π²Π΅Ρ€Ρ… Π‘. Π²Π»Π΅Π²ΠΎ

Π’. сила Ρ€Π°Π²Π½Π° Π½ΡƒΠ»ΡŽ Π“. Π²ΠΏΡ€Π°Π²ΠΎ

5. ΠŸΡ€ΡΠΌΠΎΠ»ΠΈΠ½Π΅ΠΉΠ½Ρ‹ΠΉ ΠΏΡ€ΠΎΠ²ΠΎΠ΄Π½ΠΈΠΊ c Ρ‚ΠΎΠΊΠΎΠΌ ΠΏΠΎΠΌΠ΅Ρ‰Ρ‘Π½ Π² ΠΎΠ΄Π½ΠΎΡ€ΠΎΠ΄Π½ΠΎΠ΅ ΠΌΠ°Π³Π½ΠΈΡ‚Π½ΠΎΠ΅ ΠΏΠΎΠ»Π΅ пСрпСндикулярно линиям ΠΈΠ½Π΄ΡƒΠΊΡ†ΠΈΠΈ. Как измСнится сила АмпСра, Π΄Π΅ΠΉΡΡ‚Π²ΡƒΡŽΡ‰Π°Ρ Π½Π° ΠΏΡ€ΠΎΠ²ΠΎΠ΄Π½ΠΈΠΊ, Ссли Π΅Π³ΠΎ Π΄Π»ΠΈΠ½Ρƒ ΡƒΠΌΠ΅Π½ΡŒΡˆΠΈΡ‚ΡŒ Π² 2 Ρ€Π°Π·Π°?

А. увСличится Π² 4 Ρ€Π°Π·Π° Π‘. увСличится Π² 2 Ρ€Π°Π·Π°

Π’. ΡƒΠΌΠ΅Π½ΡŒΡˆΠΈΡ‚ΡΡ Π² 4 Ρ€Π°Π·Π° Π“. ΡƒΠΌΠ΅Π½ΡŒΡˆΠΈΡ‚ΡΡ Π² 2 Ρ€Π°Π·Π°

6. ΠŸΡ€ΡΠΌΠΎΠ»ΠΈΠ½Π΅ΠΉΠ½Ρ‹ΠΉ ΠΏΡ€ΠΎΠ²ΠΎΠ΄Π½ΠΈΠΊ Π΄Π»ΠΈΠ½ΠΎΠΉ 5 см находится Π² ΠΎΠ΄Π½ΠΎΡ€ΠΎΠ΄Π½ΠΎΠΌ ΠΌΠ°Π³Π½ΠΈΡ‚Π½ΠΎΠΌ ΠΏΠΎΠ»Π΅ с ΠΈΠ½Π΄ΡƒΠΊΡ†ΠΈΠ΅ΠΉ 5 Π’Π» ΠΈ располоТСн ΠΏΠΎΠ΄ ΡƒΠ³Π»ΠΎΠΌ 30° к Π²Π΅ΠΊΡ‚ΠΎΡ€Ρƒ ΠΌΠ°Π³Π½ΠΈΡ‚Π½ΠΎΠΉ ΠΈΠ½Π΄ΡƒΠΊΡ†ΠΈΠΈ. Π§Π΅ΠΌΡƒ Ρ€Π°Π²Π½Π° сила, Π΄Π΅ΠΉΡΡ‚Π²ΡƒΡŽΡ‰Π°Ρ Π½Π° ΠΏΡ€ΠΎΠ²ΠΎΠ΄Π½ΠΈΠΊ со стороны ΠΌΠ°Π³Π½ΠΈΡ‚Π½ΠΎΠ³ΠΎ поля, Ссли сила Ρ‚ΠΎΠΊΠ° Π² ΠΏΡ€ΠΎΠ²ΠΎΠ΄Π½ΠΈΠΊΠ΅ 2 А?

А. 0,25 Н Π‘. 0,5 Н Π’. 1,5 Н Π“. 25 Н

7. ΠŸΡ€ΠΎΡ‚ΠΎΠ½ Π²Π»Π΅Ρ‚Π°Π΅Ρ‚ Π² ΠΌΠ°Π³Π½ΠΈΡ‚Π½ΠΎΠ΅ ΠΏΠΎΠ»Π΅. ΠžΠΏΡ€Π΅Π΄Π΅Π»ΠΈΡ‚Π΅ Π½Π°ΠΏΡ€Π°Π²Π»Π΅Π½ΠΈΠ΅ силы Π›ΠΎΡ€Π΅Π½Ρ†Π°, Π΄Π΅ΠΉΡΡ‚Π²ΡƒΡŽΡ‰Π΅ΠΉ Π½Π° ΠΏΡ€ΠΎΡ‚ΠΎΠ½.

А. Π²Π²Π΅Ρ€Ρ… Π‘. Π²Π½ΠΈΠ·

Π’. ΠΊ Π½Π°ΠΌ Π“. ΠΎΡ‚ нас

8. ΠœΠ°Π³Π½ΠΈΡ‚ Π²Ρ‹Π΄Π²ΠΈΠ³Π°ΡŽΡ‚ ΠΈΠ· алюминиСвого ΠΊΠΎΠ»ΡŒΡ†Π° Ρ‚Π°ΠΊ, ΠΊΠ°ΠΊ

ΠΏΠΎΠΊΠ°Π·Π°Π½ΠΎ Π½Π° рисункС. НаправлСниС Ρ‚ΠΎΠΊΠ° Π² ΠΊΠΎΠ»ΡŒΡ†Π΅ ΡƒΠΊΠ°Π·Π°Π½ΠΎ

стрСлкой. Каким полюсом ΠΌΠ°Π³Π½ΠΈΡ‚ Π²Ρ‹Π΄Π²ΠΈΠ³Π°ΡŽΡ‚ ΠΈΠ· ΠΊΠΎΠ»ΡŒΡ†Π°?

А. ΠΏΠΎΠ»ΠΎΠΆΠΈΡ‚Π΅Π»ΡŒΠ½Ρ‹ΠΌ Π‘. ΠΎΡ‚Ρ€ΠΈΡ†Π°Ρ‚Π΅Π»ΡŒΠ½Ρ‹ΠΌ

Π’. сСвСрным Π“. ΡŽΠΆΠ½Ρ‹ΠΌ

9. Π—Π° 5 мс Π² солСноидС, содСрТащСм 100 Π²ΠΈΡ‚ΠΊΠΎΠ² ΠΏΡ€ΠΎΠ²ΠΎΠ΄Π°, ΠΌΠ°Π³Π½ΠΈΡ‚Π½Ρ‹ΠΉ ΠΏΠΎΡ‚ΠΎΠΊ Ρ€Π°Π²Π½ΠΎΠΌΠ΅Ρ€Π½ΠΎ ΡƒΠ±Ρ‹Π²Π°Π΅Ρ‚ ΠΎΡ‚ 8 Π΄ΠΎ 4 ΠΌΠ’Π±. НайдитС Π­Π”Π‘ ΠΈΠ½Π΄ΡƒΠΊΡ†ΠΈΠΈ Π² Ρ€Π°ΠΌΠΊΠ΅.

А. 125 Π’ Π‘. 12,5 Π’ Π’. 8 Π’ Π“. 80 Π’

10. На рисункС ΠΏΠΎΠΊΠ°Π·Π°Π½ΠΎ ΠΈΠ·ΠΌΠ΅Π½Π΅Π½ΠΈΠ΅ силы Ρ‚ΠΎΠΊΠ° Π² ΠΊΠ°Ρ‚ΡƒΡˆΠΊΠ΅ индуктивности ΠΎΡ‚ Π²Ρ€Π΅ΠΌΠ΅Π½ΠΈ. ΠœΠΎΠ΄ΡƒΠ»ΡŒ Π­Π”Π‘ самоиндукции ΠΏΡ€ΠΈΠ½ΠΈΠΌΠ°Π΅Ρ‚ Ρ€Π°Π²Π½Ρ‹Π΅ значСния Π² ΠΏΡ€ΠΎΠΌΠ΅ΠΆΡƒΡ‚ΠΊΠ°Ρ… Π²Ρ€Π΅ΠΌΠ΅Π½ΠΈ

А. 0-l c ΠΈ l-3 c Π‘. 3-4 с ΠΈ 4-7 с

Π’. 1-3 с ΠΈ 4-7 с Π“. 0-1 с ΠΈ 3-4 с

11. Π‘ΠΈΠ»Π° Ρ‚ΠΎΠΊΠ° Π² ΠΊΠ°Ρ‚ΡƒΡˆΠΊΠ΅ ΠΈΠ½Π΄ΡƒΠΊΡ‚ΠΈΠ²Π½ΠΎΡΡ‚ΡŒΡŽ 0,25 Π“Π½ измСняСтся с Ρ‚Π΅Ρ‡Π΅Π½ΠΈΠ΅ΠΌ Π²Ρ€Π΅ΠΌΠ΅Π½ΠΈ, ΠΊΠ°ΠΊ ΠΏΠΎΠΊΠ°Π·Π°Π½ΠΎ Π½Π° Π³Ρ€Π°Ρ„ΠΈΠΊΠ΅. ΠžΠΏΡ€Π΅Π΄Π΅Π»ΠΈΡ‚Π΅ Π­Π”Π‘ самоиндукции, которая Π²ΠΎΠ·Π½ΠΈΠΊΠ°Π΅Ρ‚ Π² ΠΊΠ°Ρ‚ΡƒΡˆΠΊΠ΅.

А. 0,25 Π’ Π‘. 2,5 Π’ Π’. 1 Π’ Π“. 25 Π’

β„– 12 установитС соотвСтствиС (2 Π±Π°Π»Π»Π°)

12. Частица массой m, нСсущая заряд q, двиТСтся Π² ΠΎΠ΄Π½ΠΎΡ€ΠΎΠ΄Π½ΠΎΠΌ ΠΌΠ°Π³Π½ΠΈΡ‚Π½ΠΎΠΌ ΠΏΠΎΠ»Π΅ с ΠΈΠ½Π΄ΡƒΠΊΡ†ΠΈΠ΅ΠΉ Π’ ΠΏΠΎ окруТности радиуса R со ΡΠΊΠΎΡ€ΠΎΡΡ‚ΡŒΡŽ Ο…. Π§Ρ‚ΠΎ ΠΏΡ€ΠΎΠΈΠ·ΠΎΠΉΠ΄Ρ‘Ρ‚ с радиусом ΠΎΡ€Π±ΠΈΡ‚Ρ‹, ΠΏΠ΅Ρ€ΠΈΠΎΠ΄ΠΎΠΌ обращСния ΠΈ ΠΈΠΌΠΏΡƒΠ»ΡŒΡΠΎΠΌ частицы ΠΏΡ€ΠΈ ΡƒΠΌΠ΅Π½ΡŒΡˆΠ΅Π½ΠΈΠΈ ΠΈΠ½Π΄ΡƒΠΊΡ†ΠΈΠΈ ΠΌΠ°Π³Π½ΠΈΡ‚Π½ΠΎΠ³ΠΎ поля?

ЀизичСская Π²Π΅Π»ΠΈΡ‡ΠΈΠ½Π°

А. радиус ΠΎΡ€Π±ΠΈΡ‚Ρ‹
Π‘. ΠΏΠ΅Ρ€ΠΈΠΎΠ΄ обращСния
Π’. ΠΈΠΌΠΏΡƒΠ»ΡŒΡ частицы

Π•Π΅ ΠΈΠ·ΠΌΠ΅Π½Π΅Π½ΠΈΠ΅

1. увСличится
2. ΡƒΠΌΠ΅Π½ΡŒΡˆΠΈΡ‚ΡΡ
3. Π½Π΅ измСнится

β„– 13 Ρ€Π΅ΡˆΠΈΡ‚ΡŒ Π·Π°Π΄Π°Ρ‡Ρƒ (3 Π±Π°Π»Π»Π°)

13. Какой Π΄ΠΎΠ»ΠΆΠ½Π° Π±Ρ‹Ρ‚ΡŒ индукция ΠΎΠ΄Π½ΠΎΡ€ΠΎΠ΄Π½ΠΎΠ³ΠΎ ΠΌΠ°Π³Π½ΠΈΡ‚Π½ΠΎΠ³ΠΎ поля, Ρ‡Ρ‚ΠΎΠ±Ρ‹ двиТущийся со ΡΠΊΠΎΡ€ΠΎΡΡ‚ΡŒΡŽ 200 ΠΊΠΌ/с ΠΏΡ€ΠΎΡ‚ΠΎΠ½ описал Π² этом ΠΏΠΎΠ»Π΅ ΠΎΠΊΡ€ΡƒΠΆΠ½ΠΎΡΡ‚ΡŒ радиусом 20 см?

1-6 Π±Π°Π»Π»ΠΎΠ²

7-12 Π±Π°Π»Π»ΠΎΠ²

13-14

15-16 Π±Π°Π»Π»ΠΎΠ²

2

3

4

5

ΠšΠΎΠ½Ρ‚Ρ€ΠΎΠ»ΡŒΠ½Π°Ρ Ρ€Π°Π±ΠΎΡ‚Π° Π½Π° Ρ‚Π΅ΠΌΡƒ «ΠœΠ°Π³Π½ΠΈΡ‚Π½ΠΎΠ΅ ΠΏΠΎΠ»Π΅. ЭлСктромагнитная индукция»

ΠšΠΎΠ½Ρ‚Ρ€ΠΎΠ»ΡŒΠ½Π°Ρ Ρ€Π°Π±ΠΎΡ‚Π° Β« ΠœΠ°Π³Π½ΠΈΡ‚Π½ΠΎΠ΅ ΠΏΠΎΠ»Π΅. ЭлСктромагнитная индукция» Π’ΠΠ Π˜ΠΠΠ’ 1

1.Какова индукция ΠΌΠ°Π³Π½ΠΈΡ‚Π½ΠΎΠ³ΠΎ поля, Π² ΠΊΠΎΡ‚ΠΎΡ€ΠΎΠΌ Π½Π° ΠΏΡ€ΠΎΠ²ΠΎΠ΄Π½ΠΈΠΊ Π΄Π»ΠΈΠ½ΠΎΠΉ Π°ΠΊΡ‚ΠΈΠ²Π½ΠΎΠΉ части 5 см, дСйствуСт

сила 50 мН. Π‘ΠΈΠ»Π° Ρ‚ΠΎΠΊΠ° Π² ΠΏΡ€ΠΎΠ²ΠΎΠ΄Π½ΠΈΠΊΠ΅ 25 А. НаправлСниС Π²Π΅ΠΊΡ‚ΠΎΡ€Π° ΠΌΠ°Π³Π½ΠΈΡ‚Π½ΠΎΠΉ ΠΈΠ½Π΄ΡƒΠΊΡ†ΠΈΠΈ ΠΈ полоТСния ΠΏΡ€ΠΎΠ²ΠΎΠ΄Π½ΠΈΠΊΠ° ΠΏΠΎΠΊΠ°Π·Π°Π½Ρ‹ Π½Π° рисункС.

2.На ΠΏΡ€ΠΎΡ‚ΠΎΠ½, двиТущийся со ΡΠΊΠΎΡ€ΠΎΡΡ‚ΡŒΡŽ 100⁢ ΠΌ/с Π² ΠΎΠ΄Π½ΠΎΡ€ΠΎΠ΄Π½ΠΎΠΌ ΠΌΠ°Π³Π½ΠΈΡ‚Π½ΠΎΠΌ ΠΏΠΎΠ»Π΅ пСрпСндикулярно линиям ΠΈΠ½Π΄ΡƒΠΊΡ†ΠΈΠΈ, дСйствуСт сила Π›ΠΎΡ€Π΅Π½Ρ†Π° 0,32 Ρ… Н . Заряд ΠΏΡ€ΠΎΡ‚ΠΎΠ½Π° 1,6 Ρ… . Какова индукция поля?

3. Π‘ ΠΊΠ°ΠΊΠΎΠΉ ΡΠΊΠΎΡ€ΠΎΡΡ‚ΡŒΡŽ Π²Π»Π΅Ρ‚Π°Π΅Ρ‚ элСктрон пСрпСндикулярно линиям ΠΌΠ°Π³Π½ΠΈΡ‚Π½ΠΎΠ³ΠΎ поля с ΠΈΠ½Π΄ΡƒΠΊΡ†ΠΈΠ΅ΠΉ 8 Π’Π» , Ссли Π½Π° Π½Π΅Π³ΠΎ дСйствуСт сила ΠΌΠ°Π³Π½ΠΈΡ‚Π½ΠΎΠ³ΠΎ поля 8 Ρ… Н . Заряд элСктрона — 1,6 Ρ… .

4. Π‘ΠΈΠ»Π° Ρ‚ΠΎΠΊΠ° Π² ΠΊΠ°Ρ‚ΡƒΡˆΠΊΠ΅ измСняСтся ΠΎΡ‚ 2А Π΄ΠΎ 8 А Π·Π° врСмя 3с. ΠŸΡ€ΠΈ этом Π­Π”Π‘ самоиндукции 0,12 Π’ . ΠžΠΏΡ€Π΅Π΄Π΅Π»ΠΈΡ‚Π΅ ΠΈΠ½Π΄ΡƒΠΊΡ‚ΠΈΠ²Π½ΠΎΡΡ‚ΡŒ ΠΊΠ°Ρ‚ΡƒΡˆΠΊΠΈ?

5. ΠœΠ°Π³Π½ΠΈΡ‚Π½Ρ‹ΠΉ ΠΏΠΎΡ‚ΠΎΠΊ Ρ‡Π΅Ρ€Π΅Π· ΠΊΠΎΠ½Ρ‚ΡƒΡ€ ΠΏΡ€ΠΎΠ²ΠΎΠ΄Π½ΠΈΠΊΠ° сопротивлСниСм 6 Ρ… Ом Π·Π° 3 с. измСнился

Π½Π° 2,4 Ρ… Π’Π±. ΠžΠΏΡ€Π΅Π΄Π΅Π»ΠΈΡ‚Π΅ силу Ρ‚ΠΎΠΊΠ° Π² ΠΏΡ€ΠΎΠ²ΠΎΠ΄Π½ΠΈΠΊΠ΅, Ссли ΠΈΠ·ΠΌΠ΅Π½Π΅Π½ΠΈΠ΅ ΠΏΠΎΡ‚ΠΎΠΊΠ° происходило Ρ€Π°Π²Π½ΠΎΠΌΠ΅Ρ€Π½ΠΎ.

6. ЭлСктричСская Ρ†Π΅ΠΏΡŒ, состоящая ΠΈΠ· Ρ‡Π΅Ρ‚Ρ‹Ρ€Ρ‘Ρ… прямолинСйных Π³ΠΎΡ€ΠΈΠ·ΠΎΠ½Ρ‚Π°Π»ΡŒΠ½Ρ‹Ρ… ΠΏΡ€ΠΎΠ²ΠΎΠ΄Π½ΠΈΠΊΠΎΠ² (1β€”2, 2β€”3, 3β€”4, 4β€”1) ΠΈ источника постоянного Ρ‚ΠΎΠΊΠ°, находится Π² ΠΎΠ΄Π½ΠΎΡ€ΠΎΠ΄Π½ΠΎΠΌ ΠΌΠ°Π³Π½ΠΈΡ‚Π½ΠΎΠΌ ΠΏΠΎΠ»Π΅, Π²Π΅ΠΊΡ‚ΠΎΡ€ ΠΌΠ°Π³Π½ΠΈΡ‚Π½ΠΎΠΉ ΠΈΠ½Π΄ΡƒΠΊΡ†ΠΈΠΈ ΠΊΠΎΡ‚ΠΎΡ€ΠΎΠ³ΠΎΒ Π’Β Π½Π°ΠΏΡ€Π°Π²Π»Π΅Π½ ΠΎΡ‚ нас (см. рисунок, Π²ΠΈΠ΄ свСрху). ΠšΡƒΠ΄Π° Π½Π°ΠΏΡ€Π°Π²Π»Π΅Π½Π° сила АмпСра, Π΄Π΅ΠΉΡΡ‚Π²ΡƒΡŽΡ‰Π°Ρ Π½Π° ΠΏΡ€ΠΎΠ²ΠΎΠ΄Π½ΠΈΠΊ 1β€”2?

7. К ΠΊΠΎΠ»ΡŒΡ†Ρƒ ΠΈΠ· алюминия ΠΏΡ€ΠΈΠ±Π»ΠΈΠΆΠ°ΡŽΡ‚ ΠΌΠ°Π³Π½ΠΈΡ‚, ΠΊΠ°ΠΊ ΠΏΠΎΠΊΠ°Π·Π°Π½ΠΎ Π½Π° рисункС. ΠŸΠΎΠΊΠ°ΠΆΠΈΡ‚Π΅ Π½Π°ΠΏΡ€Π°Π²Π»Π΅Π½ΠΈΠ΅ ΠΈΠ½Π΄ΡƒΠΊΡ†ΠΈΠΎΠ½Π½ΠΎΠ³ΠΎ Ρ‚ΠΎΠΊΠ° Π² ΠΏΡ€ΠΎΠ²ΠΎΠ΄Π½ΠΈΠΊΠ΅. ΠŸΠΎΡΡΠ½ΠΈΡ‚Π΅ ΠΏΡ€Π°Π²ΠΈΠ»ΠΎ, ΠΏΠΎ ΠΊΠΎΡ‚ΠΎΡ€ΠΎΠΉ ΠΌΠΎΠΆΠ½ΠΎ ΠΎΠΏΡ€Π΅Π΄Π΅Π»ΠΈΡ‚ΡŒ Π½Π°ΠΏΡ€Π°Π²Π»Π΅Π½ΠΈΠ΅ ΠΈΠ½Π΄ΡƒΠΊΡ†ΠΈΠΎΠ½Π½ΠΎΠ³ΠΎ Ρ‚ΠΎΠΊΠ°?

8. На рисункС ΠΏΠΎΠΊΠ°Π·Π°Π½ Π³Ρ€Π°Ρ„ΠΈΠΊ измСнСния ΠΈΠ½Π΄ΡƒΠΊΡ†ΠΈΠΈ ΠΌΠ°Π³Π½ΠΈΡ‚Π½ΠΎΠ³ΠΎ поля ΠΏΡ€ΠΎΠ½ΠΈΠ·Ρ‹Π²Π°ΡŽΡ‰Π΅Π³ΠΎ ΠΊΠΎΠ½Ρ‚ΡƒΡ€ ΠΏΠ»ΠΎΡ‰Π°Π΄ΡŒΡŽ 0,01 ΠΌΒ². ΠœΠ°Π³Π½ΠΈΡ‚Π½Ρ‹Π΅ Π»ΠΈΠ½ΠΈΠΈ пСрпСндикулярны плоскости ΠΊΠΎΠ½Ρ‚ΡƒΡ€Π°. НайдитС ΠΌΠΎΠ΄ΡƒΠ»ΡŒ значСния максимального Π­Π”Π‘?

9 ΠœΠ°Π³Π½ΠΈΡ‚Π½ΠΎΠ΅ ΠΏΠΎΠ»Π΅ Ρ‡Π΅Π³ΠΎ ΠΌΠΎΠΆΠ΅Ρ‚ Π±Ρ‹Ρ‚ΡŒ нарисовано Π½Π° рисункС. ΠžΠ±ΠΎΠ·Π½Π°Ρ‡ΡŒΡ‚Π΅ полюса?

10. Найти Π½Π°ΠΏΡ€Π°Π²Π»Π΅Π½ΠΈΠ΅ силы ΠΌΠ°Π³Π½ΠΈΡ‚Π½ΠΎΠ³ΠΎ поля, Π΄Π΅ΠΉΡΡ‚Π²ΡƒΡŽΡ‰Π΅ΠΉ Π½Π° ΠΏΡ€ΠΎΠ²ΠΎΠ΄Π½ΠΈΠΊΠΈ с Ρ‚ΠΎΠΊΠΎΠΌ?

ΠšΠΎΠ½Ρ‚Ρ€ΠΎΠ»ΡŒΠ½Π°Ρ Ρ€Π°Π±ΠΎΡ‚Π° Β« ΠœΠ°Π³Π½ΠΈΡ‚Π½ΠΎΠ΅ ΠΏΠΎΠ»Π΅. ЭлСктромагнитная индукция» Π’ΠΠ Π˜ΠΠΠ’ 2

1. ΠšΠ²Π°Π΄Ρ€Π°Ρ‚Π½Π°Ρ Ρ€Π°ΠΌΠΊΠ° со стороной lΒ =Β 10 см ΠΏΠΎΠ΄ΠΊΠ»ΡŽΡ‡Π΅Π½Π° ΠΊ источнику постоянного Ρ‚ΠΎΠΊΠ° сСрСдинами своих сторон. На участкС  АБ Β Ρ‚Π΅Ρ‡Ρ‘Ρ‚ Ρ‚ΠΎΠΊΒ IΒ =Β 2 А. Π‘ΠΎΠΏΡ€ΠΎΡ‚ΠΈΠ²Π»Π΅Π½ΠΈΠ΅ всСх сторон Ρ€Π°ΠΌΠΊΠΈ ΠΎΠ΄ΠΈΠ½Π°ΠΊΠΎΠ²ΠΎ. Π’ ΠΎΠ΄Π½ΠΎΡ€ΠΎΠ΄Π½ΠΎΠΌ ΠΌΠ°Π³Π½ΠΈΡ‚Π½ΠΎΠΌ ΠΏΠΎΠ»Π΅, Π²Π΅ΠΊΡ‚ΠΎΡ€ ΠΈΠ½Π΄ΡƒΠΊΡ†ΠΈΠΈ ΠΊΠΎΡ‚ΠΎΡ€ΠΎΠ³ΠΎ Π½Π°ΠΏΡ€Π°Π²Π»Π΅Π½ пСрпСндикулярно плоскости Ρ€Π°ΠΌΠΊΠΈ, Ρ€Π΅Π·ΡƒΠ»ΡŒΡ‚ΠΈΡ€ΡƒΡŽΡ‰Π°Ρ сила АмпСра, Π΄Π΅ΠΉΡΡ‚Π²ΡƒΡŽΡ‰Π°Ρ Π½Π° Ρ€Π°ΠΌΠΊΡƒ,Β FΒ =Β 80Β ΠΌ Н. ΠžΠΏΡ€Π΅Π΄Π΅Π»ΠΈΡ‚Π΅ ΠΌΠΎΠ΄ΡƒΠ»ΡŒ Π²Π΅ΠΊΡ‚ΠΎΡ€Π° ΠΌΠ°Π³Π½ΠΈΡ‚Π½ΠΎΠΉ ΠΈΠ½Π΄ΡƒΠΊΡ†ΠΈΠΈ?

2. Участок ΠΏΡ€ΠΎΠ²ΠΎΠ΄Π½ΠΈΠΊΠ° Π΄Π»ΠΈΠ½ΠΎΠΉ 20 см находится Π² ΠΌΠ°Π³Π½ΠΈΡ‚Π½ΠΎΠΌ ΠΏΠΎΠ»Π΅ ΠΈΠ½Π΄ΡƒΠΊΡ†ΠΈΠΈ 50 ΠΌ Π’Π». Π‘ΠΈΠ»Π° элСктричСского Ρ‚ΠΎΠΊΠ° ΠΈΠ΄ΡƒΡ‰Π΅Π³ΠΎ ΠΏΠΎ ΠΏΡ€ΠΎΠ²ΠΎΠ΄Π½ΠΈΠΊΡƒ Ρ€Π°Π²Π½Π° 5 А. КакоС ΠΏΠ΅Ρ€Π΅ΠΌΠ΅Ρ‰Π΅Π½ΠΈΠ΅ ΡΠΎΠ²Π΅Ρ€ΡˆΠΈΡ‚ ΠΏΡ€ΠΎΠ²ΠΎΠ΄Π½ΠΈΠΊ Π² Π½Π°ΠΏΡ€Π°Π²Π»Π΅Π½ΠΈΠΈ дСйствия силы АмпСра, Ссли Ρ€Π°Π±ΠΎΡ‚Π° этой сила Ρ€Π°Π²Π½Π° 0,005 Π”ΠΆ? ΠŸΡ€ΠΎΠ²ΠΎΠ΄Π½ΠΈΠΊ располоТСн пСрпСндикулярно линиям ΠΌΠ°Π³Π½ΠΈΡ‚Π½ΠΎΠΉ ΠΈΠ½Π΄ΡƒΠΊΡ†ΠΈΠΈ. ΠžΡ‚Π²Π΅Ρ‚ ΠΏΡ€ΠΈΠ²Π΅Π΄ΠΈΡ‚Π΅ Π² ΠΌΠ΅Ρ‚Ρ€Π°Ρ….

3. Какая энСргия запасСна Π² ΠΊΠ°Ρ‚ΡƒΡˆΠΊΠ΅, Ссли извСстно, Ρ‡Ρ‚ΠΎ ΠΏΡ€ΠΈ ΠΏΡ€ΠΎΡ‚Π΅ΠΊΠ°Π½ΠΈΠΈ Ρ‡Π΅Ρ€Π΅Π· Π½Π΅Ρ‘ Ρ‚ΠΎΠΊΠ° 2 А, ΠΏΠΎΡ‚ΠΎΠΊ ΠΏΡ€ΠΎΠ½ΠΈΠ·Ρ‹Π²Π°ΡŽΡ‰ΠΈΠΉ Π²ΠΈΡ‚ΠΊΠΈ Π΅Ρ‘ ΠΎΠ±ΠΌΠΎΡ‚ΠΊΠΈ Ρ€Π°Π²Π΅Π½ 9 Π’Π±?

4. Π—Π° 6с ΠΌΠ°Π³Π½ΠΈΡ‚Π½Ρ‹ΠΉ ΠΏΠΎΡ‚ΠΎΠΊ ΠΏΡ€ΠΎΠ½ΠΈΠ·Ρ‹Π²Π°ΡŽΡ‰ΠΈΠΉ Ρ€Π°ΠΌΠΊΡƒ измСнился ΠΎΡ‚ 4 Π’Π± Π΄ΠΎ 40 Π’Π±. Π§Π΅ΠΌΡƒ Ρ€Π°Π²Π½ΠΎ Π·Π½Π°Ρ‡Π΅Π½ΠΈΠ΅ Π­Π”Π‘ ΠΈΠ½Π΄ΡƒΠΊΡ†ΠΈΠΈ Π² Ρ€Π°ΠΌΠΊΠ΅.

5. Найти ΠΈΠ½Π΄ΡƒΠΊΡ‚ΠΈΠ²Π½ΠΎΡΡ‚ΡŒ ΠΊΠ°Ρ‚ΡƒΡˆΠΊΠΈ, энСргия ΠΌΠ°Π³Π½ΠΈΡ‚Π½ΠΎΠ³ΠΎ поля ΠΊΠΎΡ‚ΠΎΡ€ΠΎΠΉ 0,3ΠΌ Π”ΠΆ, Π° сила ΠΈΠ½Π΄ΡƒΠΊΡ†ΠΈΠΎΠ½Π½ΠΎΠ³ΠΎ Ρ‚ΠΎΠΊΠ° 2мА?

6. ΠšΠ²Π°Π΄Ρ€Π°Ρ‚Π½Π°Ρ Ρ€Π°ΠΌΠΊΠ° располоТСна Π² ΠΎΠ΄Π½ΠΎΡ€ΠΎΠ΄Π½ΠΎΠΌ ΠΌΠ°Π³Π½ΠΈΡ‚Π½ΠΎΠΌ ΠΏΠΎΠ»Π΅ Π² плоскости Π»ΠΈΠ½ΠΈΠΉ ΠΌΠ°Π³Π½ΠΈΡ‚Π½ΠΎΠΉ ΠΈΠ½Π΄ΡƒΠΊΡ†ΠΈΠΈ Ρ‚Π°ΠΊ, ΠΊΠ°ΠΊ ΠΏΠΎΠΊΠ°Π·Π°Π½ΠΎ Π½Π° рисункС. НаправлСниС Ρ‚ΠΎΠΊΠ° Π² Ρ€Π°ΠΌΠΊΠ΅ ΠΏΠΎΠΊΠ°Π·Π°Π½ΠΎ стрСлками. ΠšΡƒΠ΄Π° Π½Π°ΠΏΡ€Π°Π²Π»Π΅Π½Π° ΠΎΡ‚Π½ΠΎΡΠΈΡ‚Π΅Π»ΡŒΠ½ΠΎ рисунка (Π²ΠΏΡ€Π°Π²ΠΎ, Π²Π»Π΅Π²ΠΎ, Π²Π²Π΅Ρ€Ρ…, Π²Π½ΠΈΠ·, ΠΊ Π½Π°Π±Π»ΡŽΠ΄Π°Ρ‚Π΅Π»ΡŽ, ΠΎΡ‚ Π½Π°Π±Π»ΡŽΠ΄Π°Ρ‚Π΅Π»Ρ) сила АмпСра, Π΄Π΅ΠΉΡΡ‚Π²ΡƒΡŽΡ‰Π°Ρ Π½Π° сторону  c dΒ  Ρ€Π°ΠΌΠΊΠΈ со стороны ΠΌΠ°Π³Π½ΠΈΡ‚Π½ΠΎΠ³ΠΎ поля? ΠžΡ‚Π²Π΅Ρ‚ Π·Π°ΠΏΠΈΡˆΠΈΡ‚Π΅ словом (словами).

7. К ΠΊΠΎΠ»ΡŒΡ†Ρƒ ΠΈΠ· алюминия ΠΏΡ€ΠΈΠ±Π»ΠΈΠΆΠ°ΡŽΡ‚ ΠΌΠ°Π³Π½ΠΈΡ‚, ΠΊΠ°ΠΊ ΠΏΠΎΠΊΠ°Π·Π°Π½ΠΎ Π½Π° рисункС. ΠŸΠΎΠΊΠ°Π·Π°Ρ‚ΡŒ Π½Π°ΠΏΡ€Π°Π²Π»Π΅Π½ΠΈΠ΅ ΠΈΠ½Π΄ΡƒΠΊΡ†ΠΈΠΎΠ½Π½ΠΎΠ³ΠΎ Ρ‚ΠΎΠΊΠ° Π² ΠΊΠΎΠ½Ρ‚ΡƒΡ€Π΅ ΠΈ ΠΎΠ±ΡŠΡΡΠ½ΠΈΡ‚ΡŒ Π΅Π³ΠΎ Π²ΠΎΠ·Π½ΠΈΠΊΠ½ΠΎΠ²Π΅Π½ΠΈΠ΅.

.

8. Π˜Π½Π΄ΡƒΠΊΡ†ΠΈΡ ΠΌΠ°Π³Π½ΠΈΡ‚Π½ΠΎΠ³ΠΎ поля, ΠΏΡ€ΠΎΠ½ΠΈΠ·Ρ‹Π²Π°ΡŽΡ‰Π΅Π³ΠΎ ΠΊΠΎΠ»ΡŒΡ†ΠΎ, измСняСтся ΠΏΠΎ Π·Π°ΠΊΠΎΠ½Ρƒ ΠΏΠΎΠΊΠ°Π·Π°Π½Π½ΠΎΠΌΡƒ Π½Π° рисункС

Π’ ΠΊΠ°ΠΊΠΎΠΉ ΠΈΠ½Ρ‚Π΅Ρ€Π²Π°Π» Π²Ρ€Π΅ΠΌΠ΅Π½ΠΈ сила Ρ‚ΠΎΠΊΠ° максимальна ΠΈ ΠΏΠΎΡ‡Π΅ΠΌΡƒ?

t c

9. ΠžΠΏΡ€Π΅Π΄Π΅Π»ΠΈΡ‚Π΅ Π½Π°ΠΏΡ€Π°Π²Π»Π΅Π½ΠΈΠ΅ силы АмпСра, Π΄Π΅ΠΉΡΡ‚Π²ΡƒΡŽΡ‰Π΅ΠΉ Π½Π° ΠΏΡ€ΠΎΠ²ΠΎΠ΄Π½ΠΈΠΊ с Ρ‚ΠΎΠΊΠΎΠΌ.

10. На рисункС ΠΏΠΎΠΊΠ°Π·Π°Π½ Π³Ρ€Π°Ρ„ΠΈΠΊ зависимости силы Ρ‚ΠΎΠΊΠ° ΠΎΡ‚ Π²Ρ€Π΅ΠΌΠ΅Π½ΠΈ для ΠΊΠ°Ρ‚ΡƒΡˆΠΊΠΈ с ΠΈΠ½Π΄ΡƒΠΊΡ‚ΠΈΠ²Π½ΠΎΡΡ‚ΡŒΡŽ 5 ΠΌ Π“Π½. Найти Π²Π΅Π»ΠΈΡ‡ΠΈΠ½Ρƒ Π­Π”Π‘ самоиндукции Π² ΠΈΠ½Ρ‚Π΅Ρ€Π²Π°Π»Π΅ Π²Ρ€Π΅ΠΌΠ΅Π½ΠΈ ΠΎΡ‚ 0 Π΄ΠΎ 6 сСкунд? t с

ΠšΠΎΠ½Ρ‚Ρ€ΠΎΠ»ΡŒΠ½Π°Ρ Ρ€Π°Π±ΠΎΡ‚Π° ΠΏΠΎ Ρ„ΠΈΠ·ΠΈΠΊΠ΅ Π­Π»Π΅ΠΊΡ‚Ρ€ΠΎΠΌΠ°Π³Π½ΠΈΡ‚Π½ΠΎΠ΅ ΠΏΠΎΠ»Π΅ для 9 класса

ΠšΠΎΠ½Ρ‚Ρ€ΠΎΠ»ΡŒΠ½Π°Ρ Ρ€Π°Π±ΠΎΡ‚Π° ΠΏΠΎ Ρ„ΠΈΠ·ΠΈΠΊΠ΅ Π­Π»Π΅ΠΊΡ‚Ρ€ΠΎΠΌΠ°Π³Π½ΠΈΡ‚Π½ΠΎΠ΅ ΠΏΠΎΠ»Π΅ для 9 класса с ΠΎΡ‚Π²Π΅Ρ‚Π°ΠΌΠΈ. ΠšΠΎΠ½Ρ‚Ρ€ΠΎΠ»ΡŒΠ½Π°Ρ Ρ€Π°Π±ΠΎΡ‚Π° прСдставлСна Π² 4 Π²Π°Ρ€ΠΈΠ°Π½Ρ‚Π°Ρ…, Π² ΠΊΠ°ΠΆΠ΄ΠΎΠΌ Π²Π°Ρ€ΠΈΠ°Π½Ρ‚Π΅ ΠΏΠΎ 9 Π·Π°Π΄Π°Π½ΠΈΠΉ.

Π’Π°Ρ€ΠΈΠ°Π½Ρ‚ 1

1. ΠšΠ²Π°Π΄Ρ€Π°Ρ‚Π½Π°Ρ Ρ€Π°ΠΌΠΊΠ° располоТСна Π² ΠΎΠ΄Π½ΠΎΡ€ΠΎΠ΄Π½ΠΎΠΌ ΠΌΠ°Π³Π½ΠΈΡ‚Π½ΠΎΠΌ ΠΏΠΎΠ»Π΅, ΠΊΠ°ΠΊ ΠΏΠΎΠΊΠ°Π·Π°Π½ΠΎ Π½Π° рисункС. НаправлСниС Ρ‚ΠΎΠΊΠ° Π² Ρ€Π°ΠΌΠΊΠ΅ ΡƒΠΊΠ°Π·Π°Π½ΠΎ стрСлками.

Π‘ΠΈΠ»Π°, Π΄Π΅ΠΉΡΡ‚Π²ΡƒΡŽΡ‰Π°Ρ Π½Π° ниТнюю сторону Ρ€Π°ΠΌΠΊΠΈ, Π½Π°ΠΏΡ€Π°Π²Π»Π΅Π½Π°

1) Π²Π½ΠΈΠ·
2) Π²Π²Π΅Ρ€Ρ…
3) ΠΈΠ· плоскости листа Π½Π° нас
4) Π² ΠΏΠ»ΠΎΡΠΊΠΎΡΡ‚ΡŒ листа ΠΎΡ‚ нас

2. Π’ ΠΎΠ΄Π½ΠΎΡ€ΠΎΠ΄Π½ΠΎΠ΅ ΠΌΠ°Π³Π½ΠΈΡ‚Π½ΠΎΠ΅ ΠΏΠΎΠ»Π΅ пСрпСндикулярно линиям ΠΌΠ°Π³Π½ΠΈΡ‚Π½ΠΎΠΉ ΠΈΠ½Π΄ΡƒΠΊΡ†ΠΈΠΈ помСстили прямолинСйный ΠΏΡ€ΠΎΠ²ΠΎΠ΄Π½ΠΈΠΊ, ΠΏΠΎ ΠΊΠΎΡ‚ΠΎΡ€ΠΎΠΌΡƒ ΠΏΡ€ΠΎΡ‚Π΅ΠΊΠ°Π΅Ρ‚ Ρ‚ΠΎΠΊ силой 8 А. ΠžΠΏΡ€Π΅Π΄Π΅Π»ΠΈΡ‚Π΅ ΠΈΠ½Π΄ΡƒΠΊΡ†ΠΈΡŽ этого поля, Ссли ΠΎΠ½ΠΎ дСйствуСт с силой 0,02 Н Π½Π° ΠΊΠ°ΠΆΠ΄Ρ‹Π΅ 5 см Π΄Π»ΠΈΠ½Ρ‹ ΠΏΡ€ΠΎΠ²ΠΎΠ΄Π½ΠΈΠΊΠ°.

1) 0,05 Π’Π»
2) 0,0005 Π’Π»
3) 80 Π’Π»
4) 0,0125 Π’Π»

3. Один Ρ€Π°Π· ΠΊΠΎΠ»ΡŒΡ†ΠΎ ΠΏΠ°Π΄Π°Π΅Ρ‚ Π½Π° стоящий Π²Π΅Ρ€Ρ‚ΠΈΠΊΠ°Π»ΡŒΠ½ΠΎ полосовой ΠΌΠ°Π³Π½ΠΈΡ‚ Ρ‚Π°ΠΊ, Ρ‡Ρ‚ΠΎ надСваСтся Π½Π° Π½Π΅Π³ΠΎ; Π²Ρ‚ΠΎΡ€ΠΎΠΉ Ρ€Π°Π· Ρ‚Π°ΠΊ, Ρ‡Ρ‚ΠΎ ΠΏΡ€ΠΎΠ»Π΅Ρ‚Π°Π΅Ρ‚ ΠΌΠΈΠΌΠΎ Π½Π΅Π³ΠΎ. ΠŸΠ»ΠΎΡΠΊΠΎΡΡ‚ΡŒ ΠΊΠΎΠ»ΡŒΡ†Π° Π² ΠΎΠ±ΠΎΠΈΡ… случаях Π³ΠΎΡ€ΠΈΠ·ΠΎΠ½Ρ‚Π°Π»ΡŒΠ½Π°.

Π’ΠΎΠΊ Π² ΠΊΠΎΠ»ΡŒΡ†Π΅ Π²ΠΎΠ·Π½ΠΈΠΊΠ°Π΅Ρ‚

1) Π² ΠΎΠ±ΠΎΠΈΡ… случаях
2) Π½ΠΈ Π² ΠΎΠ΄Π½ΠΎΠΌ ΠΈΠ· случаСв
3) Ρ‚ΠΎΠ»ΡŒΠΊΠΎ Π² ΠΏΠ΅Ρ€Π²ΠΎΠΌ случаС
4) Ρ‚ΠΎΠ»ΡŒΠΊΠΎ Π²ΠΎ Π²Ρ‚ΠΎΡ€ΠΎΠΌ случаС

4. Радиостанция Ρ€Π°Π±ΠΎΡ‚Π°Π΅Ρ‚ Π½Π° частотС 60 ΠœΠ“Ρ†. НайдитС Π΄Π»ΠΈΠ½Ρƒ элСктромагнитных Π²ΠΎΠ»Π½, ΠΈΠ·Π»ΡƒΡ‡Π°Π΅ΠΌΡ‹Ρ… Π°Π½Ρ‚Π΅Π½Π½ΠΎΠΉ радиостанции. Π‘ΠΊΠΎΡ€ΠΎΡΡ‚ΡŒ распространСния элСктромагнитных Π²ΠΎΠ»Π½ с = 3 Β· 108 ΠΌ/с.

1) 0,5 ΠΌ
2) 5 ΠΌ
3) 6 ΠΌ
4) 10 ΠΌ

5. Как измСнится элСктричСская Ρ‘ΠΌΠΊΠΎΡΡ‚ΡŒ плоского кондСнсатора, Ссли ΠΏΠ»ΠΎΡ‰Π°Π΄ΡŒ пластин ΡƒΠ²Π΅Π»ΠΈΡ‡ΠΈΡ‚ΡŒ Π² 3 Ρ€Π°Π·Π°?

1) Π½Π΅ измСнится
2) увСличится Π² 3 Ρ€Π°Π·Π°
3) ΡƒΠΌΠ΅Π½ΡŒΡˆΠΈΡ‚ΡΡ Π² 3 Ρ€Π°Π·Π°
4) срСди ΠΎΡ‚Π²Π΅Ρ‚ΠΎΠ² 1-3 Π½Π΅Ρ‚ ΠΏΡ€Π°Π²ΠΈΠ»ΡŒΠ½ΠΎΠ³ΠΎ

6. Как измСнится ΠΏΠ΅Ρ€ΠΈΠΎΠ΄ собствСнных элСктромагнитных ΠΊΠΎΠ»Π΅Π±Π°Π½ΠΈΠΉ Π² ΠΊΠΎΠ½Ρ‚ΡƒΡ€Π΅, Ссли ΠΊΠ»ΡŽΡ‡ К пСрСвСсти ΠΈΠ· полоТСния 1 Π² ΠΏΠΎΠ»ΠΎΠΆΠ΅Π½ΠΈΠ΅ 2?

1) ΡƒΠΌΠ΅Π½ΡŒΡˆΠΈΡ‚ΡΡ Π² 9 Ρ€Π°Π·
2) увСличится Π² 9 Ρ€Π°Π·
3) ΡƒΠΌΠ΅Π½ΡŒΡˆΠΈΡ‚ΡΡ Π² 3 Ρ€Π°Π·Π°
4) увСличится Π² 3 Ρ€Π°Π·Π°

7. УстановитС соотвСтствиС ΠΌΠ΅ΠΆΠ΄Ρƒ Π½Π°ΡƒΡ‡Π½Ρ‹ΠΌΠΈ открытиями ΠΈ ΡƒΡ‡Π΅Π½Ρ‹ΠΌΠΈ, ΠΊΠΎΡ‚ΠΎΡ€Ρ‹ΠΌ эти открытия ΠΏΡ€ΠΈΠ½Π°Π΄Π»Π΅ΠΆΠ°Ρ‚. К ΠΊΠ°ΠΆΠ΄ΠΎΠΉ ΠΏΠΎΠ·ΠΈΡ†ΠΈΠΈ ΠΏΠ΅Ρ€Π²ΠΎΠ³ΠΎ столбца ΠΏΠΎΠ΄Π±Π΅Ρ€ΠΈΡ‚Π΅ ΡΠΎΠΎΡ‚Π²Π΅Ρ‚ΡΡ‚Π²ΡƒΡŽΡ‰ΡƒΡŽ ΠΏΠΎΠ·ΠΈΡ†ΠΈΡŽ Π²Ρ‚ΠΎΡ€ΠΎΠ³ΠΎ ΠΈ Π·Π°ΠΏΠΈΡˆΠΈΡ‚Π΅ Π²Ρ‹Π±Ρ€Π°Π½Π½Ρ‹Π΅ Ρ†ΠΈΡ„Ρ€Ρ‹ ΠΏΠΎΠ΄ ΡΠΎΠΎΡ‚Π²Π΅Ρ‚ΡΡ‚Π²ΡƒΡŽΡ‰ΠΈΠΌΠΈ Π±ΡƒΠΊΠ²Π°ΠΌΠΈ.

НАУЧНЫЕ ОВКРЫВИЯ

А) Π‘ΠΎΠ·Π΄Π°Π» Ρ‚Π΅ΠΎΡ€ΠΈΡŽ элСктромагнитного поля
Π‘) ЗарСгистрировал элСктромагнитныС Π²ΠΎΠ»Π½Ρ‹
Π’) ОсновополоТник ΠΊΠ²Π°Π½Ρ‚ΠΎΠ²ΠΎΠΉ Ρ„ΠΈΠ·ΠΈΠΊΠΈ

УЧЕНЫЕ

1) М. Планк
2) М. Π€Π°Ρ€Π°Π΄Π΅ΠΉ
3) Π”. МаксвСлл
4) Π‘. Π―ΠΊΠΎΠ±ΠΈ
5) Π“. Π“Π΅Ρ€Ρ†

8. Если Π½Π° Π΄Π½ΠΎ тонкостСнного сосуда, Π·Π°ΠΏΠΎΠ»Π½Π΅Π½Π½ΠΎΠ³ΠΎ ΠΆΠΈΠ΄ΠΊΠΎΡΡ‚ΡŒΡŽ ΠΈ ΠΈΠΌΠ΅ΡŽΡ‰Π΅Π³ΠΎ Ρ„ΠΎΡ€ΠΌΡƒ, ΠΏΡ€ΠΈΠ²Π΅Π΄Π΅Π½Π½ΡƒΡŽ Π½Π° рисункС, ΠΏΡƒΡΡ‚ΠΈΡ‚ΡŒ Π»ΡƒΡ‡ свСта Ρ‚Π°ΠΊ, Ρ‡Ρ‚ΠΎ ΠΎΠ½, пройдя Ρ‡Π΅Ρ€Π΅Π· ΠΆΠΈΠ΄ΠΊΠΎΡΡ‚ΡŒ, ΠΏΠΎΠΏΠ°Π΄Π΅Ρ‚ Π² Ρ†Π΅Π½Ρ‚Ρ€ сосуда, Ρ‚ΠΎ Π»ΡƒΡ‡ Π²Ρ‹Ρ…ΠΎΠ΄ΠΈΡ‚ ΠΈΠ· Тидкости ΠΏΠΎΠ΄ ΡƒΠ³Π»ΠΎΠΌ 30Β° ΠΎΡ‚Π½ΠΎΡΠΈΡ‚Π΅Π»ΡŒΠ½ΠΎ повСрхности Тидкости. Каков ΠΏΠΎΠΊΠ°Π·Π°Ρ‚Π΅Π»ΡŒ прСломлСния n Тидкости, Ссли Π»ΡƒΡ‡ АО составляСт 45Β° с Π²Π΅Ρ€Ρ‚ΠΈΠΊΠ°Π»ΡŒΡŽ?

9. Π”Π΅Ρ‚Π΅ΠΊΡ‚ΠΎΡ€ ΠΏΠΎΠ»Π½ΠΎΡΡ‚ΡŒΡŽ ΠΏΠΎΠ³Π»ΠΎΡ‰Π°Π΅Ρ‚ ΠΏΠ°Π΄Π°ΡŽΡ‰ΠΈΠΉ Π½Π° Π½Π΅Π³ΠΎ свСт частотой Ξ½ = 6 Β· 1014 Π“Ρ†. Π—Π° врСмя t = 5 с Π½Π° Π΄Π΅Ρ‚Π΅ΠΊΡ‚ΠΎΡ€ ΠΏΠ°Π΄Π°Π΅Ρ‚ N = 3 Β· 105 Ρ„ΠΎΡ‚ΠΎΠ½ΠΎΠ². Какова поглощаСмая Π΄Π΅Ρ‚Π΅ΠΊΡ‚ΠΎΡ€ΠΎΠΌ ΠΌΠΎΡ‰Π½ΠΎΡΡ‚ΡŒ? ΠŸΠΎΡΡ‚ΠΎΡΠ½Π½Π°Ρ Планка 6,6 Β· 10-34 Π”ΠΆ Β· с.

Π’Π°Ρ€ΠΈΠ°Π½Ρ‚ 2

1. ΠšΠ²Π°Π΄Ρ€Π°Ρ‚Π½Π°Ρ Ρ€Π°ΠΌΠΊΠ° располоТСна Π² ΠΎΠ΄Π½ΠΎΡ€ΠΎΠ΄Π½ΠΎΠΌ ΠΌΠ°Π³Π½ΠΈΡ‚Π½ΠΎΠΌ ΠΏΠΎΠ»Π΅, ΠΊΠ°ΠΊ ΠΏΠΎΠΊΠ°Π·Π°Π½ΠΎ Π½Π° рисункС. НаправлСниС Ρ‚ΠΎΠΊΠ° Π² Ρ€Π°ΠΌΠΊΠ΅ ΡƒΠΊΠ°Π·Π°Π½ΠΎ стрСлками. Как Π½Π°ΠΏΡ€Π°Π²Π»Π΅Π½Π° сила, Π΄Π΅ΠΉΡΡ‚Π²ΡƒΡŽΡ‰Π°Ρ Π½Π° сторону ab Ρ€Π°ΠΌΠΊΠΈ со стороны ΠΌΠ°Π³Π½ΠΈΡ‚Π½ΠΎΠ³ΠΎ поля?

1) пСрпСндикулярно плоскости Ρ‡Π΅Ρ€Ρ‚Π΅ΠΆΠ°, ΠΎΡ‚ нас
2) пСрпСндикулярно плоскости Ρ‡Π΅Ρ€Ρ‚Π΅ΠΆΠ°, ΠΊ Π½Π°ΠΌ
3) Π²Π΅Ρ€Ρ‚ΠΈΠΊΠ°Π»ΡŒΠ½ΠΎ Π²Π²Π΅Ρ€Ρ…, Π² плоскости Ρ‡Π΅Ρ€Ρ‚Π΅ΠΆΠ°
4) Π²Π΅Ρ€Ρ‚ΠΈΠΊΠ°Π»ΡŒΠ½ΠΎ Π²Π½ΠΈΠ· Π² плоскости Ρ‡Π΅Ρ€Ρ‚Π΅ΠΆΠ°

2. ΠŸΡ€ΡΠΌΠΎΠ»ΠΈΠ½Π΅ΠΉΠ½Ρ‹ΠΉ ΠΏΡ€ΠΎΠ²ΠΎΠ΄Π½ΠΈΠΊ Π΄Π»ΠΈΠ½ΠΎΠΉ 20 см, ΠΏΠΎ ΠΊΠΎΡ‚ΠΎΡ€ΠΎΠΌΡƒ Ρ‚Π΅Ρ‡Π΅Ρ‚ элСктричСский Ρ‚ΠΎΠΊ силой 3 А, находится Π² ΠΎΠ΄Π½ΠΎΡ€ΠΎΠ΄Π½ΠΎΠΌ ΠΌΠ°Π³Π½ΠΈΡ‚Π½ΠΎΠΌ ΠΏΠΎΠ»Π΅ с ΠΈΠ½Π΄ΡƒΠΊΡ†ΠΈΠ΅ΠΉ 4 Π’Π» ΠΈ располоТСн ΠΏΠΎΠ΄ ΡƒΠ³Π»ΠΎΠΌ 90Β° ΠΊ Π²Π΅ΠΊΡ‚ΠΎΡ€Ρƒ ΠΌΠ°Π³Π½ΠΈΡ‚Π½ΠΎΠΉ ΠΈΠ½Π΄ΡƒΠΊΡ†ΠΈΠΈ. Π§Π΅ΠΌΡƒ Ρ€Π°Π²Π½Π° сила, Π΄Π΅ΠΉΡΡ‚Π²ΡƒΡŽΡ‰Π°Ρ Π½Π° ΠΏΡ€ΠΎΠ²ΠΎΠ΄Π½ΠΈΠΊ со стороны ΠΌΠ°Π³Π½ΠΈΡ‚Π½ΠΎΠ³ΠΎ поля?

1) 240 Н
2) 0,15 Н
3) 60 Н
4) 2,4 Н

3. ΠŸΡ€ΠΎΠ²ΠΎΠ΄ΡΡ‰Π΅Π΅ ΠΊΠΎΠ»ΡŒΡ†ΠΎ с Ρ€Π°Π·Ρ€Π΅Π·ΠΎΠΌ ΠΏΠΎΠ΄Π½ΠΈΠΌΠ°ΡŽΡ‚ Π½Π°Π΄ полосовым ΠΌΠ°Π³Π½ΠΈΡ‚ΠΎΠΌ, Π° сплошноС проводящСС ΠΊΠΎΠ»ΡŒΡ†ΠΎ ΡΠΌΠ΅Ρ‰Π°ΡŽΡ‚ Π²ΠΏΡ€Π°Π²ΠΎ.

ΠŸΡ€ΠΈ этом ΠΈΠ½Π΄ΡƒΠΊΡ†ΠΈΠΎΠ½Π½Ρ‹ΠΉ Ρ‚ΠΎΠΊ

1) Ρ‚Π΅Ρ‡Π΅Ρ‚ Ρ‚ΠΎΠ»ΡŒΠΊΠΎ Π² ΠΏΠ΅Ρ€Π²ΠΎΠΌ ΠΊΠΎΠ»ΡŒΡ†Π΅
2) Ρ‚Π΅Ρ‡Π΅Ρ‚ Ρ‚ΠΎΠ»ΡŒΠΊΠΎ Π²ΠΎ Π²Ρ‚ΠΎΡ€ΠΎΠΌ ΠΊΠΎΠ»ΡŒΡ†Π΅
3) Ρ‚Π΅Ρ‡Π΅Ρ‚ ΠΈ Π² ΠΏΠ΅Ρ€Π²ΠΎΠΌ, ΠΈ Π²ΠΎ Π²Ρ‚ΠΎΡ€ΠΎΠΌ ΠΊΠΎΠ»ΡŒΡ†Π΅
4) Π½Π΅ Ρ‚Π΅Ρ‡Π΅Ρ‚ Π½ΠΈ Π² ΠΏΠ΅Ρ€Π²ΠΎΠΌ, Π½ΠΈ Π²ΠΎ Π²Ρ‚ΠΎΡ€ΠΎΠΌ ΠΊΠΎΠ»ΡŒΡ†Π΅

4. Π”Π»ΠΈΠ½Π° элСктромагнитной Π²ΠΎΠ»Π½Ρ‹ Π² Π²ΠΎΠ·Π΄ΡƒΡ…Π΅ Ρ€Π°Π²Π½Π° 0,6 ΠΌΠΊΠΌ. Π§Π΅ΠΌΡƒ Ρ€Π°Π²Π½Π° частота ΠΊΠΎΠ»Π΅Π±Π°Π½ΠΈΠΉ Π²Π΅ΠΊΡ‚ΠΎΡ€Π° напряТСнности элСктричСского поля Π² этой Π²ΠΎΠ»Π½Π΅? Π‘ΠΊΠΎΡ€ΠΎΡΡ‚ΡŒ распространСния элСктромагнитных Π²ΠΎΠ»Π½ с = 3 Β· 108 ΠΌ/с.

1) 1014 Π“Ρ†
2) 5 Β· 1013 Π“Ρ†
3) 1013 Π“Ρ†
4) 5 Β· 1014 Π“Ρ†

5. Как измСнится элСктричСская Π΅ΠΌΠΊΠΎΡΡ‚ΡŒ плоского кондСнсатора, Ссли расстояниС ΠΌΠ΅ΠΆΠ΄Ρƒ пластинами ΡƒΠ²Π΅Π»ΠΈΡ‡ΠΈΡ‚ΡŒ Π² 2 Ρ€Π°Π·Π°?

1) Π½Π΅ измСнится
2) увСличится Π² 2 Ρ€Π°Π·Π°
3) ΡƒΠΌΠ΅Π½ΡŒΡˆΠΈΡ‚ΡΡ Π² 2 Ρ€Π°Π·Π°
4) срСди ΠΎΡ‚Π²Π΅Ρ‚ΠΎΠ² 1-3 Π½Π΅Ρ‚ ΠΏΡ€Π°Π²ΠΈΠ»ΡŒΠ½ΠΎΠ³ΠΎ

6. Как измСнится ΠΏΠ΅Ρ€ΠΈΠΎΠ΄ собствСнных элСктромагнитных ΠΊΠΎΠ»Π΅Π±Π°Π½ΠΈΠΉ Π² ΠΊΠΎΠ½Ρ‚ΡƒΡ€Π΅, Ссли ΠΊΠ»ΡŽΡ‡ К пСрСвСсти ΠΈΠ· полоТСния 1 Π² ΠΏΠΎΠ»ΠΎΠΆΠ΅Π½ΠΈΠ΅ 2?

1) ΡƒΠΌΠ΅Π½ΡŒΡˆΠΈΡ‚ΡΡ Π² 4 Ρ€Π°Π·Π°
2) увСличится Π² 4 Ρ€Π°Π·Π°
3) ΡƒΠΌΠ΅Π½ΡŒΡˆΠΈΡ‚ΡΡ Π² 2 Ρ€Π°Π·Π°
4) увСличится Π² 2 Ρ€Π°Π·Π°

7. УстановитС соотвСтствиС ΠΌΠ΅ΠΆΠ΄Ρƒ особСнностями элСктромагнитных Π²ΠΎΠ»Π½ ΠΈ ΠΈΡ… Π΄ΠΈΠ°ΠΏΠ°Π·ΠΎΠ½Π°ΠΌΠΈ. К ΠΊΠ°ΠΆΠ΄ΠΎΠΉ ΠΏΠΎΠ·ΠΈΡ†ΠΈΠΈ ΠΏΠ΅Ρ€Π²ΠΎΠ³ΠΎ столбца ΠΏΠΎΠ΄Π±Π΅Ρ€ΠΈΡ‚Π΅ ΡΠΎΠΎΡ‚Π²Π΅Ρ‚ΡΡ‚Π²ΡƒΡŽΡ‰ΡƒΡŽ ΠΏΠΎΠ·ΠΈΡ†ΠΈΡŽ Π²Ρ‚ΠΎΡ€ΠΎΠ³ΠΎ ΠΈ Π·Π°ΠΏΠΈΡˆΠΈΡ‚Π΅ Π²Ρ‹Π±Ρ€Π°Π½Π½Ρ‹Π΅ Ρ†ΠΈΡ„Ρ€Ρ‹ ΠΏΠΎΠ΄ ΡΠΎΠΎΡ‚Π²Π΅Ρ‚ΡΡ‚Π²ΡƒΡŽΡ‰ΠΈΠΌΠΈ Π±ΡƒΠΊΠ²Π°ΠΌΠΈ.

ΠžΠ‘ΠžΠ‘Π•ΠΠΠžΠ‘Π’Π˜ Π’ΠžΠ›Π

А) Π’ΠΎΠ»Π½Ρ‹ с минимальной частотой
Π‘) Π’ΠΎΠ»Π½Ρ‹, ΠΈΠ΄ΡƒΡ‰ΠΈΠ΅ ΠΎΡ‚ Π½Π°Π³Ρ€Π΅Ρ‚Ρ‹Ρ… Ρ‚Π΅Π»
Π’) Π’ΠΎΠ»Π½Ρ‹, ΠΎΠ±Π»Π°Π΄Π°ΡŽΡ‰ΠΈΠ΅ ΠΏΡ€ΠΎΠ½ΠΈΠΊΠ°ΡŽΡ‰Π΅ΠΉ ΡΠΏΠΎΡΠΎΠ±Π½ΠΎΡΡ‚ΡŒΡŽ

Π­Π›Π•ΠšΠ’Π ΠžΠœΠΠ“ΠΠ˜Π’ΠΠ«Π• Π’ΠžΠ›ΠΠ«

1) Π Π°Π΄ΠΈΠΎΠ²ΠΎΠ»Π½Ρ‹
2) Π˜Π½Ρ„Ρ€Π°ΠΊΡ€Π°ΡΠ½ΠΎΠ΅ ΠΈΠ·Π»ΡƒΡ‡Π΅Π½ΠΈΠ΅
3) Π’ΠΈΠ΄ΠΈΠΌΠΎΠ΅ ΠΈΠ·Π»ΡƒΡ‡Π΅Π½ΠΈΠ΅
4) Π£Π»ΡŒΡ‚Ρ€Π°Ρ„ΠΈΠΎΠ»Π΅Ρ‚ΠΎΠ²ΠΎΠ΅ ΠΈΠ·Π»ΡƒΡ‡Π΅Π½ΠΈΠ΅
5) РСнтгСновскоС ΠΈΠ·Π»ΡƒΡ‡Π΅Π½ΠΈΠ΅

8. Π£Ρ‡Π΅Π½ΠΈΠΊ Ρ€Π΅ΡˆΠΈΠ» ΠΈΡΠΏΠΎΠ»ΡŒΠ·ΠΎΠ²Π°Ρ‚ΡŒ Π»Π°Π·Π΅Ρ€Π½ΡƒΡŽ ΡƒΠΊΠ°Π·ΠΊΡƒ для опрСдСлСния показатСля прСломлСния нСизвСстной Тидкости. Он взял ΠΏΡ€ΡΠΌΠΎΡƒΠ³ΠΎΠ»ΡŒΠ½ΡƒΡŽ ΠΏΠ»Π°ΡΡ‚ΠΌΠ°ΡΡΠΎΠ²ΡƒΡŽ ΠΊΠΎΡ€ΠΎΠ±ΠΎΡ‡ΠΊΡƒ с ΠΏΡ€ΠΎΠ·Ρ€Π°Ρ‡Π½Ρ‹ΠΌΠΈ стСнками, Π½Π°Π»ΠΈΠ» Π² Π½Π΅Π΅ ΠΆΠΈΠ΄ΠΊΠΎΡΡ‚ΡŒ ΠΈ насыпал Π΄Π΅Ρ‚ΡΠΊΡƒΡŽ присыпку, Ρ‡Ρ‚ΠΎΠ±Ρ‹ Π»ΡƒΡ‡ стал Π²ΠΈΠ΄ΠΈΠΌΡ‹ΠΌ. Для измСрСния ΡƒΠ³Π»Π° падСния ΠΈ ΡƒΠ³Π»Π° прСломлСния ΠΎΠ½ воспользовался двумя ΠΎΠ΄ΠΈΠ½Π°ΠΊΠΎΠ²Ρ‹ΠΌΠΈ транспортирами ΠΈ ΠΎΠΏΡ€Π΅Π΄Π΅Π»ΠΈΠ», Ρ‡Ρ‚ΠΎ ΡƒΠ³ΠΎΠ» падСния 75Β° (sin75Β° = 0,97). Π§Π΅ΠΌΡƒ Ρ€Π°Π²Π΅Π½ ΠΏΠΎΠΊΠ°Π·Π°Ρ‚Π΅Π»ΡŒ прСломлСния n?

9. Π’ Ρ‚Π°Π±Π»ΠΈΡ†Π΅ ΠΏΠΎΠΊΠ°Π·Π°Π½ΠΎ, ΠΊΠ°ΠΊ измСнялся заряд кондСнсатора Π² ΠΊΠΎΠ»Π΅Π±Π°Ρ‚Π΅Π»ΡŒΠ½ΠΎΠΌ ΠΊΠΎΠ½Ρ‚ΡƒΡ€Π΅ с Ρ‚Π΅Ρ‡Π΅Π½ΠΈΠ΅ΠΌ Π²Ρ€Π΅ΠΌΠ΅Π½ΠΈ.

t, 10-6 с0123456789
q, 10-6 Кл21,420-1,42-2-1,4201,4221,42

ВычислитС Π΅ΠΌΠΊΠΎΡΡ‚ΡŒ кондСнсатора Π² ΠΊΠΎΠ½Ρ‚ΡƒΡ€Π΅, Ссли ΠΈΠ½Π΄ΡƒΠΊΡ‚ΠΈΠ²Π½ΠΎΡΡ‚ΡŒ ΠΊΠ°Ρ‚ΡƒΡˆΠΊΠΈ Ρ€Π°Π²Π½Π° 32 ΠΌΠ“Π½.

Π’Π°Ρ€ΠΈΠ°Π½Ρ‚ 3

1. ΠšΠ²Π°Π΄Ρ€Π°Ρ‚Π½Π°Ρ Ρ€Π°ΠΌΠΊΠ° располоТСна Π² ΠΎΠ΄Π½ΠΎΡ€ΠΎΠ΄Π½ΠΎΠΌ ΠΌΠ°Π³Π½ΠΈΡ‚Π½ΠΎΠΌ ΠΏΠΎΠ»Π΅, ΠΊΠ°ΠΊ ΠΏΠΎΠΊΠ°Π·Π°Π½ΠΎ Π½Π° рисункС. НаправлСниС Ρ‚ΠΎΠΊΠ° Π² Ρ€Π°ΠΌΠΊΠ΅ ΡƒΠΊΠ°Π·Π°Π½ΠΎ стрСлками.

Π‘ΠΈΠ»Π°, Π΄Π΅ΠΉΡΡ‚Π²ΡƒΡŽΡ‰Π°Ρ Π½Π° Π²Π΅Ρ€Ρ…Π½ΡŽΡŽ сторону Ρ€Π°ΠΌΠΊΠΈ, Π½Π°ΠΏΡ€Π°Π²Π»Π΅Π½Π°

1) Π²Π½ΠΈΠ·
2) Π²Π²Π΅Ρ€Ρ…
3) ΠΈΠ· плоскости листа Π½Π° нас
4) Π² ΠΏΠ»ΠΎΡΠΊΠΎΡΡ‚ΡŒ листа ΠΎΡ‚ нас

2. Π’ ΠΎΠ΄Π½ΠΎΡ€ΠΎΠ΄Π½ΠΎΠ΅ ΠΌΠ°Π³Π½ΠΈΡ‚Π½ΠΎΠ΅ ΠΏΠΎΠ»Π΅ пСрпСндикулярно линиям ΠΌΠ°Π³Π½ΠΈΡ‚Π½ΠΎΠΉ ΠΈΠ½Π΄ΡƒΠΊΡ†ΠΈΠΈ помСстили прямолинСйный ΠΏΡ€ΠΎΠ²ΠΎΠ΄Π½ΠΈΠΊ, ΠΏΠΎ ΠΊΠΎΡ‚ΠΎΡ€ΠΎΠΌΡƒ ΠΏΡ€ΠΎΡ‚Π΅ΠΊΠ°Π΅Ρ‚ Ρ‚ΠΎΠΊ силой 4 А. ΠžΠΏΡ€Π΅Π΄Π΅Π»ΠΈΡ‚Π΅ ΠΈΠ½Π΄ΡƒΠΊΡ†ΠΈΡŽ этого поля, Ссли ΠΎΠ½ΠΎ дСйствуСт с силой 0,2 Н Π½Π° ΠΊΠ°ΠΆΠ΄Ρ‹Π΅ 10 см Π΄Π»ΠΈΠ½Ρ‹ ΠΏΡ€ΠΎΠ²ΠΎΠ΄Π½ΠΈΠΊΠ°.

1) 0,5 Π’Π»
2) 0,005 Π’Π»
3) 2 Π’Π»
4) 20 Π’Π»

3. БплошноС проводящСС ΠΊΠΎΠ»ΡŒΡ†ΠΎ ΠΈΠ· Π½Π°Ρ‡Π°Π»ΡŒΠ½ΠΎΠ³ΠΎ полоТСния Π² ΠΏΠ΅Ρ€Π²ΠΎΠΌ случаС ΡΠΌΠ΅Ρ‰Π°ΡŽΡ‚ Π²Π²Π΅Ρ€Ρ…, Π° Π²ΠΎ Π²Ρ‚ΠΎΡ€ΠΎΠΌ Π²Π½ΠΈΠ·.

Π˜Π½Π΄ΡƒΠΊΡ†ΠΈΠΎΠ½Π½Ρ‹ΠΉ Ρ‚ΠΎΠΊ Π² ΠΊΠΎΠ»ΡŒΡ†Π΅

1) Ρ‚Π΅Ρ‡Π΅Ρ‚ Ρ‚ΠΎΠ»ΡŒΠΊΠΎ Π² ΠΏΠ΅Ρ€Π²ΠΎΠΌ случаС
2) Ρ‚Π΅Ρ‡Π΅Ρ‚ Ρ‚ΠΎΠ»ΡŒΠΊΠΎ Π²ΠΎ Π²Ρ‚ΠΎΡ€ΠΎΠΌ случаС
3) Ρ‚Π΅Ρ‡Π΅Ρ‚ Π² ΠΎΠ±ΠΎΠΈΡ… случаях
4) Π² ΠΎΠ±ΠΎΠΈΡ… случаях Π½Π΅ Ρ‚Π΅Ρ‡Ρ‘Ρ‚

4. На ΠΊΠ°ΠΊΡƒΡŽ Π΄Π»ΠΈΠ½Ρƒ Π²ΠΎΠ»Π½Ρ‹ Π½ΡƒΠΆΠ½ΠΎ Π½Π°ΡΡ‚Ρ€ΠΎΠΈΡ‚ΡŒ Ρ€Π°Π΄ΠΈΠΎΠΏΡ€ΠΈΠ΅ΠΌΠ½ΠΈΠΊ, Ρ‡Ρ‚ΠΎΠ±Ρ‹ ΡΠ»ΡƒΡˆΠ°Ρ‚ΡŒ Ρ€Π°Π΄ΠΈΠΎΡΡ‚Π°Π½Ρ†ΠΈΡŽ «НашС Ρ€Π°Π΄ΠΈΠΎΒ», которая Π²Π΅Ρ‰Π°Π΅Ρ‚ Π½Π° частотС 101,7 ΠœΠ“Ρ†? Π‘ΠΊΠΎΡ€ΠΎΡΡ‚ΡŒ распространСния элСктромагнитных Π²ΠΎΠ»Π½ с = 3 Β· 108 ΠΌ/с.

1) 2,950 ΠΊΠΌ
2) 2,950 ΠΌ
3) 2,950 Π΄ΠΌ
4) 2,950 см

5. Как измСнится элСктричСская Π΅ΠΌΠΊΠΎΡΡ‚ΡŒ плоского кондСнсатора, Ссли ΠΏΠ»ΠΎΡ‰Π°Π΄ΡŒ пластин ΡƒΠΌΠ΅Π½ΡŒΡˆΠΈΡ‚ΡŒ Π² 5 Ρ€Π°Π·?

1) Π½Π΅ измСнится
2) увСличится Π² 5 Ρ€Π°Π·
3) ΡƒΠΌΠ΅Π½ΡŒΡˆΠΈΡ‚ΡΡ Π² 5 Ρ€Π°Π·
4) срСди ΠΎΡ‚Π²Π΅Ρ‚ΠΎΠ² 1-3 Π½Π΅Ρ‚ ΠΏΡ€Π°Π²ΠΈΠ»ΡŒΠ½ΠΎΠ³ΠΎ.

6. Как измСнится ΠΏΠ΅Ρ€ΠΈΠΎΠ΄ собствСнных элСктромагнитных ΠΊΠΎΠ»Π΅Π±Π°Π½ΠΈΠΉ Π² ΠΊΠΎΠ½Ρ‚ΡƒΡ€Π΅, Ссли ΠΊΠ»ΡŽΡ‡ К пСрСвСсти ΠΈΠ· полоТСния 1 Π² ΠΏΠΎΠ»ΠΎΠΆΠ΅Π½ΠΈΠ΅ 2?

1) ΡƒΠΌΠ΅Π½ΡŒΡˆΠΈΡ‚ΡΡ Π² 4 Ρ€Π°Π·Π°
2) Π½Π΅ измСнится
3) ΡƒΠΌΠ΅Π½ΡŒΡˆΠΈΡ‚ΡΡ Π² 2 Ρ€Π°Π·Π°
4) увСличится Π² 2 Ρ€Π°Π·Π°

7. УстановитС соотвСтствиС ΠΌΠ΅ΠΆΠ΄Ρƒ Π½Π°ΡƒΡ‡Π½Ρ‹ΠΌΠΈ открытиями ΠΈ ΡƒΡ‡Π΅Π½Ρ‹ΠΌΠΈ, ΠΊΠΎΡ‚ΠΎΡ€Ρ‹ΠΌ эти открытия ΠΏΡ€ΠΈΠ½Π°Π΄Π»Π΅ΠΆΠ°Ρ‚. К ΠΊΠ°ΠΆΠ΄ΠΎΠΉ ΠΏΠΎΠ·ΠΈΡ†ΠΈΠΈ ΠΏΠ΅Ρ€Π²ΠΎΠ³ΠΎ столбца ΠΏΠΎΠ΄Π±Π΅Ρ€ΠΈΡ‚Π΅ ΡΠΎΠΎΡ‚Π²Π΅Ρ‚ΡΡ‚Π²ΡƒΡŽΡ‰ΡƒΡŽ ΠΏΠΎΠ·ΠΈΡ†ΠΈΡŽ Π²Ρ‚ΠΎΡ€ΠΎΠ³ΠΎ ΠΈ Π·Π°ΠΏΠΈΡˆΠΈΡ‚Π΅ Π²Ρ‹Π±Ρ€Π°Π½Π½Ρ‹Π΅ Ρ†ΠΈΡ„Ρ€Ρ‹ ΠΏΠΎΠ΄ ΡΠΎΠΎΡ‚Π²Π΅Ρ‚ΡΡ‚Π²ΡƒΡŽΡ‰ΠΈΠΌΠΈ Π±ΡƒΠΊΠ²Π°ΠΌΠΈ.

НАУЧНЫЕ ОВКРЫВИЯ

А) Π‘ΠΎΠ·Π΄Π°Π» Ρ‚Π΅ΠΎΡ€ΠΈΡŽ элСктромагнитного поля
Π‘) ЗарСгистрировал элСктромагнитныС Π²ΠΎΠ»Π½Ρ‹
Π’) Π’Ρ‹Π΄Π²ΠΈΠ½ΡƒΠ» Π³ΠΈΠΏΠΎΡ‚Π΅Π·Ρƒ ΠΎ ΠΊΠ²Π°Π½Ρ‚Π°Ρ…

УЧЕНЫЕ

1) Π‘. Π―ΠΊΠΎΠ±ΠΈ
2) Π”. МаксвСлл
3) М. Планк
4) М. Π€Π°Ρ€Π°Π΄Π΅ΠΉ
5) Π“. Π“Π΅Ρ€Ρ†

8. Если Π½Π° Π΄Π½ΠΎ тонкостСнного сосуда, Π·Π°ΠΏΠΎΠ»Π½Π΅Π½Π½ΠΎΠ³ΠΎ ΠΆΠΈΠ΄ΠΊΠΎΡΡ‚ΡŒΡŽ ΠΈ ΠΈΠΌΠ΅ΡŽΡ‰Π΅Π³ΠΎ Ρ„ΠΎΡ€ΠΌΡƒ, ΠΏΡ€ΠΈΠ²Π΅Π΄Π΅Π½Π½ΡƒΡŽ Π½Π° рисункС, ΠΏΡƒΡΡ‚ΠΈΡ‚ΡŒ Π»ΡƒΡ‡ свСта Ρ‚Π°ΠΊ, Ρ‡Ρ‚ΠΎ ΠΎΠ½, пройдя Ρ‡Π΅Ρ€Π΅Π· ΠΆΠΈΠ΄ΠΊΠΎΡΡ‚ΡŒ, ΠΏΠΎΠΏΠ°Π΄Π΅Ρ‚ Π² Ρ†Π΅Π½Ρ‚Ρ€ сосуда, Ρ‚ΠΎ Π»ΡƒΡ‡ Π²Ρ‹Ρ…ΠΎΠ΄ΠΈΡ‚ ΠΈΠ· Тидкости ΠΏΠΎΠ΄ ΡƒΠ³Π»ΠΎΠΌ 60Β° ΠΎΡ‚Π½ΠΎΡΠΈΡ‚Π΅Π»ΡŒΠ½ΠΎ Π²Π΅Ρ€Ρ‚ΠΈΠΊΠ°Π»ΠΈ. Каков ΠΏΠΎΠΊΠ°Π·Π°Ρ‚Π΅Π»ΡŒ прСломлСния n Тидкости, Ссли Π»ΡƒΡ‡ АО составляСт 45Β° с Π²Π΅Ρ€Ρ‚ΠΈΠΊΠ°Π»ΡŒΡŽ?

9. Π”Π΅Ρ‚Π΅ΠΊΡ‚ΠΎΡ€ ΠΏΠΎΠ»Π½ΠΎΡΡ‚ΡŒΡŽ ΠΏΠΎΠ³Π»ΠΎΡ‰Π°Π΅Ρ‚ ΠΏΠ°Π΄Π°ΡŽΡ‰ΠΈΠΉ Π½Π° Π½Π΅Π³ΠΎ свСт частотой Ξ½ = 5 Β· 1014 Π“Ρ†. ΠŸΠΎΠ³Π»ΠΎΡ‰Π°Π΅ΠΌΠ°Ρ ΠΌΠΎΡ‰Π½ΠΎΡΡ‚ΡŒ Ρ€Π°Π²Π½Π° Π  = 3,3 Β· 10-14 Π’Ρ‚. Бколько Ρ„ΠΎΡ‚ΠΎΠ½ΠΎΠ² ΠΏΠ°Π΄Π°Π΅Ρ‚ Π½Π° Π΄Π΅Ρ‚Π΅ΠΊΡ‚ΠΎΡ€ Π·Π° врСмя t = 5 с? ΠŸΠΎΡΡ‚ΠΎΡΠ½Π½Π°Ρ Планка 6,6 Β· 10-34 Π”ΠΆ Β· с.

Π’Π°Ρ€ΠΈΠ°Π½Ρ‚ 4

1. ΠšΠ²Π°Π΄Ρ€Π°Ρ‚Π½Π°Ρ Ρ€Π°ΠΌΠΊΠ° располоТСна Π² ΠΎΠ΄Π½ΠΎΡ€ΠΎΠ΄Π½ΠΎΠΌ ΠΌΠ°Π³Π½ΠΈΡ‚Π½ΠΎΠΌ ΠΏΠΎΠ»Π΅, ΠΊΠ°ΠΊ ΠΏΠΎΠΊΠ°Π·Π°Π½ΠΎ Π½Π° рисункС. НаправлСниС Ρ‚ΠΎΠΊΠ° Π² Ρ€Π°ΠΌΠΊΠ΅ ΡƒΠΊΠ°Π·Π°Π½ΠΎ стрСлками. Как Π½Π°ΠΏΡ€Π°Π²Π»Π΅Π½Π° сила, Π΄Π΅ΠΉΡΡ‚Π²ΡƒΡŽΡ‰Π°Ρ Π½Π° сторону dc Ρ€Π°ΠΌΠΊΠΈ со стороны ΠΌΠ°Π³Π½ΠΈΡ‚Π½ΠΎΠ³ΠΎ поля?

1) пСрпСндикулярно плоскости Ρ‡Π΅Ρ€Ρ‚Π΅ΠΆΠ°, ΠΎΡ‚ нас
2) пСрпСндикулярно плоскости Ρ‡Π΅Ρ€Ρ‚Π΅ΠΆΠ°, ΠΊ Π½Π°ΠΌ
3) Π²Π΅Ρ€Ρ‚ΠΈΠΊΠ°Π»ΡŒΠ½ΠΎ Π²Π²Π΅Ρ€Ρ…, Π² плоскости Ρ‡Π΅Ρ€Ρ‚Π΅ΠΆΠ°
4) Π²Π΅Ρ€Ρ‚ΠΈΠΊΠ°Π»ΡŒΠ½ΠΎ Π²Π½ΠΈΠ·, Π² плоскости Ρ‡Π΅Ρ€Ρ‚Π΅ΠΆΠ°

2. Π‘ ΠΊΠ°ΠΊΠΎΠΉ силой дСйствуСт ΠΎΠ΄Π½ΠΎΡ€ΠΎΠ΄Π½ΠΎΠ΅ ΠΌΠ°Π³Π½ΠΈΡ‚Π½ΠΎΠ΅ ΠΏΠΎΠ»Π΅ с ΠΈΠ½Π΄ΡƒΠΊΡ†ΠΈΠ΅ΠΉ 2,5 Π’Π» Π½Π° ΠΏΡ€ΠΎΠ²ΠΎΠ΄Π½ΠΈΠΊ Π΄Π»ΠΈΠ½ΠΎΠΉ 50 см, располоТСнный ΠΏΠΎΠ΄ ΡƒΠ³Π»ΠΎΠΌ 90Β° ΠΊ Π²Π΅ΠΊΡ‚ΠΎΡ€Ρƒ ΠΈΠ½Π΄ΡƒΠΊΡ†ΠΈΠΈ, ΠΏΡ€ΠΈ силС Ρ‚ΠΎΠΊΠ° Π² ΠΏΡ€ΠΎΠ²ΠΎΠ΄Π½ΠΈΠΊΠ΅ 2 А?

1) 250 Н
2) 1,6 Н
3) 1 Н
4) 2,5 Н

3. ΠŸΡ€ΠΎΠ²ΠΎΠ΄ΡΡ‰Π΅Π΅ ΠΊΠΎΠ»ΡŒΡ†ΠΎ с Ρ€Π°Π·Ρ€Π΅Π·ΠΎΠΌ ΠΈΠ· Π½Π°Ρ‡Π°Π»ΡŒΠ½ΠΎΠ³ΠΎ полоТСния ΠΏΠΎΠ΄Π½ΠΈΠΌΠ°ΡŽΡ‚ Π²Π²Π΅Ρ€Ρ… ΠΊ полосовому ΠΌΠ°Π³Π½ΠΈΡ‚Ρƒ, Π° сплошноС проводящСС ΠΊΠΎΠ»ΡŒΡ†ΠΎ ΠΈΠ· Π½Π°Ρ‡Π°Π»ΡŒΠ½ΠΎΠ³ΠΎ полоТСния ΡΠΌΠ΅Ρ‰Π°ΡŽΡ‚ Π²ΠΏΡ€Π°Π²ΠΎ.

ΠŸΡ€ΠΈ этом ΠΈΠ½Π΄ΡƒΠΊΡ†ΠΈΠΎΠ½Π½Ρ‹ΠΉ Ρ‚ΠΎΠΊ

1) Ρ‚Π΅Ρ‡Π΅Ρ‚ Π² ΠΎΠ±ΠΎΠΈΡ… случаях
2) Π² ΠΎΠ±ΠΎΠΈΡ… случаях Π½Π΅ Ρ‚Π΅Ρ‡Π΅Ρ‚
3) Ρ‚Π΅Ρ‡Π΅Ρ‚ Ρ‚ΠΎΠ»ΡŒΠΊΠΎ Π² ΠΏΠ΅Ρ€Π²ΠΎΠΌ случаС
4) Ρ‚Π΅Ρ‡Π΅Ρ‚ Ρ‚ΠΎΠ»ΡŒΠΊΠΎ Π²ΠΎ Π²Ρ‚ΠΎΡ€ΠΎΠΌ случаС

4. Π’ ΠΏΠ΅Ρ€Π²Ρ‹Ρ… экспСримСнтах ΠΏΠΎ ΠΈΠ·ΡƒΡ‡Π΅Π½ΠΈΡŽ распространСния элСктромагнитных Π²ΠΎΠ»Π½ Π² Π²ΠΎΠ·Π΄ΡƒΡ…Π΅ Π±Ρ‹Π»ΠΈ ΠΈΠ·ΠΌΠ΅Ρ€Π΅Π½Ρ‹ Π΄Π»ΠΈΠ½Π° Π²ΠΎΠ»Π½Ρ‹ Ξ» = 50 см ΠΈ частота излучСния Ξ½ = 500 ΠœΠ“Ρ†. На основС этих Π½Π΅Ρ‚ΠΎΡ‡Π½Ρ‹Ρ… Π·Π½Π°Ρ‡Π΅Π½ΠΈΠΉ ΡΠΊΠΎΡ€ΠΎΡΡ‚ΡŒ свСта ΠΏΡ€ΠΈΠΌΠ΅Ρ€Π½ΠΎ Ρ€Π°Π²Π½Π°

1) 100 000 км/с
2) 200 000 км/с
3) 250 000 км/с
4) 300 000 км/с

5. Как измСнится элСктричСская Π΅ΠΌΠΊΠΎΡΡ‚ΡŒ плоского кондСнсатора, Ссли расстояниС ΠΌΠ΅ΠΆΠ΄Ρƒ пластинами ΡƒΠΌΠ΅Π½ΡŒΡˆΠΈΡ‚ΡŒ Π² 4 Ρ€Π°Π·Π°?

1) Π½Π΅ измСнится
2) увСличится Π² 4 Ρ€Π°Π·Π°
3) ΡƒΠΌΠ΅Π½ΡŒΡˆΠΈΡ‚ΡΡ Π² 4 Ρ€Π°Π·Π°
4) срСди ΠΎΡ‚Π²Π΅Ρ‚ΠΎΠ² 1-3 Π½Π΅Ρ‚ ΠΏΡ€Π°Π²ΠΈΠ»ΡŒΠ½ΠΎΠ³ΠΎ

6. Как измСнится ΠΏΠ΅Ρ€ΠΈΠΎΠ΄ собствСнных элСктромагнитных ΠΊΠΎΠ»Π΅Π±Π°Π½ΠΈΠΉ Π² ΠΊΠΎΠ½Ρ‚ΡƒΡ€Π΅, Ссли ΠΊΠ»ΡŽΡ‡ К пСрСвСсти ΠΈΠ· полоТСния 1 Π² ΠΏΠΎΠ»ΠΎΠΆΠ΅Π½ΠΈΠ΅ 2?

1) ΡƒΠΌΠ΅Π½ΡŒΡˆΠΈΡ‚ΡΡ Π² 2 Ρ€Π°Π·Π°
2) увСличится Π² 2 Ρ€Π°Π·Π°
3) ΡƒΠΌΠ΅Π½ΡŒΡˆΠΈΡ‚ΡΡ Π² 4 Ρ€Π°Π·Π°
4) увСличится Π² 4 Ρ€Π°Π·Π°

7. УстановитС соотвСтствиС ΠΌΠ΅ΠΆΠ΄Ρƒ особСнностями элСктромагнитных Π²ΠΎΠ»Π½ ΠΈ ΠΈΡ… Π΄ΠΈΠ°ΠΏΠ°Π·ΠΎΠ½Π°ΠΌΠΈ. К ΠΊΠ°ΠΆΠ΄ΠΎΠΉ ΠΏΠΎΠ·ΠΈΡ†ΠΈΠΈ ΠΏΠ΅Ρ€Π²ΠΎΠ³ΠΎ столбца ΠΏΠΎΠ΄Π±Π΅Ρ€ΠΈΡ‚Π΅ ΡΠΎΠΎΡ‚Π²Π΅Ρ‚ΡΡ‚Π²ΡƒΡŽΡ‰ΡƒΡŽ ΠΏΠΎΠ·ΠΈΡ†ΠΈΡŽ Π²Ρ‚ΠΎΡ€ΠΎΠ³ΠΎ ΠΈ Π·Π°ΠΏΠΈΡˆΠΈΡ‚Π΅ Π²Ρ‹Π±Ρ€Π°Π½Π½Ρ‹Π΅ Ρ†ΠΈΡ„Ρ€Ρ‹ ΠΏΠΎΠ΄ ΡΠΎΠΎΡ‚Π²Π΅Ρ‚ΡΡ‚Π²ΡƒΡŽΡ‰ΠΈΠΌΠΈ Π±ΡƒΠΊΠ²Π°ΠΌΠΈ.

ΠžΠ‘ΠžΠ‘Π•ΠΠΠžΠ‘Π’Π˜ Π’ΠžΠ›Π

А) Π’ΠΎΠ»Π½Ρ‹ с максимальной частотой
Π‘) Π’ΠΎΠ»Π½Ρ‹, ΠΈΡΠΏΠΎΠ»ΡŒΠ·ΡƒΠ΅ΠΌΡ‹Π΅ Π² Ρ‚Π΅Π»Π΅Π²ΠΈΠ΄Π΅Π½ΠΈΠΈ ΠΈ сотовой связи
Π’) Π’ΠΎΠ»Π½Ρ‹, Π²Ρ‹Π·Ρ‹Π²Π°ΡŽΡ‰ΠΈΠ΅ ΠΏΠΈΠ³ΠΌΠ΅Π½Ρ‚Π°Ρ†ΠΈΡŽ ΠΊΠΎΠΆΠΈ

Π­Π›Π•ΠšΠ’Π ΠžΠœΠΠ“ΠΠ˜Π’ΠΠ«Π• Π’ΠžΠ›ΠΠ«

1) Π Π°Π΄ΠΈΠΎΠ²ΠΎΠ»Π½Ρ‹
2) Π˜Π½Ρ„Ρ€Π°ΠΊΡ€Π°ΡΠ½ΠΎΠ΅ ΠΈΠ·Π»ΡƒΡ‡Π΅Π½ΠΈΠ΅
3) Π’ΠΈΠ΄ΠΈΠΌΠΎΠ΅ ΠΈΠ·Π»ΡƒΡ‡Π΅Π½ΠΈΠ΅
4) Π£Π»ΡŒΡ‚Ρ€Π°Ρ„ΠΈΠΎΠ»Π΅Ρ‚ΠΎΠ²ΠΎΠ΅ ΠΈΠ·Π»ΡƒΡ‡Π΅Π½ΠΈΠ΅
5) РСнтгСновскоС ΠΈΠ·Π»ΡƒΡ‡Π΅Π½ΠΈΠ΅

8. Π£Ρ‡Π΅Π½ΠΈΠΊ Ρ€Π΅ΡˆΠΈΠ» ΠΈΡΠΏΠΎΠ»ΡŒΠ·ΠΎΠ²Π°Ρ‚ΡŒ Π»Π°Π·Π΅Ρ€Π½ΡƒΡŽ ΡƒΠΊΠ°Π·ΠΊΡƒ для опрСдСлСния показатСля прСломлСния нСизвСстной Тидкости. Он взял ΠΏΡ€ΡΠΌΠΎΡƒΠ³ΠΎΠ»ΡŒΠ½ΡƒΡŽ ΠΏΠ»Π°ΡΡ‚ΠΌΠ°ΡΡΠΎΠ²ΡƒΡŽ ΠΊΠΎΡ€ΠΎΠ±ΠΎΡ‡ΠΊΡƒ с ΠΏΡ€ΠΎΠ·Ρ€Π°Ρ‡Π½Ρ‹ΠΌΠΈ стСнками, Π½Π°Π»ΠΈΠ» Π² Π½Π΅Π΅ ΠΆΠΈΠ΄ΠΊΠΎΡΡ‚ΡŒ ΠΈ насыпал Π΄Π΅Ρ‚ΡΠΊΡƒΡŽ присыпку, Ρ‡Ρ‚ΠΎΠ±Ρ‹ Π»ΡƒΡ‡ стал Π²ΠΈΠ΄ΠΈΠΌΡ‹ΠΌ. Для измСрСния ΡƒΠ³Π»Π° падСния ΠΈ ΡƒΠ³Π»Π° прСломлСния ΠΎΠ½ воспользовался двумя ΠΎΠ΄ΠΈΠ½Π°ΠΊΠΎΠ²Ρ‹ΠΌΠΈ транспортирами, ΠΊΠΎΡ‚ΠΎΡ€Ρ‹Π΅ ΠΏΠΎΠ»ΠΎΠΆΠΈΠ» вдоль стСнки ΠΊΠΎΡ€ΠΎΠ±ΠΎΡ‡ΠΊΠΈ ΠΈ ΠΎΠΏΡ€Π΅Π΄Π΅Π»ΠΈΠ», Ρ‡Ρ‚ΠΎ ΡƒΠ³ΠΎΠ» падСния Ρ€Π°Π²Π΅Π½ 60Β°. Π§Π΅ΠΌΡƒ Ρ€Π°Π²Π΅Π½ ΠΏΠΎΠΊΠ°Π·Π°Ρ‚Π΅Π»ΡŒ прСломлСния n Тидкости?

9. Π’ Ρ‚Π°Π±Π»ΠΈΡ†Π΅ ΠΏΠΎΠΊΠ°Π·Π°Π½ΠΎ, ΠΊΠ°ΠΊ измСнялся заряд кондСнсатора Π² ΠΊΠΎΠ»Π΅Π±Π°Ρ‚Π΅Π»ΡŒΠ½ΠΎΠΌ ΠΊΠΎΠ½Ρ‚ΡƒΡ€Π΅ с Ρ‚Π΅Ρ‡Π΅Π½ΠΈΠ΅ΠΌ Π²Ρ€Π΅ΠΌΠ΅Π½ΠΈ.

t, 10-6 с024681012141618
q, 10-6 Кл02,1332,130-2,13-3-2,1302,13

ВычислитС ΠΈΠ½Π΄ΡƒΠΊΡ‚ΠΈΠ²Π½ΠΎΡΡ‚ΡŒ ΠΊΠ°Ρ‚ΡƒΡˆΠΊΠΈ, Ссли Π΅ΠΌΠΊΠΎΡΡ‚ΡŒ кондСнсатора Π² ΠΊΠΎΠ½Ρ‚ΡƒΡ€Π΅ Ρ€Π°Π²Π½Π° 100 ΠΏΠ€.

ΠžΡ‚Π²Π΅Ρ‚Ρ‹ Π½Π° ΠΊΠΎΠ½Ρ‚Ρ€ΠΎΠ»ΡŒΠ½ΡƒΡŽ Ρ€Π°Π±ΠΎΡ‚Ρƒ ΠΏΠΎ Ρ„ΠΈΠ·ΠΈΠΊΠ΅ Π­Π»Π΅ΠΊΡ‚Ρ€ΠΎΠΌΠ°Π³Π½ΠΈΡ‚Π½ΠΎΠ΅ ΠΏΠΎΠ»Π΅ для 9 класса
Π’Π°Ρ€ΠΈΠ°Π½Ρ‚ 1
1-4
2-1
3-1
4-2
5-2
6-3
7. А3 Π‘5 Π’1
8. 1,22
9. 2,376 Β· 10-14 Π’Ρ‚
Π’Π°Ρ€ΠΈΠ°Π½Ρ‚ 2
1-2
2-4
3-2
4-4
5-3
6-4
7. А1 Π‘2 Π’5
8. 1,37
9. 5 Β· 10-11 Π€
Π’Π°Ρ€ΠΈΠ°Π½Ρ‚ 3
1-3
2-1
3-3
4-2
5-3
6-4
7. А2 Π‘5 Π’3
8. 1,22
9. 5 Β· 105
Π’Π°Ρ€ΠΈΠ°Π½Ρ‚ 4
1-1
2-4
3-4
4-3
5-2
6-1
7. А5 Π‘1 Π’4
8. 1,73
9. 6,49 Β· 10-2 Π“Π½

ΠšΠΎΠ½Ρ‚Ρ€ΠΎΠ»ΡŒΠ½Π°Ρ Ρ€Π°Π±ΠΎΡ‚Π° «ΠœΠ°Π³Π½ΠΈΡ‚Π½ΠΎΠ΅ ΠΏΠΎΠ»Π΅»

1

1.Π›ΠΈΠ½ΠΈΠΈ ΠΈΠ½Π΄ΡƒΠΊΡ†ΠΈΠΈ ΠΎΠ΄Π½ΠΎΡ€ΠΎΠ΄Π½ΠΎΠ³ΠΎ ΠΌΠ°Π³Π½ΠΈΡ‚Π½ΠΎΠ³ΠΎ поля ΠΏΡ€ΠΎΠ½ΠΈΠ·Ρ‹Π²Π°ΡŽΡ‚ Ρ€Π°ΠΌΠΊΡƒ ΠΏΠ»ΠΎΡ‰Π°Π΄ΡŒΡŽ 0,5 ΠΌ2 ΠΏΠΎΠ΄ ΡƒΠ³Π»ΠΎΠΌ 30Β° ΠΊ Π΅Ρ‘ повСрхности, создавая ΠΌΠ°Π³Π½ΠΈΡ‚Π½Ρ‹ΠΉ ΠΏΠΎΡ‚ΠΎΠΊ, Ρ€Π°Π²Π½Ρ‹ΠΉ 0,2 Π’Π±. Π§Π΅ΠΌΡƒ Ρ€Π°Π²Π΅Π½ ΠΌΠΎΠ΄ΡƒΠ»ΡŒ Π²Π΅ΠΊΡ‚ΠΎΡ€Π° ΠΈΠ½Π΄ΡƒΠΊΡ†ΠΈΠΈ ΠΌΠ°Π³Π½ΠΈΡ‚Π½ΠΎΠ³ΠΎ поля? (ΠžΡ‚Π²Π΅Ρ‚ Π΄Π°Ρ‚ΡŒ Π² тСслах.)

2. ΠŸΡ€ΠΎΠ²ΠΎΠ΄Π½ΠΈΠΊ с Ρ‚ΠΎΠΊΠΎΠΌ I=10 A Π΄Π»ΠΈΠ½ΠΎΠΉ 2 ΠΌ находится Π² ΠΎΠ΄Π½ΠΎΡ€ΠΎΠ΄Π½ΠΎΠΌ ΠΌΠ°Π³Π½ΠΈΡ‚Π½ΠΎΠΌ ΠΏΠΎΠ»Π΅ с ΠΈΠ½Π΄ΡƒΠΊΡ†ΠΈΠ΅ΠΉ B=0.5Tl. ΠŸΡ€ΠΈΡ‡Π΅ΠΌ Π½Π°ΠΏΡ€Π°Π²Π»Π΅Π½ΠΈΠ΅ ΠΌΠ°Π³Π½ΠΈΡ‚Π½ΠΎΠ³ΠΎ поля составляСт 30Β° с Π½Π°ΠΏΡ€Π°Π²Π»Π΅Π½ΠΈΠ΅ΠΌ Ρ‚ΠΎΠΊΠ°. Π§Π΅ΠΌΡƒ Ρ€Π°Π²Π½Π° сила со стороны ΠΌΠ°Π³Π½ΠΈΡ‚Π½ΠΎΠ³ΠΎ поля, Π΄Π΅ΠΉΡΡ‚Π²ΡƒΡŽΡ‰Π°Ρ Π½Π° ΠΏΡ€ΠΎΠ²ΠΎΠ΄Π½ΠΈΠΊ? (ΠžΡ‚Π²Π΅Ρ‚ Π΄Π°Ρ‚ΡŒ Π² Π½ΡŒΡŽΡ‚ΠΎΠ½Π°Ρ….)

3. По Π΄Π²ΡƒΠΌ ΠΏΠ°Ρ€Π°Π»Π»Π΅Π»ΡŒΠ½Ρ‹ΠΌ Ρ‚ΠΎΠ½ΠΊΠΈΠΌ Π΄Π»ΠΈΠ½Π½Ρ‹ΠΌ ΠΏΡ€ΠΎΠ²ΠΎΠ΄Π°ΠΌ, расстояниС ΠΌΠ΅ΠΆΠ΄Ρƒ ΠΊΠΎΡ‚ΠΎΡ€Ρ‹ΠΌΠΈ Ρ€Π°Π²Π½ΠΎΒ R, Ρ‚Π΅ΠΊΡƒΡ‚ ΠΎΠ΄ΠΈΠ½Π°ΠΊΠΎΠ²Ρ‹Π΅, Π½ΠΎ ΠΏΡ€ΠΎΡ‚ΠΈΠ²ΠΎΠΏΠΎΠ»ΠΎΠΆΠ½ΠΎ Π½Π°ΠΏΡ€Π°Π²Π»Π΅Π½Π½Ρ‹Π΅ Ρ‚ΠΎΠΊΠΈ силой IΒ (см. рисунок, Π²ΠΈΠ΄ вдоль ΠΏΡ€ΠΎΠ²ΠΎΠ΄ΠΎΠ²). ΠŸΡƒΠ½ΠΊΡ‚ΠΈΡ€Π½ΠΎΠΉ Π»ΠΈΠ½ΠΈΠ΅ΠΉ ΠΈΠ·ΠΎΠ±Ρ€Π°ΠΆΠ΅Π½Π° ΠΎΠΊΡ€ΡƒΠΆΠ½ΠΎΡΡ‚ΡŒ радиусом R с Ρ†Π΅Π½Ρ‚Ρ€ΠΎΠΌ Π² Ρ‚ΠΎΡ‡ΠΊΠ΅ 3, которая находится Π½Π° ΠΎΠ΄ΠΈΠ½Π°ΠΊΠΎΠ²ΠΎΠΌ расстоянии ΠΎΡ‚ ΠΎΠ±ΠΎΠΈΡ… ΠΏΡ€ΠΎΠ²ΠΎΠ΄ΠΎΠ². Π£ΠΊΠ°ΠΆΠΈΡ‚Π΅ Π½ΠΎΠΌΠ΅Ρ€ Ρ‚ΠΎΡ‡ΠΊΠΈ (2, 3, 4, 5), Π² ΠΊΠΎΡ‚ΠΎΡ€ΠΎΠΉ Π²Π΅ΠΊΡ‚ΠΎΡ€ ΠΌΠ°Π³Π½ΠΈΡ‚Π½ΠΎΠΉ ΠΈΠ½Π΄ΡƒΠΊΡ†ΠΈΠΈ суммарного ΠΌΠ°Π³Π½ΠΈΡ‚Π½ΠΎΠ³ΠΎ поля ΠΈΠΌΠ΅Π΅Ρ‚ Ρ‚Π°ΠΊΠΈΠ΅ ΠΆΠ΅ ΠΌΠΎΠ΄ΡƒΠ»ΡŒ ΠΈ Π½Π°ΠΏΡ€Π°Π²Π»Π΅Π½ΠΈΠ΅, ΠΊΠ°ΠΊ ΠΈ Π² Ρ‚ΠΎΡ‡ΠΊΠ΅ 1.

4. К прямолинСйному Π³ΠΎΡ€ΠΈΠ·ΠΎΠ½Ρ‚Π°Π»ΡŒΠ½ΠΎΠΌΡƒ участку ΠΏΡ€ΠΎΠ²ΠΎΠ΄Π°, ΠΏΠΎ ΠΊΠΎΡ‚ΠΎΡ€ΠΎΠΌΡƒ ΠΏΡ€ΠΎΡ‚Π΅ΠΊΠ°Π΅Ρ‚ постоянный Ρ‚ΠΎΠΊ I, ΠΌΠ΅Π΄Π»Π΅Π½Π½ΠΎ поднСсли снизу постоянный ΠΌΠ°Π³Π½ΠΈΡ‚, ΠΊΠ°ΠΊ ΠΏΠΎΠΊΠ°Π·Π°Π½ΠΎ Π½Π° рисункС. ΠšΡƒΠ΄Π° Π½Π°ΠΏΡ€Π°Π²Π»Π΅Π½Π° магнитная сила, Π΄Π΅ΠΉΡΡ‚Π²ΡƒΡŽΡ‰Π°Ρ Π½Π° ΠΏΡ€ΠΎΠ²ΠΎΠ΄?

1) Π²Π²Π΅Ρ€Ρ… ↑ 2) Π²Π½ΠΈΠ· ↓

3) Β«Π½Π° нас» 4) Β«ΠΎΡ‚ нас»

5. Π’ ΠΎΠ΄Π½ΠΎΡ€ΠΎΠ΄Π½ΠΎΠΌ ΠΌΠ°Π³Π½ΠΈΡ‚Π½ΠΎΠΌ ΠΏΠΎΠ»Π΅ с ΠΈΠ½Π΄ΡƒΠΊΡ†ΠΈΠ΅ΠΉ 1.67*10-5 Tl ΠΏΡ€ΠΎΡ‚ΠΎΠ½ двиТСтся пСрпСндикулярно Π²Π΅ΠΊΡ‚ΠΎΡ€Ρƒ ΠΈΠ½Π΄ΡƒΠΊΡ†ΠΈΠΈ со ΡΠΊΠΎΡ€ΠΎΡΡ‚ΡŒΡŽ 8 km/c. ΠžΠΏΡ€Π΅Π΄Π΅Π»ΠΈΡ‚Π΅ радиус Ρ‚Ρ€Π°Π΅ΠΊΡ‚ΠΎΡ€ΠΈΠΈ ΠΏΡ€ΠΎΡ‚ΠΎΠ½Π°.

2

1.Π›ΠΈΠ½ΠΈΠΈ ΠΈΠ½Π΄ΡƒΠΊΡ†ΠΈΠΈ ΠΎΠ΄Π½ΠΎΡ€ΠΎΠ΄Π½ΠΎΠ³ΠΎ ΠΌΠ°Π³Π½ΠΈΡ‚Π½ΠΎΠ³ΠΎ поля ΠΏΡ€ΠΎΠ½ΠΈΠ·Ρ‹Π²Π°ΡŽΡ‚ Ρ€Π°ΠΌΠΊΡƒ ΠΏΠ»ΠΎΡ‰Π°Π΄ΡŒΡŽ 0,25 ΠΌ2 ΠΏΠΎΠ΄ ΡƒΠ³Π»ΠΎΠΌ 30Β° ΠΊ Π΅Ρ‘ повСрхности, создавая ΠΌΠ°Π³Π½ΠΈΡ‚Π½Ρ‹ΠΉ ΠΏΠΎΡ‚ΠΎΠΊ, Ρ€Π°Π²Π½Ρ‹ΠΉ 0,1 Π’Π±. Π§Π΅ΠΌΡƒ Ρ€Π°Π²Π΅Π½ ΠΌΠΎΠ΄ΡƒΠ»ΡŒ Π²Π΅ΠΊΡ‚ΠΎΡ€Π° ΠΈΠ½Π΄ΡƒΠΊΡ†ΠΈΠΈ ΠΌΠ°Π³Π½ΠΈΡ‚Π½ΠΎΠ³ΠΎ поля? (ΠžΡ‚Π²Π΅Ρ‚ Π΄Π°Ρ‚ΡŒ Π² тСслах.)

2. ΠŸΡ€ΠΈ силС Ρ‚ΠΎΠΊΠ° Π² ΠΏΡ€ΠΎΠ²ΠΎΠ΄Π½ΠΈΠΊΠ΅ 20 А Π½Π° участок прямого ΠΏΡ€ΠΎΠ²ΠΎΠ΄Π½ΠΈΠΊΠ° Π΄Π»ΠΈΠ½ΠΎΠΉ 50 см Π² ΠΎΠ΄Π½ΠΎΡ€ΠΎΠ΄Π½ΠΎΠΌ ΠΌΠ°Π³Π½ΠΈΡ‚Π½ΠΎΠΌ ΠΏΠΎΠ»Π΅ дСйствуСт сила АмпСра 12 Н. Π’Π΅ΠΊΡ‚ΠΎΡ€ ΠΈΠ½Π΄ΡƒΠΊΡ†ΠΈΠΈ ΠΌΠ°Π³Π½ΠΈΡ‚Π½ΠΎΠ³ΠΎ поля Π½Π°ΠΏΡ€Π°Π²Π»Π΅Π½ ΠΏΠΎΠ΄ ΡƒΠ³Π»ΠΎΠΌ 37Β° ΠΊ ΠΏΡ€ΠΎΠ²ΠΎΠ΄Π½ΠΈΠΊΡƒ (sin 37Β°=0.6 cos37Β° =0.8) ΠžΠΏΡ€Π΅Π΄Π΅Π»ΠΈΡ‚Π΅ ΠΌΠΎΠ΄ΡƒΠ»ΡŒ ΠΈΠ½Π΄ΡƒΠΊΡ†ΠΈΠΈ ΠΌΠ°Π³Π½ΠΈΡ‚Π½ΠΎΠ³ΠΎ поля. ΠžΡ‚Π²Π΅Ρ‚ Π²Ρ‹Ρ€Π°Π·ΠΈΡ‚Π΅ Π² тСслах ΠΈ ΠΎΠΊΡ€ΡƒΠ³Π»ΠΈΡ‚Π΅ Π΄ΠΎ Ρ†Π΅Π»ΠΎΠ³ΠΎ числа.

3.По Π΄Π²ΡƒΠΌ ΠΏΠ°Ρ€Π°Π»Π»Π΅Π»ΡŒΠ½Ρ‹ΠΌ Ρ‚ΠΎΠ½ΠΊΠΈΠΌ Π΄Π»ΠΈΠ½Π½Ρ‹ΠΌ ΠΏΡ€ΠΎΠ²ΠΎΠ΄Π°ΠΌ, расстояниС ΠΌΠ΅ΠΆΠ΄Ρƒ ΠΊΠΎΡ‚ΠΎΡ€Ρ‹ΠΌΠΈ Ρ€Π°Π²Π½ΠΎ R, Ρ‚Π΅ΠΊΡƒΡ‚ ΠΎΠ΄ΠΈΠ½Π°ΠΊΠΎΠ²Ρ‹Π΅, Π½ΠΎ ΠΏΡ€ΠΎΡ‚ΠΈΠ²ΠΎΠΏΠΎΠ»ΠΎΠΆΠ½ΠΎ Π½Π°ΠΏΡ€Π°Π²Π»Π΅Π½Π½Ρ‹Π΅ Ρ‚ΠΎΠΊΠΈ силой I (см. рисунок, Π²ΠΈΠ΄ вдоль ΠΏΡ€ΠΎΠ²ΠΎΠ΄ΠΎΠ²). ΠŸΡƒΠ½ΠΊΡ‚ΠΈΡ€Π½ΠΎΠΉ Π»ΠΈΠ½ΠΈΠ΅ΠΉ ΠΈΠ·ΠΎΠ±Ρ€Π°ΠΆΠ΅Π½Π° ΠΎΠΊΡ€ΡƒΠΆΠ½ΠΎΡΡ‚ΡŒ радиусом R с Ρ†Π΅Π½Ρ‚Ρ€ΠΎΠΌ Π² Ρ‚ΠΎΡ‡ΠΊΠ΅ 3, которая находится Π½Π° ΠΎΠ΄ΠΈΠ½Π°ΠΊΠΎΠ²ΠΎΠΌ расстоянии ΠΎΡ‚ ΠΎΠ±ΠΎΠΈΡ… ΠΏΡ€ΠΎΠ²ΠΎΠ΄ΠΎΠ². Π£ΠΊΠ°ΠΆΠΈΡ‚Π΅ Π½ΠΎΠΌΠ΅Ρ€ Ρ‚ΠΎΡ‡ΠΊΠΈ (1, 3, 4, 5), Π² ΠΊΠΎΡ‚ΠΎΡ€ΠΎΠΉ Π²Π΅ΠΊΡ‚ΠΎΡ€ ΠΌΠ°Π³Π½ΠΈΡ‚Π½ΠΎΠΉ ΠΈΠ½Π΄ΡƒΠΊΡ†ΠΈΠΈ суммарного ΠΌΠ°Π³Π½ΠΈΡ‚Π½ΠΎΠ³ΠΎ поля ΠΈΠΌΠ΅Π΅Ρ‚ Ρ‚Π°ΠΊΠΈΠ΅ ΠΆΠ΅ ΠΌΠΎΠ΄ΡƒΠ»ΡŒ ΠΈ Π½Π°ΠΏΡ€Π°Π²Π»Π΅Π½ΠΈΠ΅, ΠΊΠ°ΠΊ ΠΈ Π² Ρ‚ΠΎΡ‡ΠΊΠ΅ 2.

4. Π­Π»Π΅ΠΊΡ‚Ρ€ΠΎΠ½ e, Π²Π»Π΅Ρ‚Π΅Π²ΡˆΠΈΠΉ Π² Π·Π°Π·ΠΎΡ€ ΠΌΠ΅ΠΆΠ΄Ρƒ полюсами элСктромагнита, ΠΈΠΌΠ΅Π΅Ρ‚ Π³ΠΎΡ€ΠΈΠ·ΠΎΠ½Ρ‚Π°Π»ΡŒΠ½ΡƒΡŽ ΡΠΊΠΎΡ€ΠΎΡΡ‚ΡŒ , которая пСрпСндикулярна Π²Π΅ΠΊΡ‚ΠΎΡ€Ρƒ ΠΈΠ½Π΄ΡƒΠΊΡ†ΠΈΠΈ ΠΌΠ°Π³Π½ΠΈΡ‚Π½ΠΎΠ³ΠΎ поля, Π½Π°ΠΏΡ€Π°Π²Π»Π΅Π½Π½ΠΎΠΌΡƒ Π³ΠΎΡ€ΠΈΠ·ΠΎΠ½Ρ‚Π°Π»ΡŒΠ½ΠΎ (см. рисунок). Как Π½Π°ΠΏΡ€Π°Π²Π»Π΅Π½Π° Π΄Π΅ΠΉΡΡ‚Π²ΡƒΡŽΡ‰Π°Ρ Π½Π° элСктрон сила Π›ΠΎΡ€Π΅Π½Ρ†Π°

1) Π²Π΅Ρ€Ρ‚ΠΈΠΊΠ°Π»ΡŒΠ½ΠΎ Π²Π½ΠΈΠ· ↓ 2) Π²Π΅Ρ€Ρ‚ΠΈΠΊΠ°Π»ΡŒΠ½ΠΎ Π²Π²Π΅Ρ€Ρ… ↑

3) Π³ΠΎΡ€ΠΈΠ·ΠΎΠ½Ρ‚Π°Π»ΡŒΠ½ΠΎ Π²Π»Π΅Π²ΠΎ ← 4) Π³ΠΎΡ€ΠΈΠ·ΠΎΠ½Ρ‚Π°Π»ΡŒΠ½ΠΎ Π²ΠΏΡ€Π°Π²ΠΎ β†’

5. Π­Π»Π΅ΠΊΡ‚Ρ€ΠΎΠ½ Π²Π»Π΅Ρ‚Π°Π΅Ρ‚ Π² ΠΎΠ΄Π½ΠΎΡ€ΠΎΠ΄Π½ΠΎΠ΅ ΠΌΠ°Π³Π½ΠΈΡ‚Π½ΠΎΠ΅ ΠΏΠΎΠ»Π΅ с ΠΈΠ½Π΄ΡƒΠΊΡ†ΠΈΠ΅ΠΉ 4*10-4 Tl пСрпСндикулярно линиям ΠΈΠ½Π΄ΡƒΠΊΡ†ΠΈΠΈ этого поля ΠΈ двиТСтся ΠΏΠΎ окруТности радиуса R=10 mm. ВычислитС ΡΠΊΠΎΡ€ΠΎΡΡ‚ΡŒ элСктрона.

ΠšΠΎΠ½Ρ‚Ρ€ΠΎΠ»ΡŒΠ½Π°Ρ Ρ€Π°Π±ΠΎΡ‚Π° ΠΏΠΎ Ρ„ΠΈΠ·ΠΈΠΊΠ΅ ЭлСктромагнитная индукция 11 класс

ΠšΠΎΠ½Ρ‚Ρ€ΠΎΠ»ΡŒΠ½Π°Ρ Ρ€Π°Π±ΠΎΡ‚Π° ΠΏΠΎ Ρ„ΠΈΠ·ΠΈΠΊΠ΅ ЭлСктромагнитная индукция 11 класс с ΠΎΡ‚Π²Π΅Ρ‚Π°ΠΌΠΈ. ΠšΠΎΠ½Ρ‚Ρ€ΠΎΠ»ΡŒΠ½Π°Ρ Ρ€Π°Π±ΠΎΡ‚Π° Π²ΠΊΠ»ΡŽΡ‡Π°Π΅Ρ‚ 4 Π²Π°Ρ€ΠΈΠ°Π½Ρ‚Π°, Π² ΠΊΠ°ΠΆΠ΄ΠΎΠΌ Π²Π°Ρ€ΠΈΠ°Π½Ρ‚Π΅ ΠΏΠΎ 6 Π·Π°Π΄Π°Π½ΠΈΠΉ.

1 Π²Π°Ρ€ΠΈΠ°Π½Ρ‚

1. РассчитайтС Ρ€Π°Π·Π½ΠΎΡΡ‚ΡŒ ΠΏΠΎΡ‚Π΅Π½Ρ†ΠΈΠ°Π»ΠΎΠ² Π½Π° ΠΊΠΎΠ½Ρ†Π°Ρ… ΠΊΡ€Ρ‹Π»ΡŒΒ­Π΅Π² самолСта, ΠΈΠΌΠ΅ΡŽΡ‰ΠΈΡ… Π΄Π»ΠΈΠ½Ρƒ 10 ΠΌ, Ссли ΡΠΊΠΎΡ€ΠΎΡΡ‚ΡŒ само­лСта ΠΏΡ€ΠΈ Π³ΠΎΡ€ΠΈΠ·ΠΎΠ½Ρ‚Π°Π»ΡŒΠ½ΠΎΠΌ ΠΏΠΎΠ»Π΅Ρ‚Π΅ 720 ΠΊΠΌ/Ρ‡, Π° Π²Π΅Ρ€Ρ‚ΠΈΠΊΠ°Π»ΡŒΒ­Π½Π°Ρ ΡΠΎΡΡ‚Π°Π²Π»ΡΡŽΡ‰Π°Ρ ΠΈΠ½Π΄ΡƒΠΊΡ†ΠΈΠΈ ΠΌΠ°Π³Π½ΠΈΡ‚Π½ΠΎΠ³ΠΎ поля Π—Π΅ΠΌΠ»ΠΈ 0,5 β‹… 10-4 Π’Π».

2. ΠžΠΏΡ€Π΅Π΄Π΅Π»ΠΈΡ‚Π΅ ΠΈΠ½Π΄ΡƒΠΊΡ‚ΠΈΠ²Π½ΠΎΡΡ‚ΡŒ ΠΊΠ°Ρ‚ΡƒΡˆΠΊΠΈ, Ссли ΠΏΡ€ΠΈ ослаб­лСнии Π² Π½Π΅ΠΉ Ρ‚ΠΎΠΊΠ° Π½Π° 2,8 А Π·Π° 62 мс Π² ΠΊΠ°Ρ‚ΡƒΡˆΠΊΠ΅ появляСтся срСдняя Π­Π”Π‘ самоиндукции 14 Π’.

3. Π’ ΠΊΠ°Ρ‚ΡƒΡˆΠΊΠ΅, состоящСй ΠΈΠ· 75 Π²ΠΈΡ‚ΠΊΠΎΠ², ΠΌΠ°Π³Π½ΠΈΡ‚Π½Ρ‹ΠΉ ΠΏΠΎΡ‚ΠΎΠΊ Ρ€Π°Π²Π΅Π½ 4,8 β‹… 10-3 Π’Π±. Π—Π° ΠΊΠ°ΠΊΠΎΠ΅ врСмя Π΄ΠΎΠ»ΠΆΠ΅Π½ ΠΈΡΡ‡Π΅Π·Π½ΡƒΡ‚ΡŒ этот ΠΏΠΎΡ‚ΠΎΠΊ, Ρ‡Ρ‚ΠΎΠ±Ρ‹ Π² ΠΊΠ°Ρ‚ΡƒΡˆΠΊΠ΅ Π²ΠΎΠ·Π½ΠΈΠΊΠ»Π° срСдняя Π­Π”Π‘ ΠΈΠ½Π΄ΡƒΠΊΡ†ΠΈΠΈ 0,74 Π’?

4. ΠœΠ°Π³Π½ΠΈΡ‚Π½Ρ‹ΠΉ ΠΏΠΎΡ‚ΠΎΠΊ, ΠΏΡ€ΠΎΠ½ΠΈΠ·Ρ‹Π²Π°ΡŽΡ‰ΠΈΠΉ Π·Π°ΠΌΠΊΠ½ΡƒΡ‚Ρ‹ΠΉ ΠΊΠΎΠ½Ρ‚ΡƒΡ€ ΠΏΡ€ΠΎΠ²ΠΎΠ΄Π½ΠΈΠΊΠ° сопротивлСниСм 2,4 Ом, Ρ€Π°Π²Π½ΠΎΠΌΠ΅Ρ€Π½ΠΎ измС­нился Π½Π° 6 Π’Π± Π·Π° 0,5 с. Какова сила ΠΈΠ½Π΄ΡƒΠΊΡ†ΠΈΠΎΠ½Π½ΠΎΠ³ΠΎ Ρ‚ΠΎΠΊΠ° Π² этот ΠΌΠΎΠΌΠ΅Π½Ρ‚?

5. По Π³ΠΎΡ€ΠΈΠ·ΠΎΠ½Ρ‚Π°Π»ΡŒΠ½Ρ‹ΠΌ Ρ€Π΅Π»ΡŒΡΠ°ΠΌ, располоТСнным Π² Π²Π΅Ρ€Ρ‚ΠΈΒ­ΠΊΠ°Π»ΡŒΠ½ΠΎΠΌ ΠΌΠ°Π³Π½ΠΈΡ‚Π½ΠΎΠΌ ΠΏΠΎΠ»Π΅ с ΠΈΠ½Π΄ΡƒΠΊΡ†ΠΈΠ΅ΠΉ 0,01 Π’Π», ΡΠΊΠΎΠ»ΡŒΠ·ΠΈΡ‚ ΠΏΡ€ΠΎΠ²ΠΎΠ΄Π½ΠΈΠΊ Π΄Π»ΠΈΠ½ΠΎΠΉ 1 ΠΌ с постоянной ΡΠΊΠΎΡ€ΠΎΡΡ‚ΡŒΡŽ 10 ΠΌ/с. ΠšΠΎΠ½Ρ†Ρ‹ Ρ€Π΅Π»ΡŒΡΠΎΠ² Π·Π°ΠΌΠΊΠ½ΡƒΡ‚Ρ‹ Π½Π° рСзистор сопротивлСниСм 2 Ом. НайдитС количСство Ρ‚Π΅ΠΏΠ»ΠΎΡ‚Ρ‹, ΠΊΠΎΡ‚ΠΎΡ€ΠΎΠ΅ выдСлится Π² рСзисторС Π·Π° 4 с. Π‘ΠΎΠΏΡ€ΠΎΡ‚ΠΈΠ²Π»Π΅Π½ΠΈΠ΅ΠΌ Ρ€Π΅Π»ΡŒΡΠΎΠ² ΠΈ ΠΏΡ€ΠΎΠ²ΠΎΠ΄Π½ΠΈΠΊΠ° ΠΏΡ€Π΅Π½Π΅Π±Ρ€Π΅Ρ‡ΡŒ.

6. Из алюминиСвой ΠΏΡ€ΠΎΠ²ΠΎΠ»ΠΎΠΊΠΈ сСчСниСм 1 ΠΌΠΌ2 сдСлано ΠΊΠΎΠ»ΡŒΡ†ΠΎ радиусом 10 см. ΠŸΠ΅Ρ€ΠΏΠ΅Π½Π΄ΠΈΠΊΡƒΠ»ΡΡ€Π½ΠΎ плоскости ΠΊΠΎΠ»ΡŒΡ†Π° Π·Π° 0,01 с Π²ΠΊΠ»ΡŽΡ‡Π°ΡŽΡ‚ ΠΌΠ°Π³Π½ΠΈΡ‚Π½ΠΎΠ΅ ΠΏΠΎΠ»Π΅ с ΠΈΠ½Π΄ΡƒΠΊΡ†ΠΈΠ΅ΠΉ 0,01 Π’Π». НайдитС срСднСС Π·Π½Π°Ρ‡Π΅Π½ΠΈΠ΅ ΠΈΠ½Π΄ΡƒΠΊΡ†ΠΈΠΎΠ½Π½ΠΎΠ³ΠΎ Ρ‚ΠΎΠΊΠ°, Π²ΠΎΠ·Π½ΠΈΠΊΠ°ΡŽΡ‰Π΅Π³ΠΎ Π·Π° это врСмя Π² ΠΊΠΎΠ»ΡŒΡ†Π΅.

2 Π²Π°Ρ€ΠΈΠ°Π½Ρ‚

1. Π’ ΠΏΡ€ΠΎΠ²ΠΎΠ΄Π½ΠΈΠΊΠ΅ Π΄Π»ΠΈΠ½ΠΎΠΉ 30 см, двиТущСмся со ΡΠΊΠΎΡ€ΠΎΡΡ‚ΡŒΡŽ 5 ΠΌ/с пСрпСндикулярно линиям ΠΈΠ½Π΄ΡƒΠΊΡ†ΠΈΠΈ ΠΎΠ΄Π½ΠΎΡ€ΠΎΠ΄Π½ΠΎΠ³ΠΎ ΠΌΠ°Π³Π½ΠΈΡ‚Π½ΠΎΠ³ΠΎ поля, Π²ΠΎΠ·Π½ΠΈΠΊΠ°Π΅Ρ‚ Π­Π”Π‘, равная 2,4 Π’. ΠžΠΏΡ€Π΅Π΄Π΅Β­Π»ΠΈΡ‚Π΅ ΠΈΠ½Π΄ΡƒΠΊΡ†ΠΈΡŽ ΠΌΠ°Π³Π½ΠΈΡ‚Π½ΠΎΠ³ΠΎ поля.

2. Какая Π­Π”Π‘ самоиндукции Π²ΠΎΠ·Π½ΠΈΠΊΠ°Π΅Ρ‚ Π² ΠΊΠ°Ρ‚ΡƒΡˆΠΊΠ΅ с ΠΈΠ½Π΄ΡƒΠΊΡ‚ΠΈΠ²Π½ΠΎΡΡ‚ΡŒΡŽ 90 ΠΌΠ“Π½, Ссли ΠΏΡ€ΠΈ Ρ€Π°Π·ΠΌΡ‹ΠΊΠ°Π½ΠΈΠΈ Ρ†Π΅ΠΏΠΈ сила Ρ‚ΠΎΠΊΠ° Π² 10 А ΡƒΠΌΠ΅Π½ΡŒΡˆΠ°Π΅Ρ‚ΡΡ Π΄ΠΎ нуля Π·Π° 0,015 с?

3. ΠŸΡ€ΠΎΠ²ΠΎΠ΄Π½ΠΈΠΊ Π΄Π»ΠΈΠ½ΠΎΠΉ 40 см находится Π² ΠΎΠ΄Π½ΠΎΡ€ΠΎΠ΄Π½ΠΎΠΌ ΠΌΠ°Π³Β­Π½ΠΈΡ‚Π½ΠΎΠΌ ΠΏΠΎΠ»Π΅ с ΠΈΠ½Π΄ΡƒΠΊΡ†ΠΈΠ΅ΠΉ 0,8 Π’Π». ΠŸΡ€ΠΎΠ²ΠΎΠ΄Π½ΠΈΠΊ ΠΏΡ€ΠΈΡˆΠ΅Π» Π² Π΄Π²ΠΈΠΆΠ΅Π½ΠΈΠ΅ пСрпСндикулярно силовым линиям, ΠΊΠΎΠ³Π΄Π° ΠΏΠΎ Π½Π΅ΠΌΡƒ пропустили Ρ‚ΠΎΠΊ 5 А. ΠžΠΏΡ€Π΅Π΄Π΅Π»ΠΈΡ‚Π΅ Ρ€Π°Π±ΠΎΡ‚Ρƒ ΠΌΠ°Π³Π½ΠΈΡ‚Π½ΠΎΠ³ΠΎ поля, Ссли ΠΏΡ€ΠΎΠ²ΠΎΠ΄Π½ΠΈΠΊ пСрСмСстился Π½Π° 20 см.

4. ΠŸΠΎΡ‚ΠΎΠΊ ΠΌΠ°Π³Π½ΠΈΡ‚Π½ΠΎΠΉ ΠΈΠ½Π΄ΡƒΠΊΡ†ΠΈΠΈ Ρ‡Π΅Ρ€Π΅Π· ΠΏΠ»ΠΎΡ‰Π°Π΄ΡŒ ΠΏΠΎΠΏΠ΅Ρ€Π΅Ρ‡Π½ΠΎΒ­Π³ΠΎ сСчСния ΠΊΠ°Ρ‚ΡƒΡˆΠΊΠΈ с 1000 Π²ΠΈΡ‚ΠΊΠΎΠ² измСнился Π½Π° 0,002 Π’Π± Π² Ρ€Π΅Π·ΡƒΠ»ΡŒΡ‚Π°Ρ‚Π΅ измСнСния силы Ρ‚ΠΎΠΊΠ° с 4 А Π΄ΠΎ 20 А. НайдитС ΠΈΠ½Π΄ΡƒΠΊΡ‚ΠΈΠ²Π½ΠΎΡΡ‚ΡŒ ΠΊΠ°Ρ‚ΡƒΡˆΠΊΠΈ.

5. По Π΄Π²ΡƒΠΌ Π²Π΅Ρ€Ρ‚ΠΈΠΊΠ°Π»ΡŒΠ½Ρ‹ΠΌ Ρ€Π΅Π»ΡŒΡΠ°ΠΌ, расстояниС ΠΌΠ΅ΠΆΠ΄Ρƒ ΠΊΠΎΒ­Ρ‚ΠΎΡ€Ρ‹ΠΌΠΈ 50 см, Π° Π²Π΅Ρ€Ρ…Π½ΠΈΠ΅ ΠΊΠΎΠ½Ρ†Ρ‹ Π·Π°ΠΌΠΊΠ½ΡƒΡ‚Ρ‹ сопротивлСни­См 4 Ом, Π½Π°Ρ‡ΠΈΠ½Π°Π΅Ρ‚ ΡΠΊΠΎΠ»ΡŒΠ·ΠΈΡ‚ΡŒ Π²Π½ΠΈΠ· Π±Π΅Π· трСния ΠΏΡ€ΠΎΠ²ΠΎΠ΄Π½ΠΈΠΊ массой 50 Π³. Вся систСма находится Π² ΠΎΠ΄Π½ΠΎΡ€ΠΎΠ΄Π½ΠΎΠΌ ΠΌΠ°Π³Π½ΠΈΡ‚Β­Π½ΠΎΠΌ ΠΏΠΎΠ»Π΅ с ΠΈΠ½Π΄ΡƒΠΊΡ†ΠΈΠ΅ΠΉ 0,4 Π’Π», силовыС Π»ΠΈΠ½ΠΈΠΈ ΠΊΠΎΡ‚ΠΎΡ€ΠΎΠ³ΠΎ пСрпСндикулярны плоскости, проходящСй Ρ‡Π΅Ρ€Π΅Π· Ρ€Π΅Π»ΡŒΡΡ‹. НайдитС ΡΠΊΠΎΡ€ΠΎΡΡ‚ΡŒ ΡƒΡΡ‚Π°Π½ΠΎΠ²ΠΈΠ²ΡˆΠ΅Π³ΠΎΡΡ двиТСния.

6. Π Π°ΠΌΠΊΠ° Π² Ρ„ΠΎΡ€ΠΌΠ΅ ΠΊΠ²Π°Π΄Ρ€Π°Ρ‚Π° со стороной 10 см ΠΈΠΌΠ΅Π΅Ρ‚ сопро­тивлСниС 0,01 Ом. Она Ρ€Π°Π²Π½ΠΎΠΌΠ΅Ρ€Π½ΠΎ вращаСтся Π² ΠΎΠ΄Π½ΠΎΡ€ΠΎΠ΄Β­Π½ΠΎΠΌ ΠΌΠ°Π³Π½ΠΈΡ‚Π½ΠΎΠΌ ΠΏΠΎΠ»Π΅ с ΠΈΠ½Π΄ΡƒΠΊΡ†ΠΈΠ΅ΠΉ 50 ΠΌΠ’Π» Π²ΠΎΠΊΡ€ΡƒΠ³ оси, Π»Π΅Β­ΠΆΠ°Ρ‰Π΅ΠΉ Π² плоскости Ρ€Π°ΠΌΠΊΠΈ ΠΈ пСрпСндикулярной линиям ΠΈΠ½Π΄ΡƒΠΊΡ†ΠΈΠΈ. ΠžΠΏΡ€Π΅Π΄Π΅Π»ΠΈΡ‚Π΅, ΠΊΠ°ΠΊΠΎΠΉ заряд ΠΏΡ€ΠΎΡ‚Π΅Ρ‡Π΅Ρ‚ Ρ‡Π΅Ρ€Π΅Π· Ρ€Π°ΠΌΒ­ΠΊΡƒ ΠΏΡ€ΠΈ ΠΈΠ·ΠΌΠ΅Π½Π΅Π½ΠΈΠΈ ΡƒΠ³Π»Π° ΠΌΠ΅ΠΆΠ΄Ρƒ Π²Π΅ΠΊΡ‚ΠΎΡ€ΠΎΠΌ ΠΌΠ°Π³Π½ΠΈΡ‚Π½ΠΎΠΉ ΠΈΠ½Β­Π΄ΡƒΠΊΡ†ΠΈΠΈ ΠΈ Π½ΠΎΡ€ΠΌΠ°Π»ΡŒΡŽ ΠΊ Ρ€Π°ΠΌΠΊΠ΅ ΠΎΡ‚ 0 Π΄ΠΎ 30Β°.

3 Π²Π°Ρ€ΠΈΠ°Π½Ρ‚

1. ΠœΠ°Π³Π½ΠΈΡ‚Π½Ρ‹ΠΉ ΠΏΠΎΡ‚ΠΎΠΊ Π²Π½ΡƒΡ‚Ρ€ΠΈ ΠΊΠ°Ρ‚ΡƒΡˆΠΊΠΈ с числом Π²ΠΈΡ‚ΠΊΠΎΠ², Ρ€Π°Π²Π½Ρ‹ΠΌ 400, Π·Π° 0,2 с измСнился ΠΎΡ‚ 0,1 Π’Π± Π΄ΠΎ 0,9 Π’Π±. ΠžΠΏΡ€Π΅Β­Π΄Π΅Π»ΠΈΡ‚Π΅ Π­Π”Π‘ Π½Π° Π·Π°ΠΆΠΈΠΌΠ°Ρ… ΠΊΠ°Ρ‚ΡƒΡˆΠΊΠΈ.

2. Π‘ ΠΊΠ°ΠΊΠΎΠΉ ΡΠΊΠΎΡ€ΠΎΡΡ‚ΡŒΡŽ Π½Π°Π΄ΠΎ ΠΏΠ΅Ρ€Π΅ΠΌΠ΅Ρ‰Π°Ρ‚ΡŒ ΠΏΡ€ΠΎΠ²ΠΎΠ΄Π½ΠΈΠΊ Π΄Π»ΠΈΒ­Π½ΠΎΠΉ 50 см Π² ΠΎΠ΄Π½ΠΎΡ€ΠΎΠ΄Π½ΠΎΠΌ ΠΌΠ°Π³Π½ΠΈΡ‚Π½ΠΎΠΌ ΠΏΠΎΠ»Π΅ с ΠΈΠ½Π΄ΡƒΠΊΡ†ΠΈΠ΅ΠΉ 0,4 Π’Π» ΠΏΠΎΠ΄ ΡƒΠ³Π»ΠΎΠΌ 60Β° ΠΊ силовым линиям, Ρ‡Ρ‚ΠΎΠ±Ρ‹ Π² ΠΏΡ€ΠΎΠ²ΠΎΠ΄Β­Π½ΠΈΠΊΠ΅ Π²ΠΎΠ·Π½ΠΈΠΊΠ»Π° Π­Π”Π‘, равная 1 Π’?

3. ΠœΠ°Π³Π½ΠΈΡ‚Π½Ρ‹ΠΉ ΠΏΠΎΡ‚ΠΎΠΊ, ΠΏΡ€ΠΎΠ½ΠΈΠ·Ρ‹Π²Π°ΡŽΡ‰ΠΈΠΉ ΠΊΠΎΠ½Ρ‚ΡƒΡ€ ΠΏΡ€ΠΎΠ²ΠΎΠ΄Π½ΠΈΒ­ΠΊΠ°, Ρ€Π°Π²Π½ΠΎΠΌΠ΅Ρ€Π½ΠΎ ΡƒΠΌΠ΅Π½ΡŒΡˆΠΈΠ»ΡΡ Π½Π° 1,6 Π’Π±. Π—Π° ΠΊΠ°ΠΊΠΎΠ΅ врСмя из­мСнился ΠΌΠ°Π³Π½ΠΈΡ‚Π½Ρ‹ΠΉ ΠΏΠΎΡ‚ΠΎΠΊ, Ссли ΠΏΡ€ΠΈ этом Π­Π”Π‘ ΠΈΠ½Π΄ΡƒΠΊΡ†ΠΈΠΈ оказалась Ρ€Π°Π²Π½ΠΎΠΉ 3,2 Π’?

4. ΠšΠ°Ρ‚ΡƒΡˆΠΊΠ° Π΄ΠΈΠ°ΠΌΠ΅Ρ‚Ρ€ΠΎΠΌ 4 см находится Π² ΠΏΠ΅Ρ€Π΅ΠΌΠ΅Π½Π½ΠΎΠΌ ΠΌΠ°Π³Β­Π½ΠΈΡ‚Π½ΠΎΠΌ ΠΏΠΎΠ»Π΅, силовыС Π»ΠΈΠ½ΠΈΠΈ ΠΊΠΎΡ‚ΠΎΡ€ΠΎΠ³ΠΎ ΠΏΠ°Ρ€Π°Π»Π»Π΅Π»ΡŒΠ½Ρ‹ оси ΠΊΠ°Ρ‚ΡƒΡˆΠΊΠΈ. ΠŸΡ€ΠΈ ΠΈΠ·ΠΌΠ΅Π½Π΅Π½ΠΈΠΈ ΠΈΠ½Π΄ΡƒΠΊΡ†ΠΈΠΈ поля Π½Π° 1 Π’Π» Π² Ρ‚Π΅Ρ‡Π΅Β­Π½ΠΈΠ΅ 6,28 с Π² ΠΊΠ°Ρ‚ΡƒΡˆΠΊΠ΅ Π²ΠΎΠ·Π½ΠΈΠΊΠ»Π° Π­Π”Π‘ 2 Π’. Бколько Π²ΠΈΡ‚ΠΊΠΎΠ² ΠΈΠΌΠ΅Π΅Ρ‚ ΠΊΠ°Ρ‚ΡƒΡˆΠΊΠ°?

5. Плоский ΠΏΡ€ΠΎΠ²ΠΎΠ»ΠΎΡ‡Π½Ρ‹ΠΉ Π²ΠΈΡ‚ΠΎΠΊ ΠΏΠ»ΠΎΡ‰Π°Π΄ΡŒΡŽ 1 000 см2, ΠΈΠΌΠ΅ΡŽΡ‰ΠΈΠΉ сопротивлСниС 2 Ом, располоТСн Π² ΠΎΠ΄Π½ΠΎΡ€ΠΎΠ΄Π½ΠΎΠΌ ΠΌΠ°Π³Π½ΠΈΡ‚Π½ΠΎΠΌ ΠΏΠΎΠ»Π΅ с ΠΈΠ½Π΄ΡƒΠΊΡ†ΠΈΠ΅ΠΉ 0,1 Π’Π» Ρ‚Π°ΠΊΠΈΠΌ ΠΎΠ±Ρ€Π°Π·ΠΎΠΌ, Ρ‡Ρ‚ΠΎ Π΅Π³ΠΎ ΠΏΠ»ΠΎΡΠΊΠΎΡΡ‚ΡŒ пСрпСндикулярна линиям ΠΌΠ°Π³Π½ΠΈΡ‚Π½ΠΎΠΉ ΠΈΠ½Β­Π΄ΡƒΠΊΡ†ΠΈΠΈ. На ΠΊΠ°ΠΊΠΎΠΉ ΡƒΠ³ΠΎΠ» Π±Ρ‹Π» ΠΏΠΎΠ²Π΅Ρ€Π½ΡƒΡ‚ Π²ΠΈΡ‚ΠΎΠΊ, Ссли ΠΏΡ€ΠΈ этом ΠΏΠΎ Π½Π΅ΠΌΡƒ ΠΏΡ€ΠΎΡˆΠ΅Π» заряд 7,5 мКл?

6. Π’ ΠΎΠ΄Π½ΠΎΡ€ΠΎΠ΄Π½ΠΎΠΌ ΠΌΠ°Π³Π½ΠΈΡ‚Π½ΠΎΠΌ ΠΏΠΎΠ»Π΅ с ΠΈΠ½Β­Π΄ΡƒΠΊΡ†ΠΈΠ΅ΠΉ 20 ΠΌΠ’Π» располоТСны Π²Π΅Ρ€Ρ‚ΠΈΒ­ΠΊΠ°Π»ΡŒΠ½ΠΎ Π½Π° расстоянии 80 см Π΄Ρ€ΡƒΠ³ ΠΎΡ‚ Π΄Ρ€ΡƒΠ³Π° Π΄Π²Π° ΠΏΡ€ΠΎΠ²ΠΎΠ»ΠΎΡ‡Π½Ρ‹Ρ… ΠΏΡ€ΡƒΡ‚Π°, Π·Π°ΠΌΠΊΠ½ΡƒΒ­Ρ‚Ρ‹Ρ… Π½Π°Π²Π΅Ρ€Ρ…Ρƒ. ΠŸΠ»ΠΎΡΠΊΠΎΡΡ‚ΡŒ, Π² ΠΊΠΎΡ‚ΠΎΡ€ΠΎΠΉ располоТСны ΠΏΡ€ΡƒΡ‚ΡŒΡ, пСрпСндикулярна Π½Π°ΠΏΡ€Π°Π²Π»Π΅Π½ΠΈΡŽ Π»ΠΈΠ½ΠΈΠΉ ΠΈΠ½Π΄ΡƒΠΊΡ†ΠΈΠΈ ΠΌΠ°Π³Π½ΠΈΡ‚Π½ΠΎΠ³ΠΎ поля. По ΠΏΡ€ΡƒΡ‚ΡŒΡΠΌ с постоянной ΡΠΊΠΎΡ€ΠΎΡΡ‚ΡŒΡŽ 1,5 ΠΌ/с ΡΠΊΠΎΠ»ΡŒΠ·ΠΈΡ‚ Π²Π½ΠΈΠ· ΠΏΠ΅Ρ€Π΅ΠΌΡ‹Ρ‡ΠΊΠ° массой 1,2 Π³ (рис. 131).

ΠžΠΏΡ€Π΅Π΄Π΅Π»ΠΈΡ‚Π΅ Π΅Π΅ сопро­тивлСниС, считая, Ρ‡Ρ‚ΠΎ ΠΏΡ€ΠΈ Π΄Π²ΠΈΠΆΠ΅Π½ΠΈΠΈ ΠΊΠΎΠ½Ρ‚Π°ΠΊΡ‚ ΠΏΠ΅Ρ€Π΅ΠΌΡ‹Ρ‡ΠΊΠΈ с ΠΏΡ€ΡƒΡ‚ΡŒΡΠΌΠΈ Π½Π΅ Π½Π°Ρ€ΡƒΡˆΠ°Π΅Ρ‚ΡΡ. Π’Ρ€Π΅Π½ΠΈΠ΅ΠΌ ΠΏΡ€Π΅Π½Π΅Π±Ρ€Π΅Ρ‡ΡŒ.

4 Π²Π°Ρ€ΠΈΠ°Π½Ρ‚

1. ΠžΠΏΡ€Π΅Π΄Π΅Π»ΠΈΡ‚Π΅ ΠΈΠ½Π΄ΡƒΠΊΡ‚ΠΈΠ²Π½ΠΎΡΡ‚ΡŒ ΠΊΠ°Ρ‚ΡƒΡˆΠΊΠΈ, Ссли ΠΏΡ€ΠΈ ΠΈΠ·ΠΌΠ΅Π½Π΅Β­Π½ΠΈΠΈ силы Ρ‚ΠΎΠΊΠ° Π² Π½Π΅ΠΉ со ΡΠΊΠΎΡ€ΠΎΡΡ‚ΡŒΡŽ 50 А/с Π²ΠΎΠ·Π½ΠΈΠΊΠ°Π΅Ρ‚ Π­Π”Π‘ самоиндукции Π² 20 Π’.

2. ΠΠ²Ρ‚ΠΎΠΌΠΎΠ±ΠΈΠ»ΡŒ Β«Π’ΠΎΠ»Π³Π°Β» Π΅Π΄Π΅Ρ‚ со ΡΠΊΠΎΡ€ΠΎΡΡ‚ΡŒΡŽ 120 ΠΊΠΌ/Ρ‡. ΠžΠΏΡ€Π΅Π΄Π΅Π»ΠΈΡ‚Π΅ Ρ€Π°Π·Π½ΠΎΡΡ‚ΡŒ ΠΏΠΎΡ‚Π΅Π½Ρ†ΠΈΠ°Π»ΠΎΠ² Π½Π° ΠΊΠΎΠ½Ρ†Π°Ρ… ΠΏΠ΅Ρ€Π΅Π΄Β­Π½Π΅ΠΉ оси ΠΌΠ°ΡˆΠΈΠ½Ρ‹, Ссли Π΄Π»ΠΈΠ½Π° оси 180 см, Π° Π²Π΅Ρ€Ρ‚ΠΈΠΊΠ°Π»ΡŒΒ­Π½Π°Ρ ΡΠΎΡΡ‚Π°Π²Π»ΡΡŽΡ‰Π°Ρ ΠΈΠ½Π΄ΡƒΠΊΡ†ΠΈΠΈ ΠΌΠ°Π³Π½ΠΈΡ‚Π½ΠΎΠ³ΠΎ поля Π—Π΅ΠΌΠ»ΠΈ 5 β‹… 10-5 Π’Π».

3. Какая Π­Π”Π‘ самоиндукции Π²ΠΎΠ·Π½ΠΈΠΊΠ°Π΅Ρ‚ Π² ΠΊΠ°Ρ‚ΡƒΡˆΠΊΠ΅ ΠΈΠ½Π΄ΡƒΠΊΡ‚ΠΈΠ²Π½ΠΎΡΡ‚ΡŒΡŽ 68 ΠΌΠ“Π½, Ссли сила Ρ‚ΠΎΠΊΠ° Π² 3,8 А ΡƒΠ±Ρ‹Π²Π°Π΅Ρ‚ Π΄ΠΎ ну­ля Π² Π½Π΅ΠΉ Π·Π° 0,012 с?

4. ΠšΠ°ΠΊΡƒΡŽ Ρ€Π°Π±ΠΎΡ‚Ρƒ Π½Π°Π΄ΠΎ ΡΠΎΠ²Π΅Ρ€ΡˆΠΈΡ‚ΡŒ ΠΏΡ€ΠΈ ΠΏΠ΅Ρ€Π΅ΠΌΠ΅Ρ‰Π΅Π½ΠΈΠΈ Π½Π° 0,25 ΠΌ ΠΏΡ€ΠΎΠ²ΠΎΠ΄Π½ΠΈΠΊΠ° Π΄Π»ΠΈΠ½ΠΎΠΉ 0,4 мс Ρ‚ΠΎΠΊΠΎΠΌ 21 А Π² ΠΎΠ΄Π½ΠΎΡ€ΠΎΠ΄Β­Π½ΠΎΠΌ ΠΌΠ°Π³Π½ΠΈΡ‚Π½ΠΎΠΌ ΠΏΠΎΠ»Π΅ с ΠΈΠ½Π΄ΡƒΠΊΡ†ΠΈΠ΅ΠΉ 1,2 Π’Π»?

5. ΠšΠΎΠ»ΡŒΡ†ΠΎ радиусом 1 ΠΌ ΠΈ сопротивлСниСм 0,1 Ом ΠΏΠΎΠΌΠ΅Β­Ρ‰Π΅Π½ΠΎ Π² ΠΎΠ΄Π½ΠΎΡ€ΠΎΠ΄Π½ΠΎΠ΅ ΠΌΠ°Π³Π½ΠΈΡ‚Π½ΠΎΠ΅ ΠΏΠΎΠ»Π΅ с ΠΈΠ½Π΄ΡƒΠΊΡ†ΠΈΠ΅ΠΉ 0,1 Π’Π». ΠŸΠ»ΠΎΡΠΊΠΎΡΡ‚ΡŒ ΠΊΠΎΠ»ΡŒΡ†Π° пСрпСндикулярна Π²Π΅ΠΊΡ‚ΠΎΡ€Ρƒ ΠΈΠ½Π΄ΡƒΠΊΡ†ΠΈΠΈ поля. Какой заряд ΠΏΡ€ΠΎΠΉΠ΄Π΅Ρ‚ Ρ‡Π΅Ρ€Π΅Π· ΠΏΠΎΠΏΠ΅Ρ€Π΅Ρ‡Π½ΠΎΠ΅ сСчСниС ΠΊΠΎΠ»ΡŒΡ†Π° ΠΏΡ€ΠΈ исчСзновСнии поля?

6. Π Π°ΠΌΠΊΠ° Π² Ρ„ΠΎΡ€ΠΌΠ΅ равностороннСго Ρ‚Ρ€Π΅ΡƒΠ³ΠΎΠ»ΡŒΠ½ΠΈΠΊΠ° ΠΏΠΎΠΌΠ΅Ρ‰Π΅Β­Π½Π° Π² ΠΎΠ΄Π½ΠΎΡ€ΠΎΠ΄Π½ΠΎΠ΅ ΠΌΠ°Π³Π½ΠΈΡ‚Π½ΠΎΠ΅ ΠΏΠΎΠ»Π΅ с ΠΈΠ½Π΄ΡƒΠΊΡ†ΠΈΠ΅ΠΉ 0,08 Π’Π», Π½Π°Β­ΠΏΡ€Π°Π²Π»Π΅Π½Π½ΠΎΠΉ ΠΏΠΎΠ΄ ΡƒΠ³Π»ΠΎΠΌ 60Β° ΠΊ плоскости Ρ€Π°ΠΌΠΊΠΈ. НайдитС Π΄Π»ΠΈΠ½Ρƒ стороны Ρ€Π°ΠΌΠΊΠΈ, Ссли извСстно, Ρ‡Ρ‚ΠΎ ΠΏΡ€ΠΈ Ρ€Π°Π²Π½ΠΎΠΌΠ΅Ρ€Β­Π½ΠΎΠΌ исчСзновСнии поля Π² Ρ‚Π΅Ρ‡Π΅Π½ΠΈΠ΅ 0,03 с Π² Ρ€Π°ΠΌΠΊΠ΅ Π²ΠΎΠ·Π½ΠΈΠΊΠ»Π° Π­Π”Π‘ ΠΈΠ½Π΄ΡƒΠΊΡ†ΠΈΠΈ, равная 10 ΠΌΠ’.

ΠžΡ‚Π²Π΅Ρ‚Ρ‹ Π½Π° ΠΊΠΎΠ½Ρ‚Ρ€ΠΎΠ»ΡŒΠ½ΡƒΡŽ Ρ€Π°Π±ΠΎΡ‚Π° ΠΏΠΎ Ρ„ΠΈΠ·ΠΈΠΊΠ΅ ЭлСктромагнитная индукция 11 класс
1 Π²Π°Ρ€ΠΈΠ°Π½Ρ‚
1. 0,1 Π’
2. 0,31 Π“Π½
3. 0,49 с
4. 5 А
5. 0,02 Π”ΠΆ
6. 1,79 А
2 Π²Π°Ρ€ΠΈΠ°Π½Ρ‚
1. 1,6 Π’Π»
2. 60 Π’
3. 0,32 Π”ΠΆ
4. 0,125 Π“Π½
5. 50 м/с
6. 6,75 мКл
3 Π²Π°Ρ€ΠΈΠ°Π½Ρ‚
1. 1600 Π’
2. 5,8 м/с
3. 0,5 с
4. 10 000
5. 120Β°
6. 32 мОм
4 Π²Π°Ρ€ΠΈΠ°Π½Ρ‚
1. 0,4 Π“Π½
2. 0,003 Π’
3. 21,5 Π’
4. 2,52 Π”ΠΆ
5. 3,14 Кл
6. 0,13 ΠΌ

Π€ΠΈΠ·ΠΈΠΊΠ° 9 ΠŸΠ΅Ρ€Ρ‹ΡˆΠΊΠΈΠ½ ΠšΠΎΠ½Ρ‚Ρ€ΠΎΠ»ΡŒΠ½Π°Ρ Ρ€Π°Π±ΠΎΡ‚Π° 6 с ΠΎΡ‚Π²Π΅Ρ‚Π°ΠΌΠΈ

Π€ΠΈΠ·ΠΈΠΊΠ° 9 ΠŸΠ΅Ρ€Ρ‹ΡˆΠΊΠΈΠ½ ΠšΠΎΠ½Ρ‚Ρ€ΠΎΠ»ΡŒΠ½Π°Ρ Ρ€Π°Π±ΠΎΡ‚Π° 6 Β«Π­Π»Π΅ΠΊΡ‚Ρ€ΠΎΠΌΠ°Π³Π½ΠΈΡ‚Π½ΠΎΠ΅ ΠΏΠΎΠ»Π΅Β» с ΠΎΡ‚Π²Π΅Ρ‚Π°ΠΌΠΈ (4 Π²Π°Ρ€ΠΈΠ°Π½Ρ‚Π°). РСшСния Π·Π°Π΄Π°Ρ‡ ΠΈΠ· пособия Β«Π€ΠΈΠ·ΠΈΠΊΠ° 9 класс: ДидактичСскиС ΠΌΠ°Ρ‚Π΅Ρ€ΠΈΠ°Π»Ρ‹Β» (Π°Π²Ρ‚ΠΎΡ€Ρ‹: А.Π•. ΠœΠ°Ρ€ΠΎΠ½, Π•.А. ΠœΠ°Ρ€ΠΎΠ½). Π¦ΠΈΡ‚Π°Ρ‚Ρ‹ ΠΈΠ· пособия ΡƒΠΊΠ°Π·Π°Π½Ρ‹ Π² ΡƒΡ‡Π΅Π±Π½Ρ‹Ρ… цСлях. ΠžΡ‚Π²Π΅Ρ‚Ρ‹ адрСсованы родитСлям.

Π€ΠΈΠ·ΠΈΠΊΠ° 9 класс (УМК ΠŸΠ΅Ρ€Ρ‹ΡˆΠΊΠΈΠ½)


ΠšΠΎΠ½Ρ‚Ρ€ΠΎΠ»ΡŒΠ½Π°Ρ Ρ€Π°Π±ΠΎΡ‚Π° β„– 6
Π­Π»Π΅ΠΊΡ‚Ρ€ΠΎΠΌΠ°Π³Π½ΠΈΡ‚Π½ΠΎΠ΅ ΠΏΠΎΠ»Π΅

К-6. Π’Π°Ρ€ΠΈΠ°Π½Ρ‚ 1.

  1. По Π³Ρ€Π°Ρ„ΠΈΠΊΡƒ (рис. 129) ΠΎΠΏΡ€Π΅Π΄Π΅Π»ΠΈΡ‚Π΅ ΠΏΠ΅Ρ€ΠΈΠΎΠ΄, частоту ΠΈ Π°ΠΌΠΏΠ»ΠΈΡ‚ΡƒΠ΄Ρƒ ΠΊΠΎΠ»Π΅Π±Π°Π½ΠΈΠΉ силы Ρ‚ΠΎΠΊΠ°.
  2. На ΠΊΠ°ΠΊΠΎΠΉ частотС Ρ€Π°Π±ΠΎΡ‚Π°Π΅Ρ‚ радиостанция, пСрСдавая ΠΏΡ€ΠΎΠ³Ρ€Π°ΠΌΠΌΡƒ Π½Π° Π²ΠΎΠ»Π½Π΅ Π΄Π»ΠΈΠ½ΠΎΠΉ 250 ΠΌ?
  3. ΠžΠΏΡ€Π΅Π΄Π΅Π»ΠΈΡ‚Π΅ силу Ρ‚ΠΎΠΊΠ°, проходящСго ΠΏΠΎ прямолинСйному ΠΏΡ€ΠΎΠ²ΠΎΠ΄Π½ΠΈΠΊΡƒ, находящСмуся Π² ΠΎΠ΄Π½ΠΎΡ€ΠΎΠ΄Π½ΠΎΠΌ ΠΌΠ°Π³Π½ΠΈΡ‚Π½ΠΎΠΌ ΠΏΠΎΠ»Π΅ с ΠΈΠ½Π΄ΡƒΠΊΡ†ΠΈΠ΅ΠΉ 10 Π’Π», Ссли Π½Π° Π°ΠΊΡ‚ΠΈΠ²Π½ΡƒΡŽ Ρ‡Π°ΡΡ‚ΡŒ ΠΏΡ€ΠΎΠ²ΠΎΠ΄Π½ΠΈΠΊΠ° Π΄Π»ΠΈΠ½ΠΎΠΉ 40 см дСйствуСт сила 20 Н. ΠŸΡ€ΠΎΠ²ΠΎΠ΄Π½ΠΈΠΊ располоТСн пСрпСндикулярно линиям ΠΌΠ°Π³Π½ΠΈΡ‚Π½ΠΎΠΉ ΠΈΠ½Π΄ΡƒΠΊΡ†ΠΈΠΈ.
  4. ΠŸΡ€ΠΎΡ‚ΠΎΠ½ двиТСтся со ΡΠΊΠΎΡ€ΠΎΡΡ‚ΡŒΡŽ 106 ΠΌ/с пСрпСндикулярно ΠΎΠ΄Π½ΠΎΡ€ΠΎΠ΄Π½ΠΎΠΌΡƒ ΠΌΠ°Π³Π½ΠΈΡ‚Π½ΠΎΠΌΡƒ полю с ΠΈΠ½Π΄ΡƒΠΊΡ†ΠΈΠ΅ΠΉ 1 Π’Π». ΠžΠΏΡ€Π΅Π΄Π΅Π»ΠΈΡ‚Π΅ силу, Π΄Π΅ΠΉΡΡ‚Π²ΡƒΡŽΡ‰ΡƒΡŽ Π½Π° ΠΏΡ€ΠΎΡ‚ΠΎΠ½.
  5. Π­Π»Π΅ΠΊΡ‚Ρ€ΠΎΠ½ описываСт Π² ΠΎΠ΄Π½ΠΎΡ€ΠΎΠ΄Π½ΠΎΠΌ ΠΌΠ°Π³Π½ΠΈΡ‚Π½ΠΎΠΌ ΠΏΠΎΠ»Π΅ ΠΎΠΊΡ€ΡƒΠΆΠ½ΠΎΡΡ‚ΡŒ радиусом 4 ΠΌΠΌ. Π‘ΠΊΠΎΡ€ΠΎΡΡ‚ΡŒ двиТСния элСктрона Ρ€Π°Π²Π½Π° 3,5 β€’ 106 ΠΌ/с. ΠžΠΏΡ€Π΅Π΄Π΅Π»ΠΈΡ‚Π΅ ΠΈΠ½Π΄ΡƒΠΊΡ†ΠΈΡŽ ΠΌΠ°Π³Π½ΠΈΡ‚Π½ΠΎΠ³ΠΎ поля.
  6. Какова сила Ρ‚ΠΎΠΊΠ° Π² прямолинСйном ΠΏΡ€ΠΎΠ²ΠΎΠ΄Π½ΠΈΠΊΠ΅, ΠΏΠΎΠΌΠ΅Ρ‰Π΅Π½Π½ΠΎΠΌ Π² ΠΎΠ΄Π½ΠΎΡ€ΠΎΠ΄Π½ΠΎΠ΅ ΠΌΠ°Π³Π½ΠΈΡ‚Π½ΠΎΠ΅ ΠΏΠΎΠ»Π΅ пСрпСндикулярно линиям ΠΈΠ½Π΄ΡƒΠΊΡ†ΠΈΠΈ, Ссли ΠΎΠ½ Π½Π΅ ΠΏΠ°Π΄Π°Π΅Ρ‚? 1 ΠΌ Π΅Π³ΠΎ Π΄Π»ΠΈΠ½Ρ‹ ΠΈΠΌΠ΅Π΅Ρ‚ массу 3 ΠΊΠ³, Π° индукция ΠΌΠ°Π³Π½ΠΈΡ‚Π½ΠΎΠ³ΠΎ поля Ρ€Π°Π²Π½Π° 20 Π’Π».

К-6. Π’Π°Ρ€ΠΈΠ°Π½Ρ‚ 2.

  1. По Π³Ρ€Π°Ρ„ΠΈΠΊΡƒ (рис. 130) ΠΎΠΏΡ€Π΅Π΄Π΅Π»ΠΈΡ‚Π΅ ΠΏΠ΅Ρ€ΠΈΠΎΠ΄, частоту ΠΈ Π°ΠΌΠΏΠ»ΠΈΡ‚ΡƒΠ΄Ρƒ ΠΊΠΎΠ»Π΅Π±Π°Π½ΠΈΠΉ силы Ρ‚ΠΎΠΊΠ°.
  2. Π§Π΅ΠΌΡƒ Ρ€Π°Π²Π½Π° Π΄Π»ΠΈΠ½Π° Π²ΠΎΠ»Π½, посылаСмых радиостанциСй, Ρ€Π°Π±ΠΎΡ‚Π°ΡŽΡ‰Π΅ΠΉ Π½Π° частотС 1400 ΠΊΠ“Ρ†?
  3. На прямолинСйный ΠΏΡ€ΠΎΠ²ΠΎΠ΄Π½ΠΈΠΊ с Ρ‚ΠΎΠΊΠΎΠΌ, ΠΏΠΎΠΌΠ΅Ρ‰Π΅Π½Π½Ρ‹ΠΉ Π² ΠΎΠ΄Π½ΠΎΡ€ΠΎΠ΄Π½ΠΎΠ΅ ΠΌΠ°Π³Π½ΠΈΡ‚Π½ΠΎΠ΅ ΠΏΠΎΠ»Π΅ с ΠΈΠ½Π΄ΡƒΠΊΡ†ΠΈΠ΅ΠΉ 0,34 Π’Π», дСйствуСт сила 1,65 Н. ΠžΠΏΡ€Π΅Π΄Π΅Π»ΠΈΡ‚Π΅ Π΄Π»ΠΈΠ½Ρƒ ΠΏΡ€ΠΎΠ²ΠΎΠ΄Π½ΠΈΠΊΠ°, Ссли ΠΎΠ½ располоТСн пСрпСндикулярно линиям ΠΈΠ½Π΄ΡƒΠΊΡ†ΠΈΠΈ ΠΌΠ°Π³Π½ΠΈΡ‚Π½ΠΎΠ³ΠΎ поля. Π‘ΠΈΠ»Π° Ρ‚ΠΎΠΊΠ° Π² ΠΏΡ€ΠΎΠ²ΠΎΠ΄Π½ΠΈΠΊΠ΅ 14,5 А.
  4. Π­Π»Π΅ΠΊΡ‚Ρ€ΠΎΠ½ Π²Π»Π΅Ρ‚Π°Π΅Ρ‚ Π² ΠΎΠ΄Π½ΠΎΡ€ΠΎΠ΄Π½ΠΎΠ΅ ΠΌΠ°Π³Π½ΠΈΡ‚Π½ΠΎΠ΅ ΠΏΠΎΠ»Π΅ с ΠΈΠ½Π΄ΡƒΠΊΡ†ΠΈΠ΅ΠΉ 0,5 Π’Π» со ΡΠΊΠΎΡ€ΠΎΡΡ‚ΡŒΡŽ 20 000 ΠΊΠΌ/с пСрпСндикулярно линиям ΠΌΠ°Π³Π½ΠΈΡ‚Π½ΠΎΠΉ ΠΈΠ½Π΄ΡƒΠΊΡ†ΠΈΠΈ. ΠžΠΏΡ€Π΅Π΄Π΅Π»ΠΈΡ‚Π΅ силу, с ΠΊΠΎΡ‚ΠΎΡ€ΠΎΠΉ ΠΌΠ°Π³Π½ΠΈΡ‚Π½ΠΎΠ΅ ΠΏΠΎΠ»Π΅ дСйствуСт Π½Π° элСктрон.
  5. Π­Π»Π΅ΠΊΡ‚Ρ€ΠΎΠ½, двигаясь со ΡΠΊΠΎΡ€ΠΎΡΡ‚ΡŒΡŽ 3,54 β€’ 105 ΠΌ/с, ΠΏΠΎΠΏΠ°Π΄Π°Π΅Ρ‚ Π² ΠΎΠ΄Π½ΠΎΡ€ΠΎΠ΄Π½ΠΎΠ΅ ΠΌΠ°Π³Π½ΠΈΡ‚Π½ΠΎΠ΅ ΠΏΠΎΠ»Π΅ с ΠΈΠ½Π΄ΡƒΠΊΡ†ΠΈΠ΅ΠΉ 2 β€’ 10–5 Π’Π» пСрпСндикулярно линиям ΠΌΠ°Π³Π½ΠΈΡ‚Π½ΠΎΠΉ ΠΈΠ½Π΄ΡƒΠΊΡ†ΠΈΠΈ ΠΈ ΠΏΡ€ΠΎΠ΄ΠΎΠ»ΠΆΠ°Π΅Ρ‚ Π΄Π²ΠΈΠ³Π°Ρ‚ΡŒΡΡ ΠΏΠΎ окруТности радиусом 10 см. ΠžΠΏΡ€Π΅Π΄Π΅Π»ΠΈΡ‚Π΅ ΠΎΡ‚Π½ΠΎΡˆΠ΅Π½ΠΈΠ΅ заряда элСктрона ΠΊ Π΅Π³ΠΎ массС.
  6. Π‘ΠΈΠ»Π° Ρ‚ΠΎΠΊΠ° Π² Π³ΠΎΡ€ΠΈΠ·ΠΎΠ½Ρ‚Π°Π»ΡŒΠ½ΠΎ располоТСнном ΠΏΡ€ΠΎΠ²ΠΎΠ΄Π½ΠΈΠΊΠ΅ Π΄Π»ΠΈΠ½ΠΎΠΉ 10 см ΠΈ массой 2 Π³ Ρ€Π°Π²Π½Π° 10 А. Какова индукция ΠΌΠ°Π³Π½ΠΈΡ‚Π½ΠΎΠ³ΠΎ поля, Π² ΠΊΠΎΡ‚ΠΎΡ€ΠΎΠ΅ Π½ΡƒΠΆΠ½ΠΎ ΠΏΠΎΠΌΠ΅ΡΡ‚ΠΈΡ‚ΡŒ ΠΏΡ€ΠΎΠ²ΠΎΠ΄Π½ΠΈΠΊ, Ρ‡Ρ‚ΠΎΠ±Ρ‹ сила тяТСсти ΡƒΡ€Π°Π²Π½ΠΎΠ²Π΅ΡΠΈΠ»Π°ΡΡŒ силой, Π΄Π΅ΠΉΡΡ‚Π²ΡƒΡŽΡ‰Π΅ΠΉ Π½Π° ΠΏΡ€ΠΎΠ²ΠΎΠ΄Π½ΠΈΠΊ со стороны ΠΌΠ°Π³Π½ΠΈΡ‚Π½ΠΎΠ³ΠΎ поля?

К-6. Π’Π°Ρ€ΠΈΠ°Π½Ρ‚ 3.

  1. По Π³Ρ€Π°Ρ„ΠΈΠΊΡƒ (рис. 131) ΠΎΠΏΡ€Π΅Π΄Π΅Π»ΠΈΡ‚Π΅ ΠΏΠ΅Ρ€ΠΈΠΎΠ΄, частоту ΠΈ Π°ΠΌΠΏΠ»ΠΈΡ‚ΡƒΠ΄Ρƒ ΠΊΠΎΠ»Π΅Π±Π°Π½ΠΈΠΉ силы Ρ‚ΠΎΠΊΠ°.
  2. Радиостанция Π²Π΅Π΄Π΅Ρ‚ ΠΏΠ΅Ρ€Π΅Π΄Π°Ρ‡ΠΈ Π½Π° частотС 70 ΠœΠ“Ρ† (Π£ΠšΠ’). Π§Π΅ΠΌΡƒ Ρ€Π°Π²Π½Π° Π΄Π»ΠΈΠ½Π° Π²ΠΎΠ»Π½Ρ‹?
  3. Π’ ΠΎΠ΄Π½ΠΎΡ€ΠΎΠ΄Π½ΠΎΠ΅ ΠΌΠ°Π³Π½ΠΈΡ‚Π½ΠΎΠ΅ ΠΏΠΎΠ»Π΅, индукция ΠΊΠΎΡ‚ΠΎΡ€ΠΎΠ³ΠΎ 1,26 ΠΌΠ’Π», ΠΏΠΎΠΌΠ΅Ρ‰Π΅Π½ прямой ΠΏΡ€ΠΎΠ²ΠΎΠ΄Π½ΠΈΠΊ Π΄Π»ΠΈΠ½ΠΎΠΉ 20 см пСрпСндикулярно линиям ΠΌΠ°Π³Π½ΠΈΡ‚Π½ΠΎΠΉ ΠΈΠ½Π΄ΡƒΠΊΡ†ΠΈΠΈ. ΠžΠΏΡ€Π΅Π΄Π΅Π»ΠΈΡ‚Π΅ силу, Π΄Π΅ΠΉΡΡ‚Π²ΡƒΡŽΡ‰ΡƒΡŽ Π½Π° ΠΏΡ€ΠΎΠ²ΠΎΠ΄Π½ΠΈΠΊ, Ссли сила Ρ‚ΠΎΠΊΠ° Π² Π½Π΅ΠΌ 50 А.
  4. Π­Π»Π΅ΠΊΡ‚Ρ€ΠΎΠ½ двиТСтся со ΡΠΊΠΎΡ€ΠΎΡΡ‚ΡŒΡŽ 3 β€’ 106 ΠΌ/с Π² ΠΎΠ΄Π½ΠΎΡ€ΠΎΠ΄Π½ΠΎΠΌ ΠΌΠ°Π³Π½ΠΈΡ‚Π½ΠΎΠΌ ΠΏΠΎΠ»Π΅ с ΠΈΠ½Π΄ΡƒΠΊΡ†ΠΈΠ΅ΠΉ 0,1 Π’Π». Π§Π΅ΠΌΡƒ Ρ€Π°Π²Π½Π° сила, Π΄Π΅ΠΉΡΡ‚Π²ΡƒΡŽΡ‰Π°Ρ Π½Π° элСктрон, Ссли ΡƒΠ³ΠΎΠ» ΠΌΠ΅ΠΆΠ΄Ρƒ Π½Π°ΠΏΡ€Π°Π²Π»Π΅Π½ΠΈΠ΅ΠΌ скорости элСктрона ΠΈ линиями ΠΌΠ°Π³Π½ΠΈΡ‚Π½ΠΎΠΉ ΠΈΠ½Π΄ΡƒΠΊΡ†ΠΈΠΈ Ρ€Π°Π²Π΅Π½ 90Β°?
  5. Π­Π»Π΅ΠΊΡ‚Ρ€ΠΎΠ½ ΠΈ ΠΏΡ€ΠΎΡ‚ΠΎΠ½, двигаясь с ΠΎΠ΄ΠΈΠ½Π°ΠΊΠΎΠ²Ρ‹ΠΌΠΈ скоростями, ΠΏΠΎΠΏΠ°Π΄Π°ΡŽΡ‚ Π² ΠΎΠ΄Π½ΠΎΡ€ΠΎΠ΄Π½ΠΎΠ΅ ΠΌΠ°Π³Π½ΠΈΡ‚Π½ΠΎΠ΅ ΠΏΠΎΠ»Π΅ пСрпСндикулярно ΠΊ линиям ΠΈΠ½Π΄ΡƒΠΊΡ†ΠΈΠΈ. Π‘Ρ€Π°Π²Π½ΠΈΡ‚Π΅ радиусы ΠΊΡ€ΠΈΠ²ΠΈΠ·Π½Ρ‹ Re ΠΈ Rp Ρ‚Ρ€Π°Π΅ΠΊΡ‚ΠΎΡ€ΠΈΠΉ двиТСния элСктрона ΠΈ ΠΏΡ€ΠΎΡ‚ΠΎΠ½Π°.
  6. Π’ ΠΎΠ΄Π½ΠΎΡ€ΠΎΠ΄Π½ΠΎΠΌ ΠΌΠ°Π³Π½ΠΈΡ‚Π½ΠΎΠΌ ΠΏΠΎΠ»Π΅ с ΠΈΠ½Π΄ΡƒΠΊΡ†ΠΈΠ΅ΠΉ 0,25 Π’Π» Π³ΠΎΡ€ΠΈΠ·ΠΎΠ½Ρ‚Π°Π»ΡŒΠ½ΠΎ располоТСн ΠΏΡ€ΠΎΠ²ΠΎΠ΄Π½ΠΈΠΊ Π΄Π»ΠΈΠ½ΠΎΠΉ 10 см ΠΈ массой 40 Π³. Π›ΠΈΠ½ΠΈΠΈ ΠΈΠ½Π΄ΡƒΠΊΡ†ΠΈΠΈ ΠΌΠ°Π³Π½ΠΈΡ‚Π½ΠΎΠ³ΠΎ поля пСрпСндикулярны ΠΏΡ€ΠΎΠ²ΠΎΠ΄Π½ΠΈΠΊΡƒ. Какой силы Ρ‚ΠΎΠΊ Π΄ΠΎΠ»ΠΆΠ΅Π½ ΠΈΠ΄Ρ‚ΠΈ ΠΏΠΎ ΠΏΡ€ΠΎΠ²ΠΎΠ΄Π½ΠΈΠΊΡƒ, Ρ‡Ρ‚ΠΎΠ±Ρ‹ ΠΎΠ½ находился Π² равновСсии Π² ΠΌΠ°Π³Π½ΠΈΡ‚Π½ΠΎΠΌ ΠΏΠΎΠ»Π΅?

К-6. Π’Π°Ρ€ΠΈΠ°Π½Ρ‚ 4.

  1. По Π³Ρ€Π°Ρ„ΠΈΠΊΡƒ (рис. 132) ΠΎΠΏΡ€Π΅Π΄Π΅Π»ΠΈΡ‚Π΅ ΠΏΠ΅Ρ€ΠΈΠΎΠ΄, частоту ΠΈ Π°ΠΌΠΏΠ»ΠΈΡ‚ΡƒΠ΄Ρƒ ΠΊΠΎΠ»Π΅Π±Π°Π½ΠΈΠΉ силы Ρ‚ΠΎΠΊΠ°.
  2. ΠžΠΏΡ€Π΅Π΄Π΅Π»ΠΈΡ‚Π΅, Π½Π° ΠΊΠ°ΠΊΠΎΠΉ частотС Ρ€Π°Π±ΠΎΡ‚Π°Π΅Ρ‚ радиостанция, ΠΏΠ΅Ρ€Π΅Π΄Π°ΡŽΡ‰Π°Ρ ΠΏΡ€ΠΎΠ³Ρ€Π°ΠΌΠΌΡƒ Π½Π° Π²ΠΎΠ»Π½Π΅ 500 ΠΌ.
  3. ΠŸΡ€ΡΠΌΠΎΠ»ΠΈΠ½Π΅ΠΉΠ½Ρ‹ΠΉ ΠΏΡ€ΠΎΠ²ΠΎΠ΄Π½ΠΈΠΊ Π΄Π»ΠΈΠ½ΠΎΠΉ 40 см ΠΏΠΎΠΌΠ΅Ρ‰Π΅Π½ Π² ΠΎΠ΄Π½ΠΎΡ€ΠΎΠ΄Π½ΠΎΠ΅ ΠΌΠ°Π³Π½ΠΈΡ‚Π½ΠΎΠ΅ ΠΏΠΎΠ»Π΅ пСрпСндикулярно линиям ΠΈΠ½Π΄ΡƒΠΊΡ†ΠΈΠΈ. ΠžΠΏΡ€Π΅Π΄Π΅Π»ΠΈΡ‚Π΅ ΠΌΠ°Π³Π½ΠΈΡ‚Π½ΡƒΡŽ ΠΈΠ½Π΄ΡƒΠΊΡ†ΠΈΡŽ поля, Ссли Π½Π° ΠΏΡ€ΠΎΠ²ΠΎΠ΄Π½ΠΈΠΊ со стороны ΠΌΠ°Π³Π½ΠΈΡ‚Π½ΠΎΠ³ΠΎ поля дСйствуСт сила Π² 4 Н, ΠΊΠΎΠ³Π΄Π° ΠΏΠΎ Π½Π΅ΠΌΡƒ ΠΏΡ€ΠΎΡ…ΠΎΠ΄ΠΈΡ‚ Ρ‚ΠΎΠΊ 2 А.
  4. ΠŸΡ€ΠΎΡ‚ΠΎΠ½ двиТСтся Π² ΠΎΠ΄Π½ΠΎΡ€ΠΎΠ΄Π½ΠΎΠΌ ΠΌΠ°Π³Π½ΠΈΡ‚Π½ΠΎΠΌ ΠΏΠΎΠ»Π΅ с ΠΈΠ½Π΄ΡƒΠΊΡ†ΠΈΠ΅ΠΉ 5 ΠΌΠ’Π» со ΡΠΊΠΎΡ€ΠΎΡΡ‚ΡŒΡŽ 10 000 ΠΊΠΌ/с, Π½Π°ΠΏΡ€Π°Π²Π»Π΅Π½Π½ΠΎΠΉ пСрпСндикулярно линиям ΠΌΠ°Π³Π½ΠΈΡ‚Π½ΠΎΠΉ ΠΈΠ½Π΄ΡƒΠΊΡ†ΠΈΠΈ. ΠžΠΏΡ€Π΅Π΄Π΅Π»ΠΈΡ‚Π΅ силу, Π΄Π΅ΠΉΡΡ‚Π²ΡƒΡŽΡ‰ΡƒΡŽ Π½Π° ΠΏΡ€ΠΎΡ‚ΠΎΠ½.
  5. Π­Π»Π΅ΠΊΡ‚Ρ€ΠΎΠ½ Π²Π»Π΅Ρ‚Π°Π΅Ρ‚ Π² ΠΌΠ°Π³Π½ΠΈΡ‚Π½ΠΎΠ΅ ΠΏΠΎΠ»Π΅ пСрпСндикулярно линиям ΠΌΠ°Π³Π½ΠΈΡ‚Π½ΠΎΠΉ ΠΈΠ½Π΄ΡƒΠΊΡ†ΠΈΠΈ со ΡΠΊΠΎΡ€ΠΎΡΡ‚ΡŒΡŽ 107 ΠΌ/с. РассчитайтС радиус ΠΊΡ€ΠΈΠ²ΠΈΠ·Π½Ρ‹ Ρ‚Ρ€Π°Π΅ΠΊΡ‚ΠΎΡ€ΠΈΠΈ, ΠΏΠΎ ΠΊΠΎΡ‚ΠΎΡ€ΠΎΠΉ Π±ΡƒΠ΄Π΅Ρ‚ Π΄Π²ΠΈΠ³Π°Ρ‚ΡŒΡΡ элСктрон, Ссли индукция ΠΌΠ°Π³Π½ΠΈΡ‚Π½ΠΎΠ³ΠΎ поля 5,6 ΠΌΠ’Π».
  6. ΠŸΡ€ΡΠΌΠΎΠ»ΠΈΠ½Π΅ΠΉΠ½Ρ‹ΠΉ ΠΏΡ€ΠΎΠ²ΠΎΠ΄Π½ΠΈΠΊ массой 0,02 ΠΊΠ³ ΠΈ Π΄Π»ΠΈΠ½ΠΎΠΉ 50 см ΠΏΠΎΠΌΠ΅Ρ‰Π΅Π½ Π² ΠΎΠ΄Π½ΠΎΡ€ΠΎΠ΄Π½ΠΎΠ΅ ΠΌΠ°Π³Π½ΠΈΡ‚Π½ΠΎΠ΅ ΠΏΠΎΠ»Π΅ пСрпСндикулярно линиям ΠΌΠ°Π³Π½ΠΈΡ‚Π½ΠΎΠΉ ΠΈΠ½Π΄ΡƒΠΊΡ†ΠΈΠΈ. Какой Π΄ΠΎΠ»ΠΆΠ½Π° Π±Ρ‹Ρ‚ΡŒ индукция ΠΌΠ°Π³Π½ΠΈΡ‚Π½ΠΎΠ³ΠΎ поля, Ρ‡Ρ‚ΠΎΠ±Ρ‹ ΠΏΡ€ΠΎΠ²ΠΎΠ΄Π½ΠΈΠΊ висСл Π½Π΅ падая, Ссли сила Ρ‚ΠΎΠΊΠ° Π² ΠΏΡ€ΠΎΠ²ΠΎΠ΄Π½ΠΈΠΊΠ΅ 2 А?

Β 

ΠŸΡ€ΠΈΠΌΠ΅Ρ‡Π°Π½ΠΈΠ΅: Масса элСктрона 9,1 β€’ 10–31 ΠΊΠ³. Заряд элСктрона –1,6 β€’ 10–19 Кл. Масса ΠΏΡ€ΠΎΡ‚ΠΎΠ½Π° 1,67 β€’ 10–27 ΠΊΠ³. Заряд ΠΏΡ€ΠΎΡ‚ΠΎΠ½Π° +1,6 β€’ 10–19 Кл.


Β 

ΠžΠ’Π’Π•Π’Π« Π½Π° ΠΊΠΎΠ½Ρ‚Ρ€ΠΎΠ»ΡŒΠ½ΡƒΡŽ Ρ€Π°Π±ΠΎΡ‚Ρƒ:

Β 

Π’Π΅Ρ€Π½ΡƒΡ‚ΡŒΡΡ ΠΊ Бписку ΠΊΠΎΠ½Ρ‚Ρ€ΠΎΠ»ΡŒΠ½Ρ‹Ρ… Ρ€Π°Π±ΠΎΡ‚ ΠΏΠΎ Ρ„ΠΈΠ·ΠΈΠΊΠ΅ Π² 9 классС


Π’Ρ‹ смотрСли: Π€ΠΈΠ·ΠΈΠΊΠ° 9 ΠŸΠ΅Ρ€Ρ‹ΡˆΠΊΠΈΠ½ ΠšΠΎΠ½Ρ‚Ρ€ΠΎΠ»ΡŒΠ½Π°Ρ Ρ€Π°Π±ΠΎΡ‚Π° 6 Β«Π­Π»Π΅ΠΊΡ‚Ρ€ΠΎΠΌΠ°Π³Π½ΠΈΡ‚Π½ΠΎΠ΅ ΠΏΠΎΠ»Π΅Β» с ΠΎΡ‚Π²Π΅Ρ‚Π°ΠΌΠΈ. РСшСния Π·Π°Π΄Π°Ρ‡ ΠΈΠ· пособия Β«Π€ΠΈΠ·ΠΈΠΊΠ° 9 класс: ДидактичСскиС ΠΌΠ°Ρ‚Π΅Ρ€ΠΈΠ°Π»Ρ‹Β» (Π°Π²Ρ‚ΠΎΡ€Ρ‹: А.Π•. ΠœΠ°Ρ€ΠΎΠ½, Π•.А. ΠœΠ°Ρ€ΠΎΠ½). Π¦ΠΈΡ‚Π°Ρ‚Ρ‹ ΠΈΠ· пособия ΡƒΠΊΠ°Π·Π°Π½Ρ‹ Π² ΡƒΡ‡Π΅Π±Π½Ρ‹Ρ… цСлях.

ΠœΠ°Π³Π½ΠΈΡ‚Π½Ρ‹Π΅ поля — AP Physics 2

Если Π²Ρ‹ считаСтС, Ρ‡Ρ‚ΠΎ ΠΊΠΎΠ½Ρ‚Π΅Π½Ρ‚, доступный Ρ‡Π΅Ρ€Π΅Π· Π’Π΅Π±-сайт (ΠΊΠ°ΠΊ ΠΎΠΏΡ€Π΅Π΄Π΅Π»Π΅Π½ΠΎ Π² Π½Π°ΡˆΠΈΡ… Условиях обслуТивания), Π½Π°Ρ€ΡƒΡˆΠ°Π΅Ρ‚ ΠΈΠ»ΠΈ большС Π²Π°ΡˆΠΈΡ… авторских ΠΏΡ€Π°Π², сообщитС Π½Π°ΠΌ, ΠΎΡ‚ΠΏΡ€Π°Π²ΠΈΠ² письмСнноС ΡƒΠ²Π΅Π΄ΠΎΠΌΠ»Π΅Π½ΠΈΠ΅ (Β«Π£Π²Π΅Π΄ΠΎΠΌΠ»Π΅Π½ΠΈΠ΅ ΠΎ Π½Π°Ρ€ΡƒΡˆΠ΅Π½ΠΈΠΈΒ»), содСрТащСС Π² ΠΈΠ½Ρ„ΠΎΡ€ΠΌΠ°Ρ†ΠΈΡŽ, ΠΎΠΏΠΈΡΠ°Π½Π½ΡƒΡŽ Π½ΠΈΠΆΠ΅, Π½Π°Π·Π½Π°Ρ‡Π΅Π½Π½ΠΎΠΌΡƒ Π½ΠΈΠΆΠ΅ Π°Π³Π΅Π½Ρ‚Ρƒ. Если Ρ€Π΅ΠΏΠ΅Ρ‚ΠΈΡ‚ΠΎΡ€Ρ‹ унивСрситСта ΠΏΡ€Π΅Π΄ΠΏΡ€ΠΈΠΌΡƒΡ‚ дСйствия Π² ΠΎΡ‚Π²Π΅Ρ‚ Π½Π° Π°Π½ Π£Π²Π΅Π΄ΠΎΠΌΠ»Π΅Π½ΠΈΠ΅ ΠΎ Π½Π°Ρ€ΡƒΡˆΠ΅Π½ΠΈΠΈ, ΠΎΠ½ΠΎ ΠΏΡ€Π΅Π΄ΠΏΡ€ΠΈΠΌΠ΅Ρ‚ Π΄ΠΎΠ±Ρ€ΠΎΡΠΎΠ²Π΅ΡΡ‚Π½ΡƒΡŽ ΠΏΠΎΠΏΡ‹Ρ‚ΠΊΡƒ ΡΠ²ΡΠ·Π°Ρ‚ΡŒΡΡ со стороной, которая прСдоставила Ρ‚Π°ΠΊΠΎΠΉ ΠΊΠΎΠ½Ρ‚Π΅Π½Ρ‚ срСдствами самого послСднСго адрСса элСктронной ΠΏΠΎΡ‡Ρ‚Ρ‹, Ссли Ρ‚Π°ΠΊΠΎΠ²ΠΎΠΉ имССтся, прСдоставлСнного Ρ‚Π°ΠΊΠΎΠΉ стороной Varsity Tutors.

Π’Π°ΡˆΠ΅ Π£Π²Π΅Π΄ΠΎΠΌΠ»Π΅Π½ΠΈΠ΅ ΠΎ Π½Π°Ρ€ΡƒΡˆΠ΅Π½ΠΈΠΈ ΠΏΡ€Π°Π² ΠΌΠΎΠΆΠ΅Ρ‚ Π±Ρ‹Ρ‚ΡŒ ΠΎΡ‚ΠΏΡ€Π°Π²Π»Π΅Π½ΠΎ сторонС, ΠΏΡ€Π΅Π΄ΠΎΡΡ‚Π°Π²ΠΈΠ²ΡˆΠ΅ΠΉ доступ ΠΊ ΠΊΠΎΠ½Ρ‚Π΅Π½Ρ‚Ρƒ, ΠΈΠ»ΠΈ Ρ‚Ρ€Π΅Ρ‚ΡŒΠΈΠΌ Π»ΠΈΡ†Π°ΠΌ, Ρ‚Π°ΠΊΠΈΠΌ ΠΊΠ°ΠΊ Π² Π²ΠΈΠ΄Π΅ ChillingEffects.org.

ΠžΠ±Ρ€Π°Ρ‚ΠΈΡ‚Π΅ Π²Π½ΠΈΠΌΠ°Π½ΠΈΠ΅, Ρ‡Ρ‚ΠΎ Π²Ρ‹ Π±ΡƒΠ΄Π΅Ρ‚Π΅ нСсти ΠΎΡ‚Π²Π΅Ρ‚ΡΡ‚Π²Π΅Π½Π½ΠΎΡΡ‚ΡŒ Π·Π° ΡƒΡ‰Π΅Ρ€Π± (Π²ΠΊΠ»ΡŽΡ‡Π°Ρ расходы ΠΈ Π³ΠΎΠ½ΠΎΡ€Π°Ρ€Ρ‹ Π°Π΄Π²ΠΎΠΊΠ°Ρ‚Π°ΠΌ), Ссли Π²Ρ‹ сущСствСнно ΠΈΡΠΊΠ°ΠΆΠ°Ρ‚ΡŒ ΠΈΠ½Ρ„ΠΎΡ€ΠΌΠ°Ρ†ΠΈΡŽ ΠΎ Ρ‚ΠΎΠΌ, Ρ‡Ρ‚ΠΎ ΠΏΡ€ΠΎΠ΄ΡƒΠΊΡ‚ ΠΈΠ»ΠΈ дСйствиС Π½Π°Ρ€ΡƒΡˆΠ°Π΅Ρ‚ ваши авторскиС ΠΏΡ€Π°Π²Π°. Π’Π°ΠΊΠΈΠΌ ΠΎΠ±Ρ€Π°Π·ΠΎΠΌ, Ссли Π²Ρ‹ Π½Π΅ ΡƒΠ²Π΅Ρ€Π΅Π½Ρ‹, Ρ‡Ρ‚ΠΎ ΠΊΠΎΠ½Ρ‚Π΅Π½Ρ‚ находится Π½Π° Π’Π΅Π±-сайтС ΠΈΠ»ΠΈ ΠΏΠΎ ссылкС с Π½Π΅Π³ΠΎ Π½Π°Ρ€ΡƒΡˆΠ°Π΅Ρ‚ ваши авторскиС ΠΏΡ€Π°Π²Π°, Π²Π°ΠΌ слСдуСт сначала ΠΎΠ±Ρ€Π°Ρ‚ΠΈΡ‚ΡŒΡΡ ΠΊ ΡŽΡ€ΠΈΡΡ‚Ρƒ.

Π§Ρ‚ΠΎΠ±Ρ‹ ΠΎΡ‚ΠΏΡ€Π°Π²ΠΈΡ‚ΡŒ ΡƒΠ²Π΅Π΄ΠΎΠΌΠ»Π΅Π½ΠΈΠ΅, Π²Ρ‹ΠΏΠΎΠ»Π½ΠΈΡ‚Π΅ ΡΠ»Π΅Π΄ΡƒΡŽΡ‰ΠΈΠ΅ дСйствия:

Π’Ρ‹ Π΄ΠΎΠ»ΠΆΠ½Ρ‹ Π²ΠΊΠ»ΡŽΡ‡ΠΈΡ‚ΡŒ ΡΠ»Π΅Π΄ΡƒΡŽΡ‰Π΅Π΅:

ЀизичСская ΠΈΠ»ΠΈ элСктронная подпись правообладатСля ΠΈΠ»ΠΈ Π»ΠΈΡ†Π°, ΡƒΠΏΠΎΠ»Π½ΠΎΠΌΠΎΡ‡Π΅Π½Π½ΠΎΠ³ΠΎ Π΄Π΅ΠΉΡΡ‚Π²ΠΎΠ²Π°Ρ‚ΡŒ ΠΎΡ‚ ΠΈΡ… ΠΈΠΌΠ΅Π½ΠΈ; Π˜Π΄Π΅Π½Ρ‚ΠΈΡ„ΠΈΠΊΠ°Ρ†ΠΈΡ авторских ΠΏΡ€Π°Π², ΠΊΠΎΡ‚ΠΎΡ€Ρ‹Π΅, ΠΊΠ°ΠΊ утвСрТдаСтся, Π±Ρ‹Π»ΠΈ Π½Π°Ρ€ΡƒΡˆΠ΅Π½Ρ‹; ОписаниС Ρ…Π°Ρ€Π°ΠΊΡ‚Π΅Ρ€Π° ΠΈ Ρ‚ΠΎΡ‡Π½ΠΎΠ³ΠΎ мСстонахоТдСния ΠΊΠΎΠ½Ρ‚Π΅Π½Ρ‚Π°, ΠΊΠΎΡ‚ΠΎΡ€Ρ‹ΠΉ, ΠΏΠΎ Π²Π°ΡˆΠ΅ΠΌΡƒ мнСнию, Π½Π°Ρ€ΡƒΡˆΠ°Π΅Ρ‚ ваши авторскиС ΠΏΡ€Π°Π²Π°, Π² \ достаточно подробностСй, Ρ‡Ρ‚ΠΎΠ±Ρ‹ ΠΏΠΎΠ·Π²ΠΎΠ»ΠΈΡ‚ΡŒ Ρ€Π΅ΠΏΠ΅Ρ‚ΠΈΡ‚ΠΎΡ€Π°ΠΌ унивСрситСтских школ Π½Π°ΠΉΡ‚ΠΈ ΠΈ Ρ‚ΠΎΡ‡Π½ΠΎ ΠΈΠ΄Π΅Π½Ρ‚ΠΈΡ„ΠΈΡ†ΠΈΡ€ΠΎΠ²Π°Ρ‚ΡŒ этот ΠΊΠΎΠ½Ρ‚Π΅Π½Ρ‚; Π½Π°ΠΏΡ€ΠΈΠΌΠ΅Ρ€ Π½Π°ΠΌ трСбуСтся Π° ссылка Π½Π° ΠΊΠΎΠ½ΠΊΡ€Π΅Ρ‚Π½Ρ‹ΠΉ вопрос (Π° Π½Π΅ Ρ‚ΠΎΠ»ΡŒΠΊΠΎ Π½Π° Π½Π°Π·Π²Π°Π½ΠΈΠ΅ вопроса), ΠΊΠΎΡ‚ΠΎΡ€Ρ‹ΠΉ содСрТит содСрТаниС ΠΈ описаниС ΠΊ ΠΊΠ°ΠΊΠΎΠΉ ΠΊΠΎΠ½ΠΊΡ€Π΅Ρ‚Π½ΠΎΠΉ части вопроса — ΠΈΠ·ΠΎΠ±Ρ€Π°ΠΆΠ΅Π½ΠΈΡŽ, ссылкС, тСксту ΠΈ Ρ‚. Π΄. — относится ваша ΠΆΠ°Π»ΠΎΠ±Π°; Π’Π°ΡˆΠ΅ имя, адрСс, Π½ΠΎΠΌΠ΅Ρ€ Ρ‚Π΅Π»Π΅Ρ„ΠΎΠ½Π° ΠΈ адрСс элСктронной ΠΏΠΎΡ‡Ρ‚Ρ‹; ΠΈ Π’Π°ΡˆΠ΅ заявлСниС: (Π°) Π²Ρ‹ добросовСстно считаСтС, Ρ‡Ρ‚ΠΎ использованиС ΠΊΠΎΠ½Ρ‚Π΅Π½Ρ‚Π°, ΠΊΠΎΡ‚ΠΎΡ€Ρ‹ΠΉ, ΠΏΠΎ Π²Π°ΡˆΠ΅ΠΌΡƒ мнСнию, Π½Π°Ρ€ΡƒΡˆΠ°Π΅Ρ‚ ваши авторскиС ΠΏΡ€Π°Π²Π° Π½Π΅ Ρ€Π°Π·Ρ€Π΅ΡˆΠ΅Π½Ρ‹ Π·Π°ΠΊΠΎΠ½ΠΎΠΌ, Π²Π»Π°Π΄Π΅Π»ΡŒΡ†Π΅ΠΌ авторских ΠΏΡ€Π°Π² ΠΈΠ»ΠΈ Π΅Π³ΠΎ Π°Π³Π΅Π½Ρ‚ΠΎΠΌ; (Π±) Ρ‡Ρ‚ΠΎ всС информация, содСрТащаяся Π² вашСм Π£Π²Π΅Π΄ΠΎΠΌΠ»Π΅Π½ΠΈΠΈ ΠΎ Π½Π°Ρ€ΡƒΡˆΠ΅Π½ΠΈΠΈ, являСтся Ρ‚ΠΎΡ‡Π½ΠΎΠΉ, ΠΈ (c) ΠΏΠΎΠ΄ страхом наказания Π·Π° Π»ΠΆΠ΅ΡΠ²ΠΈΠ΄Π΅Ρ‚Π΅Π»ΡŒΡΡ‚Π²ΠΎ, Ρ‡Ρ‚ΠΎ Π²Ρ‹ Π»ΠΈΠ±ΠΎ Π²Π»Π°Π΄Π΅Π»Π΅Ρ† авторских ΠΏΡ€Π°Π², Π»ΠΈΠ±ΠΎ Π»ΠΈΡ†ΠΎ, ΡƒΠΏΠΎΠ»Π½ΠΎΠΌΠΎΡ‡Π΅Π½Π½ΠΎΠ΅ Π΄Π΅ΠΉΡΡ‚Π²ΠΎΠ²Π°Ρ‚ΡŒ ΠΎΡ‚ ΠΈΡ… ΠΈΠΌΠ΅Π½ΠΈ.

ΠžΡ‚ΠΏΡ€Π°Π²ΡŒΡ‚Π΅ ΠΆΠ°Π»ΠΎΠ±Ρƒ Π½Π°ΡˆΠ΅ΠΌΡƒ ΡƒΠΏΠΎΠ»Π½ΠΎΠΌΠΎΡ‡Π΅Π½Π½ΠΎΠΌΡƒ Π°Π³Π΅Π½Ρ‚Ρƒ ΠΏΠΎ адрСсу:

Π§Π°Ρ€Π»ΡŒΠ· Кон Varsity Tutors LLC
101 S. Hanley Rd, Suite 300
St. Louis, MO 63105

Или Π·Π°ΠΏΠΎΠ»Π½ΠΈΡ‚Π΅ Ρ„ΠΎΡ€ΠΌΡƒ Π½ΠΈΠΆΠ΅:

ВСст Π½Π° ΠΌΠ°Π³Π½Π΅Ρ‚ΠΈΠ·ΠΌ

1) Когда ΡŽΠΆΠ½Ρ‹Π΅ полюса Π΄Π²ΡƒΡ… стСрТнСвых ΠΌΠ°Π³Π½ΠΈΡ‚ΠΎΠ² ΡΠ±Π»ΠΈΠΆΠ°ΡŽΡ‚ΡΡ, Π²ΠΎΠ·Π½ΠΈΠΊΠ°ΡŽΡ‚:

3) ΠžΠ±Ρ‹Ρ‡Π½Ρ‹ΠΉ Ρ‚ΠΎΠΊ ΠΏΠΎ ΠΏΡ€ΠΎΠ²ΠΎΠ΄Π½ΠΈΠΊΡƒ ΠΏΠΎΠΊΠ°Π·Π°Π½ Π½Π° схСмС.КакоС Π½Π°ΠΏΡ€Π°Π²Π»Π΅Π½ΠΈΠ΅ ΠΌΠ°Π³Π½ΠΈΡ‚Π½ΠΎΠ³ΠΎ поля Π² Ρ‚ΠΎΡ‡ΠΊΠ΅ P?

5) НаправлСниС ΠΌΠ°Π³Π½ΠΈΡ‚Π½ΠΎΠ³ΠΎ поля ΠΎΡ‚:

7) ΠœΠ°Π³Π½ΠΈΡ‚Π½ΠΎΠ΅ сопротивлСниС Π² ΠΌΠ°Π³Π½ΠΈΡ‚Π½ΠΎΠΉ Ρ†Π΅ΠΏΠΈ Π°Π½Π°Π»ΠΎΠ³ΠΈΡ‡Π½ΠΎ:

9) Если ΠΏΡ€ΠΎΠ²ΠΎΠ΄Π½ΠΈΠΊ пСрСмСщаСтся Π²ΠΏΠ΅Ρ€Π΅Π΄ ΠΈ Π½Π°Π·Π°Π΄ с постоянной ΡΠΊΠΎΡ€ΠΎΡΡ‚ΡŒΡŽ Π² постоянном ΠΌΠ°Π³Π½ΠΈΡ‚Π½ΠΎΠΌ ΠΏΠΎΠ»Π΅, ΠΈΠ½Π΄ΡƒΡ†ΠΈΡ€ΠΎΠ²Π°Π½Π½ΠΎΠ΅ Π² ΠΏΡ€ΠΎΠ²ΠΎΠ΄Π½ΠΈΠΊΠ΅ напряТСниС Π±ΡƒΠ΄Π΅Ρ‚:
2) ΠœΠ°Π³Π½ΠΈΡ‚Π½ΠΎΠ΅ ΠΏΠΎΠ»Π΅ состоит ΠΈΠ·:

4) На схСмС ΠΏΠΎΠΊΠ°Π·Π°Π½ ΠΎΠ±Ρ‹Ρ‡Π½Ρ‹ΠΉ Ρ‚ΠΎΠΊ Π² ΠΏΡ€ΠΎΠ²ΠΎΠ»ΠΎΡ‡Π½ΠΎΠΉ ΠΏΠ΅Ρ‚Π»Π΅.КакоС Π½Π°ΠΏΡ€Π°Π²Π»Π΅Π½ΠΈΠ΅ ΠΌΠ°Π³Π½ΠΈΡ‚Π½ΠΎΠ³ΠΎ поля Π² Ρ†Π΅Π½Ρ‚Ρ€Π΅ ΠΏΠ΅Ρ‚Π»ΠΈ?

6) Π­Π»Π΅ΠΊΡ‚Ρ€ΠΎΠΌΠ°Π³Π½ΠΈΡ‚ ΠΈΠΌΠ΅Π» Π±Ρ‹ Π½Π°ΠΈΠ±ΠΎΠ»ΡŒΡˆΡƒΡŽ ΠΏΡ€ΠΎΡ‡Π½ΠΎΡΡ‚ΡŒ, Ссли Π±Ρ‹ Π΅Π³ΠΎ ΠΏΡ€ΠΎΠ²ΠΎΠ΄ Π±Ρ‹Π» Π½Π°ΠΌΠΎΡ‚Π°Π½ Π½Π° сСрдСчник, сдСланный ΠΈΠ·:

8) Если ΠΏΡ€ΠΎΠ²ΠΎΠ΄, ΠΏΠΎ ΠΊΠΎΡ‚ΠΎΡ€ΠΎΠΌΡƒ ΠΏΡ€ΠΎΡ‚Π΅ΠΊΠ°Π΅Ρ‚ Ρ‚ΠΎΠΊ, ΠΏΠΎΠΌΠ΅Ρ‰Π΅Π½ Π² ΠΌΠ°Π³Π½ΠΈΡ‚Π½ΠΎΠ΅ ΠΏΠΎΠ»Π΅:

10) ΠšΠ°Ρ‚ΡƒΡˆΠΊΡƒ с ΠΏΡ€ΠΎΠ²ΠΎΠ»ΠΎΠΊΠΎΠΉ ΠΏΠΎΠΌΠ΅Ρ‰Π°ΡŽΡ‚ Π² ΠΈΠ·ΠΌΠ΅Π½ΡΡŽΡ‰Π΅Π΅ΡΡ ΠΌΠ°Π³Π½ΠΈΡ‚Π½ΠΎΠ΅ ΠΏΠΎΠ»Π΅.Если количСство Π²ΠΈΡ‚ΠΊΠΎΠ² Π² ΠΊΠ°Ρ‚ΡƒΡˆΠΊΠ΅ ΡƒΠ²Π΅Π»ΠΈΡ‡ΠΈΡ‚ΡŒ, напряТСниС Π½Π° ΠΊΠ°Ρ‚ΡƒΡˆΠΊΠ΅ Π±ΡƒΠ΄Π΅Ρ‚:

Как ΠΈΠ·ΠΌΠ΅Ρ€ΠΈΡ‚ΡŒ ΠΌΠ°Π³Π½ΠΈΡ‚Π½ΠΎΠ΅ ΠΏΠΎΠ»Π΅?

ΠœΠΎΠΆΠ΅Ρ‚, стоит Π²ΠΊΠ»ΡŽΡ‡ΠΈΡ‚ΡŒ ΠΏΠ°Ρ€Ρƒ ΡƒΡ€Π°Π²Π½Π΅Π½ΠΈΠΉ. Π’ΠΎ-ΠΏΠ΅Ρ€Π²Ρ‹Ρ…, Π΄Π²Π΅ силы, Π΄Π΅ΠΉΡΡ‚Π²ΡƒΡŽΡ‰ΠΈΠ΅ Π½Π° элСктричСский заряд, ΠΌΠΎΠΆΠ½ΠΎ Π·Π°ΠΏΠΈΡΠ°Ρ‚ΡŒ ΠΊΠ°ΠΊ силу Π›ΠΎΡ€Π΅Π½Ρ†Π°.

Π”Π°, это Π²Π΅ΠΊΡ‚ΠΎΡ€Π½ΠΎΠ΅ ΠΏΡ€ΠΎΠΈΠ·Π²Π΅Π΄Π΅Π½ΠΈΠ΅ ΠΌΠ°Π³Π½ΠΈΡ‚Π½ΠΎΠΉ части силы. ΠšΡ€ΠΎΠΌΠ΅ Ρ‚ΠΎΠ³ΠΎ, Ссли Ρƒ вас Π΅ΡΡ‚ΡŒ элСктричСскоС ΠΏΠΎΠ»Π΅, ΠΈΠ·ΠΌΠ΅Π½Π΅Π½ΠΈΠ΅ элСктричСского ΠΏΠΎΡ‚Π΅Π½Ρ†ΠΈΠ°Π»Π° ΠΌΠ΅ΠΆΠ΄Ρƒ двумя Ρ‚ΠΎΡ‡ΠΊΠ°ΠΌΠΈ Π±ΡƒΠ΄Π΅Ρ‚:

Если элСктричСскоС ΠΏΠΎΠ»Π΅ являСтся постоянным ΠΊΠ°ΠΊ ΠΏΠΎ Π½Π°ΠΏΡ€Π°Π²Π»Π΅Π½ΠΈΡŽ, Ρ‚Π°ΠΊ ΠΈ ΠΏΠΎ Π²Π΅Π»ΠΈΡ‡ΠΈΠ½Π΅, Ρ‚ΠΎ Π²Π΅Π»ΠΈΡ‡ΠΈΠ½Π° измСнСния элСктричСского ΠΏΠΎΡ‚Π΅Π½Ρ†ΠΈΠ°Π»Π° Π±ΡƒΠ΄Π΅Ρ‚ просто E * с .

Π’Π΅ΠΏΠ΅Ρ€ΡŒ ΠΌΡ‹ Π³ΠΎΡ‚ΠΎΠ²Ρ‹ ΠΊ Π΄Π°Ρ‚Ρ‡ΠΈΠΊΡƒ Π₯ΠΎΠ»Π»Π°. Π’ΠΎΡ‚ нСбольшой кусок ΠΌΠ°Ρ‚Π΅Ρ€ΠΈΠ°Π»Π° с Ρ‚ΠΎΠΊΠΎΠΌ, ΠΏΠΎΠΌΠ΅Ρ‰Π΅Π½Π½Ρ‹ΠΉ Π² ΠΌΠ°Π³Π½ΠΈΡ‚Π½ΠΎΠ΅ ΠΏΠΎΠ»Π΅. ПолС Π±ΡƒΠ΄Π΅Ρ‚ Π½Π°ΠΏΡ€Π°Π²Π»Π΅Π½ΠΎ Π½Π° экран. Π‘Π°ΠΌΡ‹ΠΉ простой способ ΠΏΠΎΠΊΠ°Π·Π°Ρ‚ΡŒ этот Ρ‚ΠΈΠΏ Π²Π΅ΠΊΡ‚ΠΎΡ€Π° — ΠΏΡ€Π΅Π΄ΡΡ‚Π°Π²ΠΈΡ‚ΡŒ Π΅Π³ΠΎ ΠΊΠ°ΠΊ Β«XΒ». Π”ΡƒΠΌΠ°ΠΉΡ‚Π΅ ΠΎ Β«XΒ» ΠΊΠ°ΠΊ ΠΎ ΠΊΠΎΠ½Ρ†Π΅ стрСлки (ΠΏΠ΅Ρ€ΡŒΡ). ΠŸΠΎΠ·Π²ΠΎΠ»ΡŒΡ‚Π΅ ΠΌΠ½Π΅ просто ΠΏΠΎΠΊΠ°Π·Π°Ρ‚ΡŒ ΠΎΠ΄ΠΈΠ½ двиТущийся элСктрон Π² этом ΠΌΠ°Ρ‚Π΅Ρ€ΠΈΠ°Π»Π΅.

ΠŸΠΎΡΠΊΠΎΠ»ΡŒΠΊΡƒ Ρ‚ΠΎΠΊ Π½Π°ΠΏΡ€Π°Π²Π»Π΅Π½ Π²Π²Π΅Ρ€Ρ…, ΡΠΊΠΎΡ€ΠΎΡΡ‚ΡŒ элСктронов Π±ΡƒΠ΄Π΅Ρ‚ ΡƒΠΌΠ΅Π½ΡŒΡˆΠ°Ρ‚ΡŒΡΡ (ΠΎΡ‚Ρ€ΠΈΡ†Π°Ρ‚Π΅Π»ΡŒΠ½Ρ‹ΠΉ заряд).Однако ΠΏΡ€ΠΎΠΈΠ·Π²Π΅Π΄Π΅Π½ΠΈΠ΅ Π½Π° ΠΈ Π½Π° Π±ΡƒΠ΄Π΅Ρ‚ Π²Π²Π΅Ρ€Ρ…, ΠΏΠΎΡΠΊΠΎΠ»ΡŒΠΊΡƒ заряд ΠΎΡ‚Ρ€ΠΈΡ†Π°Ρ‚Π΅Π»ΡŒΠ½Ρ‹ΠΉ. ΠœΠ°Π³Π½ΠΈΡ‚Π½Π°Ρ сила Π½Π° этом зарядС Π±ΡƒΠ΄Π΅Ρ‚ слСва. ΠžΠ±Ρ€Π°Ρ‚ΠΈΡ‚Π΅ Π²Π½ΠΈΠΌΠ°Π½ΠΈΠ΅, Ρ‡Ρ‚ΠΎ эта сила пСрпСндикулярна ΠΊΠ°ΠΊ скорости, Ρ‚Π°ΠΊ ΠΈ ΠΌΠ°Π³Π½ΠΈΡ‚Π½ΠΎΠΌΡƒ полю.

Π§Ρ‚ΠΎ эта магнитная сила Π΄Π΅Π»Π°Π΅Ρ‚ с двиТущимся элСктроном Π² Ρ‚ΠΎΠΊΠ΅? Ясно, Ρ‡Ρ‚ΠΎ ΠΎΠ½ Π½Π΅ Π±ΡƒΠ΄Π΅Ρ‚ Π΄Π²ΠΈΠ³Π°Ρ‚ΡŒΡΡ ΠΏΠΎ прямой Π² Π½Π°ΠΏΡ€Π°Π²Π»Π΅Π½ΠΈΠΈ тСчСния. ВмСсто этого элСктрон Π±ΡƒΠ΄Π΅Ρ‚ ΠΈΠ·Π³ΠΈΠ±Π°Ρ‚ΡŒΡΡ Π²Π»Π΅Π²ΠΎ. Если всС эти элСктроны Π² Ρ‚ΠΎΠΊΠ΅ ΠΈΠ·Π³ΠΈΠ±Π°ΡŽΡ‚ΡΡ Π²Π»Π΅Π²ΠΎ, Π² ΠΊΠΎΠ½Π΅Ρ‡Π½ΠΎΠΌ ΠΈΡ‚ΠΎΠ³Π΅ Π½Π° Π»Π΅Π²ΠΎΠΉ сторонС этого ΠΌΠ°Ρ‚Π΅Ρ€ΠΈΠ°Π»Π° Π±ΡƒΠ΄ΡƒΡ‚ ΠΈΠ·Π±Ρ‹Ρ‚ΠΎΡ‡Π½Ρ‹Π΅ ΠΎΡ‚Ρ€ΠΈΡ†Π°Ρ‚Π΅Π»ΡŒΠ½Ρ‹Π΅ заряды.
ΠŸΠΎΡΠΊΠΎΠ»ΡŒΠΊΡƒ ΠΌΠ°Ρ‚Π΅Ρ€ΠΈΠ°Π» ΠΈΠΌΠ΅Π΅Ρ‚ ΠΎΠ±Ρ‰ΠΈΠΉ Π½Π΅ΠΉΡ‚Ρ€Π°Π»ΡŒΠ½Ρ‹ΠΉ заряд, Π½Π° ΠΏΡ€Π°Π²ΠΎΠΉ повСрхности Ρ‚Π°ΠΊΠΆΠ΅ Π΄ΠΎΠ»ΠΆΠ½Ρ‹ Π±Ρ‹Ρ‚ΡŒ ΠΏΠΎΠ»ΠΎΠΆΠΈΡ‚Π΅Π»ΡŒΠ½Ρ‹Π΅ заряды.

Π’ ΠΊΠΎΠ½Π΅Ρ‡Π½ΠΎΠΌ ΠΈΡ‚ΠΎΠ³Π΅ ΠΌΠ°Ρ‚Π΅Ρ€ΠΈΠ°Π» Π±ΡƒΠ΄Π΅Ρ‚ Π²Ρ‹Π³Π»ΡΠ΄Π΅Ρ‚ΡŒ Ρ‚Π°ΠΊ (я ΡΠΎΠ±ΠΈΡ€Π°ΡŽΡΡŒ Π½Π°Ρ€ΠΈΡΠΎΠ²Π°Ρ‚ΡŒ Ρ‚ΠΎΠ»ΡŒΠΊΠΎ ΠΎΠ΄ΠΈΠ½ Π²Π΅ΠΊΡ‚ΠΎΡ€ ΠΌΠ°Π³Π½ΠΈΡ‚Π½ΠΎΠ³ΠΎ поля):

Π­Ρ‚ΠΎ ΠΈΠ·ΠΎΠ±Ρ€Π°ΠΆΠ΅Π½ΠΈΠ΅ Π½Π΅ΠΌΠ½ΠΎΠ³ΠΎ слоТнСС, Ρ‡Π΅ΠΌ я Ρ…ΠΎΡ‚Π΅Π», Π½ΠΎ Π²ΠΎΡ‚ ΠΊΠ»ΡŽΡ‡Π΅Π²Ρ‹Π΅ ΠΌΠΎΠΌΠ΅Π½Ρ‚Ρ‹:

  • ΠŸΠΎΠ²Π΅Ρ€Ρ…Π½ΠΎΡΡ‚ΡŒ заряд накапливаСтся сбоку ΠΈΠ·-Π·Π° ΠΌΠ°Π³Π½ΠΈΡ‚Π½ΠΎΠΉ силы, Π΄Π΅ΠΉΡΡ‚Π²ΡƒΡŽΡ‰Π΅ΠΉ Π½Π° двиТущиСся носитСли заряда.
  • Π­Ρ‚ΠΎΡ‚ повСрхностный элСктричСский заряд создаСт элСктричСскоС ΠΏΠΎΠ»Π΅.
  • ЭлСктричСскоС ΠΏΠΎΠ»Π΅ (ΠΈΠ·-Π·Π° Π±ΠΎΠΊΠΎΠ²Ρ‹Ρ… повСрхностных зарядов — Ρ‚Π°ΠΊΠΆΠ΅ сущСствуСт элСктричСскоС ΠΏΠΎΠ»Π΅, ΠΊΠΎΡ‚ΠΎΡ€ΠΎΠ΅ Π²Ρ‹Π·Ρ‹Π²Π°Π΅Ρ‚ Ρ‚ΠΎΠΊ) ΠΎΠΊΠ°Π·Ρ‹Π²Π°Π΅Ρ‚ силу Π½Π° двиТущиСся заряды.
  • Заряды Π½Π° Π±ΠΎΠΊΠΎΠ²Ρ‹Ρ… повСрхностях Π±ΡƒΠ΄ΡƒΡ‚ Π½Π°ΠΊΠ°ΠΏΠ»ΠΈΠ²Π°Ρ‚ΡŒΡΡ Π΄ΠΎ Ρ‚Π΅Ρ… ΠΏΠΎΡ€, ΠΏΠΎΠΊΠ° Π½Π΅ появится боковая элСктричСская сила, которая Π½Π΅ΠΉΡ‚Ρ€Π°Π»ΠΈΠ·ΡƒΠ΅Ρ‚ ΠΌΠ°Π³Π½ΠΈΡ‚Π½ΡƒΡŽ силу, ΠΈ элСктроны снова Π½Π΅ Π½Π°Ρ‡Π½ΡƒΡ‚ Π΄Π²ΠΈΠ³Π°Ρ‚ΡŒΡΡ Π² Π½Π°ΠΏΡ€Π°Π²Π»Π΅Π½ΠΈΠΈ ΠΏΡ€ΠΎΠ²ΠΎΠ΄Π°.
  • Π­Ρ‚ΠΎ элСктричСскоС ΠΏΠΎΠ»Π΅ Ρ‚Π°ΠΊΠΆΠ΅ ΠΎΠ·Π½Π°Ρ‡Π°Π΅Ρ‚ ΠΈΠ·ΠΌΠ΅Π½Π΅Π½ΠΈΠ΅ элСктричСского ΠΏΠΎΡ‚Π΅Π½Ρ†ΠΈΠ°Π»Π° Π² ΠΌΠ°Ρ‚Π΅Ρ€ΠΈΠ°Π»Π΅ (ΠΊΠΎΡ‚ΠΎΡ€ΠΎΠ΅ ΠΌΡ‹ ΠΌΠΎΠΆΠ΅ΠΌ ΠΈΠ·ΠΌΠ΅Ρ€ΠΈΡ‚ΡŒ).

Если Π²Ρ‹ Π·Π½Π°Π΅Ρ‚Π΅ Ρ€Π°Π·ΠΌΠ΅Ρ€ ΠΌΠ°Ρ‚Π΅Ρ€ΠΈΠ°Π»Π° ΠΈ ΡΠΊΠΎΡ€ΠΎΡΡ‚ΡŒ элСктронов (тСхничСски Π½Π°Π·Ρ‹Π²Π°Π΅ΠΌΡƒΡŽ ΡΠΊΠΎΡ€ΠΎΡΡ‚ΡŒΡŽ Π΄Ρ€Π΅ΠΉΡ„Π°), Ρ‚ΠΎ я ΠΌΠΎΠ³Ρƒ ΡƒΡΡ‚Π°Π½ΠΎΠ²ΠΈΡ‚ΡŒ ΠΌΠ°Π³Π½ΠΈΡ‚Π½ΡƒΡŽ силу, Ρ€Π°Π²Π½ΡƒΡŽ Π±ΠΎΠΊΠΎΠ²ΠΎΠΉ элСктричСской силС.

ИзмСнСниС элСктричСского ΠΏΠΎΡ‚Π΅Π½Ρ†ΠΈΠ°Π»Π° (ΠΏΠΎΠΏΠ΅Ρ€Π΅ΠΊ ΠΌΠ°Ρ‚Π΅Ρ€ΠΈΠ°Π»Π°) ΠΌΠΎΠΆΠ½ΠΎ ΠΈΠ·ΠΌΠ΅Ρ€ΠΈΡ‚ΡŒ с ΠΏΠΎΠΌΠΎΡ‰ΡŒΡŽ Π²ΠΎΠ»ΡŒΡ‚ΠΌΠ΅Ρ‚Ρ€Π°. Если ΠΏΠΎΠΏΠ΅Ρ€Π΅Ρ‡Π½ΠΎΠ΅ элСктричСскоС ΠΏΠΎΠ»Π΅ постоянно, Ρ‚ΠΎ:

И это Π΄Π°Π΅Ρ‚ Π²Π°ΠΌ ΠΌΠ°Π³Π½ΠΈΡ‚Π½ΠΎΠ΅ ΠΏΠΎΠ»Π΅. ΠšΠΎΠ½Π΅Ρ‡Π½ΠΎ, Π²Π°ΠΌ всС Π΅Ρ‰Π΅ Π½ΡƒΠΆΠ½Π° ΡΠΊΠΎΡ€ΠΎΡΡ‚ΡŒ Π΄Ρ€Π΅ΠΉΡ„Π° элСктронов, Π½ΠΎ Π²Ρ‹ ΠΌΠΎΠΆΠ΅Ρ‚Π΅ Π΅Π΅ ΠΏΠΎΠ»ΡƒΡ‡ΠΈΡ‚ΡŒ, Ссли Π·Π½Π°Π΅Ρ‚Π΅ Ρ‚ΠΈΠΏ ΠΌΠ°Ρ‚Π΅Ρ€ΠΈΠ°Π»Π° ΠΈ Π²Π΅Π»ΠΈΡ‡ΠΈΠ½Ρƒ элСктричСского Ρ‚ΠΎΠΊΠ°. Как насчСт ΠΎΠ±Π·ΠΎΡ€Π°?

  • ΠŸΠΎΠΌΠ΅ΡΡ‚ΠΈΡ‚Π΅ ΠΌΠ°Ρ‚Π΅Ρ€ΠΈΠ°Π» Π² ΠΌΠ°Π³Π½ΠΈΡ‚Π½ΠΎΠ΅ ΠΏΠΎΠ»Π΅.
  • ΠŸΡ€ΠΎΠΏΡƒΡΡ‚ΠΈΡ‚Π΅ Ρ‚ΠΎΠΊ Ρ‡Π΅Ρ€Π΅Π· этот ΠΌΠ°Ρ‚Π΅Ρ€ΠΈΠ°Π».
  • ΠœΠ°Π³Π½ΠΈΡ‚Π½ΠΎΠ΅ ΠΏΠΎΠ»Π΅ Π±ΡƒΠ΄Π΅Ρ‚ ΡΠΎΠ·Π΄Π°Π²Π°Ρ‚ΡŒ «Π±ΠΎΠΊΠΎΠ²ΠΎΠ΅» ΠΈΠ·ΠΌΠ΅Π½Π΅Π½ΠΈΠ΅ элСктричСского ΠΏΠΎΡ‚Π΅Π½Ρ†ΠΈΠ°Π»Π° Π² ΠΌΠ°Ρ‚Π΅Ρ€ΠΈΠ°Π»Π΅, ΠΊΠΎΡ‚ΠΎΡ€ΠΎΠ΅ Π²Ρ‹ ΠΌΠΎΠΆΠ΅Ρ‚Π΅ ΠΈΠ·ΠΌΠ΅Ρ€ΠΈΡ‚ΡŒ.
  • Π˜ΡΠΏΠΎΠ»ΡŒΠ·ΡƒΡ это ΠΈΠ·ΠΌΠ΅Π½Π΅Π½ΠΈΠ΅ ΠΏΠΎΡ‚Π΅Π½Ρ†ΠΈΠ°Π»Π° ΠΈ Ρ€Π°Π·ΠΌΠ΅Ρ€Π° ΠΌΠ°Ρ‚Π΅Ρ€ΠΈΠ°Π»Π°, Π²Ρ‹ ΠΏΠΎΠ»ΡƒΡ‡ΠΈΡ‚Π΅ Π²Π΅Π»ΠΈΡ‡ΠΈΠ½Ρƒ ΠΌΠ°Π³Π½ΠΈΡ‚Π½ΠΎΠ³ΠΎ поля.

Но ΠΏΠΎΠ΄ΠΎΠΆΠ΄ΠΈΡ‚Π΅! Π£ вас Π½Π΅Ρ‚ ΠΌΠ°Π³Π½ΠΈΡ‚Π½ΠΎΠ³ΠΎ поля. Π’Ρ‹ ΠΏΠΎΠ»ΡƒΡ‡Π°Π΅Ρ‚Π΅ ΡΠΎΡΡ‚Π°Π²Π»ΡΡŽΡ‰ΡƒΡŽ ΠΌΠ°Π³Π½ΠΈΡ‚Π½ΠΎΠ³ΠΎ поля, ΠΏΠ΅Ρ€ΠΏΠ΅Π½Π΄ΠΈΠΊΡƒΠ»ΡΡ€Π½ΡƒΡŽ Π΄Π°Ρ‚Ρ‡ΠΈΠΊΡƒ. Π’ iPhone (я ΠΏΠΎΡ‡Ρ‚ΠΈ ΡƒΠ²Π΅Ρ€Π΅Π½) Π΅ΡΡ‚ΡŒ Ρ‚Ρ€ΠΈ Π΄Π°Ρ‚Ρ‡ΠΈΠΊΠ°, Ρ‚Π°ΠΊ Ρ‡Ρ‚ΠΎ Π²Ρ‹ ΠΌΠΎΠΆΠ΅Ρ‚Π΅ ΠΏΠΎΠ»ΡƒΡ‡ΠΈΡ‚ΡŒ всС Ρ‚Ρ€ΠΈ ΠΊΠΎΠΌΠΏΠΎΠ½Π΅Π½Ρ‚Π° ΠΌΠ°Π³Π½ΠΈΡ‚Π½ΠΎΠ³ΠΎ поля Π—Π΅ΠΌΠ»ΠΈ ΠΈ, Ρ‚Π°ΠΊΠΈΠΌ ΠΎΠ±Ρ€Π°Π·ΠΎΠΌ, ΠΎΠΏΡ€Π΅Π΄Π΅Π»ΠΈΡ‚ΡŒ Π½Π°ΠΏΡ€Π°Π²Π»Π΅Π½ΠΈΠ΅ ΠΌΠ°Π³Π½ΠΈΡ‚Π½ΠΎΠ³ΠΎ поля.

ΠšΠΎΠ½Π΅Ρ‡Π½ΠΎ, ΡΡƒΡ‰Π΅ΡΡ‚Π²ΡƒΡŽΡ‚ ΠΈ Π΄Ρ€ΡƒΠ³ΠΈΠ΅ ΠΌΠ΅Ρ‚ΠΎΠ΄Ρ‹ измСрСния ΠΌΠ°Π³Π½ΠΈΡ‚Π½ΠΎΠ³ΠΎ поля, Π½ΠΎ это Π΄Π²Π° Π²Π°Ρ€ΠΈΠ°Π½Ρ‚Π°, ΠΊ ΠΊΠΎΡ‚ΠΎΡ€Ρ‹ΠΌ Ρƒ вас, вСроятно, Π΅ΡΡ‚ΡŒ Π»Π΅Π³ΠΊΠΈΠΉ доступ. Π― ΠΏΠΎΠΊΠ°ΠΆΡƒ, ΠΊΠ°ΠΊ Π²Ρ‹ ΠΌΠΎΠΆΠ΅Ρ‚Π΅ ΠΈΡΠΏΠΎΠ»ΡŒΠ·ΠΎΠ²Π°Ρ‚ΡŒ эти ΠΌΠ΅Ρ‚ΠΎΠ΄Ρ‹, Ρ‡Ρ‚ΠΎΠ±Ρ‹ ΠΏΠΎΡΠΌΠΎΡ‚Ρ€Π΅Ρ‚ΡŒ Π½Π° силу Ρ€Π°Π·Π»ΠΈΡ‡Π½Ρ‹Ρ… ΠΌΠ°Π³Π½ΠΈΡ‚ΠΎΠ², Π½ΠΎ Π² Π±ΠΎΠ»Π΅Π΅ ΠΏΠΎΠ·Π΄Π½Π΅ΠΌ постС.

ΠšΠ»ΡŽΡ‡ΠΈ ΠΊ испытаниям ΠΌΠ°Π³Π½ΠΈΡ‚Π½ΠΎΠ³ΠΎ поля ΠΈ проницаСмости

Π’ ΠΊΠ°ΠΆΠ΄ΠΎΠΉ отрасли Π΅ΡΡ‚ΡŒ свои стандарты ΠΈ трСбования, ΠΊΠΎΡ‚ΠΎΡ€Ρ‹ΠΌ Π΄ΠΎΠ»ΠΆΠ½Ρ‹ ΡΠΎΠΎΡ‚Π²Π΅Ρ‚ΡΡ‚Π²ΠΎΠ²Π°Ρ‚ΡŒ ΠΊΠΎΠΌΠΏΠΎΠ½Π΅Π½Ρ‚Ρ‹, Ρ‡Ρ‚ΠΎΠ±Ρ‹ ΠΈΡ… ΠΌΠΎΠΆΠ½ΠΎ Π±Ρ‹Π»ΠΎ ΠΈΡΠΏΠΎΠ»ΡŒΠ·ΠΎΠ²Π°Ρ‚ΡŒ Π² этой отрасли. Π•ΡΡ‚ΡŒ Π½Π΅ΠΊΠΎΡ‚ΠΎΡ€Ρ‹Π΅ тСсты, ΠΊΠΎΡ‚ΠΎΡ€Ρ‹Π΅ ΡΠ²Π»ΡΡŽΡ‚ΡΡ ΠΎΠ±Ρ‰ΠΈΠΌΠΈ для Ρ€Π°Π·Π½Ρ‹Ρ… отраслСй, Π½Π°ΠΏΡ€ΠΈΠΌΠ΅Ρ€, дСгазация, Π½ΠΎ Ρ‡Π°Ρ‰Π΅ всСго ΡΡƒΡ‰Π΅ΡΡ‚Π²ΡƒΡŽΡ‚ Ρ€Π°Π·Π½Ρ‹Π΅ трСбования, ΠΊΠΎΡ‚ΠΎΡ€Ρ‹Π΅ Π΄ΠΎΠ»ΠΆΠ½Ρ‹ Π±Ρ‹Ρ‚ΡŒ Π²Ρ‹ΠΏΠΎΠ»Π½Π΅Π½Ρ‹.

НапримСр, ΠΏΡ€ΠΎΠΈΠ·Π²ΠΎΠ΄ΠΈΡ‚Π΅Π»ΠΈ Π² области ΠΌΠ΅Π΄ΠΈΡ†ΠΈΠ½Ρ‹ ΠΌΠΎΠ³ΡƒΡ‚ Π±Ρ‹Ρ‚ΡŒ заинтСрСсованы Π² испытании ΠΌΠ°Π³Π½ΠΈΡ‚Π½ΠΎΠ³ΠΎ поля ΠΈ ΠΌΠ°Π³Π½ΠΈΡ‚Π½ΠΎΠΉ проницаСмости, Ссли ΠΈΡ… продукция ΠΈΡΠΏΠΎΠ»ΡŒΠ·ΡƒΠ΅Ρ‚ΡΡ Π² Π°ΠΏΠΏΠ°Ρ€Π°Ρ‚Π΅ ΠΌΠ°Π³Π½ΠΈΡ‚Π½ΠΎ-рСзонансной Ρ‚ΠΎΠΌΠΎΠ³Ρ€Π°Ρ„ΠΈΠΈ (МРВ) ΠΈΠ»ΠΈ рядом с Π½ΠΈΠΌ.

МРВ создаСт Ρ‡Ρ€Π΅Π·Π²Ρ‹Ρ‡Π°ΠΉΠ½ΠΎ сильноС ΠΌΠ°Π³Π½ΠΈΡ‚Π½ΠΎΠ΅ ΠΏΠΎΠ»Π΅, ΠΎΠ±Ρ‹Ρ‡Π½ΠΎ ΠΎΠΊΠΎΠ»ΠΎ 1,5 ВСсла, для наблюдСния Π·Π° нСбольшими особСнностями Π°Π½Π°Ρ‚ΠΎΠΌΠΈΠΈ Ρ‡Π΅Π»ΠΎΠ²Π΅ΠΊΠ°, Π° ΠΌΠ°Ρ‚Π΅Ρ€ΠΈΠ°Π»Ρ‹ с ΠΏΠΎΡ‚Π΅Π½Ρ†ΠΈΠ°Π»ΠΎΠΌ ΠΌΠ°Π³Π½Π΅Ρ‚ΠΈΠ·ΠΌΠ° Π±ΡƒΠ΄ΡƒΡ‚ ΠΈΡΠΊΠ°ΠΆΠ°Ρ‚ΡŒ измСрСния. К Π²Π°ΡˆΠ΅ΠΌΡƒ свСдСнию, Π½ΠΎΡ€ΠΌΠ°Π»ΡŒΠ½Ρ‹ΠΉ Π±Ρ‹Ρ‚ΠΎΠ²ΠΎΠΉ ΠΌΠ°Π³Π½ΠΈΡ‚ ΠΌΠΎΠΆΠ΅Ρ‚ Π±Ρ‹Ρ‚ΡŒ ΠΎΠΊΠΎΠ»ΠΎ 0,001 ВСсла.

Π§Ρ‚ΠΎ Ρ‚Π°ΠΊΠΎΠ΅ магнитная ΠΏΡ€ΠΎΠ½ΠΈΡ†Π°Π΅ΠΌΠΎΡΡ‚ΡŒ?

ВСстированиС ΠΌΠ°Π³Π½ΠΈΡ‚Π½ΠΎΠΉ проницаСмости измСряСт сопротивлСниС ΠΌΠ°Ρ‚Π΅Ρ€ΠΈΠ°Π»Π° Ρ„ΠΎΡ€ΠΌΠΈΡ€ΠΎΠ²Π°Π½ΠΈΡŽ ΠΌΠ°Π³Π½ΠΈΡ‚Π½ΠΎΠ³ΠΎ поля ΠΈΠ»ΠΈ ΡΡ‚Π΅ΠΏΠ΅Π½ΡŒ ΠΈΠ½Π΄ΡƒΡ†ΠΈΡ€ΠΎΠ²Π°Π½Π½ΠΎΠ³ΠΎ ΠΌΠ°Π³Π½Π΅Ρ‚ΠΈΠ·ΠΌΠ°, ΠΊΠΎΡ‚ΠΎΡ€Ρ‹ΠΉ ΠΌΠ°Ρ‚Π΅Ρ€ΠΈΠ°Π» испытываСт ΠΏΠΎΠ΄ дСйствиСм ΠΌΠ°Π³Π½ΠΈΡ‚Π½ΠΎΠ³ΠΎ поля.

ΠœΠΠ“ΠΠ˜Π’ΠžΠ‘ΠšΠžΠŸ

Π”ΡƒΠΌΠ°ΠΉΡ‚Π΅ ΠΎ ΠΌΠ°Π³Π½ΠΈΡ‚Π½ΠΎΠΉ проницаСмости ΠΊΠ°ΠΊ ΠΎ проводимости. НСкоторыС ΠΌΠ°Ρ‚Π΅Ρ€ΠΈΠ°Π»Ρ‹ ΠΌΠΎΠ³ΡƒΡ‚ ΠΏΡ€ΠΎΠ²ΠΎΠ΄ΠΈΡ‚ΡŒ элСктричСство Π»ΡƒΡ‡ΡˆΠ΅, Ρ‡Π΅ΠΌ Π΄Ρ€ΡƒΠ³ΠΈΠ΅, Π½Π°ΠΏΡ€ΠΈΠΌΠ΅Ρ€, мСдь, Π·ΠΎΠ»ΠΎΡ‚ΠΎ, сСрСбро ΠΈ Ρ‚. Π” .; Π³Π΄Π΅ Π΄Ρ€ΡƒΠ³ΠΈΠ΅ ΠΌΠ°Ρ‚Π΅Ρ€ΠΈΠ°Π»Ρ‹, Ρ‚Π°ΠΊΠΈΠ΅ ΠΊΠ°ΠΊ Π±Π΅Ρ‚ΠΎΠ½, Ρ€Π΅Π·ΠΈΠ½Π° ΠΈ пластик, ΡΠ²Π»ΡΡŽΡ‚ΡΡ ΠΏΠ»ΠΎΡ…ΠΈΠΌΠΈ ΠΏΡ€ΠΎΠ²ΠΎΠ΄Π½ΠΈΠΊΠ°ΠΌΠΈ.

Π§Π΅ΠΌ Π²Ρ‹ΡˆΠ΅ магнитная ΠΏΡ€ΠΎΠ½ΠΈΡ†Π°Π΅ΠΌΠΎΡΡ‚ΡŒ ΠΌΠ°Ρ‚Π΅Ρ€ΠΈΠ°Π»Π°, Ρ‚Π΅ΠΌ большС ΠΌΠ°Ρ‚Π΅Ρ€ΠΈΠ°Π» позволяСт ΠΌΠ°Π³Π½ΠΈΡ‚Π½ΠΎΠΌΡƒ ΠΏΠΎΡ‚ΠΎΠΊΡƒ ΠΏΡ€ΠΎΡ…ΠΎΠ΄ΠΈΡ‚ΡŒ Ρ‡Π΅Ρ€Π΅Π· Π½Π΅Π³ΠΎ. Π­Ρ‚ΠΎ прСдставлСно Π² Ρ„ΠΎΡ€ΠΌΡƒΠ»Π΅:

Π˜Π·ΠΎΠ±Ρ€Π°ΠΆΠ΅Π½ΠΈΠ΅ прСдоставлСно Π’ΠΈΠΊΠΈΠΏΠ΅Π΄ΠΈΠ΅ΠΉ

B = Β΅ H

Π“Π΄Π΅ B — ΠΏΠ»ΠΎΡ‚Π½ΠΎΡΡ‚ΡŒ ΠΏΠΎΡ‚ΠΎΠΊΠ°, Β΅ — ΠΏΡ€ΠΎΠ½ΠΈΡ†Π°Π΅ΠΌΠΎΡΡ‚ΡŒ, Π° H — прилоТСнная ΠΈΠ·Π²Π½Π΅ магнитная сила. Когда Β΅ = 1, считаСтся, Ρ‡Ρ‚ΠΎ ΠΌΠ°Ρ‚Π΅Ρ€ΠΈΠ°Π» Π½Π΅ Ρ€Π΅Π°Π³ΠΈΡ€ΡƒΠ΅Ρ‚ Π½Π° ΠΌΠ°Π³Π½ΠΈΡ‚Π½ΠΎΠ΅ ΠΏΠΎΠ»Π΅, Π° Ссли Β΅> 1, ΠΌΠ°Ρ‚Π΅Ρ€ΠΈΠ°Π» намагничиваСтся Π² ΠΎΡ‚Π²Π΅Ρ‚ Π½Π° ΠΏΡ€ΠΈΠ»ΠΎΠΆΠ΅Π½Π½ΠΎΠ΅ ΠΌΠ°Π³Π½ΠΈΡ‚Π½ΠΎΠ΅ ΠΏΠΎΠ»Π΅.

На ΠΏΡ€ΠΈΠ²Π΅Π΄Π΅Π½Π½ΠΎΠΌ Π²Ρ‹ΡˆΠ΅ Π³Ρ€Π°Ρ„ΠΈΠΊΠ΅ Β΅ 0 = 1,25663706 x 10 -6 Π“Π½ / ΠΌ ΠΏΡ€ΠΎΠ½ΠΈΡ†Π°Π΅ΠΌΠΎΡΡ‚ΡŒ свободного пространства, вакуумная срСда. Β΅ d — любой ΠΌΠ°Ρ‚Π΅Ρ€ΠΈΠ°Π», ΠΊΠΎΡ‚ΠΎΡ€Ρ‹ΠΉ считаСтся Π΄ΠΈΠ°ΠΌΠ°Π³Π½ΠΈΡ‚Π½Ρ‹ΠΌ, ΠΌΠ°Ρ‚Π΅Ρ€ΠΈΠ°Π» отталкиваСтся ΠΌΠ°Π³Π½ΠΈΡ‚Π½Ρ‹ΠΌ ΠΏΠΎΠ»Π΅ΠΌ. ΠœΠ°Ρ‚Π΅Ρ€ΠΈΠ°Π»ΡŒΠ½Ρ‹ΠΌΠΈ ΠΏΡ€ΠΈΠΌΠ΅Ρ€Π°ΠΌΠΈ этого ΠΌΠΎΠ³ΡƒΡ‚ Π±Ρ‹Ρ‚ΡŒ висмут ΠΈ ΡΡƒΡ€ΡŒΠΌΠ°. Β΅ p считаСтся ΠΏΠ°Ρ€Π°ΠΌΠ°Π³Π½ΠΈΡ‚Π½Ρ‹ΠΌ ΠΌΠ°Ρ‚Π΅Ρ€ΠΈΠ°Π»ΠΎΠΌ ΠΈ Π±ΡƒΠ΄Π΅Ρ‚ ΠΈΠΌΠ΅Ρ‚ΡŒ ΠΏΠΎΠΊΠ°Π·Π°Π½ΠΈΠ΅ Β΅ большС 1. Β΅ f — это ΠΌΠ°Ρ‚Π΅Ρ€ΠΈΠ°Π», ΠΊΠΎΡ‚ΠΎΡ€Ρ‹ΠΉ являСтся Ρ„Π΅Ρ€Ρ€ΠΎΠΌΠ°Π³Π½ΠΈΡ‚Π½Ρ‹ΠΌ ΠΈ притягиваСтся ΠΊ ΠΌΠ°Π³Π½ΠΈΡ‚Π°ΠΌ, Ρ‚Π°ΠΊΠΈΠΌ ΠΊΠ°ΠΊ никСль, ΠΆΠ΅Π»Π΅Π·ΠΎ ΠΈ ΠΊΠΎΠ±Π°Π»ΡŒΡ‚.

Π—Π°Ρ‡Π΅ΠΌ ΠΏΡ€ΠΎΠ²ΠΎΠ΄ΠΈΡ‚ΡŒ испытания ΠΌΠ°Π³Π½ΠΈΡ‚Π½Ρ‹ΠΌ ΠΏΠΎΠ»Π΅ΠΌ?

Π˜ΡΠΏΡ‹Ρ‚Π°Π½ΠΈΠ΅ ΠΌΠ°Π³Π½ΠΈΡ‚Π½Ρ‹ΠΌ ΠΏΠΎΠ»Π΅ΠΌ измСряСт ΡΡ‚Π΅ΠΏΠ΅Π½ΡŒ воздСйствия ΠΌΠ°Π³Π½ΠΈΡ‚Π½ΠΎΠ³ΠΎ поля Π½Π° ΠΌΠ°Ρ‚Π΅Ρ€ΠΈΠ°Π». Π­Ρ‚ΠΎ свойство притяТСния ΠΊ ΠΌΠ°Π³Π½ΠΈΡ‚Π½ΠΎΠΌΡƒ полю извСстно ΠΊΠ°ΠΊ ΠΏΠ°Ρ€Π°ΠΌΠ°Π³Π½Π΅Ρ‚ΠΈΠ·ΠΌ.

Если ваши ΠΏΡ€ΠΎΠ΄ΡƒΠΊΡ‚Ρ‹ ΠΈΡΠΏΠΎΠ»ΡŒΠ·ΡƒΡŽΡ‚ΡΡ Π² ΠΎΠΊΡ€ΡƒΠΆΠ°ΡŽΡ‰Π΅ΠΉ срСдС, Π½Π°ΠΏΡ€ΠΈΠΌΠ΅Ρ€, Π²ΠΎΠΊΡ€ΡƒΠ³ МРВ, Π²Π°ΡˆΠ΅ΠΌΡƒ ΠΊΠ»ΠΈΠ΅Π½Ρ‚Ρƒ Π±ΡƒΠ΄Π΅Ρ‚ Π²Π°ΠΆΠ½ΠΎ Π·Π½Π°Ρ‚ΡŒ, являСтся Π»ΠΈ ваш ΠΏΡ€ΠΎΠ΄ΡƒΠΊΡ‚ ΠΏΠ°Ρ€Π°ΠΌΠ°Π³Π½ΠΈΡ‚Π½Ρ‹ΠΌ.

ΠŸΡ€ΠΎΡΡ‚Π°Ρ установка, показанная Π²Ρ‹ΡˆΠ΅ с магнитоскопом, ΠΌΠΎΠΆΠ΅Ρ‚ ΠΎΠΏΡ€Π΅Π΄Π΅Π»ΠΈΡ‚ΡŒ, ΠΊΠ°ΠΊ ваши ΠΏΡ€ΠΎΠ΄ΡƒΠΊΡ‚Ρ‹ Π±ΡƒΠ΄ΡƒΡ‚ Ρ€Π΅Π°Π³ΠΈΡ€ΠΎΠ²Π°Ρ‚ΡŒ Π² этой срСдС.Samtec ΠΏΡ€ΠΎΠ²ΠΎΠ΄ΠΈΡ‚ испытания ΠΎΡ‚ΠΎΠ±Ρ€Π°Π½Π½Ρ‹Ρ… ΠΏΡ€ΠΎΠ΄ΡƒΠΊΡ‚ΠΎΠ², Ρ‡Ρ‚ΠΎΠ±Ρ‹ ΡƒΠ±Π΅Π΄ΠΈΡ‚ΡŒΡΡ, Ρ‡Ρ‚ΠΎ ΠΎΠ½ΠΈ ΠΌΠΎΠ³ΡƒΡ‚ Ρ€Π°Π±ΠΎΡ‚Π°Ρ‚ΡŒ Π² срСдС с ΡΠΊΡΡ‚Ρ€Π΅ΠΌΠ°Π»ΡŒΠ½ΠΎΠΉ Ρ‡ΡƒΠ²ΡΡ‚Π²ΠΈΡ‚Π΅Π»ΡŒΠ½ΠΎΡΡ‚ΡŒΡŽ ΠΊ ΠΌΠ°Π³Π½ΠΈΡ‚Π½Ρ‹ΠΌ полям.

НСкоторыС популярныС ΠΌΠ΅ΠΆΠΊΠΎΠΌΠΏΠΎΠ½Π΅Π½Ρ‚Π½Ρ‹Π΅ соСдинСния RF, ΠΊΠΎΡ‚ΠΎΡ€Ρ‹Π΅ ΠΌΠΎΠ³ΡƒΡ‚ Π±Ρ‹Ρ‚ΡŒ ΠΈΠ·Π³ΠΎΡ‚ΠΎΠ²Π»Π΅Π½Ρ‹ ΠΊΠ°ΠΊ Π½Π΅ΠΌΠ°Π³Π½ΠΈΡ‚Π½Ρ‹Π΅:

  • SMA (прямой, ΠΏΠΎΠ΄ прямым ΡƒΠ³Π»ΠΎΠΌ, ΠΌΠΎΠ½Ρ‚Π°ΠΆ Π½Π° ΠΊΡ€ΠΎΠΌΠΊΠ΅)
  • SMB (ΠΏΠΎΠ΄ прямым ΡƒΠ³Π»ΠΎΠΌ)
  • MMCX (прямой, ΠΏΠΎΠ΄ прямым ΡƒΠ³Π»ΠΎΠΌ, ΠΌΠΎΠ½Ρ‚Π°ΠΆ Π½Π° ΠΊΡ€ΠΎΠΌΠΊΠ΅, высокая вибрация)
  • MCX (прямой, ΠΏΠΎΠ΄ прямым ΡƒΠ³Π»ΠΎΠΌ, ΠΊΡ€Π΅ΠΏΠ»Π΅Π½ΠΈΠ΅ Π½Π° ΠΊΡ€ΠΎΠΌΠΊΡƒ)
  • SMP ( прямой, ΠΊΡ€Π°Π΅Π²ΠΎΠ΅ ΠΊΡ€Π΅ΠΏΠ»Π΅Π½ΠΈΠ΅, ΠΏΠ΅Ρ€Π΅Ρ…ΠΎΠ΄Π½ΠΈΠΊΠΈ для ΠΏΡƒΠ»ΠΈ)
  • RF 174 (SMA, SMB, MMCX, MCX)
  • RF 178 (SMA, SMB, MMCX, MCX)
  • RF 316 (SMA, SMB, MMCX, MCX)

Если Ρƒ вас Π΅ΡΡ‚ΡŒ вопросы ΠΎΡ‚Π½ΠΎΡΠΈΡ‚Π΅Π»ΡŒΠ½ΠΎ тСстирования ΠΌΠ°Π³Π½ΠΈΡ‚Π½ΠΎΠ³ΠΎ поля, ΡΠ²ΡΠΆΠΈΡ‚Π΅ΡΡŒ с asp @ samtec.com для получСния Π΄ΠΎΠΏΠΎΠ»Π½ΠΈΡ‚Π΅Π»ΡŒΠ½ΠΎΠΉ ΠΈΠ½Ρ„ΠΎΡ€ΠΌΠ°Ρ†ΠΈΠΈ.

Π Π°Π΄ΠΈ Π·Π°Π±Π°Π²Ρ‹

Если Π²Ρ‹ зашли Ρ‚Π°ΠΊ Π΄Π°Π»Π΅ΠΊΠΎ, Ρ‚ΠΎ Π²ΠΎΡ‚ Π·Π°Π±Π°Π²Π½ΠΎΠ΅ / интСрСсноС Π²ΠΈΠ΄Π΅ΠΎ Π½Π΅ΠΊΠΎΡ‚ΠΎΡ€Ρ‹Ρ… людСй, Β«ΡΠΊΡΠΏΠ΅Ρ€ΠΈΠΌΠ΅Π½Ρ‚ΠΈΡ€ΡƒΡŽΡ‰ΠΈΡ…Β» со старым Π°ΠΏΠΏΠ°Ρ€Π°Ρ‚ΠΎΠΌ МРВ. НСкоторыС ΠΈΠ· сил, ΠΏΠΎΠΊΠ°Π·Π°Π½Π½Ρ‹Ρ… Π² этом Π²ΠΈΠ΄Π΅ΠΎ, довольно нСвСроятны ΠΈ ΠΏΠΎΠ΄Ρ‡Π΅Ρ€ΠΊΠΈΠ²Π°ΡŽΡ‚ Π²Π°ΠΆΠ½ΠΎΡΡ‚ΡŒ Π·Π°Ρ‰ΠΈΡ‚Ρ‹ Ρ‡Π΅Ρ€Π½Ρ‹Ρ… ΠΌΠ΅Ρ‚Π°Π»Π»ΠΎΠ² ΠΎΡ‚ МРВ.

ΠœΠ°Π³Π½ΠΈΡ‚Π½Ρ‹Π΅ поля ΠΈ Π»ΠΈΠ½ΠΈΠΈ ΠΌΠ°Π³Π½ΠΈΡ‚Π½ΠΎΠ³ΠΎ поля

ЦСль обучСния

К ΠΊΠΎΠ½Ρ†Ρƒ этого Ρ€Π°Π·Π΄Π΅Π»Π° Π²Ρ‹ смоТСтС:

  • ΠžΠΏΡ€Π΅Π΄Π΅Π»ΠΈΡ‚Π΅ ΠΌΠ°Π³Π½ΠΈΡ‚Π½ΠΎΠ΅ ΠΏΠΎΠ»Π΅ ΠΈ ΠΎΠΏΠΈΡˆΠΈΡ‚Π΅ силовыС Π»ΠΈΠ½ΠΈΠΈ Ρ€Π°Π·Π»ΠΈΡ‡Π½Ρ‹Ρ… ΠΌΠ°Π³Π½ΠΈΡ‚Π½Ρ‹Ρ… ΠΏΠΎΠ»Π΅ΠΉ.

Говорят, Ρ‡Ρ‚ΠΎ Π² дСтствС Π­ΠΉΠ½ΡˆΡ‚Π΅ΠΉΠ½ Π±Ρ‹Π» ΠΎΡ‡Π°Ρ€ΠΎΠ²Π°Π½ компасом, Π²ΠΎΠ·ΠΌΠΎΠΆΠ½ΠΎ, Ρ€Π°Π·ΠΌΡ‹ΡˆΠ»ΡΡ ΠΎ Ρ‚ΠΎΠΌ, ΠΊΠ°ΠΊ стрСлка ΠΎΡ‰ΡƒΡ‰Π°Π»Π° силу Π±Π΅Π· прямого физичСского ΠΊΠΎΠ½Ρ‚Π°ΠΊΡ‚Π°. Π•Π³ΠΎ ΡΠΏΠΎΡΠΎΠ±Π½ΠΎΡΡ‚ΡŒ Π³Π»ΡƒΠ±ΠΎΠΊΠΎ ΠΈ ясно ΠΌΡ‹ΡΠ»ΠΈΡ‚ΡŒ ΠΎ дСйствиях Π½Π° расстоянии, особСнно ΠΎ Π³Ρ€Π°Π²ΠΈΡ‚Π°Ρ†ΠΈΠΎΠ½Π½Ρ‹Ρ…, элСктричСских ΠΈ ΠΌΠ°Π³Π½ΠΈΡ‚Π½Ρ‹Ρ… силах, ΠΏΠΎΠ·ΠΆΠ΅ ΠΏΠΎΠ·Π²ΠΎΠ»ΠΈΠ»Π° Π΅ΠΌΡƒ ΡΠΎΠ·Π΄Π°Ρ‚ΡŒ свою Ρ€Π΅Π²ΠΎΠ»ΡŽΡ†ΠΈΠΎΠ½Π½ΡƒΡŽ Ρ‚Π΅ΠΎΡ€ΠΈΡŽ ΠΎΡ‚Π½ΠΎΡΠΈΡ‚Π΅Π»ΡŒΠ½ΠΎΡΡ‚ΠΈ. ΠŸΠΎΡΠΊΠΎΠ»ΡŒΠΊΡƒ ΠΌΠ°Π³Π½ΠΈΡ‚Π½Ρ‹Π΅ силы Π΄Π΅ΠΉΡΡ‚Π²ΡƒΡŽΡ‚ Π½Π° расстоянии, ΠΌΡ‹ опрСдСляСм ΠΌΠ°Π³Π½ΠΈΡ‚Π½ΠΎΠ΅ ΠΏΠΎΠ»Π΅ для прСдставлСния ΠΌΠ°Π³Π½ΠΈΡ‚Π½Ρ‹Ρ… сил. ГрафичСскоС прСдставлСниС Π»ΠΈΠ½ΠΈΠΉ ΠΌΠ°Π³Π½ΠΈΡ‚Π½ΠΎΠ³ΠΎ поля ΠΎΡ‡Π΅Π½ΡŒ ΠΏΠΎΠ»Π΅Π·Π½ΠΎ для Π²ΠΈΠ·ΡƒΠ°Π»ΠΈΠ·Π°Ρ†ΠΈΠΈ силы ΠΈ направлСния ΠΌΠ°Π³Π½ΠΈΡ‚Π½ΠΎΠ³ΠΎ поля.Как ΠΏΠΎΠΊΠ°Π·Π°Π½ΠΎ Π½Π° Π€ΠΈΠ³.1, Π½Π°ΠΏΡ€Π°Π²Π»Π΅Π½ΠΈΠ΅ Π»ΠΈΠ½ΠΈΠΉ ΠΌΠ°Π³Π½ΠΈΡ‚Π½ΠΎΠ³ΠΎ поля опрСдСляСтся ΠΊΠ°ΠΊ Π½Π°ΠΏΡ€Π°Π²Π»Π΅Π½ΠΈΠ΅, Π² ΠΊΠΎΡ‚ΠΎΡ€ΠΎΠΌ ΡƒΠΊΠ°Π·Ρ‹Π²Π°Π΅Ρ‚ сСвСрный ΠΊΠΎΠ½Π΅Ρ† стрСлки компаса. ΠœΠ°Π³Π½ΠΈΡ‚Π½ΠΎΠ΅ ΠΏΠΎΠ»Π΅ Ρ‚Ρ€Π°Π΄ΠΈΡ†ΠΈΠΎΠ½Π½ΠΎ Π½Π°Π·Ρ‹Π²Π°ΡŽΡ‚ ΠΏΠΎΠ»Π΅ΠΌ , , B, , .

Рис. 1. Π›ΠΈΠ½ΠΈΠΈ ΠΌΠ°Π³Π½ΠΈΡ‚Π½ΠΎΠ³ΠΎ поля ΠΎΠΏΡ€Π΅Π΄Π΅Π»ΡΡŽΡ‚ΡΡ Ρ‚Π°ΠΊ, Ρ‡Ρ‚ΠΎΠ±Ρ‹ ΠΎΠ½ΠΈ ΠΈΠΌΠ΅Π»ΠΈ Π½Π°ΠΏΡ€Π°Π²Π»Π΅Π½ΠΈΠ΅, ΠΊΠΎΡ‚ΠΎΡ€ΠΎΠ΅ ΡƒΠΊΠ°Π·Ρ‹Π²Π°Π΅Ρ‚ малСнький компас ΠΏΡ€ΠΈ Ρ€Π°Π·ΠΌΠ΅Ρ‰Π΅Π½ΠΈΠΈ Π² ΠΎΠΏΡ€Π΅Π΄Π΅Π»Π΅Π½Π½ΠΎΠΌ мСстС. (a) Если для отобраТСния ΠΌΠ°Π³Π½ΠΈΡ‚Π½ΠΎΠ³ΠΎ поля Π²ΠΎΠΊΡ€ΡƒΠ³ стСрТнСвого ΠΌΠ°Π³Π½ΠΈΡ‚Π° ΠΈΡΠΏΠΎΠ»ΡŒΠ·ΡƒΡŽΡ‚ΡΡ нСбольшиС компасы, ΠΎΠ½ΠΈ Π±ΡƒΠ΄ΡƒΡ‚ ΡƒΠΊΠ°Π·Ρ‹Π²Π°Ρ‚ΡŒ Π² ΠΏΠΎΠΊΠ°Π·Π°Π½Π½Ρ‹Ρ… направлСниях: ΠΎΡ‚ сСвСрного полюса ΠΌΠ°Π³Π½ΠΈΡ‚Π° ΠΊ ΡŽΠΆΠ½ΠΎΠΌΡƒ ΠΏΠΎΠ»ΡŽΡΡƒ ΠΌΠ°Π³Π½ΠΈΡ‚Π°.(Напомним, Ρ‡Ρ‚ΠΎ сСвСрный ΠΌΠ°Π³Π½ΠΈΡ‚Π½Ρ‹ΠΉ полюс Π—Π΅ΠΌΠ»ΠΈ Π½Π° самом Π΄Π΅Π»Π΅ являСтся ΡŽΠΆΠ½Ρ‹ΠΌ полюсом с Ρ‚ΠΎΡ‡ΠΊΠΈ зрСния опрСдСлСния полюсов стСрТнСвого ΠΌΠ°Π³Π½ΠΈΡ‚Π°.) (B) Π‘ΠΎΠ΅Π΄ΠΈΠ½Π΅Π½ΠΈΠ΅ стрСлок Π΄Π°Π΅Ρ‚ Π½Π΅ΠΏΡ€Π΅Ρ€Ρ‹Π²Π½Ρ‹Π΅ Π»ΠΈΠ½ΠΈΠΈ ΠΌΠ°Π³Π½ΠΈΡ‚Π½ΠΎΠ³ΠΎ поля. Π‘ΠΈΠ»Π° поля ΠΏΡ€ΠΎΠΏΠΎΡ€Ρ†ΠΈΠΎΠ½Π°Π»ΡŒΠ½Π° близости (ΠΈΠ»ΠΈ плотности) Π»ΠΈΠ½ΠΈΠΉ. (c) Если Π±Ρ‹ ΠΌΠΎΠΆΠ½ΠΎ Π±Ρ‹Π»ΠΎ ΠΈΡΡΠ»Π΅Π΄ΠΎΠ²Π°Ρ‚ΡŒ Π²Π½ΡƒΡ‚Ρ€Π΅Π½Π½ΡŽΡŽ Ρ‡Π°ΡΡ‚ΡŒ ΠΌΠ°Π³Π½ΠΈΡ‚Π°, Π±Ρ‹Π»ΠΎ Π±Ρ‹ ΠΎΠ±Π½Π°Ρ€ΡƒΠΆΠ΅Π½ΠΎ, Ρ‡Ρ‚ΠΎ силовыС Π»ΠΈΠ½ΠΈΠΈ ΠΎΠ±Ρ€Π°Π·ΡƒΡŽΡ‚ Π½Π΅ΠΏΡ€Π΅Ρ€Ρ‹Π²Π½Ρ‹Π΅ Π·Π°ΠΌΠΊΠ½ΡƒΡ‚Ρ‹Π΅ ΠΊΠΎΠ½Ρ‚ΡƒΡ€Ρ‹.

МалСнькиС компасы, ΠΈΡΠΏΠΎΠ»ΡŒΠ·ΡƒΠ΅ΠΌΡ‹Π΅ для ΠΏΡ€ΠΎΠ²Π΅Ρ€ΠΊΠΈ ΠΌΠ°Π³Π½ΠΈΡ‚Π½ΠΎΠ³ΠΎ поля, Π½Π΅ поврСдят Π΅Π³ΠΎ. (Π­Ρ‚ΠΎ Π°Π½Π°Π»ΠΎΠ³ΠΈΡ‡Π½ΠΎ Ρ‚ΠΎΠΌΡƒ, ΠΊΠ°ΠΊ ΠΌΡ‹ провСряли элСктричСскиС поля с нСбольшим ΠΏΡ€ΠΎΠ±Π½Ρ‹ΠΌ зарядом.Π’ ΠΎΠ±ΠΎΠΈΡ… случаях поля ΠΏΡ€Π΅Π΄ΡΡ‚Π°Π²Π»ΡΡŽΡ‚ Ρ‚ΠΎΠ»ΡŒΠΊΠΎ ΠΎΠ±ΡŠΠ΅ΠΊΡ‚, ΡΠΎΠ·Π΄Π°ΡŽΡ‰ΠΈΠΉ ΠΈΡ…, Π° Π½Π΅ Π·ΠΎΠ½Π΄, ΠΏΡ€ΠΎΠ²Π΅Ρ€ΡΡŽΡ‰ΠΈΠΉ ΠΈΡ….) На рисункС 2 ΠΏΠΎΠΊΠ°Π·Π°Π½ΠΎ, ΠΊΠ°ΠΊ ΠΌΠ°Π³Π½ΠΈΡ‚Π½ΠΎΠ΅ ΠΏΠΎΠ»Π΅ появляСтся для Ρ‚ΠΎΠΊΠΎΠ²ΠΎΠΉ ΠΏΠ΅Ρ‚Π»ΠΈ ΠΈ Π΄Π»ΠΈΠ½Π½ΠΎΠ³ΠΎ прямого ΠΏΡ€ΠΎΠ²ΠΎΠ΄Π°, Ρ‡Ρ‚ΠΎ ΠΌΠΎΠΆΠ½ΠΎ Π±Ρ‹Π»ΠΎ Π±Ρ‹ ΠΈΡΡΠ»Π΅Π΄ΠΎΠ²Π°Ρ‚ΡŒ с ΠΏΠΎΠΌΠΎΡ‰ΡŒΡŽ Π½Π΅Π±ΠΎΠ»ΡŒΡˆΠΈΡ… компасов. НСбольшой компас, ΠΏΠΎΠΌΠ΅Ρ‰Π΅Π½Π½Ρ‹ΠΉ Π² эти поля, выровняСтся ΠΏΠ°Ρ€Π°Π»Π»Π΅Π»ΡŒΠ½ΠΎ Π»ΠΈΠ½ΠΈΠΈ поля Π² этом мСстС, Π° Π΅Π³ΠΎ сСвСрный полюс Π±ΡƒΠ΄Π΅Ρ‚ ΡƒΠΊΠ°Π·Ρ‹Π²Π°Ρ‚ΡŒ Π² Π½Π°ΠΏΡ€Π°Π²Π»Π΅Π½ΠΈΠΈ B . ΠžΠ±Ρ€Π°Ρ‚ΠΈΡ‚Π΅ Π²Π½ΠΈΠΌΠ°Π½ΠΈΠ΅ Π½Π° символы, ΠΈΡΠΏΠΎΠ»ΡŒΠ·ΡƒΠ΅ΠΌΡ‹Π΅ для Π²Π²ΠΎΠ΄Π° ΠΈ Π²Ρ‹Π²ΠΎΠ΄Π° ΠΈΠ· Π±ΡƒΠΌΠ°Π³ΠΈ.

Рис. 2. МалСнькиС компасы ΠΌΠΎΠΆΠ½ΠΎ ΠΈΡΠΏΠΎΠ»ΡŒΠ·ΠΎΠ²Π°Ρ‚ΡŒ для картирования ΠΏΠΎΠ»Π΅ΠΉ, ΠΏΠΎΠΊΠ°Π·Π°Π½Π½Ρ‹Ρ… здСсь.(Π°) ΠœΠ°Π³Π½ΠΈΡ‚Π½ΠΎΠ΅ ΠΏΠΎΠ»Π΅ ΠΊΡ€ΡƒΠ³ΠΎΠ²ΠΎΠΉ Ρ‚ΠΎΠΊΠΎΠ²ΠΎΠΉ ΠΏΠ΅Ρ‚Π»ΠΈ ΠΏΠΎΡ…ΠΎΠΆΠ΅ Π½Π° ΠΌΠ°Π³Π½ΠΈΡ‚Π½ΠΎΠ΅ ΠΏΠΎΠ»Π΅ стСрТнСвого ΠΌΠ°Π³Π½ΠΈΡ‚Π°. (Π±) Π”Π»ΠΈΠ½Π½Ρ‹ΠΉ ΠΈ прямой ΠΏΡ€ΠΎΠ²ΠΎΠ΄ создаСт ΠΏΠΎΠ»Π΅ с силовыми линиями ΠΌΠ°Π³Π½ΠΈΡ‚Π½ΠΎΠ³ΠΎ поля, ΠΎΠ±Ρ€Π°Π·ΡƒΡŽΡ‰ΠΈΠΌΠΈ ΠΊΠΎΠ»ΡŒΡ†Π΅Π²Ρ‹Π΅ ΠΏΠ΅Ρ‚Π»ΠΈ. (c) Когда ΠΏΡ€ΠΎΠ²ΠΎΠ»ΠΎΠΊΠ° находится Π² плоскости Π±ΡƒΠΌΠ°Π³ΠΈ, ΠΏΠΎΠ»Π΅ пСрпСндикулярно Π±ΡƒΠΌΠ°Π³Π΅. ΠžΠ±Ρ€Π°Ρ‚ΠΈΡ‚Π΅ Π²Π½ΠΈΠΌΠ°Π½ΠΈΠ΅, Ρ‡Ρ‚ΠΎ символы, ΠΈΡΠΏΠΎΠ»ΡŒΠ·ΡƒΠ΅ΠΌΡ‹Π΅ для поля, ΡƒΠΊΠ°Π·Ρ‹Π²Π°ΡŽΡ‰Π΅Π³ΠΎ Π²Π½ΡƒΡ‚Ρ€ΡŒ (Π½Π°ΠΏΡ€ΠΈΠΌΠ΅Ρ€, хвоста стрСлки), ΠΈ поля, ΡƒΠΊΠ°Π·Ρ‹Π²Π°ΡŽΡ‰Π΅Π³ΠΎ Π½Π°Ρ€ΡƒΠΆΡƒ (Π½Π°ΠΏΡ€ΠΈΠΌΠ΅Ρ€, Π½Π°ΠΊΠΎΠ½Π΅Ρ‡Π½ΠΈΠΊΠ° стрСлки).

УстановлСниС соСдинСний: концСпция поля

ПолС — это способ отобраТСния сил, ΠΎΠΊΡ€ΡƒΠΆΠ°ΡŽΡ‰ΠΈΡ… любой ΠΎΠ±ΡŠΠ΅ΠΊΡ‚, ΠΊΠΎΡ‚ΠΎΡ€Ρ‹Π΅ ΠΌΠΎΠ³ΡƒΡ‚ Π²ΠΎΠ·Π΄Π΅ΠΉΡΡ‚Π²ΠΎΠ²Π°Ρ‚ΡŒ Π½Π° Π΄Ρ€ΡƒΠ³ΠΎΠΉ ΠΎΠ±ΡŠΠ΅ΠΊΡ‚ Π½Π° расстоянии Π±Π΅Π· Π²ΠΈΠ΄ΠΈΠΌΠΎΠΉ физичСской связи.ПолС прСдставляСт ΠΎΠ±ΡŠΠ΅ΠΊΡ‚, Π΅Π³ΠΎ Π³Π΅Π½Π΅Ρ€ΠΈΡ€ΡƒΡŽΡ‰ΠΈΠΉ. Π“Ρ€Π°Π²ΠΈΡ‚Π°Ρ†ΠΈΠΎΠ½Π½Ρ‹Π΅ поля ΠΎΡ‚ΠΎΠ±Ρ€Π°ΠΆΠ°ΡŽΡ‚ Π³Ρ€Π°Π²ΠΈΡ‚Π°Ρ†ΠΈΠΎΠ½Π½Ρ‹Π΅ силы, элСктричСскиС поля ΠΎΡ‚ΠΎΠ±Ρ€Π°ΠΆΠ°ΡŽΡ‚ элСктричСскиС силы, Π° ΠΌΠ°Π³Π½ΠΈΡ‚Π½Ρ‹Π΅ поля ΠΎΡ‚ΠΎΠ±Ρ€Π°ΠΆΠ°ΡŽΡ‚ ΠΌΠ°Π³Π½ΠΈΡ‚Π½Ρ‹Π΅ силы.

ΠžΠ±ΡˆΠΈΡ€Π½Ρ‹Π΅ исслСдования ΠΌΠ°Π³Π½ΠΈΡ‚Π½Ρ‹Ρ… ΠΏΠΎΠ»Π΅ΠΉ выявили ряд ТСстких ΠΏΡ€Π°Π²ΠΈΠ». ΠœΡ‹ ΠΈΡΠΏΠΎΠ»ΡŒΠ·ΡƒΠ΅ΠΌ Π»ΠΈΠ½ΠΈΠΈ ΠΌΠ°Π³Π½ΠΈΡ‚Π½ΠΎΠ³ΠΎ поля для прСдставлСния поля (Π»ΠΈΠ½ΠΈΠΈ — это графичСский инструмСнт, Π° Π½Π΅ физичСская ΡΡƒΡ‰Π½ΠΎΡΡ‚ΡŒ сами ΠΏΠΎ сСбС). Бвойства силовых Π»ΠΈΠ½ΠΈΠΉ ΠΌΠ°Π³Π½ΠΈΡ‚Π½ΠΎΠ³ΠΎ поля ΠΌΠΎΠΆΠ½ΠΎ ΠΎΠΏΠΈΡΠ°Ρ‚ΡŒ ΡΠ»Π΅Π΄ΡƒΡŽΡ‰ΠΈΠΌΠΈ ΠΏΡ€Π°Π²ΠΈΠ»Π°ΠΌΠΈ:

  1. НаправлСниС ΠΌΠ°Π³Π½ΠΈΡ‚Π½ΠΎΠ³ΠΎ поля касаСтся силовой Π»ΠΈΠ½ΠΈΠΈ Π² любой Ρ‚ΠΎΡ‡ΠΊΠ΅ пространства.МалСнький компас ΡƒΠΊΠ°ΠΆΠ΅Ρ‚ Π½Π°ΠΏΡ€Π°Π²Π»Π΅Π½ΠΈΠ΅ Π»ΠΈΠ½ΠΈΠΈ поля.
  2. Π‘ΠΈΠ»Π° поля ΠΏΡ€ΠΎΠΏΠΎΡ€Ρ†ΠΈΠΎΠ½Π°Π»ΡŒΠ½Π° близости Π»ΠΈΠ½ΠΈΠΉ. Он Ρ‚ΠΎΡ‡Π½ΠΎ ΠΏΡ€ΠΎΠΏΠΎΡ€Ρ†ΠΈΠΎΠ½Π°Π»Π΅Π½ количСству Π»ΠΈΠ½ΠΈΠΉ Π½Π° Π΅Π΄ΠΈΠ½ΠΈΡ†Ρƒ ΠΏΠ»ΠΎΡ‰Π°Π΄ΠΈ, пСрпСндикулярной линиям (Π½Π°Π·Ρ‹Π²Π°Π΅ΠΌΠΎΠΉ ΠΏΠ»ΠΎΡ‚Π½ΠΎΡΡ‚ΡŒΡŽ ΠΏΠ»ΠΎΡ‰Π°Π΄ΠΈ).
  3. Π‘ΠΈΠ»ΠΎΠ²Ρ‹Π΅ Π»ΠΈΠ½ΠΈΠΈ ΠΌΠ°Π³Π½ΠΈΡ‚Π½ΠΎΠ³ΠΎ поля Π½ΠΈΠΊΠΎΠ³Π΄Π° Π½Π΅ ΠΌΠΎΠ³ΡƒΡ‚ ΠΏΠ΅Ρ€Π΅ΡΠ΅ΠΊΠ°Ρ‚ΡŒΡΡ, Π° это ΠΎΠ·Π½Π°Ρ‡Π°Π΅Ρ‚, Ρ‡Ρ‚ΠΎ ΠΏΠΎΠ»Π΅ ΡƒΠ½ΠΈΠΊΠ°Π»ΡŒΠ½ΠΎ Π² любой Ρ‚ΠΎΡ‡ΠΊΠ΅ пространства.
  4. Π›ΠΈΠ½ΠΈΠΈ ΠΌΠ°Π³Π½ΠΈΡ‚Π½ΠΎΠ³ΠΎ поля Π½Π΅ΠΏΡ€Π΅Ρ€Ρ‹Π²Π½Ρ‹, ΠΎΠ±Ρ€Π°Π·ΡƒΡŽΡ‚ Π·Π°ΠΌΠΊΠ½ΡƒΡ‚Ρ‹Π΅ ΠΊΠΎΠ½Ρ‚ΡƒΡ€Ρ‹ Π±Π΅Π· Π½Π°Ρ‡Π°Π»Π° ΠΈ ΠΊΠΎΠ½Ρ†Π°. Они ΠΈΠ΄ΡƒΡ‚ ΠΎΡ‚ сСвСрного полюса ΠΊ ΡŽΠΆΠ½ΠΎΠΌΡƒ.

ПослСднСС свойство связано с Ρ‚Π΅ΠΌ, Ρ‡Ρ‚ΠΎ сСвСрный ΠΈ ΡŽΠΆΠ½Ρ‹ΠΉ полюса нСльзя Ρ€Π°Π·Π΄Π΅Π»ΠΈΡ‚ΡŒ. Π­Ρ‚ΠΎ явноС ΠΎΡ‚Π»ΠΈΡ‡ΠΈΠ΅ ΠΎΡ‚ силовых Π»ΠΈΠ½ΠΈΠΉ элСктричСского поля, ΠΊΠΎΡ‚ΠΎΡ€Ρ‹Π΅ Π½Π°Ρ‡ΠΈΠ½Π°ΡŽΡ‚ΡΡ ΠΈ Π·Π°ΠΊΠ°Π½Ρ‡ΠΈΠ²Π°ΡŽΡ‚ΡΡ ΠΏΠΎΠ»ΠΎΠΆΠΈΡ‚Π΅Π»ΡŒΠ½Ρ‹ΠΌΠΈ ΠΈ ΠΎΡ‚Ρ€ΠΈΡ†Π°Ρ‚Π΅Π»ΡŒΠ½Ρ‹ΠΌΠΈ зарядами. Если Π±Ρ‹ ΠΌΠ°Π³Π½ΠΈΡ‚Π½Ρ‹Π΅ ΠΌΠΎΠ½ΠΎΠΏΠΎΠ»ΠΈ сущСствовали, Ρ‚ΠΎ силовыС Π»ΠΈΠ½ΠΈΠΈ ΠΌΠ°Π³Π½ΠΈΡ‚Π½ΠΎΠ³ΠΎ поля Π½Π°Ρ‡ΠΈΠ½Π°Π»ΠΈΡΡŒ Π±Ρ‹ ΠΈ Π·Π°ΠΊΠ°Π½Ρ‡ΠΈΠ²Π°Π»ΠΈΡΡŒ Π½Π° Π½ΠΈΡ….

Π‘Π²ΠΎΠ΄ΠΊΠ° Ρ€Π°Π·Π΄Π΅Π»Π°

  • ΠœΠ°Π³Π½ΠΈΡ‚Π½Ρ‹Π΅ поля ΠΌΠΎΠ³ΡƒΡ‚ Π±Ρ‹Ρ‚ΡŒ графичСски прСдставлСны силовыми линиями ΠΌΠ°Π³Π½ΠΈΡ‚Π½ΠΎΠ³ΠΎ поля, свойства ΠΊΠΎΡ‚ΠΎΡ€Ρ‹Ρ… ΡΠ»Π΅Π΄ΡƒΡŽΡ‰ΠΈΠ΅:
    • ПолС касаСтся Π»ΠΈΠ½ΠΈΠΈ ΠΌΠ°Π³Π½ΠΈΡ‚Π½ΠΎΠ³ΠΎ поля.
    • ΠΠ°ΠΏΡ€ΡΠΆΠ΅Π½Π½ΠΎΡΡ‚ΡŒ поля ΠΏΡ€ΠΎΠΏΠΎΡ€Ρ†ΠΈΠΎΠ½Π°Π»ΡŒΠ½Π° Π»ΠΈΠ½Π΅ΠΉΠ½ΠΎΠΉ плотности.
    • Π›ΠΈΠ½ΠΈΠΈ поля Π½Π΅ ΠΌΠΎΠ³ΡƒΡ‚ ΠΏΠ΅Ρ€Π΅ΡΠ΅ΠΊΠ°Ρ‚ΡŒΡΡ.
    • Π›ΠΈΠ½ΠΈΠΈ поля ΠΏΡ€Π΅Π΄ΡΡ‚Π°Π²Π»ΡΡŽΡ‚ собой Π½Π΅ΠΏΡ€Π΅Ρ€Ρ‹Π²Π½Ρ‹Π΅ ΠΏΠ΅Ρ‚Π»ΠΈ.

ΠšΠΎΠ½Ρ†Π΅ΠΏΡ‚ΡƒΠ°Π»ΡŒΠ½Ρ‹Π΅ вопросы

  1. ΠžΠ±ΡŠΡΡΠ½ΠΈΡ‚Π΅, ΠΏΠΎΡ‡Π΅ΠΌΡƒ ΠΌΠ°Π³Π½ΠΈΡ‚Π½ΠΎΠ΅ ΠΏΠΎΠ»Π΅ Π½Π΅ ΠΌΠΎΠΆΠ΅Ρ‚ Π±Ρ‹Ρ‚ΡŒ ΡƒΠ½ΠΈΠΊΠ°Π»ΡŒΠ½Ρ‹ΠΌ (Ρ‚ΠΎ Π΅ΡΡ‚ΡŒ Π½Π΅ ΠΈΠΌΠ΅Ρ‚ΡŒ СдинствСнного значСния) Π² Ρ‚ΠΎΡ‡ΠΊΠ΅ пространства, Π³Π΄Π΅ силовыС Π»ΠΈΠ½ΠΈΠΈ ΠΌΠ°Π³Π½ΠΈΡ‚Π½ΠΎΠ³ΠΎ поля ΠΌΠΎΠ³ΡƒΡ‚ ΠΏΠ΅Ρ€Π΅ΡΠ΅ΠΊΠ°Ρ‚ΡŒΡΡ. (Π£Ρ‡ΠΈΡ‚Ρ‹Π²Π°ΠΉΡ‚Π΅ Π½Π°ΠΏΡ€Π°Π²Π»Π΅Π½ΠΈΠ΅ поля Π² Ρ‚Π°ΠΊΠΎΠΉ Ρ‚ΠΎΡ‡ΠΊΠ΅.)
  2. ΠŸΠ΅Ρ€Π΅Ρ‡ΠΈΡΠ»ΠΈΡ‚Π΅ сходства силовых Π»ΠΈΠ½ΠΈΠΉ ΠΌΠ°Π³Π½ΠΈΡ‚Π½ΠΎΠ³ΠΎ ΠΈ элСктричСского поля.НапримСр, Π½Π°ΠΏΡ€Π°Π²Π»Π΅Π½ΠΈΠ΅ поля касаСтся Π»ΠΈΠ½ΠΈΠΈ Π² любой Ρ‚ΠΎΡ‡ΠΊΠ΅ пространства. Π’Π°ΠΊΠΆΠ΅ ΡƒΠΊΠ°ΠΆΠΈΡ‚Π΅, Ρ‡Π΅ΠΌ ΠΎΠ½ΠΈ ΠΎΡ‚Π»ΠΈΡ‡Π°ΡŽΡ‚ΡΡ. НапримСр, элСктричСская сила ΠΏΠ°Ρ€Π°Π»Π»Π΅Π»ΡŒΠ½Π° силовым линиям элСктричСского поля, Ρ‚ΠΎΠ³Π΄Π° ΠΊΠ°ΠΊ магнитная сила, Π΄Π΅ΠΉΡΡ‚Π²ΡƒΡŽΡ‰Π°Ρ Π½Π° двиТущиСся заряды, пСрпСндикулярна силовым линиям ΠΌΠ°Π³Π½ΠΈΡ‚Π½ΠΎΠ³ΠΎ поля.
  3. Π—Π°ΠΌΠ΅Ρ‚ΠΈΠ², Ρ‡Ρ‚ΠΎ силовыС Π»ΠΈΠ½ΠΈΠΈ ΠΌΠ°Π³Π½ΠΈΡ‚Π½ΠΎΠ³ΠΎ поля стСрТнСвого ΠΌΠ°Π³Π½ΠΈΡ‚Π° Π½Π°ΠΏΠΎΠΌΠΈΠ½Π°ΡŽΡ‚ силовыС Π»ΠΈΠ½ΠΈΠΈ ΠΏΠ°Ρ€Ρ‹ Ρ€Π°Π²Π½Ρ‹Ρ… ΠΈ ΠΏΡ€ΠΎΡ‚ΠΈΠ²ΠΎΠΏΠΎΠ»ΠΎΠΆΠ½Ρ‹Ρ… зарядов, ΠΎΠΆΠΈΠ΄Π°Π΅Ρ‚Π΅ Π»ΠΈ Π²Ρ‹, Ρ‡Ρ‚ΠΎ ΠΌΠ°Π³Π½ΠΈΡ‚Π½ΠΎΠ΅ ΠΏΠΎΠ»Π΅ Π±ΡƒΠ΄Π΅Ρ‚ быстро ΡƒΠΌΠ΅Π½ΡŒΡˆΠ°Ρ‚ΡŒΡΡ Π² силС ΠΏΠΎ ΠΌΠ΅Ρ€Π΅ удалСния ΠΎΡ‚ ΠΌΠ°Π³Π½ΠΈΡ‚Π°? Π­Ρ‚ΠΎ согласуСтся с вашим ΠΎΠΏΡ‹Ρ‚ΠΎΠΌ Ρ€Π°Π±ΠΎΡ‚Ρ‹ с ΠΌΠ°Π³Π½ΠΈΡ‚Π°ΠΌΠΈ?
  4. ΠœΠ°Π³Π½ΠΈΡ‚Π½ΠΎΠ΅ ΠΏΠΎΠ»Π΅ Π—Π΅ΠΌΠ»ΠΈ ΠΏΠ°Ρ€Π°Π»Π»Π΅Π»ΡŒΠ½ΠΎ Π·Π΅ΠΌΠ»Π΅ Π²ΠΎ всСх мСстах? Если Π½Π΅Ρ‚, Ρ‚ΠΎ Π³Π΄Π΅ ΠΎΠ½Π° ΠΏΠ°Ρ€Π°Π»Π»Π΅Π»ΡŒΠ½Π° повСрхности? Π•Π³ΠΎ сила ΠΎΠ΄ΠΈΠ½Π°ΠΊΠΎΠ²Π° Π²ΠΎ всСх мСстах? Если Π½Π΅Ρ‚, Ρ‚ΠΎ Π³Π΄Π΅ ΠΎΠ½ΠΎ большС всСго?

Глоссарий

ΠΌΠ°Π³Π½ΠΈΡ‚Π½ΠΎΠ΅ ΠΏΠΎΠ»Π΅:
прСдставлСниС ΠΌΠ°Π³Π½ΠΈΡ‚Π½Ρ‹Ρ… сил
B -ΠΏΠΎΠ»Π΅:
Π΄Ρ€ΡƒΠ³ΠΎΠΉ Ρ‚Π΅Ρ€ΠΌΠΈΠ½ для обозначСния ΠΌΠ°Π³Π½ΠΈΡ‚Π½ΠΎΠ³ΠΎ поля
силовыС Π»ΠΈΠ½ΠΈΠΈ ΠΌΠ°Π³Π½ΠΈΡ‚Π½ΠΎΠ³ΠΎ поля:
графичСскоС ΠΈΠ·ΠΎΠ±Ρ€Π°ΠΆΠ΅Π½ΠΈΠ΅ силы ΠΈ направлСния ΠΌΠ°Π³Π½ΠΈΡ‚Π½ΠΎΠ³ΠΎ поля
Π½Π°ΠΏΡ€Π°Π²Π»Π΅Π½ΠΈΠ΅ силовых Π»ΠΈΠ½ΠΈΠΉ ΠΌΠ°Π³Π½ΠΈΡ‚Π½ΠΎΠ³ΠΎ поля:
Π½Π°ΠΏΡ€Π°Π²Π»Π΅Π½ΠΈΠ΅, Π½Π° ΠΊΠΎΡ‚ΠΎΡ€ΠΎΠ΅ ΡƒΠΊΠ°Π·Ρ‹Π²Π°Π΅Ρ‚ сСвСрный ΠΊΠΎΠ½Π΅Ρ† стрСлки компаса

20.3 ЭлСктромагнитная индукция — Ρ„ΠΈΠ·ΠΈΠΊΠ°

ИзмСнСниС ΠΌΠ°Π³Π½ΠΈΡ‚Π½Ρ‹Ρ… ΠΏΠΎΠ»Π΅ΠΉ

Π’ ΠΏΡ€Π΅Π΄Ρ‹Π΄ΡƒΡ‰Π΅ΠΌ Ρ€Π°Π·Π΄Π΅Π»Π΅ ΠΌΡ‹ ΡƒΠ·Π½Π°Π»ΠΈ, Ρ‡Ρ‚ΠΎ Ρ‚ΠΎΠΊ создаСт ΠΌΠ°Π³Π½ΠΈΡ‚Π½ΠΎΠ΅ ΠΏΠΎΠ»Π΅. Если ΠΏΡ€ΠΈΡ€ΠΎΠ΄Π° симмСтрична, Ρ‚ΠΎ, Π²ΠΎΠ·ΠΌΠΎΠΆΠ½ΠΎ, ΠΌΠ°Π³Π½ΠΈΡ‚Π½ΠΎΠ΅ ΠΏΠΎΠ»Π΅ ΠΌΠΎΠΆΠ΅Ρ‚ ΡΠΎΠ·Π΄Π°Ρ‚ΡŒ Ρ‚ΠΎΠΊ. Π’ 1831 Π³ΠΎΠ΄Ρƒ, ΠΏΡ€ΠΈΠΌΠ΅Ρ€Π½ΠΎ Ρ‡Π΅Ρ€Π΅Π· 12 Π»Π΅Ρ‚ послС открытия, Ρ‡Ρ‚ΠΎ элСктричСский Ρ‚ΠΎΠΊ создаСт ΠΌΠ°Π³Π½ΠΈΡ‚Π½ΠΎΠ΅ ΠΏΠΎΠ»Π΅, английский ΡƒΡ‡Π΅Π½Ρ‹ΠΉ Майкл Π€Π°Ρ€Π°Π΄Π΅ΠΉ (1791–1862) ΠΈ амСриканский ΡƒΡ‡Π΅Π½Ρ‹ΠΉ Π”ΠΆΠΎΠ·Π΅Ρ„ Π“Π΅Π½Ρ€ΠΈ (1797–1878) нСзависимо Π΄Ρ€ΡƒΠ³ ΠΎΡ‚ Π΄Ρ€ΡƒΠ³Π° продСмонстрировали, Ρ‡Ρ‚ΠΎ ΠΌΠ°Π³Π½ΠΈΡ‚Π½Ρ‹Π΅ поля ΠΌΠΎΠ³ΡƒΡ‚ ΡΠΎΠ·Π΄Π°Π²Π°Ρ‚ΡŒ Ρ‚ΠΎΠΊΠΈ.Основной процСсс Π³Π΅Π½Π΅Ρ€Π°Ρ†ΠΈΠΈ Ρ‚ΠΎΠΊΠΎΠ² с ΠΏΠΎΠΌΠΎΡ‰ΡŒΡŽ ΠΌΠ°Π³Π½ΠΈΡ‚Π½Ρ‹Ρ… ΠΏΠΎΠ»Π΅ΠΉ называСтся ΠΈΠ½Π΄ΡƒΠΊΡ†ΠΈΠ΅ΠΉ; этот процСсс Ρ‚Π°ΠΊΠΆΠ΅ Π½Π°Π·Ρ‹Π²Π°ΡŽΡ‚ ΠΌΠ°Π³Π½ΠΈΡ‚Π½ΠΎΠΉ ΠΈΠ½Π΄ΡƒΠΊΡ†ΠΈΠ΅ΠΉ, Ρ‡Ρ‚ΠΎΠ±Ρ‹ ΠΎΡ‚Π»ΠΈΡ‡ΠΈΡ‚ΡŒ Π΅Π³ΠΎ ΠΎΡ‚ ΠΈΠ½Π΄ΡƒΠΊΡ†ΠΈΠΎΠ½Π½ΠΎΠΉ зарядки, Π² ΠΊΠΎΡ‚ΠΎΡ€ΠΎΠΉ ΠΈΡΠΏΠΎΠ»ΡŒΠ·ΡƒΠ΅Ρ‚ΡΡ элСктростатичСская кулоновская сила.

Когда Π€Π°Ρ€Π°Π΄Π΅ΠΉ ΠΎΡ‚ΠΊΡ€Ρ‹Π» Ρ‚ΠΎ, Ρ‡Ρ‚ΠΎ сСйчас называСтся Π·Π°ΠΊΠΎΠ½ΠΎΠΌ ΠΈΠ½Π΄ΡƒΠΊΡ†ΠΈΠΈ ЀарадСя, ΠΊΠΎΡ€ΠΎΠ»Π΅Π²Π° Виктория спросила Π΅Π³ΠΎ, ΠΊΠ°ΠΊ ΠΌΠΎΠΆΠ½ΠΎ ΠΈΡΠΏΠΎΠ»ΡŒΠ·ΠΎΠ²Π°Ρ‚ΡŒ элСктричСство. «Мадам, — ΠΎΡ‚Π²Π΅Ρ‚ΠΈΠ» ΠΎΠ½, — Ρ‡Ρ‚ΠΎ Ρ…ΠΎΡ€ΠΎΡˆΠ΅Π³ΠΎ Π² Ρ€Π΅Π±Π΅Π½ΠΊΠ΅?Β» БСгодня Ρ‚ΠΎΠΊΠΈ, ΠΈΠ½Π΄ΡƒΡ†ΠΈΡ€ΠΎΠ²Π°Π½Π½Ρ‹Π΅ ΠΌΠ°Π³Π½ΠΈΡ‚Π½Ρ‹ΠΌΠΈ полями, Π½Π΅ΠΎΠ±Ρ…ΠΎΠ΄ΠΈΠΌΡ‹ Π½Π°ΡˆΠ΅ΠΌΡƒ тСхнологичСскому общСству. ЭлСктричСский Π³Π΅Π½Π΅Ρ€Π°Ρ‚ΠΎΡ€, ΠΊΠΎΡ‚ΠΎΡ€Ρ‹ΠΉ ΠΌΠΎΠΆΠ½ΠΎ Π½Π°ΠΉΡ‚ΠΈ Π²ΠΎ всСм, ΠΎΡ‚ Π°Π²Ρ‚ΠΎΠΌΠΎΠ±ΠΈΠ»Π΅ΠΉ Π΄ΠΎ вСлосипСдов ΠΈ Π°Ρ‚ΠΎΠΌΠ½Ρ‹Ρ… элСктростанций, ΠΈΡΠΏΠΎΠ»ΡŒΠ·ΡƒΠ΅Ρ‚ ΠΌΠ°Π³Π½Π΅Ρ‚ΠΈΠ·ΠΌ для Π³Π΅Π½Π΅Ρ€Π°Ρ†ΠΈΠΈ элСктричСского Ρ‚ΠΎΠΊΠ°.Π”Ρ€ΡƒΠ³ΠΈΠ΅ устройства, ΠΊΠΎΡ‚ΠΎΡ€Ρ‹Π΅ ΠΈΡΠΏΠΎΠ»ΡŒΠ·ΡƒΡŽΡ‚ ΠΌΠ°Π³Π½Π΅Ρ‚ΠΈΠ·ΠΌ для ΠΈΠ½Π΄ΡƒΠΊΡ†ΠΈΠΈ Ρ‚ΠΎΠΊΠΎΠ², Π²ΠΊΠ»ΡŽΡ‡Π°ΡŽΡ‚ Π² сСбя звукосниматСли Π² элСктрогитарах, трансформаторы любого Ρ€Π°Π·ΠΌΠ΅Ρ€Π°, ΠΎΠΏΡ€Π΅Π΄Π΅Π»Π΅Π½Π½Ρ‹Π΅ ΠΌΠΈΠΊΡ€ΠΎΡ„ΠΎΠ½Ρ‹, Π²ΠΎΡ€ΠΎΡ‚Π° бСзопасности аэропорта ΠΈ ΠΌΠ΅Ρ…Π°Π½ΠΈΠ·ΠΌΡ‹ дСмпфирования Π½Π° Ρ‡ΡƒΠ²ΡΡ‚Π²ΠΈΡ‚Π΅Π»ΡŒΠ½Ρ‹Ρ… химичСских вСсах.

Один ΠΈΠ· экспСримСнтов ЀарадСя для дСмонстрации ΠΌΠ°Π³Π½ΠΈΡ‚Π½ΠΎΠΉ ΠΈΠ½Π΄ΡƒΠΊΡ†ΠΈΠΈ Π·Π°ΠΊΠ»ΡŽΡ‡Π°Π»ΡΡ Π² ΠΏΠ΅Ρ€Π΅ΠΌΠ΅Ρ‰Π΅Π½ΠΈΠΈ стСрТнСвого ΠΌΠ°Π³Π½ΠΈΡ‚Π° Ρ‡Π΅Ρ€Π΅Π· ΠΏΡ€ΠΎΠ²ΠΎΠ»ΠΎΡ‡Π½ΡƒΡŽ ΠΊΠ°Ρ‚ΡƒΡˆΠΊΡƒ ΠΈ ΠΈΠ·ΠΌΠ΅Ρ€Π΅Π½ΠΈΠΈ Ρ€Π΅Π·ΡƒΠ»ΡŒΡ‚ΠΈΡ€ΡƒΡŽΡ‰Π΅Π³ΠΎ элСктричСского Ρ‚ΠΎΠΊΠ° Ρ‡Π΅Ρ€Π΅Π· ΠΏΡ€ΠΎΠ²ΠΎΠ»ΠΎΠΊΡƒ. Π‘Ρ…Π΅ΠΌΠ° этого экспСримСнта ΠΏΠΎΠΊΠ°Π·Π°Π½Π° Π½Π° рис. 20.33. Он ΠΎΠ±Π½Π°Ρ€ΡƒΠΆΠΈΠ», Ρ‡Ρ‚ΠΎ Ρ‚ΠΎΠΊ индуцируСтся Ρ‚ΠΎΠ»ΡŒΠΊΠΎ Ρ‚ΠΎΠ³Π΄Π°, ΠΊΠΎΠ³Π΄Π° ΠΌΠ°Π³Π½ΠΈΡ‚ двиТСтся ΠΎΡ‚Π½ΠΎΡΠΈΡ‚Π΅Π»ΡŒΠ½ΠΎ ΠΊΠ°Ρ‚ΡƒΡˆΠΊΠΈ.Когда ΠΌΠ°Π³Π½ΠΈΡ‚ Π½Π΅ΠΏΠΎΠ΄Π²ΠΈΠΆΠ΅Π½ ΠΏΠΎ ΠΎΡ‚Π½ΠΎΡˆΠ΅Π½ΠΈΡŽ ΠΊ ΠΊΠ°Ρ‚ΡƒΡˆΠΊΠ΅, Π² ΠΊΠ°Ρ‚ΡƒΡˆΠΊΠ΅ Π½Π΅ индуцируСтся Ρ‚ΠΎΠΊ, ΠΊΠ°ΠΊ ΠΏΠΎΠΊΠ°Π·Π°Π½ΠΎ Π½Π° рисункС 20.33. ΠšΡ€ΠΎΠΌΠ΅ Ρ‚ΠΎΠ³ΠΎ, ΠΏΠ΅Ρ€Π΅ΠΌΠ΅Ρ‰Π΅Π½ΠΈΠ΅ ΠΌΠ°Π³Π½ΠΈΡ‚Π° Π² ΠΏΡ€ΠΎΡ‚ΠΈΠ²ΠΎΠΏΠΎΠ»ΠΎΠΆΠ½ΠΎΠΌ Π½Π°ΠΏΡ€Π°Π²Π»Π΅Π½ΠΈΠΈ (сравнитС рис. 20.33 с рис. 20.33) ΠΈΠ»ΠΈ ΠΈΠ·ΠΌΠ΅Π½Π΅Π½ΠΈΠ΅ полярности ΠΌΠ°Π³Π½ΠΈΡ‚Π° (сравнитС рис. 20.33 с рис. 20.33) ΠΏΡ€ΠΈΠ²ΠΎΠ΄ΠΈΡ‚ ΠΊ возникновСнию Ρ‚ΠΎΠΊΠ° Π² ΠΏΡ€ΠΎΡ‚ΠΈΠ²ΠΎΠΏΠΎΠ»ΠΎΠΆΠ½ΠΎΠΌ Π½Π°ΠΏΡ€Π°Π²Π»Π΅Π½ΠΈΠΈ.

Рис. 20.33 Π”Π²ΠΈΠΆΠ΅Π½ΠΈΠ΅ ΠΌΠ°Π³Π½ΠΈΡ‚Π° ΠΎΡ‚Π½ΠΎΡΠΈΡ‚Π΅Π»ΡŒΠ½ΠΎ ΠΊΠ°Ρ‚ΡƒΡˆΠΊΠΈ создаСт элСктричСскиС Ρ‚ΠΎΠΊΠΈ, ΠΊΠ°ΠΊ ΠΏΠΎΠΊΠ°Π·Π°Π½ΠΎ. Π’Π°ΠΊΠΈΠ΅ ΠΆΠ΅ Ρ‚ΠΎΠΊΠΈ Π²ΠΎΠ·Π½ΠΈΠΊΠ°ΡŽΡ‚, Ссли ΠΊΠ°Ρ‚ΡƒΡˆΠΊΡƒ ΠΏΠ΅Ρ€Π΅ΠΌΠ΅Ρ‰Π°Ρ‚ΡŒ ΠΎΡ‚Π½ΠΎΡΠΈΡ‚Π΅Π»ΡŒΠ½ΠΎ ΠΌΠ°Π³Π½ΠΈΡ‚Π°.Π§Π΅ΠΌ большС ΡΠΊΠΎΡ€ΠΎΡΡ‚ΡŒ, Ρ‚Π΅ΠΌ большС Π²Π΅Π»ΠΈΡ‡ΠΈΠ½Π° Ρ‚ΠΎΠΊΠ°, ΠΈ Ρ‚ΠΎΠΊ Ρ€Π°Π²Π΅Π½ Π½ΡƒΠ»ΡŽ, ΠΊΠΎΠ³Π΄Π° Π½Π΅Ρ‚ двиТСния. Π’ΠΎΠΊ, Π²ΠΎΠ·Π½ΠΈΠΊΠ°ΡŽΡ‰ΠΈΠΉ ΠΏΡ€ΠΈ ΠΏΠ΅Ρ€Π΅ΠΌΠ΅Ρ‰Π΅Π½ΠΈΠΈ ΠΌΠ°Π³Π½ΠΈΡ‚Π° Π²Π²Π΅Ρ€Ρ…, ΠΈΠΌΠ΅Π΅Ρ‚ Π½Π°ΠΏΡ€Π°Π²Π»Π΅Π½ΠΈΠ΅, ΠΏΡ€ΠΎΡ‚ΠΈΠ²ΠΎΠΏΠΎΠ»ΠΎΠΆΠ½ΠΎΠ΅ Π½Π°ΠΏΡ€Π°Π²Π»Π΅Π½ΠΈΡŽ Ρ‚ΠΎΠΊΠ°, Π²ΠΎΠ·Π½ΠΈΠΊΠ°ΡŽΡ‰Π΅Π³ΠΎ ΠΏΡ€ΠΈ ΠΏΠ΅Ρ€Π΅ΠΌΠ΅Ρ‰Π΅Π½ΠΈΠΈ ΠΌΠ°Π³Π½ΠΈΡ‚Π° Π²Π½ΠΈΠ·.

Π’ΠΈΡ€Ρ‚ΡƒΠ°Π»ΡŒΠ½Π°Ρ Ρ„ΠΈΠ·ΠΈΠΊΠ°

Π—Π°ΠΊΠΎΠ½ ЀарадСя

ΠŸΠΎΠΏΡ€ΠΎΠ±ΡƒΠΉΡ‚Π΅ это ΠΌΠΎΠ΄Π΅Π»ΠΈΡ€ΠΎΠ²Π°Π½ΠΈΠ΅, Ρ‡Ρ‚ΠΎΠ±Ρ‹ ΡƒΠ²ΠΈΠ΄Π΅Ρ‚ΡŒ, ΠΊΠ°ΠΊ Π΄Π²ΠΈΠΆΠ΅Π½ΠΈΠ΅ ΠΌΠ°Π³Π½ΠΈΡ‚Π° создаСт Ρ‚ΠΎΠΊ Π² Ρ†Π΅ΠΏΠΈ. Π›Π°ΠΌΠΏΠΎΡ‡ΠΊΠ° загораСтся, Ρ‡Ρ‚ΠΎΠ±Ρ‹ ΠΏΠΎΠΊΠ°Π·Π°Ρ‚ΡŒ, ΠΊΠΎΠ³Π΄Π° Ρ‚Π΅Ρ‡Π΅Ρ‚ Ρ‚ΠΎΠΊ, Π° Π²ΠΎΠ»ΡŒΡ‚ΠΌΠ΅Ρ‚Ρ€ ΠΏΠΎΠΊΠ°Π·Ρ‹Π²Π°Π΅Ρ‚ ΠΏΠ°Π΄Π΅Π½ΠΈΠ΅ напряТСния Π½Π° Π»Π°ΠΌΠΏΠΎΡ‡ΠΊΠ΅.ΠŸΠΎΠΏΡ€ΠΎΠ±ΡƒΠΉΡ‚Π΅ ΠΏΠ΅Ρ€Π΅ΠΌΠ΅ΡΡ‚ΠΈΡ‚ΡŒ ΠΌΠ°Π³Π½ΠΈΡ‚ Ρ‡Π΅Ρ€Π΅Π· Ρ‡Π΅Ρ‚Ρ‹Ρ€Π΅Ρ…Π²ΠΈΡ‚ΠΊΠΎΠ²ΡƒΡŽ ΠΊΠ°Ρ‚ΡƒΡˆΠΊΡƒ ΠΈ Ρ‡Π΅Ρ€Π΅Π· Π΄Π²ΡƒΡ…Π²ΠΈΡ‚ΠΊΠΎΠ²ΡƒΡŽ ΠΊΠ°Ρ‚ΡƒΡˆΠΊΡƒ. Какая ΠΊΠ°Ρ‚ΡƒΡˆΠΊΠ° ΠΏΡ€ΠΎΠΈΠ·Π²ΠΎΠ΄ΠΈΡ‚ Π±ΠΎΠ»Π΅Π΅ высокоС напряТСниС ΠΏΡ€ΠΈ ΠΎΠ΄ΠΈΠ½Π°ΠΊΠΎΠ²ΠΎΠΉ скорости ΠΌΠ°Π³Π½ΠΈΡ‚Π°?

Π—Π°Ρ…Π²Π°Ρ‚

Если сСвСрный полюс находится Π²Π»Π΅Π²ΠΎ ΠΈ ΠΌΠ°Π³Π½ΠΈΡ‚ пСрСмСщаСтся справа Π½Π°Π»Π΅Π²ΠΎ, ΠΏΡ€ΠΈ Π²Ρ…ΠΎΠ΄Π΅ ΠΌΠ°Π³Π½ΠΈΡ‚Π° Π² ΠΊΠ°Ρ‚ΡƒΡˆΠΊΡƒ создаСтся ΠΏΠΎΠ»ΠΎΠΆΠΈΡ‚Π΅Π»ΡŒΠ½ΠΎΠ΅ напряТСниС. КакоС Π·Π½Π°ΠΊΠΎΠ²ΠΎΠ΅ напряТСниС получится, Ссли экспСримСнт ΠΏΠΎΠ²Ρ‚ΠΎΡ€ΠΈΡ‚ΡŒ с ΡŽΠΆΠ½Ρ‹ΠΌ полюсом слСва?

  1. Π—Π½Π°ΠΊ напряТСния измСнится, ΠΏΠΎΡ‚ΠΎΠΌΡƒ Ρ‡Ρ‚ΠΎ Π½Π°ΠΏΡ€Π°Π²Π»Π΅Π½ΠΈΠ΅ Ρ‚ΠΎΠΊΠ° измСнится ΠΏΡ€ΠΈ ΠΏΠ΅Ρ€Π΅ΠΌΠ΅Ρ‰Π΅Π½ΠΈΠΈ юТного полюса ΠΌΠ°Π³Π½ΠΈΡ‚Π° Π²Π»Π΅Π²ΠΎ.
  2. Π—Π½Π°ΠΊ напряТСния останСтся ΠΏΡ€Π΅ΠΆΠ½ΠΈΠΌ, ΠΏΠΎΡ‚ΠΎΠΌΡƒ Ρ‡Ρ‚ΠΎ Π½Π°ΠΏΡ€Π°Π²Π»Π΅Π½ΠΈΠ΅ Ρ‚ΠΎΠΊΠ° Π½Π΅ измСнится ΠΏΡ€ΠΈ ΠΏΠ΅Ρ€Π΅ΠΌΠ΅Ρ‰Π΅Π½ΠΈΠΈ юТного полюса ΠΌΠ°Π³Π½ΠΈΡ‚Π° Π²Π»Π΅Π²ΠΎ.
  3. Π—Π½Π°ΠΊ напряТСния измСнится, ΠΏΠΎΡ‚ΠΎΠΌΡƒ Ρ‡Ρ‚ΠΎ Π²Π΅Π»ΠΈΡ‡ΠΈΠ½Π° ΠΏΡ€ΠΎΡ‚Π΅ΠΊΠ°ΡŽΡ‰Π΅Π³ΠΎ Ρ‚ΠΎΠΊΠ° измСнится ΠΏΡ€ΠΈ ΠΏΠ΅Ρ€Π΅ΠΌΠ΅Ρ‰Π΅Π½ΠΈΠΈ юТного полюса ΠΌΠ°Π³Π½ΠΈΡ‚Π° Π²Π»Π΅Π²ΠΎ.
  4. Π—Π½Π°ΠΊ напряТСния останСтся ΠΏΡ€Π΅ΠΆΠ½ΠΈΠΌ, ΠΏΠΎΡ‚ΠΎΠΌΡƒ Ρ‡Ρ‚ΠΎ Π²Π΅Π»ΠΈΡ‡ΠΈΠ½Π° Ρ‚ΠΎΠΊΠ° Π½Π΅ измСнится ΠΏΡ€ΠΈ ΠΏΠ΅Ρ€Π΅ΠΌΠ΅Ρ‰Π΅Π½ΠΈΠΈ юТного полюса ΠΌΠ°Π³Π½ΠΈΡ‚Π° Π²Π»Π΅Π²ΠΎ.

Π˜Π½Π΄ΡƒΡ†ΠΈΡ€ΠΎΠ²Π°Π½Π½Π°Ρ элСктродвиТущая сила

Если Π² ΠΊΠ°Ρ‚ΡƒΡˆΠΊΠ΅ индуцируСтся Ρ‚ΠΎΠΊ, Π€Π°Ρ€Π°Π΄Π΅ΠΉ рассуТдал, Ρ‡Ρ‚ΠΎ Π΄ΠΎΠ»ΠΆΠ½ΠΎ Π±Ρ‹Ρ‚ΡŒ Ρ‚ΠΎ, Ρ‡Ρ‚ΠΎ ΠΎΠ½ Π½Π°Π·Π²Π°Π» элСктродвиТущСй силой , ΠΏΡ€ΠΎΡ‚Π°Π»ΠΊΠΈΠ²Π°ΡŽΡ‰Π΅ΠΉ заряды Ρ‡Π΅Ρ€Π΅Π· ΠΊΠ°Ρ‚ΡƒΡˆΠΊΡƒ. Π­Ρ‚Π° интСрпрСтация оказалась Π½Π΅Π²Π΅Ρ€Π½ΠΎΠΉ; вмСсто этого внСшний источник, Π²Ρ‹ΠΏΠΎΠ»Π½ΡΡŽΡ‰ΠΈΠΉ Ρ€Π°Π±ΠΎΡ‚Ρƒ ΠΏΠΎ ΠΏΠ΅Ρ€Π΅ΠΌΠ΅Ρ‰Π΅Π½ΠΈΡŽ ΠΌΠ°Π³Π½ΠΈΡ‚Π°, добавляСт ΡΠ½Π΅Ρ€Π³ΠΈΡŽ зарядам Π² ΠΊΠ°Ρ‚ΡƒΡˆΠΊΠ΅. ЭнСргия, добавляСмая Π½Π° Π΅Π΄ΠΈΠ½ΠΈΡ†Ρƒ заряда, измСряСтся Π² Π²ΠΎΠ»ΡŒΡ‚Π°Ρ…, поэтому элСктродвиТущая сила Π½Π° самом Π΄Π΅Π»Π΅ являСтся ΠΏΠΎΡ‚Π΅Π½Ρ†ΠΈΠ°Π»ΠΎΠΌ. К соТалСнию, Π½Π°Π·Π²Π°Π½ΠΈΠ΅ «элСктродвиТущая сила» ΠΏΡ€ΠΈΠΆΠΈΠ»ΠΎΡΡŒ, Π° вмСстС с Π½ΠΈΠΌ ΠΈ Π²ΠΎΠ·ΠΌΠΎΠΆΠ½ΠΎΡΡ‚ΡŒ ΡΠΏΡƒΡ‚Π°Ρ‚ΡŒ Π΅Π³ΠΎ с Ρ€Π΅Π°Π»ΡŒΠ½ΠΎΠΉ силой.По этой ΠΏΡ€ΠΈΡ‡ΠΈΠ½Π΅ ΠΌΡ‹ ΠΈΠ·Π±Π΅Π³Π°Π΅ΠΌ Ρ‚Π΅Ρ€ΠΌΠΈΠ½Π° элСктродвиТущая сила ΠΈ просто ΠΈΡΠΏΠΎΠ»ΡŒΠ·ΡƒΠ΅ΠΌ сокращСниС эдс , ΠΊΠΎΡ‚ΠΎΡ€ΠΎΠ΅ ΠΈΠΌΠ΅Π΅Ρ‚ матСматичСский символ Ξ΅.Ξ΅. Π­Π”Π‘ ΠΌΠΎΠΆΠ΅Ρ‚ Π±Ρ‹Ρ‚ΡŒ ΠΎΠΏΡ€Π΅Π΄Π΅Π»Π΅Π½Π° ΠΊΠ°ΠΊ ΡΠΊΠΎΡ€ΠΎΡΡ‚ΡŒ, с ΠΊΠΎΡ‚ΠΎΡ€ΠΎΠΉ энСргия отбираСтся ΠΎΡ‚ источника Π½Π° Π΅Π΄ΠΈΠ½ΠΈΡ†Ρƒ Ρ‚ΠΎΠΊΠ°, ΠΏΡ€ΠΎΡ‚Π΅ΠΊΠ°ΡŽΡ‰Π΅Π³ΠΎ ΠΏΠΎ Ρ†Π΅ΠΏΠΈ. Π’Π°ΠΊΠΈΠΌ ΠΎΠ±Ρ€Π°Π·ΠΎΠΌ, Π­Π”Π‘ — это энСргия Π½Π° Π΅Π΄ΠΈΠ½ΠΈΡ†Ρƒ заряда , добавлСнная источником, которая контрастируСт с напряТСниСм, ΠΊΠΎΡ‚ΠΎΡ€ΠΎΠ΅ прСдставляСт собой ΡΠ½Π΅Ρ€Π³ΠΈΡŽ Π½Π° Π΅Π΄ΠΈΠ½ΠΈΡ†Ρƒ заряда , Π²Ρ‹ΡΠ²ΠΎΠ±ΠΎΠΆΠ΄Π°Π΅ΠΌΡƒΡŽ , ΠΊΠΎΠ³Π΄Π° заряды проходят Ρ‡Π΅Ρ€Π΅Π· Ρ†Π΅ΠΏΡŒ.

Π§Ρ‚ΠΎΠ±Ρ‹ ΠΏΠΎΠ½ΡΡ‚ΡŒ, ΠΏΠΎΡ‡Π΅ΠΌΡƒ Π² ΠΊΠ°Ρ‚ΡƒΡˆΠΊΠ΅ Π²ΠΎΠ·Π½ΠΈΠΊΠ°Π΅Ρ‚ Π­Π”Π‘ ΠΈΠ·-Π·Π° двиТущСгося ΠΌΠ°Π³Π½ΠΈΡ‚Π°, рассмотрим рисунок 20.34, Π½Π° ΠΊΠΎΡ‚ΠΎΡ€ΠΎΠΌ ΠΏΠΎΠΊΠ°Π·Π°Π½ стСрТнСвой ΠΌΠ°Π³Π½ΠΈΡ‚, двиТущийся Π²Π½ΠΈΠ· ΠΎΡ‚Π½ΠΎΡΠΈΡ‚Π΅Π»ΡŒΠ½ΠΎ ΠΏΡ€ΠΎΠ²ΠΎΠ»ΠΎΡ‡Π½ΠΎΠΉ ΠΏΠ΅Ρ‚Π»ΠΈ. ΠŸΠ΅Ρ€Π²ΠΎΠ½Π°Ρ‡Π°Π»ΡŒΠ½ΠΎ Ρ‡Π΅Ρ€Π΅Π· ΠΏΠ΅Ρ‚Π»ΡŽ проходят сСмь силовых Π»ΠΈΠ½ΠΈΠΉ ΠΌΠ°Π³Π½ΠΈΡ‚Π½ΠΎΠ³ΠΎ поля (см. Π˜Π·ΠΎΠ±Ρ€Π°ΠΆΠ΅Π½ΠΈΠ΅ слСва). ΠŸΠΎΡΠΊΠΎΠ»ΡŒΠΊΡƒ ΠΌΠ°Π³Π½ΠΈΡ‚ удаляСтся ΠΎΡ‚ ΠΊΠ°Ρ‚ΡƒΡˆΠΊΠΈ, Ρ‚ΠΎΠ»ΡŒΠΊΠΎ ΠΏΡΡ‚ΡŒ силовых Π»ΠΈΠ½ΠΈΠΉ ΠΌΠ°Π³Π½ΠΈΡ‚Π½ΠΎΠ³ΠΎ поля проходят Ρ‡Π΅Ρ€Π΅Π· ΠΏΠ΅Ρ‚Π»ΡŽ Π·Π° ΠΊΠΎΡ€ΠΎΡ‚ΠΊΠΎΠ΅ врСмя Ξ”tΞ”t (см. Π˜Π·ΠΎΠ±Ρ€Π°ΠΆΠ΅Π½ΠΈΠ΅ справа). Π’Π°ΠΊΠΈΠΌ ΠΎΠ±Ρ€Π°Π·ΠΎΠΌ, ΠΊΠΎΠ³Π΄Π° происходит ΠΈΠ·ΠΌΠ΅Π½Π΅Π½ΠΈΠ΅ количСства силовых Π»ΠΈΠ½ΠΈΠΉ ΠΌΠ°Π³Π½ΠΈΡ‚Π½ΠΎΠ³ΠΎ поля, проходящих Ρ‡Π΅Ρ€Π΅Π· ΠΎΠ±Π»Π°ΡΡ‚ΡŒ, ΠΎΠΏΡ€Π΅Π΄Π΅Π»ΡΠ΅ΠΌΡƒΡŽ ΠΏΡ€ΠΎΠ²ΠΎΠ»ΠΎΡ‡Π½ΠΎΠΉ ΠΏΠ΅Ρ‚Π»Π΅ΠΉ, Π² ΠΏΡ€ΠΎΠ²ΠΎΠ»ΠΎΡ‡Π½ΠΎΠΉ ΠΏΠ΅Ρ‚Π»Π΅ индуцируСтся Π­Π”Π‘. ΠŸΠΎΠ΄ΠΎΠ±Π½Ρ‹Π΅ экспСримСнты ΠΏΠΎΠΊΠ°Π·Ρ‹Π²Π°ΡŽΡ‚, Ρ‡Ρ‚ΠΎ навСдСнная Π­Π”Π‘ ΠΏΡ€ΠΎΠΏΠΎΡ€Ρ†ΠΈΠΎΠ½Π°Π»ΡŒΠ½Π° скорости измСнСния ΠΌΠ°Π³Π½ΠΈΡ‚Π½ΠΎΠ³ΠΎ поля.ΠœΠ°Ρ‚Π΅ΠΌΠ°Ρ‚ΠΈΡ‡Π΅ΡΠΊΠΈ ΠΌΡ‹ Π²Ρ‹Ρ€Π°ΠΆΠ°Π΅ΠΌ это ΠΊΠ°ΠΊ

Ξ΅βˆΞ”BΞ”t, Ξ΅βˆΞ”BΞ”t,

20,24

Π³Π΄Π΅ Ξ”BΞ”B — ΠΈΠ·ΠΌΠ΅Π½Π΅Π½ΠΈΠ΅ Π²Π΅Π»ΠΈΡ‡ΠΈΠ½Ρ‹ ΠΌΠ°Π³Π½ΠΈΡ‚Π½ΠΎΠ³ΠΎ поля Π·Π° врСмя Ξ”tΞ”t, Π° A — ΠΏΠ»ΠΎΡ‰Π°Π΄ΡŒ ΠΏΠ΅Ρ‚Π»ΠΈ.

Рис. 20.34 Π‘Ρ‚Π΅Ρ€ΠΆΠ½Π΅Π²ΠΎΠΉ ΠΌΠ°Π³Π½ΠΈΡ‚ двиТСтся Π²Π½ΠΈΠ· ΠΎΡ‚Π½ΠΎΡΠΈΡ‚Π΅Π»ΡŒΠ½ΠΎ ΠΏΡ€ΠΎΠ²ΠΎΠ»ΠΎΡ‡Π½ΠΎΠΉ ΠΏΠ΅Ρ‚Π»ΠΈ, Ρ‚Π°ΠΊ Ρ‡Ρ‚ΠΎ количСство силовых Π»ΠΈΠ½ΠΈΠΉ ΠΌΠ°Π³Π½ΠΈΡ‚Π½ΠΎΠ³ΠΎ поля, проходящих Ρ‡Π΅Ρ€Π΅Π· ΠΏΠ΅Ρ‚Π»ΡŽ, со Π²Ρ€Π΅ΠΌΠ΅Π½Π΅ΠΌ ΡƒΠΌΠ΅Π½ΡŒΡˆΠ°Π΅Ρ‚ΡΡ. Π­Ρ‚ΠΎ Π²Ρ‹Π·Ρ‹Π²Π°Π΅Ρ‚ Π² ΠΊΠΎΠ½Ρ‚ΡƒΡ€Π΅ Π­Π”Π‘, ΡΠΎΠ·Π΄Π°ΡŽΡ‰ΡƒΡŽ элСктричСский Ρ‚ΠΎΠΊ.

ΠžΠ±Ρ€Π°Ρ‚ΠΈΡ‚Π΅ Π²Π½ΠΈΠΌΠ°Π½ΠΈΠ΅, Ρ‡Ρ‚ΠΎ силовыС Π»ΠΈΠ½ΠΈΠΈ ΠΌΠ°Π³Π½ΠΈΡ‚Π½ΠΎΠ³ΠΎ поля, Π»Π΅ΠΆΠ°Ρ‰ΠΈΠ΅ Π² плоскости ΠΏΡ€ΠΎΠ²ΠΎΠ»ΠΎΡ‡Π½ΠΎΠΉ ΠΏΠ΅Ρ‚Π»ΠΈ, Π½Π° самом Π΄Π΅Π»Π΅ Π½Π΅ проходят Ρ‡Π΅Ρ€Π΅Π· ΠΏΠ΅Ρ‚Π»ΡŽ, ΠΊΠ°ΠΊ ΠΏΠΎΠΊΠ°Π·Π°Π½ΠΎ ΠΊΡ€Π°ΠΉΠ½ΠΈΠΌ Π»Π΅Π²Ρ‹ΠΌ Π²ΠΈΡ‚ΠΊΠΎΠΌ Π½Π° рисункС 20.35. На этом рисункС стрСлка, выходящая ΠΈΠ· ΠΏΠ΅Ρ‚Π»ΠΈ, прСдставляСт собой Π²Π΅ΠΊΡ‚ΠΎΡ€, Π²Π΅Π»ΠΈΡ‡ΠΈΠ½Π° ΠΊΠΎΡ‚ΠΎΡ€ΠΎΠ³ΠΎ Ρ€Π°Π²Π½Π° ΠΏΠ»ΠΎΡ‰Π°Π΄ΠΈ ΠΏΠ΅Ρ‚Π»ΠΈ, Π° Π½Π°ΠΏΡ€Π°Π²Π»Π΅Π½ΠΈΠ΅ пСрпСндикулярно плоскости ΠΏΠ΅Ρ‚Π»ΠΈ. На рисункС 20.35 пСтля ΠΏΠΎΠ²Π΅Ρ€Π½ΡƒΡ‚Π° ΠΎΡ‚ ΞΈ = 90 Β° ΞΈ = 90 Β°. Π΄ΠΎ ΞΈ = 0 Β°, ΞΈ = 0 Β° Π²ΠΊΠ»Π°Π΄ силовых Π»ΠΈΠ½ΠΈΠΉ ΠΌΠ°Π³Π½ΠΈΡ‚Π½ΠΎΠ³ΠΎ поля Π² Π­Π”Π‘ увСличиваСтся. Π’Π°ΠΊΠΈΠΌ ΠΎΠ±Ρ€Π°Π·ΠΎΠΌ, для создания Π­Π”Π‘ Π² ΠΏΡ€ΠΎΠ²ΠΎΠ»ΠΎΡ‡Π½ΠΎΠΉ ΠΏΠ΅Ρ‚Π»Π΅ Π²Π°ΠΆΠ½Π° ΡΠΎΡΡ‚Π°Π²Π»ΡΡŽΡ‰Π°Ρ ΠΌΠ°Π³Π½ΠΈΡ‚Π½ΠΎΠ³ΠΎ поля, которая Π½Π° пСрпСндикулярна плоскости ΠΏΠ΅Ρ‚Π»ΠΈ Π½Π° , Ρ‚ΠΎ Π΅ΡΡ‚ΡŒ BcosΞΈ.BcosΞΈ.

Π­Ρ‚ΠΎ Π°Π½Π°Π»ΠΎΠ³ паруса Π½Π° Π²Π΅Ρ‚Ρ€Ρƒ.ΠŸΡ€Π΅Π΄ΡΡ‚Π°Π²ΡŒΡ‚Π΅, Ρ‡Ρ‚ΠΎ проводящая пСтля — это парус, Π° ΠΌΠ°Π³Π½ΠΈΡ‚Π½ΠΎΠ΅ ΠΏΠΎΠ»Π΅ — ΠΊΠ°ΠΊ Π²Π΅Ρ‚Π΅Ρ€. Π§Ρ‚ΠΎΠ±Ρ‹ ΠΌΠ°ΠΊΡΠΈΠΌΠΈΠ·ΠΈΡ€ΠΎΠ²Π°Ρ‚ΡŒ силу Π²Π΅Ρ‚Ρ€Π° Π½Π° парусС, парус ΠΎΡ€ΠΈΠ΅Π½Ρ‚ΠΈΡ€ΡƒΡŽΡ‚ Ρ‚Π°ΠΊ, Ρ‡Ρ‚ΠΎΠ±Ρ‹ Π²Π΅ΠΊΡ‚ΠΎΡ€ Π΅Π³ΠΎ повСрхности ΡƒΠΊΠ°Π·Ρ‹Π²Π°Π» Π² Ρ‚ΠΎΠΌ ΠΆΠ΅ Π½Π°ΠΏΡ€Π°Π²Π»Π΅Π½ΠΈΠΈ, Ρ‡Ρ‚ΠΎ ΠΈ Π²Π΅Ρ‚Π΅Ρ€, ΠΊΠ°ΠΊ Π² самой ΠΏΡ€Π°Π²ΠΎΠΉ ΠΏΠ΅Ρ‚Π»Π΅ Π½Π° рис. 20.35. Когда парус Π²Ρ‹Ρ€ΠΎΠ²Π½Π΅Π½ Ρ‚Π°ΠΊ, Ρ‡Ρ‚ΠΎ Π²Π΅ΠΊΡ‚ΠΎΡ€ Π΅Π³ΠΎ повСрхности пСрпСндикулярСн Π²Π΅Ρ‚Ρ€Ρƒ, ΠΊΠ°ΠΊ Π² ΠΊΡ€Π°ΠΉΠ½Π΅ΠΉ Π»Π΅Π²ΠΎΠΉ ΠΏΠ΅Ρ‚Π»Π΅ Π½Π° рис. 20.35, Ρ‚ΠΎΠ³Π΄Π° Π²Π΅Ρ‚Π΅Ρ€ Π½Π΅ ΠΎΠΊΠ°Π·Ρ‹Π²Π°Π΅Ρ‚ силы Π½Π° парус.

Π’Π°ΠΊΠΈΠΌ ΠΎΠ±Ρ€Π°Π·ΠΎΠΌ, с ΡƒΡ‡Π΅Ρ‚ΠΎΠΌ ΡƒΠ³Π»Π° Π½Π°ΠΊΠ»ΠΎΠ½Π° ΠΌΠ°Π³Π½ΠΈΡ‚Π½ΠΎΠ³ΠΎ поля ΠΊ ΠΏΠ»ΠΎΡ‰Π°Π΄ΠΈ, ΠΏΡ€ΠΎΠΏΠΎΡ€Ρ†ΠΈΠΎΠ½Π°Π»ΡŒΠ½ΠΎΡΡ‚ΡŒ EβˆΞ”B / Ξ”tEβˆΞ”B / Ξ”t становится Ρ€Π°Π²Π½ΠΎΠΉ

EβˆΞ”BcosΞΈΞ”t.EβˆΞ”BcosΞΈΞ”t.

20,25

Рис. 20.35 ΠœΠ°Π³Π½ΠΈΡ‚Π½ΠΎΠ΅ ΠΏΠΎΠ»Π΅ Π»Π΅ΠΆΠΈΡ‚ Π² плоскости ΠΊΡ€Π°ΠΉΠ½Π΅ΠΉ Π»Π΅Π²ΠΎΠΉ ΠΏΠ΅Ρ‚Π»ΠΈ, поэтому Π² этом случаС ΠΎΠ½ΠΎ Π½Π΅ ΠΌΠΎΠΆΠ΅Ρ‚ Π³Π΅Π½Π΅Ρ€ΠΈΡ€ΠΎΠ²Π°Ρ‚ΡŒ Π­Π”Π‘. Когда пСтля поворачиваСтся Ρ‚Π°ΠΊ, Ρ‡Ρ‚ΠΎ ΡƒΠ³ΠΎΠ» ΠΌΠ°Π³Π½ΠΈΡ‚Π½ΠΎΠ³ΠΎ поля с Π²Π΅ΠΊΡ‚ΠΎΡ€ΠΎΠΌ, пСрпСндикулярным области ΠΏΠ΅Ρ‚Π»ΠΈ, увСличиваСтся Π΄ΠΎ 90 Β° 90 Β° (см. ΠšΡ€Π°ΠΉΠ½ΡŽΡŽ ΠΏΡ€Π°Π²ΡƒΡŽ ΠΏΠ΅Ρ‚Π»ΡŽ), ΠΌΠ°Π³Π½ΠΈΡ‚Π½ΠΎΠ΅ ΠΏΠΎΠ»Π΅ вносит ΠΌΠ°ΠΊΡΠΈΠΌΠ°Π»ΡŒΠ½Ρ‹ΠΉ Π²ΠΊΠ»Π°Π΄ Π² Π­Π”Π‘ Π² ΠΏΠ΅Ρ‚Π»Π΅. Π’ΠΎΡ‡ΠΊΠΈ ΠΏΠΎΠΊΠ°Π·Ρ‹Π²Π°ΡŽΡ‚, Π³Π΄Π΅ силовыС Π»ΠΈΠ½ΠΈΠΈ ΠΌΠ°Π³Π½ΠΈΡ‚Π½ΠΎΠ³ΠΎ поля ΠΏΠ΅Ρ€Π΅ΡΠ΅ΠΊΠ°ΡŽΡ‚ ΠΏΠ»ΠΎΡΠΊΠΎΡΡ‚ΡŒ, ΠΎΠΏΡ€Π΅Π΄Π΅Π»ΡΠ΅ΠΌΡƒΡŽ ΠΏΠ΅Ρ‚Π»Π΅ΠΉ.

Π”Ρ€ΡƒΠ³ΠΎΠΉ способ ΡƒΠΌΠ΅Π½ΡŒΡˆΠΈΡ‚ΡŒ количСство силовых Π»ΠΈΠ½ΠΈΠΉ ΠΌΠ°Π³Π½ΠΈΡ‚Π½ΠΎΠ³ΠΎ поля, проходящих Ρ‡Π΅Ρ€Π΅Π· ΠΏΡ€ΠΎΠ²ΠΎΠ΄ΡΡ‰ΡƒΡŽ ΠΏΠ΅Ρ‚Π»ΡŽ Π½Π° РисункС 20.35 Π½Π΅ для пСрСмСщСния ΠΌΠ°Π³Π½ΠΈΡ‚Π°, Π° для ΡƒΠΌΠ΅Π½ΡŒΡˆΠ΅Π½ΠΈΡ Ρ€Π°Π·ΠΌΠ΅Ρ€Π° ΠΏΠ΅Ρ‚Π»ΠΈ. ЭкспСримСнты ΠΏΠΎΠΊΠ°Π·Ρ‹Π²Π°ΡŽΡ‚, Ρ‡Ρ‚ΠΎ ΠΈΠ·ΠΌΠ΅Π½Π΅Π½ΠΈΠ΅ ΠΏΠ»ΠΎΡ‰Π°Π΄ΠΈ проводящСй ΠΏΠ΅Ρ‚Π»ΠΈ Π² ΡΡ‚Π°Π±ΠΈΠ»ΡŒΠ½ΠΎΠΌ ΠΌΠ°Π³Π½ΠΈΡ‚Π½ΠΎΠΌ ΠΏΠΎΠ»Π΅ Π²Ρ‹Π·Ρ‹Π²Π°Π΅Ρ‚ Π² ΠΏΠ΅Ρ‚Π»Π΅ Π­Π”Π‘. Π’Π°ΠΊΠΈΠΌ ΠΎΠ±Ρ€Π°Π·ΠΎΠΌ, Π­Π”Π‘, создаваСмая Π² проводящСй ΠΏΠ΅Ρ‚Π»Π΅, ΠΏΡ€ΠΎΠΏΠΎΡ€Ρ†ΠΈΠΎΠ½Π°Π»ΡŒΠ½Π° скорости измСнСния произвСдСния пСрпСндикулярного ΠΌΠ°Π³Π½ΠΈΡ‚Π½ΠΎΠ³ΠΎ поля ΠΈ ΠΏΠ»ΠΎΡ‰Π°Π΄ΠΈ ΠΏΠ΅Ρ‚Π»ΠΈ

. Ξ΅βˆΞ” [(BcosΞΈ) A] Ξ”t, Ξ΅βˆΞ” [(BcosΞΈ) A] Ξ”t,

20,26

, Π³Π΄Π΅ BcosΞΈBcosΞΈ — пСрпСндикулярноС ΠΌΠ°Π³Π½ΠΈΡ‚Π½ΠΎΠ΅ ΠΏΠΎΠ»Π΅, Π° A — ΠΏΠ»ΠΎΡ‰Π°Π΄ΡŒ ΠΊΠΎΠ½Ρ‚ΡƒΡ€Π°.ΠŸΡ€ΠΎΠ΄ΡƒΠΊΡ‚ BAcosΞΈBAcosΞΈ ΠΎΡ‡Π΅Π½ΡŒ Π²Π°ΠΆΠ΅Π½. Оно ΠΏΡ€ΠΎΠΏΠΎΡ€Ρ†ΠΈΠΎΠ½Π°Π»ΡŒΠ½ΠΎ количСству силовых Π»ΠΈΠ½ΠΈΠΉ ΠΌΠ°Π³Π½ΠΈΡ‚Π½ΠΎΠ³ΠΎ поля, ΠΊΠΎΡ‚ΠΎΡ€Ρ‹Π΅ проходят пСрпСндикулярно Ρ‡Π΅Ρ€Π΅Π· ΠΏΠΎΠ²Π΅Ρ€Ρ…Π½ΠΎΡΡ‚ΡŒ ΠΏΠ»ΠΎΡ‰Π°Π΄ΡŒΡŽ A . Π’ΠΎΠ·Π²Ρ€Π°Ρ‰Π°ΡΡΡŒ ΠΊ нашСй Π°Π½Π°Π»ΠΎΠ³ΠΈΠΈ с парусом, ΠΎΠ½ Π±ΡƒΠ΄Π΅Ρ‚ ΠΏΡ€ΠΎΠΏΠΎΡ€Ρ†ΠΈΠΎΠ½Π°Π»Π΅Π½ силС Π²Π΅Ρ‚Ρ€Π° Π½Π° парусС. Он называСтся ΠΌΠ°Π³Π½ΠΈΡ‚Π½Ρ‹ΠΌ ΠΏΠΎΡ‚ΠΎΠΊΠΎΠΌ ΠΈ обозначаСтся ΠΊΠ°ΠΊ ΦΦ.

Ξ¦ = BAcosΞΈΞ¦ = BAcosΞΈ

20,27

Π•Π΄ΠΈΠ½ΠΈΡ†Π΅ΠΉ измСрСния ΠΌΠ°Π³Π½ΠΈΡ‚Π½ΠΎΠ³ΠΎ ΠΏΠΎΡ‚ΠΎΠΊΠ° являСтся Π’Π΅Π±Π΅Ρ€ (Π’Π±), Ρ‚ΠΎ Π΅ΡΡ‚ΡŒ ΠΌΠ°Π³Π½ΠΈΡ‚Π½ΠΎΠ΅ ΠΏΠΎΠ»Π΅ Π½Π° Π΅Π΄ΠΈΠ½ΠΈΡ†Ρƒ ΠΏΠ»ΠΎΡ‰Π°Π΄ΠΈ, ΠΈΠ»ΠΈ Π’Π» / ΠΌ 2 . Π’Π΅Π±Π΅Ρ€ — это Ρ‚Π°ΠΊΠΆΠ΅ Π²ΠΎΠ»ΡŒΡ‚-сСкунда (Vs).

Π˜Π½Π΄ΡƒΡ†ΠΈΡ€ΠΎΠ²Π°Π½Π½Π°Ρ Π­Π”Π‘ фактичСски ΠΏΡ€ΠΎΠΏΠΎΡ€Ρ†ΠΈΠΎΠ½Π°Π»ΡŒΠ½Π° скорости измСнСния ΠΌΠ°Π³Π½ΠΈΡ‚Π½ΠΎΠ³ΠΎ ΠΏΠΎΡ‚ΠΎΠΊΠ° Ρ‡Π΅Ρ€Π΅Π· ΠΏΡ€ΠΎΠ²ΠΎΠ΄ΡΡ‰ΡƒΡŽ ΠΏΠ΅Ρ‚Π»ΡŽ.

Ξ΅βˆΞ”Ξ¦Ξ”tΞ΅βˆΞ”Ξ¦Ξ”t

20,28

НаконСц, для ΠΊΠ°Ρ‚ΡƒΡˆΠΊΠΈ, ΠΈΠ·Π³ΠΎΡ‚ΠΎΠ²Π»Π΅Π½Π½ΠΎΠΉ ΠΈΠ· ΠΏΠ΅Ρ‚Π΅Π»ΡŒ N , Π­Π”Π‘ Π² N Ρ€Π°Π· сильнСС, Ρ‡Π΅ΠΌ для ΠΎΠ΄ΠΈΠ½ΠΎΡ‡Π½ΠΎΠΉ ΠΏΠ΅Ρ‚Π»ΠΈ. Π’Π°ΠΊΠΈΠΌ ΠΎΠ±Ρ€Π°Π·ΠΎΠΌ, Π­Π”Π‘, индуцированная ΠΈΠ·ΠΌΠ΅Π½ΡΡŽΡ‰ΠΈΠΌΡΡ ΠΌΠ°Π³Π½ΠΈΡ‚Π½Ρ‹ΠΌ ΠΏΠΎΠ»Π΅ΠΌ Π² ΠΊΠ°Ρ‚ΡƒΡˆΠΊΠ΅ ΠΈΠ· ΠΏΠ΅Ρ‚Π΅Π»ΡŒ N , составляСт

Ρ∝NΞ”BcosΞΈΞ”tA.Ρ∝NΞ”BcosΞΈΞ”tA.

ПослСдний вопрос, Π½Π° ΠΊΠΎΡ‚ΠΎΡ€Ρ‹ΠΉ Π½ΡƒΠΆΠ½ΠΎ ΠΎΡ‚Π²Π΅Ρ‚ΠΈΡ‚ΡŒ, ΠΏΡ€Π΅ΠΆΠ΄Π΅ Ρ‡Π΅ΠΌ ΠΌΡ‹ смоТСм ΠΏΡ€Π΅ΠΎΠ±Ρ€Π°Π·ΠΎΠ²Π°Ρ‚ΡŒ ΠΏΡ€ΠΎΠΏΠΎΡ€Ρ†ΠΈΠΎΠ½Π°Π»ΡŒΠ½ΠΎΡΡ‚ΡŒ Π² ΡƒΡ€Π°Π²Π½Π΅Π½ΠΈΠ΅: Β«Π’ ΠΊΠ°ΠΊΠΎΠΌ Π½Π°ΠΏΡ€Π°Π²Π»Π΅Π½ΠΈΠΈ Ρ‚Π΅Ρ‡Π΅Ρ‚ Ρ‚ΠΎΠΊ?Β» Русский ΡƒΡ‡Π΅Π½Ρ‹ΠΉ Π“Π΅Π½Ρ€ΠΈΡ… Π›Π΅Π½Ρ† (1804–1865) объяснил, Ρ‡Ρ‚ΠΎ Ρ‚ΠΎΠΊ Ρ‚Π΅Ρ‡Π΅Ρ‚ Π² Ρ‚ΠΎΠΌ Π½Π°ΠΏΡ€Π°Π²Π»Π΅Π½ΠΈΠΈ, ΠΊΠΎΡ‚ΠΎΡ€ΠΎΠ΅ создаСт ΠΌΠ°Π³Π½ΠΈΡ‚Π½ΠΎΠ΅ ΠΏΠΎΠ»Π΅, ΠΊΠΎΡ‚ΠΎΡ€ΠΎΠ΅ пытаСтся ΡΠΎΡ…Ρ€Π°Π½ΠΈΡ‚ΡŒ постоянный ΠΏΠΎΡ‚ΠΎΠΊ Π² ΠΊΠΎΠ½Ρ‚ΡƒΡ€Π΅.НапримСр, снова рассмотрим рисунок 20.34. Π”Π²ΠΈΠΆΠ΅Π½ΠΈΠ΅ стСрТнСвого ΠΌΠ°Π³Π½ΠΈΡ‚Π° ΠΏΡ€ΠΈΠ²ΠΎΠ΄ΠΈΡ‚ ΠΊ ΡƒΠΌΠ΅Π½ΡŒΡˆΠ΅Π½ΠΈΡŽ количСства Π½Π°ΠΏΡ€Π°Π²Π»Π΅Π½Π½Ρ‹Ρ… Π²Π²Π΅Ρ€Ρ… силовых Π»ΠΈΠ½ΠΈΠΉ ΠΌΠ°Π³Π½ΠΈΡ‚Π½ΠΎΠ³ΠΎ поля, ΠΊΠΎΡ‚ΠΎΡ€Ρ‹Π΅ проходят Ρ‡Π΅Ρ€Π΅Π· ΠΏΠ΅Ρ‚Π»ΡŽ. Π‘Π»Π΅Π΄ΠΎΠ²Π°Ρ‚Π΅Π»ΡŒΠ½ΠΎ, Π² ΠΊΠΎΠ½Ρ‚ΡƒΡ€Π΅ гСнСрируСтся Π­Π”Π‘, которая направляСт Ρ‚ΠΎΠΊ Π² Π½Π°ΠΏΡ€Π°Π²Π»Π΅Π½ΠΈΠΈ, ΠΊΠΎΡ‚ΠΎΡ€ΠΎΠ΅ создаСт большС Π½Π°ΠΏΡ€Π°Π²Π»Π΅Π½Π½Ρ‹Ρ… Π²Π²Π΅Ρ€Ρ… Π»ΠΈΠ½ΠΈΠΉ ΠΌΠ°Π³Π½ΠΈΡ‚Π½ΠΎΠ³ΠΎ поля. Π˜ΡΠΏΠΎΠ»ΡŒΠ·ΡƒΡ ΠΏΡ€Π°Π²ΠΈΠ»ΠΎ ΠΏΡ€Π°Π²ΠΎΠΉ Ρ€ΡƒΠΊΠΈ, ΠΌΡ‹ Π²ΠΈΠ΄ΠΈΠΌ, Ρ‡Ρ‚ΠΎ этот Ρ‚ΠΎΠΊ Π΄ΠΎΠ»ΠΆΠ΅Π½ Ρ‚Π΅Ρ‡ΡŒ Π² Π½Π°ΠΏΡ€Π°Π²Π»Π΅Π½ΠΈΠΈ, ΠΏΠΎΠΊΠ°Π·Π°Π½Π½ΠΎΠΌ Π½Π° рисункС. Π§Ρ‚ΠΎΠ±Ρ‹ Π²Ρ‹Ρ€Π°Π·ΠΈΡ‚ΡŒ Ρ‚ΠΎΡ‚ Ρ„Π°ΠΊΡ‚, Ρ‡Ρ‚ΠΎ навСдСнная Π­Π”Π‘ дСйствуСт ΠΊΠ°ΠΊ противодСйствиС измСнСнию ΠΌΠ°Π³Π½ΠΈΡ‚Π½ΠΎΠ³ΠΎ ΠΏΠΎΡ‚ΠΎΠΊΠ° Ρ‡Π΅Ρ€Π΅Π· ΠΏΡ€ΠΎΠ²ΠΎΠ»ΠΎΡ‡Π½ΡƒΡŽ ΠΏΠ΅Ρ‚Π»ΡŽ, Π² ΠΏΡ€ΠΎΠΏΠΎΡ€Ρ†ΠΈΠΎΠ½Π°Π»ΡŒΠ½ΠΎΡΡ‚ΡŒ Ξ΅βˆΞ”Ξ¦ / Ξ”t вводится Π·Π½Π°ΠΊ минус.) Π²Π½ΡƒΡ‚Ρ€ΠΈ ΠΊΠ°Ρ‚ΡƒΡˆΠΊΠΈ, Π½Π°ΠΏΡ€Π°Π²Π»Π΅Π½Π½ΠΎΠΉ Π²Π»Π΅Π²ΠΎ. Π­Ρ‚ΠΎ Π±ΡƒΠ΄Π΅Ρ‚ ΠΏΡ€ΠΎΡ‚ΠΈΠ²ΠΎΠ΄Π΅ΠΉΡΡ‚Π²ΠΎΠ²Π°Ρ‚ΡŒ ΡƒΠ²Π΅Π»ΠΈΡ‡Π΅Π½ΠΈΡŽ ΠΌΠ°Π³Π½ΠΈΡ‚Π½ΠΎΠ³ΠΎ ΠΏΠΎΡ‚ΠΎΠΊΠ°, Π½Π°ΠΏΡ€Π°Π²Π»Π΅Π½Π½ΠΎΠ³ΠΎ Π²ΠΏΡ€Π°Π²ΠΎ. Π§Ρ‚ΠΎΠ±Ρ‹ ΡƒΠ²ΠΈΠ΄Π΅Ρ‚ΡŒ, Π² ΠΊΠ°ΠΊΠΎΠΌ Π½Π°ΠΏΡ€Π°Π²Π»Π΅Π½ΠΈΠΈ Π΄ΠΎΠ»ΠΆΠ΅Π½ Ρ‚Π΅Ρ‡ΡŒ Ρ‚ΠΎΠΊ, Π½Π°ΠΏΡ€Π°Π²ΡŒΡ‚Π΅ большой ΠΏΠ°Π»Π΅Ρ† ΠΏΡ€Π°Π²ΠΎΠΉ Ρ€ΡƒΠΊΠΈ Π² ΠΆΠ΅Π»Π°Π΅ΠΌΠΎΠΌ Π½Π°ΠΏΡ€Π°Π²Π»Π΅Π½ΠΈΠΈ ΠΌΠ°Π³Π½ΠΈΡ‚Π½ΠΎΠ³ΠΎ поля B β†’ ΠΊΠ°Ρ‚ΡƒΡˆΠΊΠ°, B β†’ ΠΊΠ°Ρ‚ΡƒΡˆΠΊΠ°, ΠΈ Ρ‚ΠΎΠΊ Π±ΡƒΠ΄Π΅Ρ‚ Ρ‚Π΅Ρ‡ΡŒ Π² Π½Π°ΠΏΡ€Π°Π²Π»Π΅Π½ΠΈΠΈ, ΡƒΠΊΠ°Π·Π°Π½Π½ΠΎΠΌ сгибаниСм Π²Π°ΡˆΠΈΡ… ΠΏΠ°Π»ΡŒΡ†Π΅Π² ΠΏΡ€Π°Π²ΠΎΠΉ Ρ€ΡƒΠΊΠΈ. Π­Ρ‚ΠΎ ΠΏΠΎΠΊΠ°Π·Π°Π½ΠΎ ΠΈΠ·ΠΎΠ±Ρ€Π°ΠΆΠ΅Π½ΠΈΠ΅ΠΌ ΠΏΡ€Π°Π²ΠΎΠΉ Ρ€ΡƒΠΊΠΈ Π² Π²Π΅Ρ€Ρ…Π½Π΅ΠΌ ряду рисунка 20.36. Π’Π°ΠΊΠΈΠΌ ΠΎΠ±Ρ€Π°Π·ΠΎΠΌ, Ρ‚ΠΎΠΊ Π΄ΠΎΠ»ΠΆΠ΅Π½ Ρ‚Π΅Ρ‡ΡŒ Π² Π½Π°ΠΏΡ€Π°Π²Π»Π΅Π½ΠΈΠΈ, ΠΏΠΎΠΊΠ°Π·Π°Π½Π½ΠΎΠΌ Π½Π° рисункС 4 (Π°).

На РисункС 4 (b) Π½Π°ΠΏΡ€Π°Π²Π»Π΅Π½ΠΈΠ΅, Π² ΠΊΠΎΡ‚ΠΎΡ€ΠΎΠΌ двиТСтся ΠΌΠ°Π³Π½ΠΈΡ‚, ΠΈΠ·ΠΌΠ΅Π½Π΅Π½ΠΎ Π½Π° ΠΎΠ±Ρ€Π°Ρ‚Π½ΠΎΠ΅.Π’ ΠΊΠ°Ρ‚ΡƒΡˆΠΊΠ΅ Π½Π°ΠΏΡ€Π°Π²Π»Π΅Π½Π½ΠΎΠ΅ Π²ΠΏΡ€Π°Π²ΠΎ ΠΌΠ°Π³Π½ΠΈΡ‚Π½ΠΎΠ΅ ΠΏΠΎΠ»Π΅ B β†’ magB β†’ mag ΠΈΠ·-Π·Π° двиТущСгося ΠΌΠ°Π³Π½ΠΈΡ‚Π° ΡƒΠΌΠ΅Π½ΡŒΡˆΠ°Π΅Ρ‚ΡΡ. Π—Π°ΠΊΠΎΠ½ Π›Π΅Π½Ρ†Π° гласит, Ρ‡Ρ‚ΠΎ, Ρ‡Ρ‚ΠΎΠ±Ρ‹ ΠΏΡ€ΠΎΡ‚ΠΈΠ²ΠΎΡΡ‚ΠΎΡΡ‚ΡŒ этому ΡƒΠΌΠ΅Π½ΡŒΡˆΠ΅Π½ΠΈΡŽ, Π­Π”Π‘ Π±ΡƒΠ΄Π΅Ρ‚ ΡƒΠΏΡ€Π°Π²Π»ΡΡ‚ΡŒ Ρ‚ΠΎΠΊΠΎΠΌ, ΠΊΠΎΡ‚ΠΎΡ€Ρ‹ΠΉ создаСт Π΄ΠΎΠΏΠΎΠ»Π½ΠΈΡ‚Π΅Π»ΡŒΠ½ΠΎΠ΅ Π½Π°ΠΏΡ€Π°Π²Π»Π΅Π½Π½ΠΎΠ΅ Π²ΠΏΡ€Π°Π²ΠΎ ΠΌΠ°Π³Π½ΠΈΡ‚Π½ΠΎΠ΅ ΠΏΠΎΠ»Π΅ B β†’ ΠΊΠ°Ρ‚ΡƒΡˆΠΊΠ° B β†’ ΠΊΠ°Ρ‚ΡƒΡˆΠΊΠ° Π² ΠΊΠ°Ρ‚ΡƒΡˆΠΊΠ΅. ΠžΠΏΡΡ‚ΡŒ ΠΆΠ΅, Π½Π°ΠΏΡ€Π°Π²ΡŒΡ‚Π΅ большой ΠΏΠ°Π»Π΅Ρ† ΠΏΡ€Π°Π²ΠΎΠΉ Ρ€ΡƒΠΊΠΈ Π² ΠΆΠ΅Π»Π°Π΅ΠΌΠΎΠΌ Π½Π°ΠΏΡ€Π°Π²Π»Π΅Π½ΠΈΠΈ ΠΌΠ°Π³Π½ΠΈΡ‚Π½ΠΎΠ³ΠΎ поля, ΠΈ Ρ‚ΠΎΠΊ Π±ΡƒΠ΄Π΅Ρ‚ Ρ‚Π΅Ρ‡ΡŒ Π² Π½Π°ΠΏΡ€Π°Π²Π»Π΅Π½ΠΈΠΈ, ΡƒΠΊΠ°Π·Π°Π½Π½ΠΎΠΌ сгибаниСм Π²Π°ΡˆΠΈΡ… ΠΏΠ°Π»ΡŒΡ†Π΅Π² ΠΏΡ€Π°Π²ΠΎΠΉ Ρ€ΡƒΠΊΠΈ (рис. 4 (b)).

НаконСц, Π½Π° РисункС 4 (c) ΠΌΠ°Π³Π½ΠΈΡ‚ ΠΏΠ΅Ρ€Π΅Π²Π΅Ρ€Π½ΡƒΡ‚, Ρ‚Π°ΠΊ Ρ‡Ρ‚ΠΎ ΡŽΠΆΠ½Ρ‹ΠΉ полюс находится Π±Π»ΠΈΠΆΠ΅ всСго ΠΊ ΠΊΠ°Ρ‚ΡƒΡˆΠΊΠ΅.Π’Π΅ΠΏΠ΅Ρ€ΡŒ ΠΌΠ°Π³Π½ΠΈΡ‚Π½ΠΎΠ΅ ΠΏΠΎΠ»Π΅ B β†’ magB β†’ mag Π½Π°ΠΏΡ€Π°Π²Π»Π΅Π½ΠΎ Π½Π° ΠΌΠ°Π³Π½ΠΈΡ‚, Π° Π½Π΅ Π½Π° ΠΊΠ°Ρ‚ΡƒΡˆΠΊΡƒ. Когда ΠΌΠ°Π³Π½ΠΈΡ‚ приблиТаСтся ΠΊ ΠΊΠ°Ρ‚ΡƒΡˆΠΊΠ΅, ΠΎΠ½ Π²Ρ‹Π·Ρ‹Π²Π°Π΅Ρ‚ ΡƒΠ²Π΅Π»ΠΈΡ‡Π΅Π½ΠΈΠ΅ Π½Π°ΠΏΡ€Π°Π²Π»Π΅Π½Π½ΠΎΠ³ΠΎ Π²Π»Π΅Π²ΠΎ ΠΌΠ°Π³Π½ΠΈΡ‚Π½ΠΎΠ³ΠΎ поля Π² ΠΊΠ°Ρ‚ΡƒΡˆΠΊΠ΅. Π—Π°ΠΊΠΎΠ½ Π›Π΅Π½Ρ†Π° гласит, Ρ‡Ρ‚ΠΎ Π­Π”Π‘, индуцированная Π² ΠΊΠ°Ρ‚ΡƒΡˆΠΊΠ΅, Π±ΡƒΠ΄Π΅Ρ‚ ΡƒΠΏΡ€Π°Π²Π»ΡΡ‚ΡŒ Ρ‚ΠΎΠΊΠΎΠΌ Π² Π½Π°ΠΏΡ€Π°Π²Π»Π΅Π½ΠΈΠΈ, ΠΊΠΎΡ‚ΠΎΡ€ΠΎΠ΅ создаСт ΠΌΠ°Π³Π½ΠΈΡ‚Π½ΠΎΠ΅ ΠΏΠΎΠ»Π΅, Π½Π°ΠΏΡ€Π°Π²Π»Π΅Π½Π½ΠΎΠ΅ Π²ΠΏΡ€Π°Π²ΠΎ. Π­Ρ‚ΠΎ Π±ΡƒΠ΄Π΅Ρ‚ ΠΏΡ€ΠΎΡ‚ΠΈΠ²ΠΎΠ΄Π΅ΠΉΡΡ‚Π²ΠΎΠ²Π°Ρ‚ΡŒ ΡƒΠ²Π΅Π»ΠΈΡ‡Π΅Π½ΠΈΡŽ ΠΌΠ°Π³Π½ΠΈΡ‚Π½ΠΎΠ³ΠΎ ΠΏΠΎΡ‚ΠΎΠΊΠ°, Π½Π°ΠΏΡ€Π°Π²Π»Π΅Π½Π½ΠΎΠ³ΠΎ Π²Π»Π΅Π²ΠΎ ΠΈΠ·-Π·Π° ΠΌΠ°Π³Π½ΠΈΡ‚Π°. ΠŸΠΎΠ²Ρ‚ΠΎΡ€Π½ΠΎΠ΅ использованиС ΠΏΡ€Π°Π²ΠΈΠ»Π° ΠΏΡ€Π°Π²ΠΎΠΉ Ρ€ΡƒΠΊΠΈ, ΠΊΠ°ΠΊ ΠΏΠΎΠΊΠ°Π·Π°Π½ΠΎ Π½Π° рисункС, ΠΏΠΎΠΊΠ°Π·Ρ‹Π²Π°Π΅Ρ‚, Ρ‡Ρ‚ΠΎ Ρ‚ΠΎΠΊ Π΄ΠΎΠ»ΠΆΠ΅Π½ Ρ‚Π΅Ρ‡ΡŒ Π² Π½Π°ΠΏΡ€Π°Π²Π»Π΅Π½ΠΈΠΈ, ΠΏΠΎΠΊΠ°Π·Π°Π½Π½ΠΎΠΌ Π½Π° рисункС 4 (c).

Рис. 20.36. Π—Π°ΠΊΠΎΠ½ Π›Π΅Π½Ρ†Π° гласит, Ρ‡Ρ‚ΠΎ Π­Π”Π‘, индуцированная ΠΌΠ°Π³Π½ΠΈΡ‚Π½Ρ‹ΠΌ ΠΏΠΎΠ»Π΅ΠΌ, Π±ΡƒΠ΄Π΅Ρ‚ ΡƒΠΏΡ€Π°Π²Π»ΡΡ‚ΡŒ Ρ‚ΠΎΠΊΠΎΠΌ, ΠΊΠΎΡ‚ΠΎΡ€Ρ‹ΠΉ сопротивляСтся измСнСнию ΠΌΠ°Π³Π½ΠΈΡ‚Π½ΠΎΠ³ΠΎ ΠΏΠΎΡ‚ΠΎΠΊΠ° Π² Ρ†Π΅ΠΏΠΈ. Π­Ρ‚ΠΎ ΠΏΠΎΠΊΠ°Π·Π°Π½ΠΎ Π½Π° панСлях (Π°) — (с) для Ρ€Π°Π·Π»ΠΈΡ‡Π½Ρ‹Ρ… ΠΎΡ€ΠΈΠ΅Π½Ρ‚Π°Ρ†ΠΈΠΉ ΠΈ скоростСй ΠΌΠ°Π³Π½ΠΈΡ‚Π°. ΠŸΡ€Π°Π²Ρ‹Π΅ Ρ€ΡƒΠΊΠΈ справа ΠΏΠΎΠΊΠ°Π·Ρ‹Π²Π°ΡŽΡ‚, ΠΊΠ°ΠΊ ΠΏΡ€ΠΈΠΌΠ΅Π½ΠΈΡ‚ΡŒ ΠΏΡ€Π°Π²ΠΈΠ»ΠΎ ΠΏΡ€Π°Π²ΠΎΠΉ Ρ€ΡƒΠΊΠΈ, Ρ‡Ρ‚ΠΎΠ±Ρ‹ Π½Π°ΠΉΡ‚ΠΈ, Π² ΠΊΠ°ΠΊΠΎΠΌ Π½Π°ΠΏΡ€Π°Π²Π»Π΅Π½ΠΈΠΈ Π½Π°Π²Π΅Π΄Π΅Π½Π½Ρ‹ΠΉ Ρ‚ΠΎΠΊ Ρ‚Π΅Ρ‡Π΅Ρ‚ Π²ΠΎΠΊΡ€ΡƒΠ³ ΠΊΠ°Ρ‚ΡƒΡˆΠΊΠΈ.

Π’ΠΈΡ€Ρ‚ΡƒΠ°Π»ΡŒΠ½Π°Ρ Ρ„ΠΈΠ·ΠΈΠΊΠ°

ЭлСктромагнитная лаборатория ЀарадСя

Π­Ρ‚ΠΎ ΠΌΠΎΠ΄Π΅Π»ΠΈΡ€ΠΎΠ²Π°Π½ΠΈΠ΅ ΠΏΡ€Π΅Π΄Π»Π°Π³Π°Π΅Ρ‚ нСсколько дСйствий.А ΠΏΠΎΠΊΠ° Ρ‰Π΅Π»ΠΊΠ½ΠΈΡ‚Π΅ Π²ΠΊΠ»Π°Π΄ΠΊΡƒ Pickup Coil, которая прСдставляСт собой стСрТнСвой ΠΌΠ°Π³Π½ΠΈΡ‚, ΠΊΠΎΡ‚ΠΎΡ€Ρ‹ΠΉ Π²Ρ‹ ΠΌΠΎΠΆΠ΅Ρ‚Π΅ ΠΏΠ΅Ρ€Π΅ΠΌΠ΅Ρ‰Π°Ρ‚ΡŒ Ρ‡Π΅Ρ€Π΅Π· ΠΊΠ°Ρ‚ΡƒΡˆΠΊΡƒ. Когда Π²Ρ‹ это сдСлаСтС, Π²Ρ‹ ΡƒΠ²ΠΈΠ΄ΠΈΡ‚Π΅, ΠΊΠ°ΠΊ элСктроны двиТутся Π² ΠΊΠ°Ρ‚ΡƒΡˆΠΊΠ΅, ΠΈ загорится Π»Π°ΠΌΠΏΠΎΡ‡ΠΊΠ°, ΠΈΠ»ΠΈ Π²ΠΎΠ»ΡŒΡ‚ΠΌΠ΅Ρ‚Ρ€ ΠΏΠΎΠΊΠ°ΠΆΠ΅Ρ‚ напряТСниС Π½Π° рСзисторС. ΠžΠ±Ρ€Π°Ρ‚ΠΈΡ‚Π΅ Π²Π½ΠΈΠΌΠ°Π½ΠΈΠ΅, Ρ‡Ρ‚ΠΎ Π²ΠΎΠ»ΡŒΡ‚ΠΌΠ΅Ρ‚Ρ€ позволяСт Π²Π°ΠΌ Π²ΠΈΠ΄Π΅Ρ‚ΡŒ Π·Π½Π°ΠΊ напряТСния ΠΏΡ€ΠΈ ΠΏΠ΅Ρ€Π΅ΠΌΠ΅Ρ‰Π΅Π½ΠΈΠΈ ΠΌΠ°Π³Π½ΠΈΡ‚Π°. Π’Ρ‹ Ρ‚Π°ΠΊΠΆΠ΅ ΠΌΠΎΠΆΠ΅Ρ‚Π΅ ΠΎΡΡ‚Π°Π²ΠΈΡ‚ΡŒ стСрТнСвой ΠΌΠ°Π³Π½ΠΈΡ‚ Π² ΠΏΠΎΠΊΠΎΠ΅ ΠΈ ΠΏΠ΅Ρ€Π΅ΠΌΠ΅ΡΡ‚ΠΈΡ‚ΡŒ ΠΊΠ°Ρ‚ΡƒΡˆΠΊΡƒ, хотя Π½Π°Π±Π»ΡŽΠ΄Π°Ρ‚ΡŒ Π·Π° Ρ€Π΅Π·ΡƒΠ»ΡŒΡ‚Π°Ρ‚Π°ΠΌΠΈ слоТнСС.

ΠŸΡ€ΠΎΠ²Π΅Ρ€ΠΊΠ° Π·Π°Ρ…Π²Π°Ρ‚Π°

Π‘ΠΎΡ€ΠΈΠ΅Π½Ρ‚ΠΈΡ€ΡƒΠΉΡ‚Π΅ стСрТнСвой ΠΌΠ°Π³Π½ΠΈΡ‚ Ρ‚Π°ΠΊ, Ρ‡Ρ‚ΠΎΠ±Ρ‹ сСвСрный полюс Π±Ρ‹Π» Π½Π°ΠΏΡ€Π°Π²Π»Π΅Π½ Π²ΠΏΡ€Π°Π²ΠΎ, ΠΈ помСститС ΠΏΡ€ΠΈΠ΅ΠΌΠ½ΡƒΡŽ ΠΊΠ°Ρ‚ΡƒΡˆΠΊΡƒ справа ΠΎΡ‚ стСрТнСвого ΠΌΠ°Π³Π½ΠΈΡ‚Π°.Π’Π΅ΠΏΠ΅Ρ€ΡŒ пСрСмСститС стСрТнСвой ΠΌΠ°Π³Π½ΠΈΡ‚ ΠΊ ΠΊΠ°Ρ‚ΡƒΡˆΠΊΠ΅ ΠΈ посмотритС, Π² ΠΊΠ°ΠΊΠΎΠΌ Π½Π°ΠΏΡ€Π°Π²Π»Π΅Π½ΠΈΠΈ двиТутся элСктроны. Π­Ρ‚ΠΎ такая ΠΆΠ΅ ситуация, ΠΊΠ°ΠΊ ΠΏΠΎΠΊΠ°Π·Π°Π½ΠΎ Π½ΠΈΠΆΠ΅. Π’ΠΎΠΊ ΠΏΡ€ΠΈ ΠΌΠΎΠ΄Π΅Π»ΠΈΡ€ΠΎΠ²Π°Π½ΠΈΠΈ Ρ‚Π΅Ρ‡Π΅Ρ‚ Π² Ρ‚ΠΎΠΌ ΠΆΠ΅ Π½Π°ΠΏΡ€Π°Π²Π»Π΅Π½ΠΈΠΈ, Ρ‡Ρ‚ΠΎ ΠΈ ΠΏΠΎΠΊΠ°Π·Π°Π½ΠΎ Π½ΠΈΠΆΠ΅? ΠžΠ±ΡŠΡΡΠ½ΠΈΡ‚Π΅, ΠΏΠΎΡ‡Π΅ΠΌΡƒ Π΄Π° ΠΈΠ»ΠΈ ΠΏΠΎΡ‡Π΅ΠΌΡƒ Π½Π΅Ρ‚.
  1. Π”Π°, Ρ‚ΠΎΠΊ Π² ΠΌΠΎΠ΄Π΅Π»ΠΈΡ€ΠΎΠ²Π°Π½ΠΈΠΈ Ρ‚Π΅Ρ‡Π΅Ρ‚, ΠΊΠ°ΠΊ ΠΏΠΎΠΊΠ°Π·Π°Π½ΠΎ, ΠΏΠΎΡ‚ΠΎΠΌΡƒ Ρ‡Ρ‚ΠΎ Π½Π°ΠΏΡ€Π°Π²Π»Π΅Π½ΠΈΠ΅ Ρ‚ΠΎΠΊΠ° ΠΏΡ€ΠΎΡ‚ΠΈΠ²ΠΎΠΏΠΎΠ»ΠΎΠΆΠ½ΠΎ Π½Π°ΠΏΡ€Π°Π²Π»Π΅Π½ΠΈΡŽ ΠΏΠΎΡ‚ΠΎΠΊΠ° элСктронов.
  2. НСт, Ρ‚ΠΎΠΊ Π² ΠΌΠΎΠ΄Π΅Π»ΠΈΡ€ΠΎΠ²Π°Π½ΠΈΠΈ Ρ‚Π΅Ρ‡Π΅Ρ‚ Π² ΠΏΡ€ΠΎΡ‚ΠΈΠ²ΠΎΠΏΠΎΠ»ΠΎΠΆΠ½ΠΎΠΌ Π½Π°ΠΏΡ€Π°Π²Π»Π΅Π½ΠΈΠΈ, ΠΏΠΎΡ‚ΠΎΠΌΡƒ Ρ‡Ρ‚ΠΎ Π½Π°ΠΏΡ€Π°Π²Π»Π΅Π½ΠΈΠ΅ Ρ‚ΠΎΠΊΠ° совпадаСт с Π½Π°ΠΏΡ€Π°Π²Π»Π΅Π½ΠΈΠ΅ΠΌ ΠΏΠΎΡ‚ΠΎΠΊΠ° элСктронов.

Watch Physics

НавСдСнный Ρ‚ΠΎΠΊ Π² ΠΏΡ€ΠΎΠ²ΠΎΠ΄Π΅

Π’ этом Π²ΠΈΠ΄Π΅ΠΎ ΠΎΠ±ΡŠΡΡΠ½ΡΠ΅Ρ‚ΡΡ, ΠΊΠ°ΠΊ ΠΌΠΎΠΆΠ½ΠΎ ΠΈΠ½Π΄ΡƒΡ†ΠΈΡ€ΠΎΠ²Π°Ρ‚ΡŒ Ρ‚ΠΎΠΊ Π² прямом ΠΏΡ€ΠΎΠ²ΠΎΠ΄Π΅, пСрСмСщая Π΅Π³ΠΎ Ρ‡Π΅Ρ€Π΅Π· ΠΌΠ°Π³Π½ΠΈΡ‚Π½ΠΎΠ΅ ΠΏΠΎΠ»Π΅. Π›Π΅ΠΊΡ‚ΠΎΡ€ ΠΈΡΠΏΠΎΠ»ΡŒΠ·ΡƒΠ΅Ρ‚ пСрСкрСстноС ΠΏΡ€ΠΎΠΈΠ·Π²Π΅Π΄Π΅Π½ΠΈΠ΅ , Ρ‚ΠΈΠΏ Π²Π΅ΠΊΡ‚ΠΎΡ€Π½ΠΎΠ³ΠΎ умноТСния. НС Π²ΠΎΠ»Π½ΡƒΠΉΡ‚Π΅ΡΡŒ, Ссли Π²Ρ‹ Π½Π΅ Π·Π½Π°ΠΊΠΎΠΌΡ‹ с этим, ΠΎΠ½ Π² основном ΠΎΠ±ΡŠΠ΅Π΄ΠΈΠ½ΡΠ΅Ρ‚ ΠΏΡ€Π°Π²ΠΈΠ»ΠΎ ΠΏΡ€Π°Π²ΠΎΠΉ Ρ€ΡƒΠΊΠΈ для опрСдСлСния силы, Π΄Π΅ΠΉΡΡ‚Π²ΡƒΡŽΡ‰Π΅ΠΉ Π½Π° заряды Π² ΠΏΡ€ΠΎΠ²ΠΎΠ΄Π΅, с ΡƒΡ€Π°Π²Π½Π΅Π½ΠΈΠ΅ΠΌ F = qvBsinΞΈ.F = qvBsinΞΈ.

ΠŸΡ€ΠΎΠ²Π΅Ρ€ΠΊΠ° Π·Π°Ρ…Π²Π°Ρ‚Π°

Какая Π­Π”Π‘ создаСтся ΠΏΠΎ прямому ΠΏΡ€ΠΎΠ²ΠΎΠ΄Ρƒ 0.Ρ‡Π΅Ρ€Π΅Π· ΠΎΠ΄Π½ΠΎΡ€ΠΎΠ΄Π½ΠΎΠ΅ ΠΌΠ°Π³Π½ΠΈΡ‚Π½ΠΎΠ΅ ΠΏΠΎΠ»Π΅ (0,30 Π’Π») αΊ‘ ? ΠŸΡ€ΠΎΠ²ΠΎΠ΄ Π»Π΅ΠΆΠΈΡ‚ Π² Π½Π°ΠΏΡ€Π°Π²Π»Π΅Π½ΠΈΠΈ Ε· . ΠšΡ€ΠΎΠΌΠ΅ Ρ‚ΠΎΠ³ΠΎ, ΠΊΠ°ΠΊΠΎΠΉ ΠΊΠΎΠ½Π΅Ρ† ΠΏΡ€ΠΎΠ²ΠΎΠ΄Π° ΠΈΠΌΠ΅Π΅Ρ‚ Π±ΠΎΠ»Π΅Π΅ высокий ΠΏΠΎΡ‚Π΅Π½Ρ†ΠΈΠ°Π» — ΠΏΡƒΡΡ‚ΡŒ Π½ΠΈΠΆΠ½ΠΈΠΉ ΠΊΠΎΠ½Π΅Ρ† ΠΏΡ€ΠΎΠ²ΠΎΠ΄Π° находится ΠΏΠΎΠ΄ ΡƒΠ³Π»ΠΎΠΌ y = 0, Π° Π²Π΅Ρ€Ρ…Π½ΠΈΠΉ ΠΊΠΎΠ½Π΅Ρ† — ΠΏΠΎΠ΄ ΡƒΠ³Π»ΠΎΠΌ y = 0,5 ΠΌ)?

  1. 0,15 Π’ ΠΈ Π½ΠΈΠΆΠ½ΠΈΠΉ ΠΊΠΎΠ½Π΅Ρ† ΠΏΡ€ΠΎΠ²ΠΎΠ΄Π° Π±ΡƒΠ΄Π΅Ρ‚ ΠΈΠΌΠ΅Ρ‚ΡŒ Π±ΠΎΠ»Π΅Π΅ высокий ΠΏΠΎΡ‚Π΅Π½Ρ†ΠΈΠ°Π»
  2. 0,15 Π’ ΠΈ Π²Π΅Ρ€Ρ…Π½ΠΈΠΉ ΠΊΠΎΠ½Π΅Ρ† ΠΏΡ€ΠΎΠ²ΠΎΠ΄Π° Π±ΡƒΠ΄Π΅Ρ‚ ΠΈΠΌΠ΅Ρ‚ΡŒ Π±ΠΎΠ»Π΅Π΅ высокий ΠΏΠΎΡ‚Π΅Π½Ρ†ΠΈΠ°Π»
  3. 0,075 Π’ ΠΈ Π½ΠΈΠΆΠ½ΠΈΠΉ ΠΊΠΎΠ½Π΅Ρ† ΠΏΡ€ΠΎΠ²ΠΎΠ΄Π° Π±ΡƒΠ΄Π΅Ρ‚ ΠΈΠΌΠ΅Ρ‚ΡŒ Π±ΠΎΠ»Π΅Π΅ высокий ΠΏΠΎΡ‚Π΅Π½Ρ†ΠΈΠ°Π»
  4. 0.075 Π’ ΠΈ Π½Π° Π²Π΅Ρ€Ρ…Π½Π΅ΠΌ ΠΊΠΎΠ½Ρ†Π΅ ΠΏΡ€ΠΎΠ²ΠΎΠ΄Π° Π±ΡƒΠ΄Π΅Ρ‚ Π±ΠΎΠ»Π΅Π΅ высокий ΠΏΠΎΡ‚Π΅Π½Ρ†ΠΈΠ°Π»

Π Π°Π±ΠΎΡ‡ΠΈΠΉ ΠΏΡ€ΠΈΠΌΠ΅Ρ€

Π­Π”Π‘, индуцированная Π² проводящСй ΠΊΠ°Ρ‚ΡƒΡˆΠΊΠ΅ двиТущимся ΠΌΠ°Π³Π½ΠΈΡ‚ΠΎΠΌ

ΠŸΡ€Π΅Π΄ΡΡ‚Π°Π²ΡŒΡ‚Π΅, Ρ‡Ρ‚ΠΎ ΠΌΠ°Π³Π½ΠΈΡ‚Π½ΠΎΠ΅ ΠΏΠΎΠ»Π΅ ΠΏΡ€ΠΎΡ…ΠΎΠ΄ΠΈΡ‚ Ρ‡Π΅Ρ€Π΅Π· ΠΊΠ°Ρ‚ΡƒΡˆΠΊΡƒ Π² Π½Π°ΠΏΡ€Π°Π²Π»Π΅Π½ΠΈΠΈ, ΡƒΠΊΠ°Π·Π°Π½Π½ΠΎΠΌ Π½Π° рисункС 20.37. Π”ΠΈΠ°ΠΌΠ΅Ρ‚Ρ€ ΠΊΠ°Ρ‚ΡƒΡˆΠΊΠΈ 2,0 см. Если ΠΌΠ°Π³Π½ΠΈΡ‚Π½ΠΎΠ΅ ΠΏΠΎΠ»Π΅ измСнится с 0,020 Π΄ΠΎ 0,010 Π’Π» Π·Π° 34 с, ΠΊΠ°ΠΊΠΎΠ²Ρ‹ Π½Π°ΠΏΡ€Π°Π²Π»Π΅Π½ΠΈΠ΅ ΠΈ Π²Π΅Π»ΠΈΡ‡ΠΈΠ½Π° ΠΈΠ½Π΄ΡƒΡ†ΠΈΡ€ΠΎΠ²Π°Π½Π½ΠΎΠ³ΠΎ Ρ‚ΠΎΠΊΠ°? ΠŸΡ€Π΅Π΄ΠΏΠΎΠ»ΠΎΠΆΠΈΠΌ, Ρ‡Ρ‚ΠΎ ΠΊΠ°Ρ‚ΡƒΡˆΠΊΠ° ΠΈΠΌΠ΅Π΅Ρ‚ сопротивлСниС 0,1 Ом.

Рисунок 20.37 ΠšΠ°Ρ‚ΡƒΡˆΠΊΠ°, Ρ‡Π΅Ρ€Π΅Π· ΠΊΠΎΡ‚ΠΎΡ€ΡƒΡŽ ΠΏΡ€ΠΎΡ…ΠΎΠ΄ΠΈΡ‚ ΠΌΠ°Π³Π½ΠΈΡ‚Π½ΠΎΠ΅ ΠΏΠΎΠ»Π΅ B .

БтратСгия

Π˜ΡΠΏΠΎΠ»ΡŒΠ·ΡƒΠΉΡ‚Π΅ ΡƒΡ€Π°Π²Π½Π΅Π½ΠΈΠ΅ Ξ΅ = βˆ’NΔΦ / Ξ”tΞ΅ = βˆ’NΔΦ / Ξ”t, Ρ‡Ρ‚ΠΎΠ±Ρ‹ Π½Π°ΠΉΡ‚ΠΈ Π½Π°Π²Π΅Π΄Π΅Π½Π½ΡƒΡŽ Π­Π”Π‘ Π² ΠΊΠ°Ρ‚ΡƒΡˆΠΊΠ΅, Π³Π΄Π΅ Ξ”t = 34sΞ”t = 34s. ΠŸΠΎΠ΄ΡΡ‡ΠΈΡ‚Π°Π² количСство ΠΏΠ΅Ρ‚Π΅Π»ΡŒ Π² солСноидС, Π½Π°Ρ…ΠΎΠ΄ΠΈΠΌ, Ρ‡Ρ‚ΠΎ Ρƒ Π½Π΅Π³ΠΎ 16 ΠΏΠ΅Ρ‚Π΅Π»ΡŒ, поэтому N = 16.N = 16. Π˜ΡΠΏΠΎΠ»ΡŒΠ·ΡƒΠΉΡ‚Π΅ ΡƒΡ€Π°Π²Π½Π΅Π½ΠΈΠ΅ Ξ¦ = BAcosΞΈΞ¦ = BAcosΞΈ для расчСта ΠΌΠ°Π³Π½ΠΈΡ‚Π½ΠΎΠ³ΠΎ ΠΏΠΎΡ‚ΠΎΠΊΠ°

Ξ¦ = BAcosΞΈ = BΟ€ (d2) 2, Ξ¦ = BAcosΞΈ = BΟ€ (d2) 2,

20,30

, Π³Π΄Π΅ d — Π΄ΠΈΠ°ΠΌΠ΅Ρ‚Ρ€ солСноида, Π° ΠΌΡ‹ использовали cos0 Β° = 1. cos0 Β° = 1. ΠŸΠΎΡΠΊΠΎΠ»ΡŒΠΊΡƒ ΠΏΠ»ΠΎΡ‰Π°Π΄ΡŒ солСноида Π½Π΅ мСняСтся, ΠΈΠ·ΠΌΠ΅Π½Π΅Π½ΠΈΠ΅ ΠΌΠ°Π³Π½ΠΈΡ‚Π½ΠΎΠ³ΠΎ ΠΏΠΎΡ‚ΠΎΠΊΠ° Ρ‡Π΅Ρ€Π΅Π· солСноид составляСт

ΔΦ = Ξ”BΟ€ (d2) 2.ΔΦ = Ξ”BΟ€ (d2) 2.

20,31

Найдя Π­Π”Π‘, ΠΌΡ‹ ΠΌΠΎΠΆΠ΅ΠΌ ΠΈΡΠΏΠΎΠ»ΡŒΠ·ΠΎΠ²Π°Ρ‚ΡŒ Π·Π°ΠΊΠΎΠ½ Ома, Ξ΅ = IR, Ξ΅ = IR, Ρ‡Ρ‚ΠΎΠ±Ρ‹ Π½Π°ΠΉΡ‚ΠΈ Ρ‚ΠΎΠΊ.

НаконСц, Π·Π°ΠΊΠΎΠ½ Π›Π΅Π½Ρ†Π° гласит, Ρ‡Ρ‚ΠΎ Ρ‚ΠΎΠΊ Π΄ΠΎΠ»ΠΆΠ΅Π½ ΡΠΎΠ·Π΄Π°Π²Π°Ρ‚ΡŒ ΠΌΠ°Π³Π½ΠΈΡ‚Π½ΠΎΠ΅ ΠΏΠΎΠ»Π΅, ΠΊΠΎΡ‚ΠΎΡ€ΠΎΠ΅ прСпятствуСт ΡƒΠΌΠ΅Π½ΡŒΡˆΠ΅Π½ΠΈΡŽ ΠΏΡ€ΠΈΠ»ΠΎΠΆΠ΅Π½Π½ΠΎΠ³ΠΎ ΠΌΠ°Π³Π½ΠΈΡ‚Π½ΠΎΠ³ΠΎ поля. Π’Π°ΠΊΠΈΠΌ ΠΎΠ±Ρ€Π°Π·ΠΎΠΌ, Ρ‚ΠΎΠΊ Π΄ΠΎΠ»ΠΆΠ΅Π½ ΡΠΎΠ·Π΄Π°Π²Π°Ρ‚ΡŒ ΠΌΠ°Π³Π½ΠΈΡ‚Π½ΠΎΠ΅ ΠΏΠΎΠ»Π΅ справа.

РСшСниС

ОбъСдинСниС ΡƒΡ€Π°Π²Π½Π΅Π½ΠΈΠΉ Ξ΅ = βˆ’NΔΦ / Ξ”tΞ΅ = βˆ’NΔΦ / Ξ”t ΠΈ Ξ¦ = BAcosΞΈΞ¦ = BAcosΞΈ Π΄Π°Π΅Ρ‚

Ξ΅ = βˆ’NΔΦΔt = βˆ’NΞ”BΟ€d24Ξ”t.Ξ΅ = βˆ’NΔΦΔt = βˆ’NΞ”BΟ€d24Ξ”t.

20,32

РСшая Π·Π°ΠΊΠΎΠ½ Ома для Ρ‚ΠΎΠΊΠ° ΠΈ ΠΈΡΠΏΠΎΠ»ΡŒΠ·ΡƒΡ этот Ρ€Π΅Π·ΡƒΠ»ΡŒΡ‚Π°Ρ‚, ΠΏΠΎΠ»ΡƒΡ‡Π°Π΅ΠΌ

I = Ξ΅R = βˆ’NΞ”BΟ€d24RΞ”t = βˆ’16 (βˆ’0,010T) Ο€ (0,020 ΠΌ) 24 (0,10 Ом) (34 с) = 15 мкА.I = Ξ΅R = βˆ’NΞ”BΟ€d24RΞ”t = βˆ’16 (βˆ’0,010T) Ο€ (0,020 ΠΌ) 24 (0,10 Ом) (34 с) = 15 мкА.

20.33

Π—Π°ΠΊΠΎΠ½ Π›Π΅Π½Ρ†Π° гласит, Ρ‡Ρ‚ΠΎ Ρ‚ΠΎΠΊ Π΄ΠΎΠ»ΠΆΠ΅Π½ ΡΠΎΠ·Π΄Π°Π²Π°Ρ‚ΡŒ ΠΌΠ°Π³Π½ΠΈΡ‚Π½ΠΎΠ΅ ΠΏΠΎΠ»Π΅ справа. Π’Π°ΠΊΠΈΠΌ ΠΎΠ±Ρ€Π°Π·ΠΎΠΌ, ΠΌΡ‹ направляСм большой ΠΏΠ°Π»Π΅Ρ† ΠΏΡ€Π°Π²ΠΎΠΉ Ρ€ΡƒΠΊΠΈ Π²ΠΏΡ€Π°Π²ΠΎ ΠΈ сТимаСм ΠΏΠ°Π»ΡŒΡ†Ρ‹ ΠΏΡ€Π°Π²ΠΎΠΉ Ρ€ΡƒΠΊΠΈ Π²ΠΎΠΊΡ€ΡƒΠ³ солСноида. Π’ΠΎΠΊ Π΄ΠΎΠ»ΠΆΠ΅Π½ Ρ‚Π΅Ρ‡ΡŒ Π² Ρ‚ΠΎΠΌ Π½Π°ΠΏΡ€Π°Π²Π»Π΅Π½ΠΈΠΈ, Π² ΠΊΠΎΡ‚ΠΎΡ€ΠΎΠΌ ΡƒΠΊΠ°Π·Ρ‹Π²Π°ΡŽΡ‚ наши ΠΏΠ°Π»ΡŒΡ†Ρ‹, поэтому ΠΎΠ½ Π²Ρ…ΠΎΠ΄ΠΈΡ‚ Π² Π»Π΅Π²Ρ‹ΠΉ ΠΊΠΎΠ½Π΅Ρ† солСноида ΠΈ Π²Ρ‹Ρ…ΠΎΠ΄ΠΈΡ‚ ΠΈΠ· ΠΏΡ€Π°Π²ΠΎΠ³ΠΎ ΠΊΠΎΠ½Ρ†Π°.

ΠžΠ±ΡΡƒΠΆΠ΄Π΅Π½ΠΈΠ΅

Π”Π°Π²Π°ΠΉΡ‚Π΅ посмотрим, ΠΈΠΌΠ΅Π΅Ρ‚ Π»ΠΈ смысл Π·Π½Π°ΠΊ минус Π² Π·Π°ΠΊΠΎΠ½Π΅ ΠΈΠ½Π΄ΡƒΠΊΡ†ΠΈΠΈ ЀарадСя. ΠžΠΏΡ€Π΅Π΄Π΅Π»ΠΈΡ‚Π΅ Π½Π°ΠΏΡ€Π°Π²Π»Π΅Π½ΠΈΠ΅ ΠΌΠ°Π³Π½ΠΈΡ‚Π½ΠΎΠ³ΠΎ поля ΠΊΠ°ΠΊ ΠΏΠΎΠ»ΠΎΠΆΠΈΡ‚Π΅Π»ΡŒΠ½ΠΎΠ΅. Π­Ρ‚ΠΎ ΠΎΠ·Π½Π°Ρ‡Π°Π΅Ρ‚, Ρ‡Ρ‚ΠΎ ΠΈΠ·ΠΌΠ΅Π½Π΅Π½ΠΈΠ΅ ΠΌΠ°Π³Π½ΠΈΡ‚Π½ΠΎΠ³ΠΎ поля ΠΎΡ‚Ρ€ΠΈΡ†Π°Ρ‚Π΅Π»ΡŒΠ½ΠΎΠ΅, ΠΊΠ°ΠΊ ΠΌΡ‹ ΠΎΠ±Π½Π°Ρ€ΡƒΠΆΠΈΠ»ΠΈ Π²Ρ‹ΡˆΠ΅. Π—Π½Π°ΠΊ минус Π² Π·Π°ΠΊΠΎΠ½Π΅ ΠΈΠ½Π΄ΡƒΠΊΡ†ΠΈΠΈ ЀарадСя ΠΎΡ‚Ρ€ΠΈΡ†Π°Π΅Ρ‚ ΠΎΡ‚Ρ€ΠΈΡ†Π°Ρ‚Π΅Π»ΡŒΠ½ΠΎΠ΅ ΠΈΠ·ΠΌΠ΅Π½Π΅Π½ΠΈΠ΅ ΠΌΠ°Π³Π½ΠΈΡ‚Π½ΠΎΠ³ΠΎ поля, оставляя Π½Π°ΠΌ ΠΏΠΎΠ»ΠΎΠΆΠΈΡ‚Π΅Π»ΡŒΠ½Ρ‹ΠΉ Ρ‚ΠΎΠΊ. Π‘Π»Π΅Π΄ΠΎΠ²Π°Ρ‚Π΅Π»ΡŒΠ½ΠΎ, Ρ‚ΠΎΠΊ Π΄ΠΎΠ»ΠΆΠ΅Π½ Ρ‚Π΅Ρ‡ΡŒ Π² Π½Π°ΠΏΡ€Π°Π²Π»Π΅Π½ΠΈΠΈ ΠΌΠ°Π³Π½ΠΈΡ‚Π½ΠΎΠ³ΠΎ поля, Ρ‡Ρ‚ΠΎ ΠΌΡ‹ ΠΈ ΠΎΠ±Π½Π°Ρ€ΡƒΠΆΠΈΠ»ΠΈ.

Π’Π΅ΠΏΠ΅Ρ€ΡŒ ΠΏΠΎΠΏΡ€ΠΎΠ±ΡƒΠΉΡ‚Π΅ ΠΎΠΏΡ€Π΅Π΄Π΅Π»ΠΈΡ‚ΡŒ ΠΏΠΎΠ»ΠΎΠΆΠΈΡ‚Π΅Π»ΡŒΠ½ΠΎΠ΅ Π½Π°ΠΏΡ€Π°Π²Π»Π΅Π½ΠΈΠ΅ ΠΊΠ°ΠΊ Π½Π°ΠΏΡ€Π°Π²Π»Π΅Π½ΠΈΠ΅, ΠΏΡ€ΠΎΡ‚ΠΈΠ²ΠΎΠΏΠΎΠ»ΠΎΠΆΠ½ΠΎΠ΅ Π½Π°ΠΏΡ€Π°Π²Π»Π΅Π½ΠΈΡŽ ΠΌΠ°Π³Π½ΠΈΡ‚Π½ΠΎΠ³ΠΎ поля, Ρ‚ΠΎ Π΅ΡΡ‚ΡŒ ΠΏΠΎΠ»ΠΎΠΆΠΈΡ‚Π΅Π»ΡŒΠ½ΠΎΠ΅ Π½Π°ΠΏΡ€Π°Π²Π»Π΅Π½ΠΈΠ΅ находится слСва Π½Π° рисункС 20.37. Π’ этом случаС Π²Ρ‹ ΠΎΠ±Π½Π°Ρ€ΡƒΠΆΠΈΡ‚Π΅ ΠΎΡ‚Ρ€ΠΈΡ†Π°Ρ‚Π΅Π»ΡŒΠ½Ρ‹ΠΉ Ρ‚ΠΎΠΊ. Но ΠΏΠΎΡΠΊΠΎΠ»ΡŒΠΊΡƒ ΠΏΠΎΠ»ΠΎΠΆΠΈΡ‚Π΅Π»ΡŒΠ½ΠΎΠ΅ Π½Π°ΠΏΡ€Π°Π²Π»Π΅Π½ΠΈΠ΅ находится Π²Π»Π΅Π²ΠΎ, ΠΎΡ‚Ρ€ΠΈΡ†Π°Ρ‚Π΅Π»ΡŒΠ½Ρ‹ΠΉ Ρ‚ΠΎΠΊ Π΄ΠΎΠ»ΠΆΠ΅Π½ Ρ‚Π΅Ρ‡ΡŒ Π²ΠΏΡ€Π°Π²ΠΎ, Ρ‡Ρ‚ΠΎ снова согласуСтся с Ρ‚Π΅ΠΌ, Ρ‡Ρ‚ΠΎ ΠΌΡ‹ ΠΎΠ±Π½Π°Ρ€ΡƒΠΆΠΈΠ»ΠΈ с ΠΏΠΎΠΌΠΎΡ‰ΡŒΡŽ Π·Π°ΠΊΠΎΠ½Π° Π›Π΅Π½Ρ†Π°.

Π Π°Π±ΠΎΡ‡ΠΈΠΉ ΠΏΡ€ΠΈΠΌΠ΅Ρ€

ΠœΠ°Π³Π½ΠΈΡ‚Π½Π°Ρ индукция ΠΈΠ·-Π·Π° измСнСния Ρ€Π°Π·ΠΌΠ΅Ρ€Π° Ρ†Π΅ΠΏΠΈ

Π‘Ρ…Π΅ΠΌΠ°, показанная Π½Π° рисункС 20.38, состоит ΠΈΠ· U-ΠΎΠ±Ρ€Π°Π·Π½ΠΎΠ³ΠΎ ΠΏΡ€ΠΎΠ²ΠΎΠ΄Π° с рСзистором, ΠΊΠΎΠ½Ρ†Ρ‹ ΠΊΠΎΡ‚ΠΎΡ€ΠΎΠ³ΠΎ соСдинСны ΡΠΊΠΎΠ»ΡŒΠ·ΡΡ‰ΠΈΠΌ токопроводящим стСрТнСм. ΠœΠ°Π³Π½ΠΈΡ‚Π½ΠΎΠ΅ ΠΏΠΎΠ»Π΅, Π·Π°ΠΏΠΎΠ»Π½ΡΡŽΡ‰Π΅Π΅ ΠΎΠ±Π»Π°ΡΡ‚ΡŒ, ΠΎΠ³Ρ€Π°Π½ΠΈΡ‡Π΅Π½Π½ΡƒΡŽ ΠΊΠΎΠ½Ρ‚ΡƒΡ€ΠΎΠΌ, ΠΈΠΌΠ΅Π΅Ρ‚ постоянноС Π·Π½Π°Ρ‡Π΅Π½ΠΈΠ΅ 0.01 T. Если ΡΡ‚Π΅Ρ€ΠΆΠ΅Π½ΡŒ тянут Π²ΠΏΡ€Π°Π²ΠΎ со ΡΠΊΠΎΡ€ΠΎΡΡ‚ΡŒΡŽ v = 0,50 ΠΌ / с, v = 0,50 ΠΌ / с, ΠΊΠ°ΠΊΠΎΠΉ Ρ‚ΠΎΠΊ индуцируСтся Π² Ρ†Π΅ΠΏΠΈ ΠΈ Π² ΠΊΠ°ΠΊΠΎΠΌ Π½Π°ΠΏΡ€Π°Π²Π»Π΅Π½ΠΈΠΈ ΠΎΠ½ Ρ‚Π΅Ρ‡Π΅Ρ‚?

Рисунок 20.38 Π‘Ρ…Π΅ΠΌΠ° ΠΏΠΎΠ»Π·ΡƒΠ½ΠΊΠ°. ΠœΠ°Π³Π½ΠΈΡ‚Π½ΠΎΠ΅ ΠΏΠΎΠ»Π΅ постоянно, ΠΈ ΡˆΡ‚ΠΎΠΊ тянСтся Π²ΠΏΡ€Π°Π²ΠΎ со ΡΠΊΠΎΡ€ΠΎΡΡ‚ΡŒΡŽ v . Π˜Π·ΠΌΠ΅Π½ΡΡŽΡ‰Π°ΡΡΡ ΠΎΠ±Π»Π°ΡΡ‚ΡŒ, Π·Π°ΠΊΠ»ΡŽΡ‡Π΅Π½Π½Π°Ρ Π² Ρ†Π΅ΠΏΠΈ, Π²Ρ‹Π·Ρ‹Π²Π°Π΅Ρ‚ Π² Ρ†Π΅ΠΏΠΈ Π­Π”Π‘.

БтратСгия

ΠœΡ‹ снова ΠΈΡΠΏΠΎΠ»ΡŒΠ·ΡƒΠ΅ΠΌ Π·Π°ΠΊΠΎΠ½ ΠΈΠ½Π΄ΡƒΠΊΡ†ΠΈΠΈ ЀарадСя, E = βˆ’NΔΦΔt, E = βˆ’NΔΦΔt, хотя Π½Π° этот Ρ€Π°Π· ΠΌΠ°Π³Π½ΠΈΡ‚Π½ΠΎΠ΅ ΠΏΠΎΠ»Π΅ остаСтся постоянным ΠΈ ΠΏΠ»ΠΎΡ‰Π°Π΄ΡŒ, ограничСнная ΠΊΠΎΠ½Ρ‚ΡƒΡ€ΠΎΠΌ, измСняСтся.Π‘Ρ…Π΅ΠΌΠ° состоит ΠΈΠ· ΠΎΠ΄Π½ΠΎΠ³ΠΎ ΠΊΠΎΠ½Ρ‚ΡƒΡ€Π°, поэтому N = 1.N = 1. Π‘ΠΊΠΎΡ€ΠΎΡΡ‚ΡŒ измСнСния ΠΏΠ»ΠΎΡ‰Π°Π΄ΠΈ Ξ”AΞ”t = vβ„“.Ξ”AΞ”t = vβ„“. Π’Π°ΠΊΠΈΠΌ ΠΎΠ±Ρ€Π°Π·ΠΎΠΌ, ΡΠΊΠΎΡ€ΠΎΡΡ‚ΡŒ измСнСния ΠΌΠ°Π³Π½ΠΈΡ‚Π½ΠΎΠ³ΠΎ ΠΏΠΎΡ‚ΠΎΠΊΠ° составляСт

ΔΦΔt = Ξ” (BAcosΞΈ) Ξ”t = BΞ”AΞ”t = Bvβ„“, ΔΦΔt = Ξ” (BAcosΞΈ) Ξ”t = BΞ”AΞ”t = Bvβ„“,

20,34

, Π³Π΄Π΅ ΠΌΡ‹ использовали Ρ‚ΠΎΡ‚ Ρ„Π°ΠΊΡ‚, Ρ‡Ρ‚ΠΎ ΡƒΠ³ΠΎΠ» ΞΈΞΈ ΠΌΠ΅ΠΆΠ΄Ρƒ Π²Π΅ΠΊΡ‚ΠΎΡ€ΠΎΠΌ ΠΏΠ»ΠΎΡ‰Π°Π΄ΠΈ ΠΈ ΠΌΠ°Π³Π½ΠΈΡ‚Π½Ρ‹ΠΌ ΠΏΠΎΠ»Π΅ΠΌ Ρ€Π°Π²Π΅Π½ 0 Β°. Зная Π­Π”Π‘, ΠΌΡ‹ ΠΌΠΎΠΆΠ΅ΠΌ Π½Π°ΠΉΡ‚ΠΈ Ρ‚ΠΎΠΊ, ΠΈΡΠΏΠΎΠ»ΡŒΠ·ΡƒΡ Π·Π°ΠΊΠΎΠ½ Ома. Π§Ρ‚ΠΎΠ±Ρ‹ Π½Π°ΠΉΡ‚ΠΈ Π½Π°ΠΏΡ€Π°Π²Π»Π΅Π½ΠΈΠ΅ Ρ‚ΠΎΠΊΠ°, ΠΌΡ‹ примСняСм Π·Π°ΠΊΠΎΠ½ Π›Π΅Π½Ρ†Π°.

РСшСниС

Π—Π°ΠΊΠΎΠ½ ΠΈΠ½Π΄ΡƒΠΊΡ†ΠΈΠΈ ЀарадСя Π΄Π°Π΅Ρ‚

E = βˆ’NΔΦΔt = βˆ’Bvβ„“.E = βˆ’NΔΦΔt = βˆ’Bvβ„“.

20,35

РСшСниС Π·Π°ΠΊΠΎΠ½Π° Ома для Ρ‚ΠΎΠΊΠ° ΠΈ использованиС ΠΏΡ€Π΅Π΄Ρ‹Π΄ΡƒΡ‰Π΅Π³ΠΎ Ρ€Π΅Π·ΡƒΠ»ΡŒΡ‚Π°Ρ‚Π° для Π­Π”Π‘ Π΄Π°Π΅Ρ‚

I = ER = βˆ’Bvβ„“R = — (0,010T) (0,50 ΠΌ / с) (0,10 ΠΌ) 20Ξ© = 25 ΠΌΠΊA I = ER = βˆ’Bvβ„“R = — (0,010T) (0,50 ΠΌ / с) (0,10 ΠΌ) 20Ξ© = 25 мкА.

20,36

По ΠΌΠ΅Ρ€Π΅ скольТСния стСрТня Π²ΠΏΡ€Π°Π²ΠΎ ΠΌΠ°Π³Π½ΠΈΡ‚Π½Ρ‹ΠΉ ΠΏΠΎΡ‚ΠΎΠΊ, проходящий Ρ‡Π΅Ρ€Π΅Π· ΠΊΠΎΠ½Ρ‚ΡƒΡ€, увСличиваСтся. Π—Π°ΠΊΠΎΠ½ Π›Π΅Π½Ρ†Π° Π³ΠΎΠ²ΠΎΡ€ΠΈΡ‚ Π½Π°ΠΌ, Ρ‡Ρ‚ΠΎ ΠΈΠ½Π΄ΡƒΡ†ΠΈΡ€ΠΎΠ²Π°Π½Π½Ρ‹ΠΉ Ρ‚ΠΎΠΊ создаст ΠΌΠ°Π³Π½ΠΈΡ‚Π½ΠΎΠ΅ ΠΏΠΎΠ»Π΅, ΠΊΠΎΡ‚ΠΎΡ€ΠΎΠ΅ Π±ΡƒΠ΄Π΅Ρ‚ ΠΏΡ€ΠΎΡ‚ΠΈΠ²ΠΎΠ΄Π΅ΠΉΡΡ‚Π²ΠΎΠ²Π°Ρ‚ΡŒ этому ΡƒΠ²Π΅Π»ΠΈΡ‡Π΅Π½ΠΈΡŽ. Π’Π°ΠΊΠΈΠΌ ΠΎΠ±Ρ€Π°Π·ΠΎΠΌ, ΠΌΠ°Π³Π½ΠΈΡ‚Π½ΠΎΠ΅ ΠΏΠΎΠ»Π΅, создаваСмоС ΠΈΠ½Π΄ΡƒΡ†ΠΈΡ€ΠΎΠ²Π°Π½Π½Ρ‹ΠΌ Ρ‚ΠΎΠΊΠΎΠΌ, Π΄ΠΎΠ»ΠΆΠ½ΠΎ Π½Π°Ρ…ΠΎΠ΄ΠΈΡ‚ΡŒΡΡ Π½Π° страницС.Π‘Π³ΠΈΠ±Π°Π½ΠΈΠ΅ ΠΏΠ΅Ρ‚Π»ΠΈ ΠΏΠ°Π»ΡŒΡ†Π°ΠΌΠΈ ΠΏΡ€Π°Π²ΠΎΠΉ Ρ€ΡƒΠΊΠΈ ΠΏΠΎ часовой стрСлкС заставляСт большой ΠΏΠ°Π»Π΅Ρ† ΠΏΡ€Π°Π²ΠΎΠΉ Ρ€ΡƒΠΊΠΈ ΡƒΠΊΠ°Π·Ρ‹Π²Π°Ρ‚ΡŒ Π½Π° страницу, Ρ‡Ρ‚ΠΎ являСтся ΠΆΠ΅Π»Π°Π΅ΠΌΡ‹ΠΌ Π½Π°ΠΏΡ€Π°Π²Π»Π΅Π½ΠΈΠ΅ΠΌ ΠΌΠ°Π³Π½ΠΈΡ‚Π½ΠΎΠ³ΠΎ поля. Π’Π°ΠΊΠΈΠΌ ΠΎΠ±Ρ€Π°Π·ΠΎΠΌ, Ρ‚ΠΎΠΊ Π΄ΠΎΠ»ΠΆΠ΅Π½ Ρ‚Π΅Ρ‡ΡŒ ΠΏΠΎ Ρ†Π΅ΠΏΠΈ ΠΏΠΎ часовой стрСлкС.

ΠžΠ±ΡΡƒΠΆΠ΄Π΅Π½ΠΈΠ΅

БохраняСтся Π»ΠΈ энСргия Π² этой Ρ†Π΅ΠΏΠΈ? Π’Π½Π΅ΡˆΠ½ΠΈΠΉ Π°Π³Π΅Π½Ρ‚ Π΄ΠΎΠ»ΠΆΠ΅Π½ Ρ‚ΡΠ½ΡƒΡ‚ΡŒ ΡΡ‚Π΅Ρ€ΠΆΠ΅Π½ΡŒ с достаточной силой, Ρ‡Ρ‚ΠΎΠ±Ρ‹ просто ΡƒΡ€Π°Π²Π½ΠΎΠ²Π΅ΡΠΈΡ‚ΡŒ силу Π½Π° ΠΏΡ€ΠΎΠ²ΠΎΠ΄Π΅ с Ρ‚ΠΎΠΊΠΎΠΌ Π² ΠΌΠ°Π³Π½ΠΈΡ‚Π½ΠΎΠΌ ΠΏΠΎΠ»Π΅ — вспомнитС, Ρ‡Ρ‚ΠΎ F = Iβ„“BsinΞΈ.F = Iβ„“BsinΞΈ. Π‘ΠΊΠΎΡ€ΠΎΡΡ‚ΡŒ, с ΠΊΠΎΡ‚ΠΎΡ€ΠΎΠΉ эта сила дСйствуСт Π½Π° ΡΡ‚Π΅Ρ€ΠΆΠ΅Π½ΡŒ, Π΄ΠΎΠ»ΠΆΠ½Π° ΡƒΡ€Π°Π²Π½ΠΎΠ²Π΅ΡˆΠΈΠ²Π°Ρ‚ΡŒΡΡ ΡΠΊΠΎΡ€ΠΎΡΡ‚ΡŒΡŽ, с ΠΊΠΎΡ‚ΠΎΡ€ΠΎΠΉ Ρ†Π΅ΠΏΡŒ рассСиваСт ΠΌΠΎΡ‰Π½ΠΎΡΡ‚ΡŒ.Π˜ΡΠΏΠΎΠ»ΡŒΠ·ΡƒΡ F = Iβ„“BsinΞΈ, F = Iβ„“BsinΞΈ, сила, нСобходимая для протягивания ΠΏΡ€ΠΎΠ²ΠΎΠ»ΠΎΠΊΠΈ с постоянной ΡΠΊΠΎΡ€ΠΎΡΡ‚ΡŒΡŽ v , Ρ€Π°Π²Π½Π°

. Fpull = Iβ„“BsinΞΈ = Iβ„“B, Fpull = Iβ„“BsinΞΈ = Iβ„“B,

20,37

, Π³Π΄Π΅ ΠΌΡ‹ использовали Ρ‚ΠΎΡ‚ Ρ„Π°ΠΊΡ‚, Ρ‡Ρ‚ΠΎ ΡƒΠ³ΠΎΠ» ΞΈΞΈ ΠΌΠ΅ΠΆΠ΄Ρƒ Ρ‚ΠΎΠΊΠΎΠΌ ΠΈ ΠΌΠ°Π³Π½ΠΈΡ‚Π½Ρ‹ΠΌ ΠΏΠΎΠ»Π΅ΠΌ составляСт 90 Β° 0,90 Β°. ΠŸΠΎΠ΄ΡΡ‚Π°Π²Π»ΡΡ ΠΏΡ€ΠΈΠ²Π΅Π΄Π΅Π½Π½ΠΎΠ΅ Π²Ρ‹ΡˆΠ΅ Π²Ρ‹Ρ€Π°ΠΆΠ΅Π½ΠΈΠ΅ для Ρ‚ΠΎΠΊΠ° Π² это ΡƒΡ€Π°Π²Π½Π΅Π½ΠΈΠ΅, ΠΏΠΎΠ»ΡƒΡ‡Π°Π΅ΠΌ

Fpull = Iβ„“B = βˆ’Bvβ„“R (β„“B) = — B2vβ„“2R. Fpull = Iβ„“B = βˆ’Bvβ„“R (β„“B) = — B2vβ„“2R.

20,38

Π‘ΠΈΠ»Π°, создаваСмая Π°Π³Π΅Π½Ρ‚ΠΎΠΌ, тянущим ΡΡ‚Π΅Ρ€ΠΆΠ΅Π½ΡŒ, Ρ€Π°Π²Π½Π° Fpullv, ΠΈΠ»ΠΈ Fpullv, ΠΈΠ»ΠΈ

ΠŸΠΎΡ‚ΡΠ½ΠΈΡ‚Π΅ = Fpullv = βˆ’B2v2β„“2R.ΠŸΠΎΡ‚ΡΠ½ΠΈΡ‚Π΅ = Fpullv = βˆ’B2v2β„“2R.

20,39

ΠœΠΎΡ‰Π½ΠΎΡΡ‚ΡŒ, рассСиваСмая схСмой, составляСт

Pdissipated = I2R = (- Bvβ„“R) 2R = B2v2β„“2R. Pdissipated = I2R = (- Bvβ„“R) 2R = B2v2β„“2R.

20,40

Π’Π°ΠΊΠΈΠΌ ΠΎΠ±Ρ€Π°Π·ΠΎΠΌ, ΠΌΡ‹ Π²ΠΈΠ΄ΠΈΠΌ, Ρ‡Ρ‚ΠΎ Ppull + Pdissipated = 0, Ppull + Pdissipated = 0, Ρ‡Ρ‚ΠΎ ΠΎΠ·Π½Π°Ρ‡Π°Π΅Ρ‚, Ρ‡Ρ‚ΠΎ ΠΌΠΎΡ‰Π½ΠΎΡΡ‚ΡŒ сохраняСтся Π² систСмС, состоящСй ΠΈΠ· Ρ†Π΅ΠΏΠΈ ΠΈ Π°Π³Π΅Π½Ρ‚Π°, ΠΊΠΎΡ‚ΠΎΡ€Ρ‹ΠΉ тянСт ΡΡ‚Π΅Ρ€ΠΆΠ΅Π½ΡŒ. Π’Π°ΠΊΠΈΠΌ ΠΎΠ±Ρ€Π°Π·ΠΎΠΌ, Π² этой систСмС сохраняСтся энСргия.

ACT Научный практичСский тСст 101_CrackACT.com

Указания: Π—Π° ΠΊΠ°ΠΆΠ΄Ρ‹ΠΌ ΠΎΡ‚Ρ€Ρ‹Π²ΠΊΠΎΠΌ слСдуСт нСсколько вопросов.ΠŸΡ€ΠΎΡ‡ΠΈΡ‚Π°Π² ΠΎΡ‚Ρ€Ρ‹Π²ΠΎΠΊ, Π²Ρ‹Π±Π΅Ρ€ΠΈΡ‚Π΅ Π»ΡƒΡ‡ΡˆΠΈΠΉ ΠΎΡ‚Π²Π΅Ρ‚ Π½Π° ΠΊΠ°ΠΆΠ΄Ρ‹ΠΉ вопрос ΠΈ Π·Π°ΠΊΡ€Π°ΡΡŒΡ‚Π΅ ΡΠΎΠΎΡ‚Π²Π΅Ρ‚ΡΡ‚Π²ΡƒΡŽΡ‰ΠΈΠΉ ΠΎΠ²Π°Π» Π² своСм ΠΎΡ‚Π²Π΅Ρ‚Π½ΠΎΠΌ Π΄ΠΎΠΊΡƒΠΌΠ΅Π½Ρ‚Π΅. Π’Ρ‹ ΠΌΠΎΠΆΠ΅Ρ‚Π΅ ΡΡΡ‹Π»Π°Ρ‚ΡŒΡΡ Π½Π° эти ΠΎΡ‚Ρ€Ρ‹Π²ΠΊΠΈ сколько ΡƒΠ³ΠΎΠ΄Π½ΠΎ Ρ€Π°Π·.

Π’Π°ΠΌ НЕ Ρ€Π°Π·Ρ€Π΅ΡˆΠ°Π΅Ρ‚ΡΡ ΠΈΡΠΏΠΎΠ»ΡŒΠ·ΠΎΠ²Π°Ρ‚ΡŒ ΠΊΠ°Π»ΡŒΠΊΡƒΠ»ΡΡ‚ΠΎΡ€ Π² этом тСстС.

PASSAGE II

ΠœΠ°Π³Π½ΠΈΡ‚Π½ΠΎΠ΅ ΠΏΠΎΠ»Π΅ Π—Π΅ΠΌΠ»ΠΈ — ΠΎΠ΄Π½ΠΎ ΠΈΠ· самых Π·Π½Π°Ρ‡ΠΈΡ‚Π΅Π»ΡŒΠ½Ρ‹Ρ… ΠΏΡ€ΠΈΡ€ΠΎΠ΄Π½Ρ‹Ρ… явлСний Π½Π° ΠΏΠ»Π°Π½Π΅Ρ‚Π΅. На протяТСнии Π²Π΅ΠΊΠΎΠ² это ΠΏΠΎΠ»Π΅ использовалось для ΠΏΠΎΠΌΠΎΡ‰ΠΈ Π² Π½Π°Π²ΠΈΠ³Π°Ρ†ΠΈΠΈ ΠΈ исслСдованиях ΠΈ Π±Ρ‹Π»ΠΎ ΠΆΠΈΠ·Π½Π΅Π½Π½ΠΎ Π²Π°ΠΆΠ½Ρ‹ΠΌ для ΠΌΠ½ΠΎΠ³ΠΈΡ… ΠΊΡ€ΡƒΠΏΠ½Ρ‹Ρ… ΠΎΡ‚ΠΊΡ€Ρ‹Ρ‚ΠΈΠΉ. ΠœΠ°Π³Π½ΠΈΡ‚Π½ΠΎΠ΅ ΠΏΠΎΠ»Π΅ Π—Π΅ΠΌΠ»ΠΈ простираСтся Π½Π° нСсколько тысяч миль Π² космос.Он Π·Π°Ρ‰ΠΈΡ‰Π°Π΅Ρ‚ Π—Π΅ΠΌΠ»ΡŽ ΠΎΡ‚ солнСчного Π²Π΅Ρ‚Ρ€Π°, Π·Π°Ρ‰ΠΈΡ‰Π°Π΅Ρ‚ ΠΏΠ»Π°Π½Π΅Ρ‚Ρƒ ΠΎΡ‚ опасных частиц высокой энСргии ΠΈ Ρ€Π°Π΄ΠΈΠ°Ρ†ΠΈΠΈ. Π’ΠΎΡ‡Π½Ρ‹ΠΉ источник ΠΌΠ°Π³Π½ΠΈΡ‚Π½ΠΎΠ³ΠΎ поля Π—Π΅ΠΌΠ»ΠΈ Π½Π΅ ΠΎΠΏΡ€Π΅Π΄Π΅Π»Π΅Π½. Π‘Π»Π΅Π΄ΡƒΡŽΡ‰ΠΈΠ΅ Π΄Π²Π° ΡƒΡ‡Π΅Π½Ρ‹Ρ… ΠΏΡ‹Ρ‚Π°ΡŽΡ‚ΡΡ ΠΎΠ±ΡŠΡΡΠ½ΠΈΡ‚ΡŒ это явлСниС.

Π£Ρ‡Π΅Π½Ρ‹ΠΉ 1

ΠœΠ°Π³Π½ΠΈΡ‚Π½ΠΎΠ΅ ΠΏΠΎΠ»Π΅ Π—Π΅ΠΌΠ»ΠΈ ΠΏΠΎΡ…ΠΎΠΆΠ΅ Π½Π° ΠΌΠ°Π³Π½ΠΈΡ‚Π½ΠΎΠ΅ ΠΏΠΎΠ»Π΅ стСрТнСвого ΠΌΠ°Π³Π½ΠΈΡ‚Π°, Π½Π°ΠΊΠ»ΠΎΠ½Π΅Π½Π½ΠΎΠ³ΠΎ Π½Π° 11 градусов ΠΎΡ‚ оси вращСния Π—Π΅ΠΌΠ»ΠΈ. ΠœΠ°Π³Π½ΠΈΡ‚Π½ΠΎΠ΅ ΠΏΠΎΠ»Π΅ стСрТнСвого ΠΌΠ°Π³Π½ΠΈΡ‚Π° ΠΈΠ»ΠΈ любого Π΄Ρ€ΡƒΠ³ΠΎΠ³ΠΎ Ρ‚ΠΈΠΏΠ° постоянного ΠΌΠ°Π³Π½ΠΈΡ‚Π° создаСтся скоординированными двиТСниями элСктронов Π²Π½ΡƒΡ‚Ρ€ΠΈ Π°Ρ‚ΠΎΠΌΠΎΠ² ΠΆΠ΅Π»Π΅Π·Π°.ΠŸΡ€ΠΈΠ½ΡΡ‚ΠΎ ΡΡ‡ΠΈΡ‚Π°Ρ‚ΡŒ, Ρ‡Ρ‚ΠΎ ядро ​​ЗСмли состоит ΠΈΠ· ΠΌΠ΅Ρ‚Π°Π»Π»ΠΎΠ². Π’Π½ΡƒΡ‚Ρ€Π΅Π½Π½Π΅Π΅ ядро ​​составляСт 70% ΡˆΠΈΡ€ΠΈΠ½Ρ‹ Π›ΡƒΠ½Ρ‹ ΠΈ состоит ΠΈΠ· Ρ‚Π²Π΅Ρ€Π΄ΠΎΠ³ΠΎ ΠΆΠ΅Π»Π΅Π·Π½ΠΎΠ³ΠΎ ΡˆΠ°Ρ€Π°, ΠΊΠΎΡ‚ΠΎΡ€Ρ‹ΠΉ проявляСт свойства Ρ„Π΅Ρ€Ρ€ΠΎΠΌΠ°Π³Π½Π΅Ρ‚ΠΈΠ·ΠΌΠ° (СстСствСнная магнитная тСндСнция ΠΆΠ΅Π»Π΅Π·Π°). Π―Π΄Ρ€ΠΎ ΠΈΠΌΠ΅Π΅Ρ‚ собствСнноС Π²Ρ€Π°Ρ‰Π΅Π½ΠΈΠ΅ ΠΈ ΠΎΠΊΡ€ΡƒΠΆΠ΅Π½ΠΎ Β«ΠΌΠΎΡ€Π΅ΠΌΒ» расплавлСнной ΠΏΠΎΡ€ΠΎΠ΄Ρ‹. ΠœΠ°Π³Π½ΠΈΡ‚Π½ΠΎΠ΅ ΠΏΠΎΠ»Π΅ растСт ΠΈ ослабСваСт, Π° полюса Π—Π΅ΠΌΠ»ΠΈ Π΄Ρ€Π΅ΠΉΡ„ΡƒΡŽΡ‚ ΠΈ ΠΈΠ½ΠΎΠ³Π΄Π° ΠΏΠ΅Ρ€Π΅Π²ΠΎΡ€Π°Ρ‡ΠΈΠ²Π°ΡŽΡ‚ΡΡ ΠΏΠΎ ΠΌΠ΅Ρ€Π΅ измСнСния вращСния ядра. Полюса ΠΌΠ°Π³Π½ΠΈΡ‚Π½ΠΎΠ³ΠΎ поля ΠΌΠ½ΠΎΠ³ΠΎΠΊΡ€Π°Ρ‚Π½ΠΎ Β«ΠΏΠ΅Ρ€Π΅Π²Π΅Ρ€Π½ΡƒΠ»ΠΈΡΡŒΒ» ΠΈΠ·-Π·Π° ΠΊΠΎΠ»Π΅Π±Π°Π½ΠΈΠΉ вращСния Ρ‚Π²Π΅Ρ€Π΄ΠΎΠ³ΠΎ Π²Π½ΡƒΡ‚Ρ€Π΅Π½Π½Π΅Π³ΠΎ сСрдСчника.Π”Ρ€ΡƒΠ³ΠΈΠ΅ колСбания ΠΌΠ°Π³Π½ΠΈΡ‚Π½ΠΎΠ³ΠΎ поля, ΠΊΠΎΡ‚ΠΎΡ€Ρ‹Π΅ ΠΌΠΎΠ³ΡƒΡ‚ ΠΏΡ€ΠΎΠΈΡΡ…ΠΎΠ΄ΠΈΡ‚ΡŒ Π΅ΠΆΠ΅Π΄Π½Π΅Π²Π½ΠΎ, Π² Π·Π½Π°Ρ‡ΠΈΡ‚Π΅Π»ΡŒΠ½ΠΎΠΉ стСпСни ΡΠ²Π»ΡΡŽΡ‚ΡΡ Ρ€Π΅Π·ΡƒΠ»ΡŒΡ‚Π°Ρ‚ΠΎΠΌ Π²ΠΌΠ΅ΡˆΠ°Ρ‚Π΅Π»ΡŒΡΡ‚Π²Π° солнСчного Π²Π΅Ρ‚Ρ€Π°.

Π£Ρ‡Π΅Π½Ρ‹ΠΉ 2

ΠœΠ°Π³Π½ΠΈΡ‚Π½ΠΎΠ΅ ΠΏΠΎΠ»Π΅ Π—Π΅ΠΌΠ»ΠΈ ΠΎΠ±ΡŠΡΡΠ½ΡΠ΅Ρ‚ΡΡ Π΄ΠΈΠ½Π°ΠΌΠΎ-эффСктом Ρ†ΠΈΡ€ΠΊΡƒΠ»ΠΈΡ€ΡƒΡŽΡ‰Π΅Π³ΠΎ элСктричСского Ρ‚ΠΎΠΊΠ° Π² расплавлСнном внСшнСм ядрС. ЭлСктричСскиС Ρ‚ΠΎΠΊΠΈ Π²Ρ‹Π·Ρ‹Π²Π°ΡŽΡ‚ ΠΌΠ°Π³Π½ΠΈΡ‚Π½Ρ‹Π΅ поля; поэтому Ρ†ΠΈΡ€ΠΊΡƒΠ»ΠΈΡ€ΡƒΡŽΡ‰ΠΈΠ΅ элСктричСскиС Ρ‚ΠΎΠΊΠΈ Π² расплавлСнном мСталличСском ядрС Π—Π΅ΠΌΠ»ΠΈ ΡΠ²Π»ΡΡŽΡ‚ΡΡ источником ΠΌΠ°Π³Π½ΠΈΡ‚Π½ΠΎΠ³ΠΎ поля. Когда проводящая ΠΆΠΈΠ΄ΠΊΠΎΡΡ‚ΡŒ Ρ‚Π΅Ρ‡Π΅Ρ‚ Ρ‡Π΅Ρ€Π΅Π· ΡΡƒΡ‰Π΅ΡΡ‚Π²ΡƒΡŽΡ‰Π΅Π΅ ΠΌΠ°Π³Π½ΠΈΡ‚Π½ΠΎΠ΅ ΠΏΠΎΠ»Π΅, ΠΈΠ½Π΄ΡƒΡ†ΠΈΡ€ΡƒΡŽΡ‚ΡΡ элСктричСскиС Ρ‚ΠΎΠΊΠΈ, создавая Π΄Ρ€ΡƒΠ³ΠΎΠ΅ ΠΌΠ°Π³Π½ΠΈΡ‚Π½ΠΎΠ΅ ΠΏΠΎΠ»Π΅.Когда это ΠΌΠ°Π³Π½ΠΈΡ‚Π½ΠΎΠ΅ ΠΏΠΎΠ»Π΅ усиливаСт исходноС ΠΌΠ°Π³Π½ΠΈΡ‚Π½ΠΎΠ΅ ΠΏΠΎΠ»Π΅, создаСтся Π΄ΠΈΠ½Π°ΠΌΠΎ-машина, которая ΠΏΠΎΠ΄Π΄Π΅Ρ€ΠΆΠΈΠ²Π°Π΅Ρ‚ сСбя.

ΠΠ°Ρ…ΠΎΠ΄ΡΡΡŒ Π½Π° горячСм ΠΆΠ΅Π»Π΅Π·Π½ΠΎΠΌ Π²Π½ΡƒΡ‚Ρ€Π΅Π½Π½Π΅ΠΌ ядрС, расплавлСнноС внСшнСС ядро ​​ЗСмли взбалтываСтся ΠΈ двиТСтся. Π’Π½Π΅ΡˆΠ½Π΅Π΅ ядро ​​такТС ΠΈΠΌΠ΅Π΅Ρ‚ Ρ†ΠΈΠΊΠ»ΠΎΠ½Ρ‹ ΠΈΠ»ΠΈ Π²ΠΎΠ΄ΠΎΠ²ΠΎΡ€ΠΎΡ‚Ρ‹, ΠΏΠΈΡ‚Π°Π΅ΠΌΡ‹Π΅ эффСктами ΠšΠΎΡ€ΠΈΠΎΠ»ΠΈΡΠ° вращСния Π—Π΅ΠΌΠ»ΠΈ. Π­Ρ‚ΠΈ слоТныС ΠΈ нСпрСдсказуСмыС двиТСния Π³Π΅Π½Π΅Ρ€ΠΈΡ€ΡƒΡŽΡ‚ Ρ„Π»ΡƒΠΊΡ‚ΡƒΠΈΡ€ΡƒΡŽΡ‰Π΅Π΅ ΠΌΠ°Π³Π½ΠΈΡ‚Π½ΠΎΠ΅ ΠΏΠΎΠ»Π΅. Π’Π½Π΅ΡˆΠ½Π΅Π΅ ядро ​​бурлит, закручиваСтся ΠΈ Ρ‚ΡƒΡ€Π±ΡƒΠ»Π΅Π½Ρ‚Π½ΠΎ, Ρ‡Ρ‚ΠΎ Π±Ρ‹Π»ΠΎ ΠΎΠ±Π½Π°Ρ€ΡƒΠΆΠ΅Π½ΠΎ ΠΏΠΎ постоянным измСнСниям ΠΈ Ρ€Π°Π·Π²ΠΎΡ€ΠΎΡ‚Π°ΠΌ полярности Π½Π° протяТСнии всСй истории ΠΏΠ»Π°Π½Π΅Ρ‚Ρ‹.ΠšΡ€ΠΎΠΌΠ΅ Ρ‚ΠΎΠ³ΠΎ, ΠΆΠ΅Π»Π΅Π·ΠΎ ΠΈΠΌΠ΅Π΅Ρ‚ ΠΎΡΠΎΠ±ΡƒΡŽ характСристику. Когда ΠΎΠ½ΠΎ горячСС, Ρ‡Π΅ΠΌ 1043 К, Π΅Π³ΠΎ Ρ‚Π΅ΠΌΠΏΠ΅Ρ€Π°Ρ‚ΡƒΡ€Π° ΠšΡŽΡ€ΠΈ, ΠΆΠ΅Π»Π΅Π·ΠΎ тСряСт свои ΠΌΠ°Π³Π½ΠΈΡ‚Π½Ρ‹Π΅ свойства. Π‘Π»Π΅Π΄ΠΎΠ²Π°Ρ‚Π΅Π»ΡŒΠ½ΠΎ, ΠΌΠ°Π³Π½ΠΈΡ‚Π½ΠΎΠ΅ ΠΏΠΎΠ»Π΅ Π—Π΅ΠΌΠ»ΠΈ Π²Ρ‹Π·Π²Π°Π½ΠΎ Π½Π΅ Π½Π°ΠΌΠ°Π³Π½ΠΈΡ‡Π΅Π½Π½Ρ‹ΠΌΠΈ отлоТСниями ΠΆΠ΅Π»Π΅Π·Π°, Π° Π² основном элСктричСскими Ρ‚ΠΎΠΊΠ°ΠΌΠΈ Π² ΠΆΠΈΠ΄ΠΊΠΎΠΌ внСшнСм ядрС.

1. КакоС ΠΈΠ· ΡΠ»Π΅Π΄ΡƒΡŽΡ‰ΠΈΡ… ΡƒΡ‚Π²Π΅Ρ€ΠΆΠ΄Π΅Π½ΠΈΠΉ ΠΎ ядрС Π—Π΅ΠΌΠ»ΠΈ ΠΏΠΎΠ΄Ρ€Π°Π·ΡƒΠΌΠ΅Π²Π°Π»ΠΎΡΡŒ Π£Ρ‡Π΅Π½Ρ‹ΠΌ 2?

A. Π’Π½ΡƒΡ‚Ρ€Π΅Π½Π½Π΅Π΅ ядро ​​ЗСмли прСдставляСт собой Π²Ρ€Π°Ρ‰Π°ΡŽΡ‰ΡƒΡŽΡΡ массу ΠΈΠ· ΠΆΠ΅Π»Π΅Π·Π°.
B. Π’Π½ΡƒΡ‚Ρ€Π΅Π½Π½Π΅Π΅ ядро ​​ЗСмли нСсСт ΠΏΠΎΠ»Π½ΡƒΡŽ ΠΎΡ‚Π²Π΅Ρ‚ΡΡ‚Π²Π΅Π½Π½ΠΎΡΡ‚ΡŒ Π·Π° ΠΌΠ°Π³Π½ΠΈΡ‚Π½ΠΎΠ΅ ΠΏΠΎΠ»Π΅.
C. Π―Π΄Ρ€ΠΎ Π—Π΅ΠΌΠ»ΠΈ ΠΆΠ΅Π»Π΅Π·Π½ΠΎΠ΅ ΠΈ ΠΎΠ±Π»Π°Π΄Π°Π΅Ρ‚ свойствами, ΠΏΠΎΠ΄ΠΎΠ±Π½Ρ‹ΠΌΠΈ стСрТнСвому ΠΌΠ°Π³Π½ΠΈΡ‚Ρƒ.
D. Π―Π΄Ρ€ΠΎ Π—Π΅ΠΌΠ»ΠΈ ΠΈΠΌΠ΅Π΅Ρ‚ Ρ‚Π΅ΠΌΠΏΠ΅Ρ€Π°Ρ‚ΡƒΡ€Ρƒ Π²Ρ‹ΡˆΠ΅ 1043 К.

2. Π’ Π½Π°ΡƒΡ‡Π½ΠΎΠΉ ΡΡ‚Π°Ρ‚ΡŒΠ΅ Π³ΠΎΠ²ΠΎΡ€ΠΈΠ»ΠΎΡΡŒ: Β«Π‘ 1848 Π³ΠΎΠ΄Π°, ΠΊΠΎΠ³Π΄Π° Π±Ρ‹Π»Π° Π²ΠΏΠ΅Ρ€Π²Ρ‹Π΅ ΠΈΠ·ΠΌΠ΅Ρ€Π΅Π½Π° сила ΠΌΠ°Π³Π½ΠΈΡ‚Π½ΠΎΠ³ΠΎ поля Π—Π΅ΠΌΠ»ΠΈ, ΠΏΠΎΠ»Π΅ потСряло 10% своСй напряТСнности. » Какая ΠΈΠ· Ρ‚ΠΎΡ‡Π΅ΠΊ зрСния ΡƒΡ‡Π΅Π½Ρ‹Ρ…, Ссли таковая имССтся, согласна (согласна) с этим ΡƒΡ‚Π²Π΅Ρ€ΠΆΠ΄Π΅Π½ΠΈΠ΅ΠΌ?

F. Волько ΡƒΡ‡Π΅Π½Ρ‹ΠΉ 1.
G. Волько ΡƒΡ‡Π΅Π½Ρ‹ΠΉ 2.
H. Π£Ρ‡Π΅Π½Ρ‹ΠΉ 1 ΠΈ 2.
J.Ни Π£Ρ‡Π΅Π½Ρ‹ΠΉ 1, Π½ΠΈ 2.

3. Π˜ΡΡΠ»Π΅Π΄ΠΎΠ²Π°Ρ‚Π΅Π»ΠΈ Π·Π°ΠΌΠ΅Ρ‡Π°ΡŽΡ‚, Ρ‡Ρ‚ΠΎ вулканичСскиС ΠΏΠΎΡ€ΠΎΠ΄Ρ‹ Π΄Π΅ΠΌΠΎΠ½ΡΡ‚Ρ€ΠΈΡ€ΡƒΡŽΡ‚ рСгулярныС ΠΈ прСдсказуСмыС измСнСния Π² своих ΠΌΠ°Π³Π½ΠΈΡ‚Π½Ρ‹Ρ… свойствах Π² зависимости ΠΎΡ‚ ΠΈΡ… возраста. Π‘ ΠΊΠ°ΠΊΠΈΠΌ ΠΈΠ· ΡΠ»Π΅Π΄ΡƒΡŽΡ‰ΠΈΡ… ΡƒΡ‚Π²Π΅Ρ€ΠΆΠ΄Π΅Π½ΠΈΠΉ ΠΎ вариациях ΠΎΠ±Π° ΡƒΡ‡Π΅Π½Ρ‹Ρ…, скорСС всСго, согласятся?

A. ЭлСктричСскиС Ρ‚ΠΎΠΊΠΈ Π² ΠΆΠΈΠ΄ΠΊΠΎΠΌ внСшнСм ядрС Π²Ρ‹Π·Ρ‹Π²Π°ΡŽΡ‚ ΠΈΠ·ΠΌΠ΅Π½Π΅Π½ΠΈΠ΅ ΠΌΠ°Π³Π½ΠΈΡ‚Π½Ρ‹Ρ… свойств вулканичСской ΠΏΠΎΡ€ΠΎΠ΄Ρ‹.
B. Π£Ρ€ΠΎΠ²Π½ΠΈ солнСчного Π²Π΅Ρ‚Ρ€Π° Π²ΠΎ врСмя создания Π³ΠΎΡ€Π½Ρ‹Ρ… ΠΏΠΎΡ€ΠΎΠ΄ ΠΎΠΏΡ€Π΅Π΄Π΅Π»ΡΡŽΡ‚ ΠΌΠ°Π³Π½ΠΈΡ‚Π½Ρ‹Π΅ свойства Π³ΠΎΡ€Π½Ρ‹Ρ… ΠΏΠΎΡ€ΠΎΠ΄.
C. Π Π°Π·Π»ΠΈΡ‡Π½Ρ‹Π΅ ΠΌΠ°Π³Π½ΠΈΡ‚Π½Ρ‹Π΅ свойства вулканичСских ΠΏΠΎΡ€ΠΎΠ΄ ΡΠ²Π»ΡΡŽΡ‚ΡΡ Ρ€Π΅Π·ΡƒΠ»ΡŒΡ‚Π°Ρ‚ΠΎΠΌ состояния ΠΊΠΎΠ»Π΅Π±Π»ΡŽΡ‰Π΅Π³ΠΎΡΡ ΠΌΠ°Π³Π½ΠΈΡ‚Π½ΠΎΠ³ΠΎ поля Π—Π΅ΠΌΠ»ΠΈ Π² Ρ‚ΠΎ врСмя, ΠΊΠΎΠ³Π΄Π° ΠΏΠΎΡ€ΠΎΠ΄Ρ‹ ΠΎΡ…Π»Π°ΠΆΠ΄Π°Π»ΠΈΡΡŒ.
D. ΠœΠ°Π³Π½ΠΈΡ‚Π½ΠΎΠ΅ ΠΏΠΎΠ»Π΅ Π—Π΅ΠΌΠ»ΠΈ Π·Π°Ρ‰ΠΈΡ‰Π°Π΅Ρ‚ Π΅Π΅ ΠΎΡ‚ солнСчного Π²Π΅Ρ‚Ρ€Π°, Ρ‚Π΅ΠΌ самым измСняя ΠΌΠ°Π³Π½ΠΈΡ‚Π½Ρ‹Π΅ свойства вулканичСских ΠΏΠΎΡ€ΠΎΠ΄.

4. Π£Ρ‡Π΅Π½Ρ‹Π΅ 1 ΠΈ 2, скорСС всСго, Π½Π΅ согласятся с ΠΊΠ°ΠΊΠΈΠΌ ΠΈΠ· ΡΠ»Π΅Π΄ΡƒΡŽΡ‰ΠΈΡ… ΡƒΡ‚Π²Π΅Ρ€ΠΆΠ΄Π΅Π½ΠΈΠΉ?

F. Π‘ΠΈΠ»Π° ΠΌΠ°Π³Π½ΠΈΡ‚Π½ΠΎΠ³ΠΎ поля Π—Π΅ΠΌΠ»ΠΈ колСблСтся со Π²Ρ€Π΅ΠΌΠ΅Π½Π΅ΠΌ.
G. ΠŸΠΎΠ»ΡΡ€Π½ΠΎΡΡ‚ΡŒ ΠΌΠ°Π³Π½ΠΈΡ‚Π½ΠΎΠ³ΠΎ поля Π—Π΅ΠΌΠ»ΠΈ ΠΌΠΎΠΆΠ΅Ρ‚ ΠΌΠ΅Π½ΡΡ‚ΡŒΡΡ со Π²Ρ€Π΅ΠΌΠ΅Π½Π΅ΠΌ.
H. Π’Π½ΡƒΡ‚Ρ€Π΅Π½Π½Π΅Π΅ ядро ​​ЗСмли ΠΎΠΊΡ€ΡƒΠΆΠ΅Π½ΠΎ ΠΆΠΈΠ΄ΠΊΠΈΠΌ внСшним ядром.
J. Π’Π½ΡƒΡ‚Ρ€Π΅Π½Π½Π΅Π΅ ядро ​​ЗСмли ΠΎΠ±Π»Π°Π΄Π°Π΅Ρ‚ ΠΌΠ°Π³Π½ΠΈΡ‚Π½Ρ‹ΠΌΠΈ свойствами.

5. Π Π°Π·Π»ΠΈΡ‡Π°ΡŽΡ‚ΡΡ Π»ΠΈ Π£Ρ‡Π΅Π½Ρ‹Π΅ Π² своСм описании ΠΌΠ°Π³Π½ΠΈΡ‚Π½ΠΎΠ³ΠΎ поля Π—Π΅ΠΌΠ»ΠΈ?

A. Π”Π°; Π£Ρ‡Π΅Π½Ρ‹ΠΉ 1 ΡƒΡ‚Π²Π΅Ρ€ΠΆΠ΄Π°Π΅Ρ‚, Ρ‡Ρ‚ΠΎ ΠΌΠ°Π³Π½ΠΈΡ‚Π½ΠΎΠ΅ ΠΏΠΎΠ»Π΅ Π—Π΅ΠΌΠ»ΠΈ ΠΈΠΌΠ΅Π΅Ρ‚ большоС Π·Π½Π°Ρ‡Π΅Π½ΠΈΠ΅, Π° Π£Ρ‡Π΅Π½Ρ‹ΠΉ 2 — Π½Π΅Ρ‚.
B. Π”Π°; Π£Ρ‡Π΅Π½Ρ‹ΠΉ 1 ΡƒΡ‚Π²Π΅Ρ€ΠΆΠ΄Π°Π΅Ρ‚, Ρ‡Ρ‚ΠΎ ΠΌΠ°Π³Π½ΠΈΡ‚Π½ΠΎΠ΅ ΠΏΠΎΠ»Π΅ Π—Π΅ΠΌΠ»ΠΈ создаСтся скоординированными двиТСниями элСктронов Π²Π½ΡƒΡ‚Ρ€ΠΈ Π°Ρ‚ΠΎΠΌΠΎΠ² ΠΆΠ΅Π»Π΅Π·Π°, Π° Π£Ρ‡Π΅Π½Ρ‹ΠΉ 2 — Π½Π΅Ρ‚.
C. НСт; Оба ΡƒΡ‡Π΅Π½Ρ‹Ρ… ΡƒΡ‚Π²Π΅Ρ€ΠΆΠ΄Π°ΡŽΡ‚, Ρ‡Ρ‚ΠΎ ΠΌΠ°Π³Π½ΠΈΡ‚Π½ΠΎΠ΅ ΠΏΠΎΠ»Π΅ Π—Π΅ΠΌΠ»ΠΈ создаСтся ΠΈΠ· Ρ†ΠΈΡ€ΠΊΡƒΠ»ΠΈΡ€ΡƒΡŽΡ‰ΠΈΡ… элСктричСских Ρ‚ΠΎΠΊΠΎΠ² Π² расплавлСнном внСшнСм ядрС Π—Π΅ΠΌΠ»ΠΈ.
D. НСт; Ни ΠΎΠ΄ΠΈΠ½ Π£Ρ‡Π΅Π½Ρ‹ΠΉ Π½Π΅ обсуТдаСт ΠΏΠΎΠ΄Ρ€ΠΎΠ±Π½ΠΎ ΠΌΠ°Π³Π½ΠΈΡ‚Π½ΠΎΠ΅ ΠΏΠΎΠ»Π΅ Π—Π΅ΠΌΠ»ΠΈ.

6. ΠŸΡ€Π΅Π΄ΠΏΠΎΠ»ΠΎΠΆΠΈΠΌ, Π±Ρ‹Π» ΠΈΠ·ΠΎΠ±Ρ€Π΅Ρ‚Π΅Π½ Π½ΠΎΠ²Ρ‹ΠΉ Ρ‚ΠΈΠΏ Π΄Π°Ρ‚Ρ‡ΠΈΠΊΠ°, ΠΊΠΎΡ‚ΠΎΡ€Ρ‹ΠΉ ΠΌΠΎΠΆΠ΅Ρ‚ ΠΎΠ±Π½Π°Ρ€ΡƒΠΆΠΈΠ²Π°Ρ‚ΡŒ элСктричСскиС Ρ‚ΠΎΠΊΠΈ Π²ΠΎ внСшнСм сСрдСчникС. Π­Ρ‚Π° новая тСхнология ΠΌΠΎΠ³Π»Π° Π±Ρ‹:

F. ΡΠΎΠΎΡ‚Π²Π΅Ρ‚ΡΡ‚Π²ΠΎΠ²Π°Ρ‚ΡŒ Ρ‚ΠΎΠ»ΡŒΠΊΠΎ Ρ‚ΠΎΡ‡ΠΊΠ΅ зрСния Π£Ρ‡Π΅Π½ΠΎΠ³ΠΎ 1.
G. согласуСтся Ρ‚ΠΎΠ»ΡŒΠΊΠΎ с Ρ‚ΠΎΡ‡ΠΊΠΎΠΉ зрСния ΡƒΡ‡Π΅Π½ΠΎΠ³ΠΎ 2.
H. Π΄ΠΎΠ»ΠΆΠ½Ρ‹ ΡΠΎΠΎΡ‚Π²Π΅Ρ‚ΡΡ‚Π²ΠΎΠ²Π°Ρ‚ΡŒ Ρ‚ΠΎΡ‡ΠΊΠ΅ зрСния ΡƒΡ‡Π΅Π½Ρ‹Ρ… 1 ΠΈ 2.
J. Π½Π΅ ΠΈΠΌΠ΅ΡŽΡ‚ ΠΎΡ‚Π½ΠΎΡˆΠ΅Π½ΠΈΡ Π½ΠΈ ΠΊ ΠΎΠ΄Π½ΠΎΠΉ Ρ‚ΠΎΡ‡ΠΊΠ΅ зрСния ΡƒΡ‡Π΅Π½Ρ‹Ρ….

7. Богласно Π£Ρ‡Π΅Π½ΠΎΠΌΡƒ 1, ΠΊΠ°ΠΊΠΎΠ΅ ΠΈΠ· ΡΠ»Π΅Π΄ΡƒΡŽΡ‰ΠΈΡ… ΠΏΡ€Π΅Π΄ΠΏΠΎΠ»ΠΎΠΆΠ΅Π½ΠΈΠΉ ΠΎΠ± источникС ΠΌΠ°Π³Π½ΠΈΡ‚Π½ΠΎΠ³ΠΎ поля Π—Π΅ΠΌΠ»ΠΈ являСтся основным нСдостатком Ρ‚Π΅ΠΎΡ€ΠΈΠΈ Π£Ρ‡Π΅Π½ΠΎΠ³ΠΎ 2?

A.

Leave a Reply

Π”ΠΎΠ±Π°Π²ΠΈΡ‚ΡŒ ΠΊΠΎΠΌΠΌΠ΅Π½Ρ‚Π°Ρ€ΠΈΠΉ

Π’Π°Ρˆ адрСс email Π½Π΅ Π±ΡƒΠ΄Π΅Ρ‚ ΠΎΠΏΡƒΠ±Π»ΠΈΠΊΠΎΠ²Π°Π½. ΠžΠ±ΡΠ·Π°Ρ‚Π΅Π»ΡŒΠ½Ρ‹Π΅ поля ΠΏΠΎΠΌΠ΅Ρ‡Π΅Π½Ρ‹ *